2ちゃんねる ■掲示板に戻る■ 全部 1- 最新50    

■ このスレッドは過去ログ倉庫に格納されています

不等式への招待 第9章

1 :不等式ヲタ ( ゚∀゚):2017/09/13(水) 11:20:03.95 ID:i1anpb+k.net
ある人は蝶を集め、ある人は切手を収集し、ある人は不等式を集める…
          ___          ----- 参考文献〔3〕 P.65 -----
    |┃三 ./  ≧ \   
    |┃   |::::  \ ./ | 
    |┃ ≡|::::: (● (● |  不等式と聞ゐちゃぁ
____.|ミ\_ヽ::::... .ワ......ノ     黙っちゃゐられねゑ…
    |┃=__    \           ハァハァ
    |┃ ≡ )  人 \ ガラッ

【まとめWiki】 http://wiki.livedoor.jp/loveinequality/

【過去スレ】
・不等式スレッド (第1章) http://science3.2ch.net/test/read.cgi/math/1072510082/
・不等式への招待 第2章 http://science6.2ch.net/test/read.cgi/math/1105911616/
・不等式への招待 第3章 http://science6.2ch.net/test/read.cgi/math/1179000000/
・不等式への招待 第4章 http://science6.2ch.net/test/read.cgi/math/1245060000/
・不等式への招待 第5章 http://uni.2ch.net/test/read.cgi/math/1287932216/
・不等式への招待 第6章 http://uni.2ch.net/test/read.cgi/math/1332950303/
・不等式への招待 第7章 http://rio2016.2ch.net/test/read.cgi/math/1362834879/
・不等式への招待 第8章 http://rio2016.2ch.net/test/read.cgi/math/1498378859/
・過去スレのミラー置き場 http://cid-d357afbb34f5b26f.skydrive.live.com/browse.aspx/.Public/

【姉妹サイト】
キャスフィ 高校数学板 不等式スレ  http://www.casphy.com/bbs/test/read.cgi/highmath/1169210077/
キャスフィ 高校数学板 不等式スレ2 http://www.casphy.com/bbs/test/read.cgi/highmath/1359202700/

【wikiなど】
Inequality (mathematics)
https://en.wikipedia.org/wiki/Inequality_(mathematics)
List of inequalities
https://en.wikipedia.org/wiki/List_of_inequalities
List of triangle inequalities
https://en.wikipedia.org/wiki/List_of_triangle_inequalities
Wolfram MathWorld
http://mathworld.wolfram.com/topics/Inequalities.html

2 :不等式ヲタ ( ゚∀゚):2017/09/13(水) 11:20:42.17 ID:i1anpb+k.net
【不等式の和書】

[1] 不等式(数学クラシックス11),Hardy, Littlewood, Polya,丸善出版,2003年
   http://amazon.jp/o/ASIN/4431710566
[2] 不等式(数学選書),大関信雄・青木雅計,槇書店,1967年(絶版)
[3] 不等式への招待(数学ゼミナール6),大関信雄・大関清太,近代科学社,1987年
   http://amazon.jp/dp/4844372661
[4] 不等式入門(数学のかんどころシリーズ9),大関清太,共立出版,2012年
   http://www.kyoritsu-pub.co.jp/bookdetail/9784320019898
[5] 不等式入門(数学ライブラリー教養篇4),渡部隆一,森北出版,2005年
   http://amazon.jp/o/ASIN/4627010494
[6] 不等式の工学への応用、海津聰、森北出版,2004年
   http://amazon.jp/o/ASIN/4627075812
[7] 不等式(モノグラフ4),染取弘,科学新興新社,1990年
   http://amazon.jp/o/ASIN/4894281740
[8] 不等式 〜 21世紀の代数的不等式論 〜,安藤哲哉,数学書房,2012年
  http://amazon.jp/dp/4903342700、(正誤表+補遺)http://www.math.s.chiba-u.ac.jp/~ando/
[9] 美しい不等式の世界: 数学オリンピックの問題を題材として,佐藤淳郎(訳),朝倉書店,2013年
  http://amazon.jp/dp/4254111371
[10] 思考力を鍛える不等式(大学への数学・別冊)、栗田哲也、東京出版、2014年
  http://www.amazon.co.jp/dp/4887422091

【不等式のpdf】

[1] Vasile Cîrtoaje、Mathematical Inequalities. http://ac.upg-ploiesti.ro/vcirtoaje/vcirtoaje.php
[2] 柳田五夫、初等的な不等式TUVなど. http://izumi-math.jp/I_Yanagita/I_Yanagita.html

【おもな埋蔵地】

[1] JIPAM https://www.emis.de/journals/JIPAM/
[2] MIA http://mia.ele-math.com/volume/20
[3] AoPS https://artofproblemsolving.com/community
[4] Maths problems http://webee.technion.ac.il/people/aditya/www.kalva.demon.co.uk/index.html
[5] IMO http://www.imo-official.org/problems.aspx
[6] American Mathematical Monthly Problems http://www.mat.uniroma2.it/~tauraso/AMM/amm.html

※ その他の参考文献などは、まとめWikiを参照 http://wiki.livedoor.jp/loveinequality/

3 :不等式ヲタ ( ゚∀゚):2017/09/13(水) 11:21:03.13 ID:i1anpb+k.net
諸君 私は不等式が好きだ
諸君 私は不等式が大好きだ

改造が好きだ 改良が好きだ 拡張が好きだ

AM-GMで Cauchyで Holderで Jensenで Schurで
Chebyshevで rearrangementで Bernoulliで
Muirheadで Karamataで Maclaurinで ぬるぽビッチで

この地上に現れるありとあらゆる不等式が大好きだ

大小順をそろえた歩兵の横隊を 並べ替え不等式で蹂躙するのが好きだ
恐慌状態の新兵が分母にAM-GMを誤用して 不等号の向きを何度も何度も間違えている様など感動すら覚える

糸口の見つからない不等式に滅茶苦茶に悩まされるのが好きだ
必死に悩んだ不等式が成立しない例を挙げられていく様はとても悲しいものだ

君達は一体何を望んでいる? 更なる不等式を望むか?

     『不等式! 不等式! 不等式!』

よろしい ならば証明だ!

              rv―v―、 r-v-v
        r、      ノ     も( ノ ま (      ,ィx
      (\\(^}   ) !! 厳. っ ( ) だ (   /)///7
      {^ヽ^ヽ {   )  し. と ( ) だ (  / 'ヽ /
        \ `Y ノ}_  ハ  く  ノ 乂  ノ  {. 〈 /
           〉,r彡ハ  _>  < /  ま ( 人_ノ〉
           V   ∨ !! 改 も () !! だ( /  7 /
          'v   V  良 っ ().  だ(/    /
             'v   V .を と 人_,ノ〈    /
           V   V rfテ弐ミk /   }'
                'v   ',仔r=r弌リ'   /
                'v   '({ ヾ二フ,j'   /
               }    j个ー‐个ト,  /
             }> / />ュ<ト、\ノ{
                _j/  / | / :| | \\
              _,>、__, イ>\/ _」/\ ̄{_
          /  /:::::| \/__,>|:::::∧  {
            /|  ./:::::/  厂     |::::::::∧ |\
         / :| /:::::/   |o      〔::::::::::::∧.|  \
       / / /:::::/   :|o    丿:|:::::::::::::∧    \

4 :132人目の素数さん:2017/09/13(水) 20:06:57.03 ID:i1anpb+k.net
[第8章 469]

> for reals
> [1] (a^2+2)(b^2+2)(c^2+2) >= (1+a+b)(1+b+c)(1+c+a)
> [2] ((a^2+3)(b^2+3)(c^2+3))^2 >= 512(a+b)(b+c)(c+a)
>
> for nonnegarives
> [3] (a^2+2)(b^2+2)(c^2+2) >= 3(a+b+c)^2+(abc-1)^2
> [4] (x^2+2)(y^2+2)(z^2+2) >= 4(x^2+y^2+z^2)+5(xy+yz+zx)+(xyz-1)^2
> [5] (a^2+2)(b^2+2)(c^2+2) >= 4(a^2+b^2+c^2)+5(ab+bc+ca)+(abc(a-1)^2(b-1)^2(c-1)^2)^(1/3)
>
> AOPS
> [1], [2] : c6h588096p3481394
> [3] : c6h4830p15309
> [4], [5] : c6h581954p3438879
>
> 他にもいろいろ

5 :132人目の素数さん:2017/09/13(水) 20:07:13.99 ID:i1anpb+k.net
[第8章 977、991]
> [疑問]-----------------------------------------------
> a, b, c >0 に対して、
> M(a,b,c) ≧ (a^2+ab+b^2)(b^2+bc+c^2)(c^2+ca+a^2) ≧ m(a,b,c)
> -----------------------------------------------------
>
> AM-GMで m(a,b,c) = 27(abc)^2 を得るけど、もっとキツく締め上げたいのでござる。
>
> L = a^2b + b^2c + c^2a
> R = ab^2 + bc^2 + ca^2
>
> (a^2+ab+b^2)(b^2+bc+c^2)(c^2+ca+a^2)
> = L^2 + LR + R^2
> = (s^2)(t^2) - (s^3)u - t^3
>
>
> とりあえず少し進展したのでパピコ。 Caushyの拡張より、
>
> (a^2+ab+b^2)(b^2+bc+c^2)(c^2+ca+a^2)
> = (ab+b^2+a^2)(b^2+bc+c^2)(a^2+c^2+ca)
> ≧ (ab+bc+ca)^3
> = t^3
>
> AM-GMで 27(abc)^2 = 27u^2 としたよりもマシになった。
>
>  m(a,b,c) = (ab+bc+ca)^3 ≧ 27(abc)^2
>
> が、以下のように分割すると、非負値の和ばかりで、ずいぶんとゆるゆるなうんちでござる。
>
> (a^2+ab+b^2)(b^2+bc+c^2)(c^2+ca+a^2) - t^3
> = (s^2)(t^2) - (s^3)u - 2(t^3)
> = (t^2-3su)F_0 + 2suF_0 + (u^2)F_{-2} + u(st-9u)
> ≧ 0

6 :132人目の素数さん:2017/09/13(水) 23:45:33.05 ID:i1anpb+k.net
>>5
(a^2+ab+b^2)(b^2+bc+c^2)(c^2+ca+a^2) ≧ (a^2+b^2)(b^2+c^2)(c^2+a^2) ≧ {(a-b)(b-c)(c-a)}^2.
(a^2+ab+b^2)(b^2+bc+c^2)(c^2+ca+a^2) ≧ 3(a^2b+b^2c+c^2a)(ab^2+bc^2+ca^2) ≧ (ab+bc+ca)^3 ≧ 3abc(a+b+c)(ab+bc+ca) ≧ 27(abc)^2.

7 :132人目の素数さん:2017/09/13(水) 23:57:53.62 ID:i1anpb+k.net
>>5
[第2章 136-138]
> 非負実数 a, b, c に対して
> (a^2+ab+b^2)(b^2+bc+c^2)(c^2+ca+a^2)
> ≧ (27/64)[(a+b)(b+c)(c+a)]^2 ≧ (1/3)[(a+b+c)(ab+bc+ca)]^2 ≧ (ab+bc+ca)^3

12年前に自分が作っていた模様…

8 :132人目の素数さん:2017/09/14(木) 02:17:46.00 ID:PaGBsMiT.net
ウインナーコーヒーにウインナーが入ってないのと同じ
すべては騙し

9 :132人目の素数さん:2017/09/14(木) 07:18:42.24 ID:mi/0+iqR.net
>>4
[1]コーシーで
(1+aa+1)(1+1+bb)≧(1+a+b)^2,etc.

[2]AM-GMで
(aa+3)(bb+3)=(aa+1+1+1)(1+bb+1+1)≧(a+b+1+1)^2 ≧ 4(a+b)(1+1),
 [第8章.994]

>>6
p = aab+bbc+cca,q = abb+bcc+caa とおくと
アイゼンシュタイン整数で
(aa+ab+bb)(bb+bc+cc)(cc+ca+aa)= pp+pq+qq
=(3/4)(p+q)^2 + (1/4)(p-q)^2,
≧ 3pq +(1/4)竸2,
=(a-b)(b-c)(c-a)= q-p,

次は(1+1+1)(aab+bbc+cca)(abb+bcc+caa)の3つでコーシー

10 :132人目の素数さん:2017/09/14(木) 09:50:04.36 ID:Ew79uiN/.net
>>5
M(a,b,c) について、

(a^2+ab+b^2)(b^2+bc+c^2)(c^2+ca+a^2)
≦ ab(a+b) + bc(b+c) + ca(c+a)

(a^2+ab+b^2)(b^2+bc+c^2)(c^2+ca+a^2)
≦ (a^2+2b^2+c^2+ab+bc)(b^2+2c^2+a^2+bc+ca)(c^2+2a^2+b^2+ca+ab)/8
≦ (1/27)*(2a^2+2b^2+2c^2+ab+bc+ca)^3

きれいな式で押さえたいんだが… ('A`)

11 :132人目の素数さん:2017/09/14(木) 09:54:59.15 ID:Ew79uiN/.net
>>6-7
(a^2+ab+b^2)(b^2+bc+c^2)(c^2+ca+a^2) ≧ (27/64)[(a+b)(b+c)(c+a)]^2 ≧ (1/3)[(a+b+c)(ab+bc+ca)]^2 ≧ (ab+bc+ca)^3
(a^2+ab+b^2)(b^2+bc+c^2)(c^2+ca+a^2) ≧ (a^2+b^2)(b^2+c^2)(c^2+a^2) ≧ {(a-b)(b-c)(c-a)}^2.
(a^2+ab+b^2)(b^2+bc+c^2)(c^2+ca+a^2) ≧ 3(a^2b+b^2c+c^2a)(ab^2+bc^2+ca^2) ≧ (ab+bc+ca)^3 ≧ 3abc(a+b+c)(ab+bc+ca) ≧ 27(abc)^2.

[疑問]
右辺の 3(a^2b+b^2c+c^2a)(ab^2+bc^2+ca^2) は、
 (27/64)[(a+b)(b+c)(c+a)]^2 や (a^2+b^2)(b^2+c^2)(c^2+a^2)
より大きそうなんだけど、うまく証明できない。
s, t, u に置き換えて差を計算してみたけど、次数が高くて…

12 :132人目の素数さん:2017/09/14(木) 10:36:49.22 ID:Ew79uiN/.net
>>5
m(a,b,c)について、

(a^2+ab+b^2)(b^2+bc+c^2)(c^2+ca+a^2) ≧ 3(√3)*(a^2-bc)(b^2-ca)(c^2-ab)

ゆるゆるな希ガス…。

13 :132人目の素数さん:2017/09/14(木) 18:07:49.55 ID:mi/0+iqR.net
>>10
(aa+ab+bb)(bb+bc+cc)(cc+ca+aa)
= pp+pq+qq
≦(p+q)^2
={ab(a+b)+bc(b+c)+ca(c+a)}^2
≦(9/2)(aa+bb)(bb+cc)(cc+aa),
ゆるゆる〜

(aa+ab+bb)≦(3/2)(aa+bb),
を巡回的に掛けて
(aa+ab+bb)(bb+bc+cc)(cc+ca+aa)≦(27/8)(aa+bb)(bb+cc)(cc+aa),

>>11
・a=b のときは
3pq ≧(27/64)(st-u)^2 ≧(1/3)sstt,

・(a,b,c)=(0,1,8)のときは
 p=8,q=64,s=9,t=8,u=0,
(27/64)(st-u)^2 >(1/3)sstt ≧ 3pq,

大小は定まらず。

14 :132人目の素数さん:2017/09/14(木) 23:04:50.69 ID:Ew79uiN/.net
ゆるゆると言えば、2009 BMO。
http://www.bmoc.maths.org/home/bmo2-2010.pdf

 a, b, c>0 に対して、4(a+b+c)^3 > a^2b + b^2c + c^2a.

こんなの見たら改良せざるをえない ( ゚∀゚)ウヒョッ!

 a, b, c>0 に対して、4(a+b+c)^3 ≧ a^2b + b^2c + c^2a + abc.

15 :132人目の素数さん:2017/09/15(金) 01:04:08.15 ID:sd0p5uuD.net
>>4 (1) について、a, b, c≧0 として、次の2式

(a^2+2)(b^2+2)(c^2+2) ≧ (1+a+b)(1+b+c)(1+c+a)
(a^2+2)(b^2+2)(c^2+2) ≧ 9(ab+bc+ca)

を合体させたくなるが、右辺の2式の大小は?

(1+a+b)(1+b+c)(1+c+a) - 9(ab+bc+ca)
= s^2 + 2s + 1 + st - 8t - u

次数がバラバラなので困る… ('A`)

16 :132人目の素数さん:2017/09/15(金) 01:55:39.88 ID:kh+vJCky.net
>>14
それが元々のカナダMO-1995 でござるよ。(但し右辺を27倍)
イギリスMO-2009 はそれを緩めたのでござるな。

0≦a≦b,c としてよい。
4(a+b+c)^3−27(aab+bbc+cca+abc)
= 9a(aa+bb+cc-ab-bc-ca)+(4b+c-5a)(a+b-2c)^2≧0
等号成立は(a,b,c)=(0,2,1)とその巡回。

文献[8] 安藤「不等式」例題2.2.12(7) p.56

17 :132人目の素数さん:2017/09/15(金) 08:58:22.73 ID:sd0p5uuD.net
>>16
なるほど。その因数分解は自力では作れそうにない。

18 :132人目の素数さん:2017/09/15(金) 15:15:34.62 ID:sd0p5uuD.net
>>15
(1+a+b)(1+b+c)(1+c+a) - 9(ab+bc+ca)
= s^2 + 2s + 1 + st - 8t - u
= (s+1)^2 + (st-9u) + 8(u-t)
= (s-1)^2 + (st-9u) + 4(2u-2t+s)

s, t, u ではうまくいかない。

19 :132人目の素数さん:2017/09/15(金) 17:23:22.02 ID:sd0p5uuD.net
a, b, c の基本対称式 s, t, u に関する不等式で定数を含むものは、いくつくらい挙げられますか?

20 :132人目の素数さん:2017/09/15(金) 18:35:21.72 ID:kh+vJCky.net
>>17
と思い込んでたが、実はカナダMOの方も緩かったでござる...orz


[第8章.950]
[第8章.754(1)]に追加
a,b,c >0 のとき
(aa+2bb)(bb+2cc)(cc+2aa)≧(1/27){(a+2b)(b+2c)(c+2a)}^2 ≧ (ab+bc+ca)^3,

(略証)
27(中辺 - 右辺)=(3st+)^2 - 27t^3
= 9(ss-3t)tt + 6st + 刧
≧ (9-4√3)(ss-3t)tt
≧ 0,

〔補題〕
a,b,c ≧0,=(a-b)(b-c)(c-a)のとき
|凵b≦(2/√3)(ss-3t)t/s,
等号は{a,b,c} = {1,1,1} {0,√3 -1,√3 +1} など。

[第3章.727、737-739]
Casphy! - 高校数学 - 不等式1 - 339

21 :132人目の素数さん:2017/09/15(金) 21:57:12.20 ID:sd0p5uuD.net
>>20
補題の証明で、t(s^2 -3t) - ((√3)/2)s|處 の計算のところだけど、

> t(s^2 -3t) - ((√3)/2)s|處
> = 3m^2・(x^2 +xy +y^2) + m・{4x^3 + 3(1-(√3)/2)xy(x+y) +2y^3} + x(x+y){x - ((√3 -1)/2)y}^2
> ≧0

m・{4x^3 + 3(1-(√3)/2)xy(x+y) +2y^3} じゃなくて、
m・{4x^3 + 3(2-(√3)/2)xy(x+y) +2y^3} じゃないのかな?
        ↑ココ

22 :132人目の素数さん:2017/09/15(金) 22:35:56.91 ID:sd0p5uuD.net
>>20
> = 9(ss-3t)tt + 6st + 刧
> ≧ (9-4√3)(ss-3t)tt

|凵b≦(2/√3)(ss-3t)t/s をどう使えばいいのか教えてください。

23 :132人目の素数さん:2017/09/15(金) 23:01:33.10 ID:tFaz6efz.net
>>15
>>18
3u=a+b+c, 3v^2=ab+bc+ca, w^3=abc とおくと u >= v >= w
L-R = -w^3 + (9u - 24) v^2 + 9u^2 + 6u + 1 >= (8u - 15) v^2 + 6u + 1
・8u - 15 >= 0 のとき
明らか
・8u - 15 <= 0 のとき
>= (8u - 15) u^2 + 6u + 1
= (u -1)^2 (8u+1)
>= 0

ここまでやらなくとも L-R を u, v^2, w^3 で表したとき w^3 について線形関数になっているから元の不等式を c=b と c=0 の二つの場合について調べればよい
次数が揃ってなくてそのまま一変数には持ち込めないから逆に面倒かもしれないが

24 :132人目の素数さん:2017/09/15(金) 23:30:57.87 ID:kh+vJCky.net
>>21
[第3章.739]でござるな。
たかたじけない。その通りでござった...orz

この補題は x,y を固定して m の関数と見るのがミソでござる。
つまり、a、b、cを同じ幅で一斉にずらすのでござる。
そのとき、 と ss-3t = F_0 は mによらず一定で、
t = 3mm + 2m(2x+y) + x(x+y)は単調増加、
ss/tt = 3/t + F_0/tt は単調減少、t/s は単調増加。 
∴ m=0 の場合だけ考えれば十分でござるよ。(*)

* ただし a=b=c(x=y=0)の場合は一定となるので除く。

>22
 刧 の項は捨て、
 s ≧ -s|凵b≧ -(2/√3)(ss-3t)t,

25 :132人目の素数さん:2017/09/16(土) 03:02:21.71 ID:wzvOStf3.net
>>23
UVW-method のありがたみが身に沁みました。
自力で (8u-15) v^2 + 6u + 1 まで変形できるか自信ないですが。
合体!
(a^2+2)(b^2+2)(c^2+2) ≧ (1+a+b)(1+b+c)(1+c+a) ≧ 9(ab+bc+ca)


>>24
言葉で説明されて初めて、その置き換えの意味に気づきました。

|處 ≦ (2/√3)(ss-3t)t/s     ← [>>20]
|處 ≦ (ss - 3t)s/{√(9+6√3)}  ← [第8章 261-262]

2√(3+2√3) > 3 なので、残念ながら右辺の大小は定まりませんな。


お二方とも有難うございます。 <(_ _)>

26 :132人目の素数さん:2017/09/16(土) 07:10:08.65 ID:wzvOStf3.net
=(a-b)(b-c)(c-a) に関する評価式を、過去ログから抽出してみた。

竸2 ≦ (a^2+b^2)(b^2+c^2)(c^2+a^2)  ← [>>6]
|處 ≦ (2/√3)(ss-3t)t/s         ← [>>20]
|處 ≦ (ss - 3t)s/{√(9+6√3)}      ← [第8章 261-262]
|處 ≦ (2/√3)(t/s)(s^2 -3t)       ← [第3章 739]
|處 ≦ st-9u                 ← [第4章 624-626]
|處 ≦ {(a+b+c)^3 -27abc}/(6√3)    ← [第5章 893]

≦ F_1 = s^3 - 4st + 9u        ← [第5章 283]
a^4 + b^4 + c^4 + s ≧ (1/27)s^4   ← [第5章 569]

[第5章 763]
納n=1,2] {a^(2n) +b^(2n) +c^(2n) -(ab)^n -(bc)^n -(ca)^n} ≧ (3/2)|處
納n=1,4] {a^(2n) +b^(2n) +c^(2n) -(ab)^n -(bc)^n -(ca)^n} ≧ 3(1+a+b+c +a^2 +b^2 +c^2)|處

27 :132人目の素数さん:2017/09/16(土) 07:59:14.82 ID:SrzKiM05.net
>>25

|凵b≦(2/√3)(ss-3t)(t/s),  >>20

|凵b≦(2/√27)(ss-3t)^(3/2),

 ss/3t =φ =(1+√5)/2 = 1.6180340
で交差する。そこで、これらから

|凵b≦(2/√27)(ss-3t)^1.2763932(3t/s)^0.4472136 ≦(2/√27)(1/φ)s(ss-3t),

としてみる。ここに、
 (2/√27)(1/φ)= 0.237881393
 残念ながら 楠瀬の定数 1/√(9+6√3) = 0.227083346 より大きい。

指数は
 3/2 - 1/(2√5)= 1.2763932
 1/√5 = 0.4472136

28 :132人目の素数さん:2017/09/16(土) 10:00:20.29 ID:SrzKiM05.net
>>25

|凵b≦(2/√3)(ss-3t)(t/s),  >>20
|凵b≦(2/√27)・0.58997984・s(ss-3t), (楠瀬)  
|凵b≦(2/√27)(ss-3t)^(3/2),
|凵b≦(2/√27)(ss-3t)^(3/2),
の4つで相乗平均すると、
(ss-3t)(3t)≦(1/4)s^4 =(s/√2)^4 より

|凵b≦ 0.876413973(2/√27)(ss-3t)^(5/4)(3t)^(1/4)≦ 0.876413973√(2/27)s(ss-3t),

ここに、
 0.619718263(2/√27)= 0.23852967
 これも 楠瀬の定数 1/√(9+6√3)= 0.227083346 より大きい。

 3/√(12+8√3)= 0.58997984
 {3/√(12+8√3)}^(1/4)= 0.876413973

29 :132人目の素数さん:2017/09/16(土) 18:14:24.29 ID:wEi2rRS2.net
任意の非負実数 a, b, c に対して次の不等式が成り立つ
 (1) (a^3+b^3+c^3-3abc) >= A |(a-b)(b-c)(c-a)|
 (2) (a^3+b^3+c^3) >= A |(a-b)(b-c)(c-a)|
 (3) (a+b+c)^3 >= B |(a-b)(b-c)(c-a)|
 (4) (a+b+c)^3 - 27abc >= B |(a-b)(b-c)(c-a)|
, A=sqrt(9+6sqrt(3)), B=6sqrt(3). 両方とも A, B が最良
abc の項はあってもなくても係数は同じだからもっといい不等式が作れそうなきがする

30 :132人目の素数さん:2017/09/16(土) 20:22:13.82 ID:wzvOStf3.net
>>26
> ≦ F_1 = s^3 - 4st + 9u        ← [第5章 283]

この証明が分かりませぬ…。過去ログに載っていないような。
ずっと考えていたんですが、緩い評価しかできませんですた。 ('A`)

L = a^2b + b^2c + c^2a
R = ab^2 + bc^2 + ca^2

ssu/t ≦ L,R ≦ s(ss-2t)/3 (左側は差をとる。右側はCauchyで。)

= (a-b)(b-c)(c-a)
= (a+2b)(b+2c)(c+2a) - 3st
= (2L + 4R) + 9u - 3st
≦ 6*s(ss-2t)/3 + 9u - 3st
= 2s^3 - 7st + 9u
= (s^3 - 4st + 9u) + s(s^2 - 3t)
= F_1 + sF_0

∴ ≦ F_1 + sF_0

ぬるぬるでござった…。もっと厳しく!もっとキツく!!

31 :132人目の素数さん:2017/09/16(土) 21:49:20.51 ID:wEi2rRS2.net
>>30
a=1, b=2, c=0 で成り立たない
過去ログでは三角形の辺となっているから a=x+y… とおくと
L^2 - R^2 = 4(xy^2+yz^2+zx^2-3xyz)(x^2y+yz^2+zx^2-3xyz) >= 0

32 :132人目の素数さん:2017/09/16(土) 21:56:59.31 ID:wzvOStf3.net
>>31
条件を見落としていました。ありがとうございます。

33 :132人目の素数さん:2017/09/17(日) 01:29:44.86 ID:8YPByAqq.net
>>30

左側は差をとる。
Rt -ssu =(aabb・b + bbcc・c + ccaa・a)-(aabb・c + bbcc・a + ccaa・b)
 ={(4aab^3 + bbc^3 + 2cca^3)/7 - aabbc}+ cyclic.
 ≧0,
Lt -ssu =(aabb・a + bbcc・b + ccaa・c)-(aabb・c + bbcc・a + ccaa・b)
 ={(4bba^3 + 2ccb^3 + aac^3)/7 - aabbc}+ cyclic.
 ≧0,

右側も差をとる。
s(ss-2t)- 3R = a(a-c)^2 + b(b-a)^2 + c(c-b)^2 ≧ 0,
s(ss-2t)- 3L = a(a-b)^2 + b(b-c)^2 + c(c-a)^2 ≧ 0,

= R-L
 = 2R -(L+R)
 ≦ 2s(ss-2t)/3 -(st-3u)
 =(2s^3 -7st +9u)/3
 =(F_1 + s F_0)/3,


>>31
反例は a=1,1/φ < b < φ,c=0 ですね。

(st-9u)^2 - 刧 =(R+L-6u)^2 -(R-L)^2 = 4(R-3u)(L-3u)≧0,

34 :132人目の素数さん:2017/09/17(日) 01:46:43.71 ID:8YPByAqq.net
>>25 >>28

|凵b≦(2/√3)(ss-3t)(t/s),  >>20
|凵b≦(2/√27)(ss-3t)^(3/2),
|凵b≦(2/√27)(ss-3t)^(3/2),
の3つで相乗平均すると、
(ss-3t)(3t)≦(1/4)s^4 =(s/√2)^4 より

|凵b≦(2/√27)(ss-3t)^(4/3)(3t/s)^(1/3)≦(2/√27)(1/4)^(1/3)s(ss-3t),

ここに、
 (2/√27)(1/4)^(1/3)= 0.2424719191
 当然ながら、これも楠瀬の定数 1/√(9+6√3)= 0.227083346 より大きい。

35 :132人目の素数さん:2017/09/17(日) 03:00:46.38 ID:8YPByAqq.net
>>33 (修正)
 1/φ<b<φ かつ b≠1

-----------------------------------

〔補題〕
x,y,z>0 のとき
x^3 + y^3 + z^3 + {√(27/2)- 3}xyz ≧(1/√2)(xx+yy+zz)^(3/2),

36 :132人目の素数さん:2017/09/17(日) 05:12:53.96 ID:8YPByAqq.net
>>4

[4]
 ab+bc+ca=t とおく。
(aa+2)(bb+2)(cc+2)-4(aa+bb+cc)-5(ab+bc+ca)-(abc-1)^2
 = 2(aabb+bbcc+ccaa)+2abc -5(ab+bc+ca)t+ 7
 ={(4-√6)/3}(aabb+bbcc+ccaa)
 +{(2+√6)/3}(aabb+bbcc+ccaa + ab+bc+ca)+2abc
 -{(17+√6)/3}(ab+bc+ca)+ 7
 ≧{(4-√6)/9}tt
 + 2{(2+√6)/3}{(ab)^(3/2)+(bc)^(3/2)+(ca)^(3/2)}+ 2abc (←補題>>35)
 -{(17+√6)/3}t + 7
 ≧{(4-√6)/9}tt +{2(√2+√3)/3}t^(3/2) -{(17+√6)/3}t +7
 ={[(4-√6)/9]t +(14/√27)√t +(7/3)}(√t - √3)^2
 ≧ 0,

[3]も[4]から出る。

37 :132人目の素数さん:2017/09/17(日) 05:43:48.86 ID:DVCxyTo5.net
最近やった問題の類題を見つけたのでメモ。
(5)を参考にすれば、>>5の M(a,b,c) が作れる鴨…

【問題】-----------------------------------------------
(1)
a, b, c >0 に対して、
a + (ab)^(1/2) + (abc)^(1/3) ≦ 3*[a*{(a+b)/2}*{(a+b+c)/3}]^(1/3)

(2)
a, b >0 に対して、QM + HM ≧ AM + GM

(3)
a, b, c >0 に対して、9*AM ≧ 8*GM + M_3 (M_3は3乗平均とする)

(4)
a, b, c >0 に対して、3*A(a,b,c) ≧ H(a,b) + H(b,c) + H(c,a)

(5)
a, b, c ≧0 に対して、
4(a+b+c)^6 ≧ 243(a^2-ab+b^2)(b^2-bc+c^2)(c^2-ca+a^2)

-----------------------------------------------------
参考(1) [第8章 972、2016 TOT]
a, b, c >0 に対して、a + (ab)^(1/2) + (abc)^(1/3) ≦ (4/3)*(a+b+c)

参考(2)(3) [第8章 755,727,782,794、出典不明]
a, b, c >0 に対して、2*QM + 3*GM ≦ 5*AM
a, b, c >0 に対して、AM + HM ≧ 5*GM/{16^(1/3)}

出典:>>2 柳田pdf
(1) P.10 問17、初等的な不等式T(問題)
(2) P.13 問7、初等的な不等式T(問題)
(3) P.256 問103、初等的な不等式V
(4) 佐藤[9] P.16 問1.56
(5) P.15 問6、初等的な不等式U

38 :132人目の素数さん:2017/09/17(日) 08:42:36.72 ID:8YPByAqq.net
>>37

(2)
GG, AA, QQ は等差数列
 Q + G ≦ √{2(QQ+GG)}= 2A

H,G,A は等比数列
 QQ-GG = 2(AA-GG)= 2A(A-H),

∴(Q-G)/(A-H)= 2A/(Q+G)≧ 1,

(4)演習問題1.55

39 :132人目の素数さん:2017/09/17(日) 12:24:37.44 ID:DVCxyTo5.net
>>37 (4)
> a, b, c >0 に対して、3*A(a,b,c) ≧ H(a,b) + H(b,c) + H(c,a)

不等式! 改造せずにはいられないッ!

(a+b+c)/3 ≧ 3/{2/(a+b) + 2/(b+c) + 2/(c+a)} ≧ {2ab/(a+b) + 2bc/(b+c) + 2ca/(c+a)}/3 ≧ 3abc/(ab+bc+ca)


                         ゚.ノヽ
                        、-'   `;_' '
                        (,(~ヽ'~
                        i`'}
                        | i'
                     。/   !
                    /},-'' ,,ノ
              _,,...,-‐-、/    i
             <,,-==、   ,,-,/
            {~''~>`v-''`ー゙`'~
             レ_ノ

       , 彡 三 ミ
キタ━━━( ( ((..゚∀゚)) ) )━━━━!!!!!!
       ヾヽミ 三彡, ソ
      / )ミ18彡ノ
      / (ミ 彡゛
         \(
          ))
          (

40 :132人目の素数さん:2017/09/17(日) 16:37:46.84 ID:DVCxyTo5.net
>>37 (1)の改造。

a, b, c, d >0 に対して、
a + (ab)^(1/2) ≦ 2*[a*{(a+b)/2}]^(1/2),
a + (ab)^(1/2) + (abc)^(1/3) + (abcd)^(1/4) ≦ 4*[a*{(a+b)/2}*{(a+b+c)/3*{(a+b+c+d)/4}]^(1/4)

5文字のときは、うまくいかなかった(証明できなかった)でござるが、本当にダメなんだらうか?

41 :132人目の素数さん:2017/09/17(日) 16:55:01.63 ID:tfwTgk//.net
>>40
Show that for all nonnegative a[1], …, a[n],
 (a[1] + (a[1]a[2])^(1/2) + … + (a[1]…a[n])^(1/n)) / n <= (a[1] * (a[1] + a[2])/2 * … * (a[1] + … + a[n])/n)^(1/n).
【Kiran Kedlaya】

42 :132人目の素数さん:2017/09/17(日) 20:09:20.92 ID:DVCxyTo5.net
>>41
なんと!帰納法を使うのかなあ…

43 :132人目の素数さん:2017/09/17(日) 22:47:41.62 ID:DVCxyTo5.net
>>37 (5)の解答例で、aを最小数として、b=a+p、c=a+qとおいて代入して展開していますが、他の回答はないですかね?

44 :132人目の素数さん:2017/09/17(日) 22:57:20.94 ID:8YPByAqq.net
>>37 (1)
>>40
G1=a,G2=√(ab),G3=(abc)^(1/3),…とおく。
コーシーで
(a+a)(a+b)≧(G1+G2)^2,
(a+a+a)(a+G2+b)(a+b+c)≧(G1+G2+G3)^3,
(a+a+a+a)(a+a+b+b)(a+b+G3+c)(a+b+c+d)≧(G1+G2+G3+G4)^4,

>>39
チェビシェフで
{c(a+b)+a(b+c)+b(c+a)} {2/(a+b)+2/(b+c)+2/(c+a)}≦ 6(a+b+c),
∴ 1/H(A(a,b),A(b,c),A(c,a))≦ s/t,

{(a+b)+(b+c)+(c+a)} {2ab/(a+b)+2bc/(b+c)+2ca/(c+a)}≦ 6(ab+bc+ca),
∴ A(H(a,b),H(b,c),H(c,a))≦ t/s,
 
よって
A(a,b,c))= A(A(a,b),A(b,c),A(c,a))
≧ H(A(a,b),A(b,c),A(c,a))
≧ A(H(a,b),H(b,c),H(c,a))
≧ H(H(a,b),H(b,c),H(c,a))= H(a,b,c),

45 :132人目の素数さん:2017/09/17(日) 23:17:44.48 ID:DVCxyTo5.net
>>44
さらに改造。

A(a,b,c))= A(A(a,b),A(b,c),A(c,a))
≧ H(A(a,b),A(b,c),A(c,a))
≧ A(H(a,b),H(b,c),H(c,a))
≧ G(G(a,b),G(b,c),G(c,a)) = G(a,b,c) ← ココ
≧ H(H(a,b),H(b,c),H(c,a)) = H(a,b,c),

もう一つ、私の計算に間違いがなければ…

G(A(a,b),A(b,c),A(c,a))
≧ A(G(a,b),G(b,c),G(c,a))
≧ G(G(a,b),G(b,c),G(c,a)) = G(a,b,c)
≧ H(G(a,b),G(b,c),G(c,a))
≧ G(H(a,b),H(b,c),H(c,a))

そして、この2つを合体させようと思いつつも、
>>37 (5)の m = min{a,b,c} を用いない解法を考えながら、
>>5 の M(a,b,c)に使えそうな次の問題の証明を考え中なのであった…。

[問題]--------------------------------
a, b, c >0 かつ a+b+c=2 に対して、
(a^2+ab+b^2)(b^2+bc+c^2)(c^2+ca+a^2) ≦3
-------------------------------------

これは、次数を揃えるには、右辺に {(a+b+c)/2}^6 を掛ければよいのかな?

46 :132人目の素数さん:2017/09/18(月) 03:34:39.74 ID:D0klnohz.net
>>44
まとめて
A(a,b,c)≧ H(A(a,b),A(b,c),A(c,a))
≧(ab+bc+ca)/(a+b+c)
≧ A(H(a,b),H(b,c),H(c,a))
≧ H(a,b,c),

>>45
(上)
(a,b,c)=(1,1,G^3)の場合を考えると…

G < (√13 -1)/6  ⇒ H(A,A,A) > A(H,H,H) > G
(√13 -1)/6 < G < (√13 +1)/2 ⇒ H(A,A,A) > G > A(H,H,H)
(√13 +1)/2 < G ⇒ G > H(A,A,A)> A(H,H,H)

(√13 -1)/6 = 0.434258546
(√13 +1)/2 = 2.802775638

(下)
等号成立は(a,b,c)=(1,1,0)とその巡回 {(1,1,1)では不成立}

47 :132人目の素数さん:2017/09/18(月) 06:59:00.88 ID:jimNgAXP.net
>>46
ぐはっ…、申し訳ない。(焼き土下座略)

48 :132人目の素数さん:2017/09/18(月) 08:57:46.12 ID:D0klnohz.net
>>37 (3)

(M_3)^3 = (a^3+b^3+c^3)/3
= 9A^3 -8G^3 -(st-9u)
≦ 9A^3 - 8A^3,

(9A-8G)^3 - (M_3)^3
≧(9A-8G)^3 - (9A^3 -8G^3)
= 72(10A^3 -27AAG +24AGG -7G^3)
= 72(10A-7G)(A-G)^2
≧ 0,
∴ 9A-8G ≧ M_3,
ただし、s = a+b+c = 3A、 u = abc = G^3、


>>39
 近畿地方は昨晩通過しますた(ミサイルぢゃなくて台風18号) 快晴でつ。

49 :132人目の素数さん:2017/09/18(月) 12:02:58.60 ID:jimNgAXP.net
>>41 について、載っている書名とか分かりませんか? 証明方法が知りたいでござる。

>>48
問題>>37(3)は、面白いでござるな。
示すべき不等式が 9A-8G ≧ M_3で、9A^3 - 8G^3 ≧ (M_3)^3 も成立するという。

50 :132人目の素数さん:2017/09/18(月) 13:20:10.34 ID:S+ljv5Ld.net
>>43
a+b+c=3, a>=b>=c とすると c=3-a-b, a+b-3 <= 0, 2a+b-3 >= 0, a+2b-3 >= 0
示すべき不等式は 12 >= (a^2-ab+b^2)(c^2-ca+a^2)(b^2-bc+c^2)
R(a,b,c)=(a^2-ab+b^2)(a^2-ac+c^2)(b^2-bc+c^2)
= (a^2-ab+b^2) (a^2+(a+b-3)(2a+b-3)) (b^2+(a+b-3)(a+2b-3))
<= (a^2-ab+b^2) a^2 b^2
= R(a, b, 0)
よって c=0 のときに不等式を示せばよい
L(a, b, 0) - R(a, b, 0)
= 12((a+b)/3)^6 - (a^2-ab+b^2) a^2 b^2
= ((2a-b)^2 (a-2b)^2 (a^2+11ab+b^2))/243
>= 0

等号成立が (0, 1, 2) だから模範解答の BW が無難かと

51 :132人目の素数さん:2017/09/18(月) 13:24:41.29 ID:S+ljv5Ld.net
>>49
Kiran Kedlaya, Proof of a Mixed Arithmetic-Mean, Geometric-Mean Inequality, The American Mathematical Monthly, Vol.101, No.4., (Apr., 1994), pp.355-357
平均同士の mixture は結構研究されているみたい(この不等式自体20年も前の研究)
掘るとAM-GMバージョンとか色々と出てくる

52 :132人目の素数さん:2017/09/18(月) 13:44:41.39 ID:S+ljv5Ld.net
>>43
>>50
a+b+c=3 なんてする必要なかった
R(a,b,c) = 4(a+b+c)^6 >= 4(a+b)^6 >= R(a,b,0)
L(a,b,c)
=243 (a^2-ab+b^2) (b^2-(b-c)) (a^2-c(a-c))
<= 243 (a^2-ab+b^2) a^2 b^2
=L(a,b,0)
あとは c=0 のとき不等式を示せばよい

なんて遠回りをしていたんだ

53 :132人目の素数さん:2017/09/18(月) 16:14:25.00 ID:jimNgAXP.net
>>51
ありがとうございます。
これはAMMの記事ですかな?知りたい情報がすぐにネットで読めるとは、よい時代になりましたなあ。

>>52
キタ━(゚∀゚)━!!!
これで計算が一気に簡単になります。

54 :132人目の素数さん:2017/09/18(月) 17:16:27.77 ID:jimNgAXP.net
>>45
下側を改造厨が改造中。

  A(A(a,b),A(b,c),A(c,a)) = A(a,b,c)
≧ G(A(a,b),A(b,c),A(c,a))
≧ (st/9)^(1/3)
≧ (t/3)^(1/2)
≧ A(G(a,b),G(b,c),G(c,a))
≧ G(G(a,b),G(b,c),G(c,a)) = G(a,b,c)
≧ H(G(a,b),G(b,c),G(c,a))
≧ (3u/s)^(1/2)
≧ (9uu/st)^(1/3)
≧ G(H(a,b),H(b,c),H(c,a))
≧ H(H(a,b),H(b,c),H(c,a)) = H(a,b,c)

大丈夫かな?

55 :132人目の素数さん:2017/09/18(月) 19:16:51.97 ID:D0klnohz.net
>>49

A(a,b,c)= A,G(a,b,c)= G,M_3(a,b,c)= M_3 と略記する。

A ≧ M_3(M_3,G,…,G)≧ A(M_3,G,…,G)=(M_3 + 8G)/9,
          8個           8個
でござるな。

或いは、f(x)= x^3 が下に凸であることから、A≦G≦M に対して
 (A^3 - G^3)/(A-G)≦ 3AA ≦(M^3 - A^3)/(M-A),

これと 8(A^3-G^3)≧ M^3 - A^3 より
 8(A-G)≧ M-A,

- - - - - - - - - - - - - - - - - - - - - - - -

>>46  最後
 (2/3,2/3,2/3)では不成立 

>>48 3行目あたり
 ≦ 9A^3 - 8G^3

56 :132人目の素数さん:2017/09/19(火) 03:19:34.92 ID:SAZ57hNz.net
>>54

 H(A,A,A)≧ t/s ≧ A(H,H,H),  >>46
と組み合わせれば
… ≧(st/9)^(1/3)≧ H(A,A,A)≧ t/s ≧ A(H,H,H)≧(9uu/st)^(1/3)≧ …
かな。


(現代語訳)
はじめの A,G,H で大勢は決するんよ。
あとの A,G,H は狭い範囲内のことなんで微調整やなぁ。


>>55 中ほど
…、G≦A≦M に対して

57 :132人目の素数さん:2017/09/19(火) 04:49:32.31 ID:WLVpxM+c.net
>>37(3)
いま検索中に見つけたんだけど、一体ヘルダーをどう使っているのだろうか?
https://artofproblemsolving.com/community/c6h441200p2486211

58 :132人目の素数さん:2017/09/19(火) 08:50:01.21 ID:JePqYnoo.net
4(2X)+1Y<=(4^(3/2)+1^(3/2))^(2/3)((2X)^3+Y^3)^(1/3).

59 :132人目の素数さん:2017/09/19(火) 10:39:16.47 ID:FC0nfxCf.net
>>57
(1 + 8)(1/3 + 8/3)((a^3+b^3+c^3) + 24abc) >= (((a^3+b^3+c^3)/3)^(1/3) + 8(abc)^(1/3))^3

60 :132人目の素数さん:2017/09/19(火) 12:07:24.52 ID:SAZ57hNz.net
n≧3 とする。(*)

(a+b+c)^n ≧ a^n + b^n + c^n + 3(N-1)(abc)^(n/3),
A^n ≧(1/N){(M_n)^n + G^n + … + G^n},
                N-1 個
N = 3^(n-1)とおいた。

コーシーあるいは f(x)=x^n の凸性から
A^n ≧(1/N){(M_n)^n + G^n + … + G^n}≧{(M_n + G + … + G)/N}^n
∴ A ≧(M_n + G + … + G)/N,
等号成立は M_n = G、a=b=c のとき。

* n=2のときも成り立つが緩い。

61 :132人目の素数さん:2017/09/19(火) 15:01:45.36 ID:WLVpxM+c.net
>>58-59
なるほど、さんくす。ようやく理解できた。

62 :132人目の素数さん:2017/09/19(火) 16:47:58.83 ID:WLVpxM+c.net
さっそく、>>37(3)の類題を作ってみた。

【類題】 (自作なので間違っていたらゴメソ)
a,b,c>0 の相加平均、相乗平均、2乗平均を A、G、Qとする。
(1) 3A ≧ 2G+Q
(2) 9A ≧ 2G+5Q

63 :132人目の素数さん:2017/09/19(火) 17:19:21.61 ID:WLVpxM+c.net
>>62
(2)は係数が気に入らない。

64 :132人目の素数さん:2017/09/19(火) 23:33:13.01 ID:5nXOTef3.net
>>62
AM >= pGM + qQM
(1) (0.6666666666666666, 0.3333333333333333)
(2) (0.2222222222222222, 0.5555555555555556)
最適値は ((45-7sqrt(7))^(1/3)/9, 5/9) = (0.3311784484968155, 0.5555555555555556)
(3) (1-sqrt(2)/3 sqrt(2)/3) = (0.5285954792089682, 0.4714045207910317)
(4) (5^(2/3)/6, 1/2) = (0.4873362897021443, 0.5)

65 :132人目の素数さん:2017/09/20(水) 00:39:12.82 ID:tjSj4nRi.net
a, b, c>0 とする。(a+b+c)^3 について。

[a] (a+b+c)^3 ≧ (27/8)*(a+b)(b+c)(c+a)
[b] (a+b+c)^3 ≧ 27(a+b-c)(b+c-a)(c+a-b)
[c] (a+b+c)^3 ≧ (27/4)*(a^2b + b^2c + c^2a + abc)

[A] (a+b+c)^3 ≦ 9(a^3+b^3+c^3)
[B] (a+b+c)^3 ≦ (a^5-a^2+3)(b^5-b^2+3)(c^5-c^2+3)
[C] (a+b+c)^3 ≦ (a^3 +2)(b^3 +2)(c^3 +2)

[a] と [b] は合体できるでござる。
[d] (a+b+c)^3 ≧ (27/8)*(a+b)(b+c)(c+a) ≧ 27(a+b-c)(b+c-a)(c+a-b)


[疑問]
(1) [c]の右辺と、[a], [b]の右辺との大小は定まるか?
(2) [A], {B}, {C} の右辺の大小は定まるか?


参考---------------------
[b] [安藤、不等式 P.29]
[c] >>16 [1999 CMO]

[B] [第1章 352]
[C] [第1章 367]

66 :132人目の素数さん:2017/09/20(水) 00:43:50.97 ID:tjSj4nRi.net
>>65
[疑問](1)の訂正。
[a]の右辺と[c]の右辺の大小は一定でないことは、既に確認済み。
反例は、c=1 のときに、(a-1)(b-1)の正負で大小が変わる。

色々計算していたら、調べ終わったことをすっかり忘れていました。

67 :132人目の素数さん:2017/09/20(水) 00:49:55.65 ID:tjSj4nRi.net
>>66
重ね重ねすみません。
>>65の訂正は無かったことに。ホント申し訳ないです。
別の問題とゴチャゴチャになっていました。

比較して大小が定まらなかったのは、次式の右辺の大小でした。
(a^2+2)(b^2+2)(c^2+2) ≧ 3(a+b+c)^2
(a^2+2)(b^2+2)(c^2+2) ≧ (1+a+b)(1+b+c)(1+c+a)

行き詰まったら別の問題へと、あれこれ弄っていたら、ゴチャゴチャになってしまいました。
ひとつ解決するごとに、ちゃんと清書しておくべきですね (切腹AA略)

68 :132人目の素数さん:2017/09/20(水) 02:48:42.32 ID:LREZjNa8.net
>>62 >>64

(1)(p,q)=(2/3,1/3)

  (p,q)=(3/5,2/5)   >>37 参考[第8章.755、808]

は緩く、最良値は

(3)(p,q)=(1 -(√2)/3,(√2)/3)

でござるな。

 (p,q) =({1+√(1/3)}/3,{2-√(1/3)}/3)=(0.525783423,0.474216577)
 等号成立は(a,b,c)=(1,1,1)と(1,1,(1+√3)/2)

は無理?

69 :132人目の素数さん:2017/09/20(水) 03:09:27.79 ID:tjSj4nRi.net
>>35
この補題の証明を教えてください <(_ _)>

70 :132人目の素数さん:2017/09/20(水) 03:18:19.58 ID:tjSj4nRi.net
[1] (a^2+2)(b^2+2)(c^2+2) ≧ (1+a+b)(1+b+c)(1+c+a) ≧ 9(ab+bc+ca)
[2] (a^2+2)(b^2+2)(c^2+2) ≧ 3(a+b+c)^2        ≧ 9(ab+bc+ca)
[3] (a^2+2)(b^2+2)(c^2+2) ≧ (2a+2b+2c-abc)^2


(疑問)
[3]の右辺 (2a+2b+2c-abc)^2 と [1],[2]の右辺の3式の大小は定まりますか?
試しにc=0にすると、[3]の右辺の方が大きいようで、定まりそうな希ガス。
差を s,t,u で計算したが無理で、u,v,wで試したが使い慣れていないせいか行き詰まりました。


(参考)
[1] >>4,>>15,>>23
[2] 第8章456
[3] lhs - rhs = 2(ab+bc+ca-abc)^2 ≧0

[1],[2]の中辺の (1+a+b)(1+b+c)(1+c+a) と 3(a+b+c)^2 の大小は一定でないでござる。
c=1 のときに (a-1)(b-1)の正負で大小が変わるから。

71 :132人目の素数さん:2017/09/20(水) 03:35:09.68 ID:tjSj4nRi.net
>>68
その最良値は、どのようにして求めるのですか?
考え方を教えてください。

-------------------------------------------
>>62を、>>57-59の方法で証明するでござる。

AM-GMより
(a+b+c)^2 = 2(ab+bc+ca) + (a^2+b^2+c^2) ≧ 3(2G^2 + Q^2)

Cauchyより
3A = a+b+c = {(2+1)(2G^2 + Q^2)}^(1/2) ≧ 2G+Q
9A = 3(a+b+c) = {(2+25)(2G^2 + Q^2)}^(1/2) ≧ 2G+5Q

72 :132人目の素数さん:2017/09/20(水) 06:27:39.72 ID:LREZjNa8.net
>>65

[疑問1]

[b]/27 =(a+b-c)(b+c-a)(c+a-b)= abc - F1(a,b,c)≦ abc,

∴ [a]≧[b]、[c]≧[b]

[a]=(27/8)(st-u)=([c]+[c~])/2,
[c]=(27/4)(L+u),    >>30
[c~]=(27/4)(R+u),    >>30
   
[a]と[c]の大小は定まらず。


[疑問2]

(a^5 -aa +3)-(a^3 +2)=(a^3 -1)(aa-1)≧0,
より
[B]≧[A],[C]

[A]と[C]の大小は定まらず。

73 :132人目の素数さん:2017/09/20(水) 06:45:32.55 ID:tjSj4nRi.net
>>19
(a-1)^2 + (b-1)^2 + (c-1)^2 ≧0 より、s^2 - 2s - 2t + 3 ≧0 とか…

74 :132人目の素数さん:2017/09/20(水) 06:51:53.25 ID:tjSj4nRi.net
>>72
ありがとうございます。

75 :132人目の素数さん:2017/09/20(水) 08:07:59.55 ID:tjSj4nRi.net
>>11
(a^2+ab+b^2)(b^2+bc+c^2)(c^2+ca+a^2) ≧ 3(a^2b+b^2c+c^2a)(ab^2+bc^2+ca^2

この証明を教えてください。
以前まとめたはずが見当たらず、自力で取り組んで挫折しました…。

76 :132人目の素数さん:2017/09/20(水) 08:20:14.90 ID:LREZjNa8.net
>>65
をまとめて

(a^5 -aa+3)(b^5 -bb+3)(c^5 -cc+3)
≧{(a^3 +2)(b^3 +2)(c^3 +2),9(a^3 +b^3 +c^3)}
≧(a+b+c)^3
≧{(27/8)(st-u),(27/4)(L+u),(27/4)(R+u)
≧ 27abc
≧ 27{abc - F_1(a,b,c)}
= 27(a+b-c)(b+c-a)(c+a-b),

77 :132人目の素数さん:2017/09/20(水) 08:27:01.63 ID:tjSj4nRi.net
>>75
自己解決しました。見たことあるような無いような…。

(a^2+ab+b^2)(b^2+bc+c^2)(c^2+ca+a^2) - 3(a^2b+b^2c+c^2a)(ab^2+bc^2+ca^2 = 竸2

78 :132人目の素数さん:2017/09/20(水) 08:42:38.53 ID:LREZjNa8.net
>>77

>>5-9 のあたり

79 :132人目の素数さん:2017/09/20(水) 08:45:19.06 ID:tjSj4nRi.net
>>78
こんなところに…

80 :132人目の素数さん:2017/09/20(水) 09:50:01.45 ID:CEB00Ztu.net
(Q-G)/(A-G)<=9/(6-r3).

81 :132人目の素数さん:2017/09/20(水) 12:31:22.95 ID:ZYqrdhH5.net
>>68
成り立ちます

>>69
L-R = Aw^3 + B(u, v^2)
A = sqrt(27/2), B は u, v の関数
よって (a, 1, 0), (a, 1, 1) のときに不等式を示せばよい
あとは微分

>>70
いずれも定まらない
a, b を十分大きくとって c = 2(a+b)/(ab-2) とすると [3] の右辺は必ず 0 となる
一方他の辺はすべて非負値を取りうるから考えられるとしたら [3] の右辺が最小
一方 a を大きく b = c = 0 としたら明らかに [3] の右辺が最大
よって定まらない

>>71
一個目について
L-R = Aw^3 + B(u, v^2)
A = -p, B は u, v の関数
よって (a, 1, 0), (a, 1, 1) のときに不等式を示せばよい
このときの p, q の領域の端点が最適
ただ解析はさほど知識がないので端点を求められず適当な値を当てはめて妥協したのが >>64

82 :132人目の素数さん:2017/09/20(水) 18:19:49.70 ID:tjSj4nRi.net
ありがとうございます。

83 :132人目の素数さん:2017/09/20(水) 23:59:51.69 ID:602tS/G4.net
[5] Given real numbers a, b, c satisfying a + b + c = 3 and abc >= -4. Prove that
3(abc+4) >= 5(ab+bc+ca).
[8] Given three circles (O[1]R[1]), (O[2]R[2]), (O[3]R[3]) which are pairwise externally tangent to each other at A, B, C. Let r be the radius of the incircle of ABC. Prove that
r <= (R[1] + R[2] + R[3]) / (6sqrt(3)).
[9] Given positive numbers x, y, z satisfying
x^2 + y^2 - 2z^2 + 2xy + yz + zx <= 0.
Find the minimum value of the expression
P = (x^4 + y^4) / z^4 + (y^4 + z^4) / x^4 + (z^4 + x^4) / y^4.
[10] Find the maximum value of the expression
T = (a+b)/(c+d) * ((a+c)/(a+d) + (b+d)/(b+c))
where a, b, c, d belong to [1/2, 2/3].
【Mathematics and Youth Magazine Problems - Sep 2017, Issue 483】

原文のままだけど [8] は R[1] とかが円の半径なのかな?

84 :132人目の素数さん:2017/09/21(木) 02:33:40.98 ID:qsDFDKvR.net
>>70

[第8章.456]
 Asia-Pacific MO-2004改
 文献 [9] 佐藤(訳)、問題3.85改 p.140

[1]
(中辺)= 1 +2s +(ss+t)+(st-u)≧{1 +√(t/3)+√(t/3)}^3 ≧ 9t,

[2]
(aa+2)(bb+2)(cc+2)- 3(a+b+c)^2
= 3(a+b+c)^2 +(abc-1)^2 + 2(ab-1)^2 + 2(bc-1)^2 + 2(ca-1)^2 +(2G+1)(G-1)^2
 +{aa +bb +cc + 3GG - 2(ab+bc+ca)},

{aa + bb + cc +3GG -2(ab+bc+ca)}≧ ss -4t +9u/s = F_1/s ≧ 0,
 [第8章.388(3)、403、432]

あるいは f(x)= exp(2x)は下に凸だから Popoviciu を適用する。
 文献[9]佐藤(訳)、演習問題1.90 p.41

[3](aa+2)(bb+2)(cc+2)=(2s-u)^2 + 2(t-2)^2,

{a+√(-2)}{b+√(-2)}{c+√(-2)}= -(2s -u)+(t-2){√(-2),}
のノルムをとる。

85 :132人目の素数さん:2017/09/21(木) 04:45:31.57 ID:qsDFDKvR.net
>>83

[9]
(x+y+2z)(x+y-z)=(x+y)^2 +(x+y)z -2zz ≦ 0,
題意より x+y+2z > 0
∴ z ≧ x+y,  (反三角不等式)
w ={2z/(x+y)}^4 ≧ 16,

P(x,y,z)≧ P((x+y)/2,(x+y)/2,z)
= 2 + 2{(x+y)/2z}^4 + 2{2z/(x+y)}^4
= 2 + 2/w + 2w  (← w>1 で単調増加)
≧ 2 + 1/8 + 32
= 34 + 1/8,

86 :132人目の素数さん:2017/09/21(木) 07:20:31.67 ID:V1vTJB/f.net
>>4 の (a^2+2)(b^2+2)(c^2+2) を弄っていて、妙なものができたんだけど…
(1), (2)はよくあるけど、(2)を見た後の(3)がなんとも気持ち悪いのでござる。

(1) (a^2+2)(b^2+2)(c^2+2) ≧ (ab+2)(bc+2)(ca+2)
(2) (a^2+2)(b^2+2)(c^2+2) ≧ (2√2)*(a+b)(b+c)(c+a)
(3) (a^2+2)(b^2+2)(c^2+2) ≧ 8*√{(a+b)(b+c)(c+a)}

87 :132人目の素数さん:2017/09/21(木) 07:24:43.09 ID:V1vTJB/f.net
[問題]
a, b, c>0に対して a^2 + b^2 + c^2 + 2abc + 1 ≧ 2(ab+bc+ca)


出典:https://artofproblemsolving.com/community/c6h155443
タイトルが easy なのに、非同次は難しい。
あと、cosに置き換えるのは、おかしいと思う。

88 :132人目の素数さん:2017/09/21(木) 08:45:07.32 ID:qsDFDKvR.net
>>86

(1)
{a+√(-2)}{b-√(-2)}=(ab+2)+(a-b)√(-2),
(aa+2)(bb+2)≧(ab+2)^2

(2)
{a+√(-2)}{b+√(-2)}=(ab-2)+(a+b)√(-2),
(aa+2)(bb+2)≧ 2(a+b)^2,
(左辺)≧(16√2)/9・st ≧(16√6)/9・t^(3/2)

(3)
(1+aa+1)(1+1+bb)
≧(1+a+b)^2   (コーシー)
= 4(a+b)+(1-a-b)^2
≧ 4(a+b),    (AM-GM)
(左辺)≧(16/3)√(2st)≧(16/3)√(2√3)・t^(3/4),

(4)
(aa+1+1)(1+bb+1)(1+1+cc)
≧{a^(2/3)+ b^(2/3)+ c^(2/3)}^3  (コーシー)
≧ ss +4t +6GG,

気持ち悪い…

89 :132人目の素数さん:2017/09/21(木) 09:23:42.72 ID:qsDFDKvR.net
>>88

(4)
(aa+1+1)(1+bb+1)(1+1+cc)
≧(a+b+1)(1+b+c)(a+1+c)    (コーシー)
≧{a^(2/3)+ b^(2/3)+ c^(2/3)}^3  (コーシー)
≧ ss +4t +6u^(2/3)
≧ 8t +(9u + F_1)/s,

90 :132人目の素数さん:2017/09/21(木) 14:27:27.61 ID:V1vTJB/f.net
>>4 [4]
(a^2+2)(b^2+2)(c^2+2) ≧ 4(a^2+b^2+c^2) + 5(ab+bc+ca) + (abc-1)^2

下のリンクでは、b=c の場合と c=0 の場合に分けて証明しているのですが、
その場合分けだけでいい理由が分かりません。なぜでしょうか?
http://artofproblemsolving.com/community/c6h581954p3439830

91 :132人目の素数さん:2017/09/21(木) 15:40:54.75 ID:V1vTJB/f.net
>>36
難しすぎでござる…

92 :132人目の素数さん:2017/09/21(木) 17:20:48.74 ID:V1vTJB/f.net
>>76
> (a^5 -aa+3)(b^5 -bb+3)(c^5 -cc+3) ≧ 9(a^3 +b^3 +c^3)

この証明を教えてください。

93 :132人目の素数さん:2017/09/21(木) 17:39:36.89 ID:V1vTJB/f.net
>>76
> ≧(a+b+c)^3
> ≧{(27/8)(st-u),(27/4)(L+u),(27/4)(R+u)
> ≧ 27abc

(a+b+c)^3 ≧ a^3 + b^3 + c^3 + 24abc ≧ 27abc もありますね。

(a^3 + b^3 + c^3 + 24abc) - (27/8)(st-u)
= F_1 - 19(st-9u)/8

大小は定まりそうになさそう?

94 :132人目の素数さん:2017/09/21(木) 22:17:34.65 ID:V1vTJB/f.net
>>11
a, b, c >0 に対して、
(1) 3(a^2b+b^2c+c^2a)(ab^2+bc^2+ca^2) ≧ (ab+bc+ca)^3 ←>>11
(2) 3(a^2b+b^2c+c^2a)(ab^2+bc^2+ca^2) ≧ abc(a+b+c)^3 ←出典は下に。

右辺の2式の大小は定まらないですよね?
一息つく間もないでござるな。

(ab+bc+ca)^3 - abc(a+b+c)^3 = u^2 F_{-2} - uF_1

出典
(2) Old and New Inequalities 問42

95 :132人目の素数さん:2017/09/21(木) 23:59:22.71 ID:LqEF6rzR.net
>>90
そこが肝

次の強力な定理が成り立つ(厳密には存在性が必要)
i) u, v^2 を固定すると w^3 は c=0 または c=b のときのみ最大・最小となりうる
ii) u, w^3 を固定すると v^2 は c=0 または c=b のときのみ最大・最小となりうる
iii) v^2, w^3 を固定すると u は c=0 または c=b のときのみ最大・最小となりうる

リンク先は違う置換の方法だけど、一般的な(係数と次数を調整した)置換 3u=a+b+c, 3v^2=ab+bc+ca, w^3=abc をすると不等式は
(2-12u)w^3+18v^4-15v^2+7 >= 0
となる。これは u, v^2 を固定すると w^3 についての一次関数。一次関数は区間の端点、つまり w^3 が最大・最小となるときに関数値が最小となる。定理 i) から右辺は c=0 または c=b のときに最小となりうるのでこのときだけ不等式を示せばよい

定理自体は uvw とは関係なく ABC Theorem として知られる。(洋書文献[3],p.155)
この定理を用いると五次以下の対称不等式は (a, b, 0), (a, b, b) のとき、さらに斉次のときは (a, 1, 0), (a, 1, 1) のときに不等式を示せばよいことがわかる

96 :132人目の素数さん:2017/09/22(金) 00:01:35.33 ID:wfEa8qDX.net
>>94
c=0 とすると正の値を取りうることは明らか
abc = 1 とすると L -R は簡単に因数分解できて符号は因数 ab+bc+ca-a-b-c で決まる
b = c -> 0 とすれば a -> inf でこれは負となる
実際 L-R = -(a^2-bc)(b^2-ca)(c^2-ab) となり符号は定まらない

97 :132人目の素数さん:2017/09/22(金) 01:13:33.43 ID:dxvc1idi.net
>>40-42

5文字のときもコーシーで   >>44

(a+a+a+a+a)(a+a+G2+b+b)(a+G2+b+C'+c)(a+b+C'+D'+d)(a+b+c+d+e)≧(G1+G2+G3+G4+G5)^5,

ここに
 G2 = √(ab)≦(a+b)/2,
 G3 =(abc)^(1/3)≦(a+b+c)/3,
 C ' = √(G3・c)≦(G3+c)/2 ≦(a+b+4c)/6,
 D ' =(G3・ccd)^(1/4)≦(G3+2c+d)/4 ≦(a+b+7c+3d)/12,

今回は非対角要素まで補正した。もちろん、実対称ではござるが。

98 :132人目の素数さん:2017/09/22(金) 02:12:22.64 ID:dxvc1idi.net
>>94

(1)    コーシーで (>>9 最後)

3LR - t^3
=(1+1+1)(aab+bbc+cca)(abb+bcc+caa)-(ab+bc+ca)^3
= 3u(s^3 -4st +9u)+ 2(t^3 -4stu +9uu) +2u(st-9u)
= 3u F_1(a,b,c)+ 2 F_1(ab,bc,ca)+ 2u(st-9u),

(2)
3LR - us^3
= 2u(s^3 -4st +9u)+ 3(t^3 -4stu +9uu) +2u(st-9u)
= 2u F_1(a,b,c)+ 3 F_1(ab,bc,ca)+ 2u(st-9u),

F_1(ab,bc,ca)= uu F_{-2}(a,b,c)

大小は定まらず。

99 :132人目の素数さん:2017/09/22(金) 06:37:35.19 ID:dxvc1idi.net
>>94 続き

F_1(ab,bc,ca)- u F_1(a,b,c)
=(t^3 -4stu +9uu)- u(s^3 -4st +9u)
= t^3 - us^3
=(ab)^3 +(bc)^3 +(ca)^3 -abc(a^3 + b^3 + c^3)
= aab・abb + bbc・bcc + cca・caa - aab・caa - bbc・abb - cca・bcc

の符号は決まらず。

100 :132人目の素数さん:2017/09/22(金) 14:32:29.99 ID:qCni587U.net
>>95-96
詳しくありがとうございます。(未だ理解できていませんが)
洋書文献[3],p.155を調べてみます。

101 :132人目の素数さん:2017/09/22(金) 14:40:37.63 ID:qCni587U.net
追加。
右辺が >>4 >>70 >>86 >>88 >> 90 の右辺のいずれかと大小関係があるのかはチェックしきれていないが

(a^2+2)(b^2+2)(c^2+2) ≧ (1/2)*(a+√2)(b+√2)(c+√2)(abc+2√2) ≧ 16abc√2

102 :132人目の素数さん:2017/09/22(金) 14:51:31.66 ID:5a1XuEA5.net
>>92
これかなり強い不等式な気がするけどどうなんだろう
成り立ちそうだけど証明できない

103 :132人目の素数さん:2017/09/22(金) 15:52:23.29 ID:qCni587U.net
>>98
> (2)
> 3LR - us^3
> = 2u(s^3 -4st +9u)+ 3(t^3 -4stu +9uu) +2u(st-9u)
> = 2u F_1(a,b,c)+ 3 F_1(ab,bc,ca)+ 2u(st-9u),

Schur の使い方に F_1(ab,bc,ca) を利用するとは!!
これは、次数が高くなったときに、(su, t, u^2) → (s,t,u) と置き換えているんですね。

104 :132人目の素数さん:2017/09/22(金) 15:53:06.94 ID:qCni587U.net
>>103
訂正

> > (2)
> > 3LR - us^3
> > = 2u(s^3 -4st +9u)+ 3(t^3 -4stu +9uu) +2u(st-9u)
> > = 2u F_1(a,b,c)+ 3 F_1(ab,bc,ca)+ 2u(st-9u),
>
> Schur の使い方に F_1(ab,bc,ca) を利用するとは!!
> これは、次数が高くなったときに、(su, t, u^2) → (s,t,u) と置き換えているんですね。
> これは、次数が高くなったときに、(su, t, u^2) → (s,t,u) と置き換えているんですね。

105 :132人目の素数さん:2017/09/22(金) 15:53:54.27 ID:qCni587U.net
すみません、誤送信です

(誤) (su, t, u^2) → (s,t,u)
(正) (su, t, u^2) → (t,s,u)

106 :132人目の素数さん:2017/09/22(金) 16:04:30.09 ID:dxvc1idi.net
>>76 >>92 >>102

(a^5-aa+3)(b^5-bb+3)(c^5-cc+3)≧(a+b+c)^3

USAMO-2004 A5

と比べたら、ずっと難しい希ガス。

107 :132人目の素数さん:2017/09/23(土) 05:55:49.72 ID:uwLyIFub.net
[問題] a, b, c, d ≧0 のとき、
(1) {(a+b)(b+c)(c+d)(d+a)}^3 ≧ 16(abcd)^2*(a+b+c+d)^4
(2) (a+b)(b+c)(c+d)(d+a) ≧ (a+b+c+d)(abc+bcd+cda+dab)
(3) (a+b)(b+c)(c+d)(d+a) ≧ (a+b+c-d)(b+c+d-a)(c+d+a-b)(d+a+b-c)

似たようなやつを集めたでござる。

疑問
[1] (2)と(3)の右辺の大小はどうなんだろう。
[2] (a+b)(b+c)(c+d)(d+a)の入った類題が他にあれば教えてください。
[3] a,b,c,d>0の面白いのがあれば、教えてください。

Old and New Inequalities 90など

108 :132人目の素数さん:2017/09/23(土) 11:30:08.53 ID:NoROM9hj.net
>>107

(2)
(a+b)(b+c)(c+d)(d+a)-(a+b+c+d)(abc+bcd+cda+dab)=(ac-bd)^2 ≧ 0,

等号成立は ac=bd

s = a+b+c+d,
t = ab+ac+ad+bc+bd+cd,
u = abc+bcd+cda+dab,
v = abcd,
とおく。
Newtonの不等式
(t/6)/(s/4)≧(u/4)/(t/6)≧ v/(u/4),
から t を消して
 u ≧(16vvs)^(1/3),

変数の1つが飛びぬけて大きいとき、(3)は負。

∴(2)≧(1),(3)

(1)と(3)の大小は定まらず。

109 :132人目の素数さん:2017/09/23(土) 15:28:49.51 ID:NoROM9hj.net
>>40-42 >>97

5文字のときもコーシーで   >>44

2文字混合のみで可能でござった...orz

(a+a+a+a+a)(a+m22+m23+m24+b)(a+m32+m33+m34+c)(a+m42+m43+m44+d)(a+b+c+d+e)≧(G1+G2+G3+G4+G5)^5,

ここに
 m22 =(aaab)^(1/4)≦(3a+b)/4,
 m23 = m32 =(aab)^(1/3)≦(2a+b)/3,
 m24 = m42 =(abbb)^(1/4)≦(a+3b)/4,
 m33 =(b^5・c)^(1/6)≦(5b+c)/6,
 m34 = m43 = √(bc)≦(b+c)/2,
 m44 =(bbbc)^(1/4)≦(3b+c)/4,

110 :132人目の素数さん:2017/09/23(土) 15:30:29.43 ID:NoROM9hj.net
>>40-42 >>97

6文字のときもコーシーで   >>44

(a+a+a+a+a+a)(a+m22+a+G2+m25+b)(a+a+m33+b+m35+c)(a+G2+b+m44+m45+d)(a+m52+m53+m54+m55+e)(a+b+c+d+e+f)≧(G1+G2+G3+G4+G5+G6)^6,

ここに
 G2 = √(ab)=(a+b)/2,
 m22 =(a^3・b^7)^(1/10)≦(3a+7b)/10,
 m25 = m52 =(abbbb)^(1/5)≦(a+4b)/5,
 m33 =(bbbcc)^(1/5)≦(3b+2c)/5,
 m35 = m53 =(bbccc)^(1/5)≦(2b+3c)/5,
 m44 =(c^9・d)^(1/10)≦(9c+d)/10,
 m45 = m54 =(cccdd)^(1/5)≦(3c+2d)/5,
 m55 =(dddde)^(1/5)≦(4d+e)/5,

111 :132人目の素数さん:2017/09/24(日) 00:17:36.33 ID:+GWHsYBd.net
>>109-110

m_{i,1}= m_{1,i}= a_1,

 … 略 …

m_{i,n-1}= m_{n-1,i}=(a_{i-1})^{(i-1)/(n-1)}(a_i)^{(n-i)/(n-1)}

m_{i,n}= m_{n,i}= a_i

だが、残りをどうするか...

112 :132人目の素数さん:2017/09/24(日) 01:33:03.23 ID:mlF1dqOj.net
>>108
(2)と(3)の右辺の大小は、どのようにして分かるのですか?

113 :132人目の素数さん:2017/09/24(日) 11:48:50.52 ID:+GWHsYBd.net
>>112

>>107 より
(1)=(16vv・s^4)^(1/3),

(2)= su = 16v + cd(a-b)^2 + … + ab(c-d)^2,

(3)=(a+b+c-d)(b+c+d-a)(c+d+a-b)(d+a+b-c),
  = 16v -(a+b+c+d)(a+b-c-d)(a-b+c-d)(a-b-c+d)
  = 16v +(a+b+c+d){(c+d-a-b)(a-b)^2 + … +(a+b-c-d)(c-d)^2}/3

さて、どうするか…

114 :132人目の素数さん:2017/09/24(日) 12:54:01.61 ID:+GWHsYBd.net
>>76 >>92 >>102 >>106

x^5 -xx+3 ≧ 3(x^5 +4)/5,

でも使うのでござるか?

115 :132人目の素数さん:2017/09/24(日) 15:20:49.88 ID:mlF1dqOj.net
>>107
(a+b)(b+c)(c+d)(d+a) ≦ (1/16)*(a+b+c+d)^4
(a+b)(b+c)(c+d)(d+a) ≧ (a+b+c-d)(b+c+d-a)(c+d+a-b)(d+a+b-c)
(a+b)(b+c)(c+d)(d+a) ≧ (a+b+c+d)(abc+bcd+cda+dab) ≧ {16(abcd)^2*(a+b+c+d)^4}^(1/3)
(a+b)(b+c)(c+d)(d+a) ≧ (a+b+c+d)(abc+bcd+cda+dab) ≧ 4(ab+bc+cd+da)(abc+bcd+cda+dab)/(abcd)

とりあえず、ここまで。
(a+b+c+d)(abc+bcd+cda+dab) ≧ (a+b+c-d)(b+c+d-a)(c+d+a-b)(d+a+b-c) が証明できない。

116 :132人目の素数さん:2017/09/24(日) 15:23:35.45 ID:mlF1dqOj.net
>>115
{16(abcd)^2*(a+b+c+d)^4}^(1/3) と 4(ab+bc+cd+da)(abc+bcd+cda+dab)/(abcd) の大小は、次数が高くて挫折…

117 :132人目の素数さん:2017/09/24(日) 15:24:44.42 ID:mlF1dqOj.net
>>115
4つ目の式の訂正。
(誤) (a+b)(b+c)(c+d)(d+a) ≧ (a+b+c+d)(abc+bcd+cda+dab) ≧ 4(ab+bc+cd+da)(abc+bcd+cda+dab)/(abcd)
(正) (a+b)(b+c)(c+d)(d+a) ≧ (a+b+c+d)(abc+bcd+cda+dab) ≧ 4(ab+bc+cd+da)(abc+bcd+cda+dab)/(a+b+c+d)

118 :132人目の素数さん:2017/09/24(日) 19:00:55.77 ID:mlF1dqOj.net
[1999 USAMO]
x, y, z>1 のとき、x^(xx+2yz)*y^(yy+2zx)*z^(zz+2xy) ≧ (xyz)^(xy+yz+zx)

これって、x, y, z >0 でいいのでは?

119 :132人目の素数さん:2017/09/24(日) 19:03:09.38 ID:N4qenZps.net
>>前スレ972-
第7章919-921
Carlemanの不等式

>>前スレ990
e

120 :132人目の素数さん:2017/09/24(日) 19:54:57.16 ID:mlF1dqOj.net
>>118
やっぱ要りますね。すまそ。

121 :132人目の素数さん:2017/09/24(日) 21:18:47.14 ID:wAMa4j2r.net
>>107
Vasile の不等式 (Algebraic Inequalities Old and New Methods, pp.271, 6.3 problem 4a)
Σx^3 + 3Σxyz >= Σxy(x+y)
より強い(Vasile は n 変数で成り立つ)

解答自体は d=1 として三変数に持ち込んでuvwで終わりだけど、よい証明が見つからない
https://artofproblemsolving.com/community/c6h1288719p6809821
https://artofproblemsolving.com/community/c6h605279p3595439
スレッドあっても解答ないのが見つかるだけ
等号成立は (a,a,0,0), (a,a,a,0), (a,a,a,a) and sym perm…

疑問[2]
あんまないけど
https://artofproblemsolving.com/community/c6h1184068p5743836

122 :132人目の素数さん:2017/09/24(日) 23:56:40.15 ID:mlF1dqOj.net
>>121
ありがとうございます

123 :132人目の素数さん:2017/09/25(月) 02:16:02.08 ID:IHMzyGPJ.net
以下の同値変形で、赤い矢印の所、おかしくないですか?
https://i.imgur.com/jKBqkLb.jpg

Suppa_Inequalities from the word 1995-2005 P.67

124 :132人目の素数さん:2017/09/25(月) 08:38:56.80 ID:IHMzyGPJ.net
>>121https://artofproblemsolving.com/community/c6h1184068p5743836 より。

a, b, c, d >0のとき、
(1) (8a+b)(8b+a)(8c+d)(8d+c) ≦ (6561/4)*(a^2+b^2)(c^2+d^2)
(2) (8a+b)(8b+c)(8c+d)(8d+a) ≦ (6561/32)*(ac+bd)(a+b+c+d)^2
(3) (5a+b)(5b+a)(5c+d)(5d+c) ≦ (81/2)*(ac+bd)(a+b+c+d)^2

(1)は 2(pa+qb)(qa+pb) ≦ (p+q)^2 (a^2+b^2) で。
(2)(3)は分かりませぬ…。

125 :132人目の素数さん:2017/09/25(月) 19:25:26.05 ID:IHMzyGPJ.net
実数a,b,c,dに対して、(1+a^2)(1+b^2)(1+c^2)(1+d^2) > a+b+c+d

この証明で、左辺を平方和に変形するときに、画像の赤い矢印の部分の符号はマイナスになりませんか?
https://i.imgur.com/YDlFNqJ.jpg

126 :132人目の素数さん:2017/09/26(火) 09:40:42.11 ID:EHLZfEpP.net
>>123
 0<x,y,z≦1 で等比数列のとき、不成立?


>>125
(1+aa)(1+bb)(1+cc)(1+dd)
≧ 1 +(aa+bb+cc+dd)
=(a+b+c+d)+(1/2 -a)^2 +(1/2 -b)^2 +(1/2-c)^2 +(1/2-d)^2
≧ a+b+c+d,

127 :132人目の素数さん:2017/09/26(火) 10:09:49.45 ID:DASjoAs7.net
>>126
なんと! そんなに簡単に証明できたんですね。ありがとうございます。
実にゆるゆるな不等式ですね。

128 :132人目の素数さん:2017/09/26(火) 10:10:20.62 ID:EHLZfEpP.net
>>125

〔類題〕
(1+aa)≧ 2a,
(3+aa)(3+bb)≧ 8(a+b),
(5+aa)(5+bb)(5+cc)≧ 72(a+b+c),
(7+aa)(7+bb)(7+cc)(7+dd)≧ 1024(a+b+c+d),
(9+aa)(9+bb)(9+cc)(9+dd)(9+ee)≧ 20000(a+b+c+d+e),

129 :132人目の素数さん:2017/09/26(火) 10:24:08.94 ID:DASjoAs7.net
>>128
つまり (1+aa)(1+bb)(1+cc)(1+dd) ≧ λ(a+b+c+d) は、λ = 1024/{343(√7)} が最良値でござるか?

130 :132人目の素数さん:2017/09/26(火) 13:58:53.45 ID:EHLZfEpP.net
>>37 >>119

>>37 参考(1)は Carleman でござるか。

等号成立の位置が異なるゆえ

(1)の右辺と Carleman の右辺の大小は定まらぬ…

131 :132人目の素数さん:2017/09/26(火) 16:44:04.97 ID:DASjoAs7.net
[問題]
a,b,c∈Rに対して、(a^3+b^3+c^3)^2 + 4(abc)^2 ≧ 2(a^3b^3+b^3c^3+c^3a^3)

ゆるゆるの極みでは? どう改造するか…

出典 : crux 2839
http://cms.math.ca/crux/v30/n4/page235-256.pdf, P.244

132 :132人目の素数さん:2017/09/27(水) 00:54:28.91 ID:CgrFdG5y.net
a,b,c∈Rに対して、
(1) (a^8-a^2+3)(b^8-b^2+3)(c^8-c^2+3) ≧ (a^2+b^2+c^2)^3

a, b, c≧0に対して,
(2) (a^5-a^3+3)(b^5-b^3+3)(c^5-c^3+3) ≧ 9(a^2+b^2+c^2)
(3) (2a^-2a+3)(2b^2-2b+3)(2c^2-2c+3) ≧ 9(a^2+b^2+c^2)
--------------------------------------------------

(1)
x^8-x^2+3 ≧ x^6+2 より、

(a^8-a^2+3)(b^8-b^2+3)(c^8-c^2+3)
≧ (a^6+2)(b^6+2)(c^6+2)
= (a^6+1+1)(1+b^6+1)(1+1+c^6)
≧ (a^2+b^2+c^2)^3

(2)
(x^5-x^3+3)^3 ≧ 9(x^6+2) より、

{(a^5-a^3+3)(b^5-b^3+3)(c^5-c^3+3)}^3
≧ (9^3)(a^6+2)(b^6+2)(c^6+2)
= (9^3)(a^6+1+1)(1+b^6+1)(1+1+c^6)
≧ (9^3)(a^2+b^2+c^2)^3

(3)
(2x^2-2x^2+3)^3 ≧ 9(x^6+2) が十分大きな x で成り立たないから、同じ方法は使えないでござる

133 :132人目の素数さん:2017/09/27(水) 06:10:06.91 ID:8Sv8e3rv.net
>>131

a^6 + b^6 + c^6 + 3(abc)^2 ≧ 2{(ab)^3 +(bc)^3 +(ca)^3},

AA + BB + CC + 3GG ≧ 2(AB+BC+CA), >>84

と同じだが...

134 :132人目の素数さん:2017/09/27(水) 13:34:23.43 ID:8Sv8e3rv.net
>>124 (2)(3)

k>0 とする。
(ka+b)(kb+c)(kc+d)(kd+a)/(k+1)^4 は k=1 で最大となり、両側で単調。
k=2 が成立てば k>2 も成立つはず...

(2a+b)(2b+a)(2c+d)(2d+c)≦(81/32)(ac+bd)(a+b+c+d)^2,

135 :132人目の素数さん:2017/09/27(水) 13:36:02.71 ID:8Sv8e3rv.net
>>134 訂正

(2a+b)(2b+c)(2c+d)(2d+a)≦(81/32)…

136 :132人目の素数さん:2017/09/27(水) 16:39:57.10 ID:8Sv8e3rv.net
>>134-135

k<5 では不成立でござった。。。死んでお詫びを…(AA略)

反例:
 k=4 (a,b,c,d)=(1,1/64,1/2048,√(1/2))

137 :132人目の素数さん:2017/09/27(水) 22:04:26.24 ID:8Sv8e3rv.net
>>134

kについて降べきの順(?)に並べると
(ka+b)(kb+c)(kc+d)(kd+a)/(k+1)^4 = abcd +{(aabc+bbcd+ccda+ddab-4abcd)k^3 +(abbc+bccd+cdda+daab+aacc+bbdd-6abcd)k^2 +(abcc+bcdd+cdaa+dabb-4abcd)k}/(k+1)^4,

ここで、k/(k+1)^4,kk/(k+1)^4 は単調減少、k^3/(k+1)^3 も k≧3 で単調減少でござる。
∴ k が 1〜2 の辺りで最大か。

138 :132人目の素数さん:2017/09/28(木) 07:56:21.57 ID:Zf2OsKu8.net
>>92
> a,b,c>0 に対して、(a^5 -aa+3)(b^5 -bb+3)(c^5 -cc+3) ≧ 9(a^3 +b^3 +c^3)

>>114の x^5 -x^2 +3 ≧ 3(x^5 +4)/5 より、
(a^5 -a^2 +3)(b^5 -b^2 +3)(b^5 -b^2 +3) ≧ (27/125)*(a^5+4)(b^5+4)(c^5+4)

したがって、以下が成り立てばよいのだが、分からない…
(a^5+4)(b^5+4)(c^5+4) ≧ (125/3)*(a^3 +b^3 +c^3) … ★

>>132 の方法を使うには、x^5 +4 ≧ (125/3)*(a^9 +2) が成り立てばいいが、成り立たず。
遡って、x^5 -x^2 +3 ≧ 9(x^9 +2) が成り立てばいいが、これも成り立たず。

お手上げでござる。

139 :132人目の素数さん:2017/09/29(金) 18:52:53.17 ID:ccDcp6Fb.net
>>92
(a^5-a^2+3)^3 >= 3(2a^9+3a^3+4)
とかから a^3 -> a と置き換えて不等式に当てはめると
 (2a^3+3a+4)(2b^3+3b+4)(2c^3+3c+4) >= 27(a+b+c)^3
を示せばよい。

いま (a^5-a^2+3)^3 >= ra^9+(27-3r)+2r は r <= 8.98395 に対して成り立つ
特にもとの不等式に適用できそうなのは m = 9/2^(2/3) = 5.66964 <= r の範囲
r = 8, 7, 6, m に対して書き下してみると
・(8a^3+3a+16)(8b^3+3b+16)(8c^3+3c+16) >= 729(a+b+c)^3
・(7a^3+6a+14)(7b^3+6b+14)(7c^3+6c+14) >= 729(a+b+c)^3
・(2a^3+3a+4)(2b^3+3b+4)(2c^3+3c+4) >= 27(a+b+c)^3
・Π(a^3+ka+2) >= 4(a+b+c)^3 where k = 3(2^(2/3)-1)
このいずれかの不等式を示せばよい

r=6 のときはきれいで頑張れば示せそう
r=m の限界値も不等式自体は悪くないからうまく示せそう
(uvw で表すと唯一 u^3 の項が消える)

あとは任せたでござる

140 :132人目の素数さん:2017/09/30(土) 17:59:19.43 ID:QFGPsK6W.net
>>139

a^9 -3a^3 +2 = (a^3 +2)(a^3 -1)^2 ≧ 0 ゆえ

r a^9 +(27-3r)a^3 + 2r は r について単調増加

>>114 と比べると

・r < 6.59 のとき、a^5 -aa +3 ≧ 3(a^5 +4)/5 ≧{r a^9 +(27-3r)a^3 + 2r}^(1/3),

・r > 6.59 のとき、大小定まらず。

141 :132人目の素数さん:2017/10/01(日) 01:08:20.63 ID:wkWWtqrc.net
>>132
> a,b,c∈Rに対して、
> (3) (2a^2-2a+3)(2b^2-2b+3)(2c^2-2c+3) ≧ 9(a^2+b^2+c^2)
> --------------------------------------------------
>
> (2x^2-2x^2+3)^3 ≧ 9(x^6+2) が十分大きな x で成り立たないから、同じ方法は使えないでござる

x=2a-1, y=2b-1, z=2c-1 とおくと、x, y, z ≧-1で、示すべき不等式は
(x^2+5)(y^2+5)(z^2+5) ≧ 18{(x+1)^2 + (y+1)^2 + (z+1)^2}

等号が x=y=z=1 で成立することに注意して、未定係数法で力ずくで平方和に分解。

lhs - rhs
= (u-s/3)^2 + (8/9)(s^2-3t) + (28/9)(t^2-3su) + 6(s-3)^2 + (17/9)(t-3)^2
≧0
  ┏━━━┓
  ┃ Q.E.D. ┃
  ┗━┳━┛
( ゚∀゚) ノ


この方法を >>92 に使おうと思ったが、よい置き換えが思いつかぬでござる。

142 :132人目の素数さん:2017/10/01(日) 14:40:49.65 ID:wkWWtqrc.net
[初等的な不等式U P.65 問36]
a, b, c≧0に対して、{(a+b)(b+c)(c+a)}^2 ≧ 4(a^2+bc)(b^2+ca)(c^2+ab)

模範解答は、aを最小として b=a+p、c=a+q を代入して差をとり、
aの6次式の係数がすべて0以上であることを確認していますが、
手計算じゃ大変だから、別解があれば教えてください。

s,t,uで計算してSchurを考えたけど (自分では)うまくいかず、
Lhs - Rhs = (st)^2 - 4(s^3)u + 22stu - 4t^3 - 31u^2

次に b を中央の数として (a+b)(b+c) - 2(b^2+ca) = (a-b)(b-c) ≧0 より
Lhs ≧ 2(a+b)(b+c)(c+a)^2(b^2+ca) だから、
(a+b)(b+c)(c+a)^2 ≧ 2(a^2+bc)(c^2+ab)
が成り立つことが示せれば…と計算して挫折

さらに(c+a)^2 = (c-a)^2 + 4ca より、 Lhs ≧ 8ca(a+b)(b+c)(b^2+ca) として、
2(a+b)(b+c) ≧ (a^2+bc)(c^2+ab)
が成り立つことが示せれば…と計算して挫折

>>52みたいな、いい方法ないかなあ…

143 :132人目の素数さん:2017/10/01(日) 14:55:35.03 ID:wkWWtqrc.net
>>142
(誤) 2(a+b)(b+c) ≧ (a^2+bc)(c^2+ab)
(正) 2ca(a+b)(b+c) ≧ (a^2+bc)(c^2+ab)

144 :132人目の素数さん:2017/10/01(日) 15:12:37.49 ID:wkWWtqrc.net
>>142の不等式を改造するでござる。

a, b, c≧0に対して、
(64/27)(a^2+ab+b^2)(b^2+bc+c^2)(c^2+ca+a^2)
≧ {(a+b)(b+c)(c+a)}^2       ←[>>7]
≧ 4(a^2+bc)(b^2+ca)(c^2+ab)
≧ 4abc(a+b)(b+c)(c+a)

参考 [>>11]
(64/27)(a^2+ab+b^2)(b^2+bc+c^2)(c^2+ca+a^2)
≧ [(a+b)(b+c)(c+a)]^2
≧ (64/81)[(a+b+c)(ab+bc+ca)]^2
≧ (64/27)(ab+bc+ca)^3

(a^2+bc)(b^2+ca)(c^2+ab) と (16/81)[(a+b+c)(ab+bc+ca)]^2 ≧ (16/27)(ab+bc+ca)^3
の大小も気になるでござる

145 :132人目の素数さん:2017/10/01(日) 15:18:28.63 ID:wkWWtqrc.net
>>142
> a, b, c≧0に対して、{(a+b)(b+c)(c+a)}^2 ≧ 4(a^2+bc)(b^2+ca)(c^2+ab)

こんな不等式もあるでござるよ。
https://artofproblemsolving.com/community/c6h498985p2804163
a, b, c≧0に対して、{(a+b)(b+c)(c+a)}^3 ≧ 64abc(a^2+bc)(b^2+ca)(c^2+ab)

左辺に (a+b)(b+c)(c+a) が余分に掛かっているので、
AM-GMで (a+b)(b+c)(c+a) ≧8abc を試しに削ってみたら、
(a+b)(b+c)(c+a)}^2 ≧ 8(a^2+bc)(b^2+ca)(c^2+ab)

無謀でござった…

146 :132人目の素数さん:2017/10/01(日) 15:27:39.96 ID:wkWWtqrc.net
>>141
一応、aの6次式を書いておく。
-------------------------------------------------------
対称性から a を最小数として、b = a+p、c = a+q (p, q ≧0)とおくと、

{(a+b)(b+c)(c+a)}^2 - 4(a^2+bc)(b^2+ca)(c^2+ab)
= 32a^6 + 64(p+q)a^5
 + 40(p^2 + 3pq + q^2)a^4
 + 4(2p^3 + 17p^2q + 17pq^2 + 2q^3)a^3
 + 4pq(3p^2 + 8pq + 3q^2)a^2
 + 4p^2q^2(p+q)a + p^2q^2(p-q)^2
≧0
-------------------------------------------------------

147 :132人目の素数さん:2017/10/01(日) 15:45:01.19 ID:5dXe4Li1.net
>>141

a = A^(3/2),b = B^(3/2),c = C^(3/2)と置換えるでござるよ。

(A^5 -AA +3)-(2aa -2a +3)
=(A^5 -AA +3)-(2A^3 -2A√A +3)
= 2(3A^5 +4A√A -7A^3)/7 +(A^5 +6A√A -7AA)/7
≧ 0,   (← AM-GM)

(A^5-AA+3)(B^5-BB+3)(C^5-CC+3)
≧(2aa-2a+3)(2bb-2b+3)(2cc-2c+3)≧ 9(aa+bb+cc)      >>141
= 9(A^3 +B^3 +C^3),

Q.E.D.

148 :132人目の素数さん:2017/10/01(日) 17:24:36.83 ID:wkWWtqrc.net
>>147
なんと! うまい方法があるものですね。かたじけのうござる。

149 :132人目の素数さん:2017/10/01(日) 18:28:58.87 ID:5dXe4Li1.net
>>141 から明らかだが…

s = a+b+c,t = ab+bc+ca,u = abc とおくと

(2aa-2a+3)(2bb-2b+3)(2cc-2c+3)- 9(aa+bb+cc)
= 2(s/3 -t +2u)^2
 +(4/9)(ss-3t)
 +(7/9){(2a-1)^2・(b-c)^2 +(2b-1)^2・(c-a)^2 +(2c-1)^2・(a-b)^2}
 + 3(s-3)^2
 +(34/9)(s-t)^2
≧ 0,

150 :132人目の素数さん:2017/10/01(日) 19:43:50.52 ID:5dXe4Li1.net
>>145

リンク先の解答:
{(a+b)(a+c)}^2 ={a(b+c)+(aa+bc)}^2 ≧ 4a(b+c)(aa+bc),
巡回的に掛ける。
(perfect_square,2012/Sep/19)

151 :132人目の素数さん:2017/10/01(日) 23:40:32.50 ID:wkWWtqrc.net
>>142
こんなのもあるみたい。
https://artofproblemsolving.com/community/c6h463340p2597719
a, b, c≧0に対して、
(ab+bc+ca)(a^4+b^4+c^4) ≧ (9/8)(a^2+bc)(b^2+ca)(c^2+ab) ≧ (ab+bc+ca){(ab)^2+(bc)^2+(ca)^2}

152 :132人目の素数さん:2017/10/02(月) 00:54:35.23 ID:+8fPBhp3.net
>>146>>142 へのレス。

153 :132人目の素数さん:2017/10/02(月) 01:25:53.50 ID:4CFPUmbD.net
>>142

s,t,u で計算してSchurを考えれば

Lhs - Rhs -32uu =(st-u)^2 -4{(s^3)u -6stu +t^3 +16uu}
=[(s^3 -4st +9u)(t^3 -4stu +9uu)/uu + 3ss(tt-3su)/u + 3(tt/u)(ss-3t) + 9(st-9u)]uu/st
=[F_1・F_{-2}+ 3ss F_{-1}+ 3(tt/u)F_0 + 9(st-9u)]uu/st
≧ 0,
となり申す。ここに、
F_0 = ss-3t,
F_1 = s^3 -4st +9u,
F_{-1}=(tt -3su)/u,
F_{-2}=(t^3 -4stu +9uu)/uu,

154 :132人目の素数さん:2017/10/02(月) 02:15:56.86 ID:+8fPBhp3.net
>>153
おぉ! Schurでやれたんですね。すんばらすぃ!

155 :132人目の素数さん:2017/10/02(月) 02:36:12.34 ID:+8fPBhp3.net
似たような式がたくさん出てきたので、まとめるナリ。
a, b, c≧0
= (a-b)(b-c)(c-a)
△ = (a+b)(b+c)(c+a)
D = (a^2+bc)(b^2+ca)(c^2+ab)

(1) (64/27)(a^2+ab+b^2)(b^2+bc+c^2)(c^2+ca+a^2) ≧ △^2 ≧ {(8/9)(a+b+c)(ab+bc+ca)}^2 ≧ {(4/3)(ab+bc+ca)}^3
(2) {(2/3)(a+b+c)}^3 ≧ △ ≧ 24abc(aa+bb+cc)/{(a+b+c)^2}
(3) (1/512){(a^2+3)(b^2+3)(c^2+3)}^2 ≧ △
(4) △^2 ≧ 4D
(5) △^3 ≧ 64abcD

[1] (△/2)^2 ≧ D ≧ abc△
[2] {(△/4)^3}/(abc) ≧ D
[3] (8/9)(ab+bc+ca)(a^4+b^4+c^4) ≧ D ≧ (8/9)(ab+bc+ca){(ab)^2+(bc)^2+(ca)^2}


(参考)---------------------
(1) 第2章 136-138
(2) 第8章 687
(3) 第8章 469
(4) >>142
(5) >>145

[1] >>142 (証明>>153)、>>144
[2] >>145
[3] >>151

156 :132人目の素数さん:2017/10/02(月) 03:07:00.08 ID:+8fPBhp3.net
>>151
右側をSchurで。左はできなかった。

9(a^2+bc)(b^2+ca)(c^2+ab) - 8(ab+bc+ca){(ab)^2+(bc)^2+(ca)^2}
= 9(s^3u - 6stu + t^3 + 8u^2) - 8t(t^2 - 2su)
= 9s^3u - 38stu + t^3 +72u^2
= tuF_{-1} + 8uF_1 + suF_0
= (u^2)F_{-2} + 7uF_1 + 2suF_0
≧0

157 :132人目の素数さん:2017/10/02(月) 07:37:42.09 ID:+8fPBhp3.net
>>155
[1], [3] の右辺について、D ≧ (8/9)(ab+bc+ca){(ab)^2+(bc)^2+(ca)^2} ≧ abc△
∵ Lhs-Rhs = (u^2)F_{-2} + 7tuF_{-1} ≧0

158 :132人目の素数さん:2017/10/02(月) 08:05:49.34 ID:+8fPBhp3.net
>>155
[1], [2] の左辺について、AM-GMより {(△/4)^3}/(abc) ≧ (△/2)^2 ≧ D

あとは (8/9)(ab+bc+ca)(a^4+b^4+c^4) と {(△/4)^3}/(abc) ≧ (△/2)^2 の大小ですな。

159 :132人目の素数さん:2017/10/02(月) 08:10:02.16 ID:+8fPBhp3.net
>>158
勘違いでした、すみません…

160 :132人目の素数さん:2017/10/02(月) 09:26:50.65 ID:+8fPBhp3.net
>>155
D ≦ {(△/4)^3}/(abc)
D ≦ (△/2)^2 ≦ (8/9)(ab+bc+ca)(a^4+b^4+c^4)
D ≧ (8/9)(ab+bc+ca){(ab)^2+(bc)^2+(ca)^2} ≧ abc△

上段と中段の右辺の大小は定まらない。

161 :132人目の素数さん:2017/10/02(月) 12:05:28.97 ID:+8fPBhp3.net
>>132>>92
問題再掲
(1) a, b, c∈Rに対して、(a^8-a^2+3)(b^8-b^2+3)(c^8-c^2+3) ≧ (a^2+b^2+c^2)^3 [答>>132]
(2) a, b, c≧0 に対して、(a^5-a^3+3)(b^5-b^3+3)(c^5-c^3+3) ≧ 9(a^2+b^2+c^2) [答>>132]
(3) a, b, c≧0 に対して、(2a^-2a+3)(2b^2-2b+3)(2c^2-2c+3) ≧ 9(a^2+b^2+c^2) [答>>141]
(4) a, b, c≧0 に対して、(a^5-a^3+3)(b^5-b^3+3)(c^5-c^3+3) ≧ 9(a^2+b^2+c^2) [答>>147]

類題
(5) a, b, c≧0 に対して、(a^4-a+3)(b^4-b+3)(c^4-c+3) ≧ 9(a^3+b^3+c^3)
https://artofproblemsolving.com/community/c6t322f6h541764_again

解法パターンも出尽くしたでござるかな?

162 :132人目の素数さん:2017/10/02(月) 14:35:42.57 ID:4CFPUmbD.net
>>161 の類題

リンク先の解答

(a^4 -a+3)^2 ≧ 3(a^6 +2),

(左辺)^2 ≧ 27(a^6 +2)(b^6 +2)(c^6 +2)
≧ 81(a^6 +b^6 +1)(1+1+c^6)    (*)
≧ 81(a^3+b^3+c^3)^2     (コーシー)

*(a-1)(b-1)≧ 0 としても一般性を失わない。
(x-1)(y-1)=(a^6 -1)(b^6 -1)≧ 0,
(x+2)(y+2)= 3(x+y+1)+(x-1)(y-1)≧ 3(x+y+1),

〔系〕
n≧1 のとき
{a^(3+n)-a^n +3}{b^(3+n) -b^n +3}{c^(3+n)-c^n +3}≧ 9(a^3+b^3+c^3),

∵ {x^(3+n)- x^n} -{x^4 -x}= x(x^3 -1){x^(n-1) -1}≧ 0,

163 :132人目の素数さん:2017/10/02(月) 14:57:41.73 ID:+8fPBhp3.net
忘れないうちにmemo。 何て発音するのか分からんけど。

【Turkevici's Inequality】
a, b, c, d ≧0
a^4 + b^4 + c^4 + d^4 + 2abcd ≧ (ab)^2 + (bc)^2 + (cd)^2 + (da)^2 + (ac)^2 + (bd)^2

164 :132人目の素数さん:2017/10/02(月) 19:39:25.11 ID:fUaxBEz1.net
>>92 >>138

>>161 の類題(5)
(a^4 -a+3)(b^4 -b+3)(c^4 -c+3)≧ 9(a^3 +b^3 +c^3),
を使うのが簡単でござったな。  >>162 〔系〕

>>153
 △^2 ≧ 4D + 32(abc)^2,

>>155
[1]D ≧ abc・Δ
チェビシェフより
(aa+bc)(bb+ca)= a(a+c)・b(b+c)+(a+b)c・(a-b)^2 ≧ a(a+c)・b(b+c),
あるいは
(aa+bc)(bb+ca)= √(ab)c(a+b)^2 + ab{√(ab)-c}^2 +{a^3 +b^3 -(aa+bb)√(ab)}c ≧ √(ab)c(a+b)^2,
を巡回的に掛ける。


>>163
リンク先の解答:
aa,bb ≧ cc ≧ dd としても一般性を失わない。
(左辺)-(右辺)=(aa-cc)^2 +(bb-cc)^2 +(aa-dd)^2 +(bb-dd)^2 - 2(ab-cd)^2
≧(aa-dd)^2 +(bb-dd)^2 - 2(ab-cd)^2
≧(1/2)(aa+bb - 2dd)^2 - 2(ab-cd)^2
≧ 2(ab-cd)^2 -2(ab-cd)^2
= 0,
(can_hang2007,2008/Nov/16)

165 :132人目の素数さん:2017/10/02(月) 22:37:01.98 ID:+8fPBhp3.net
>>164
> (a^4 -a+3)(b^4 -b+3)(c^4 -c+3)≧ 9(a^3 +b^3 +c^3),
> を使うのが簡単でござったな。  >>162 〔系〕

たしかに! 言われるまで気づかなかったナリ。

166 :132人目の素数さん:2017/10/03(火) 01:29:34.82 ID:K9DRTZfC.net
>>162
(a^4 -a +3)^2 ≧ 3(a^6 +2),
(a^4 -a^3 +1)^3 ≧(a^9 +2)/3,

>>163-164
リ、リンクが無ゑ…
http://artofproblemsolving.com/community/c6h239742s1
_5_lines_for_Turkevicis_inequality

167 :132人目の素数さん:2017/10/03(火) 02:04:55.44 ID:K9DRTZfC.net
>>166
略証(念のため)
(a^4 -a +3)^2 - 3(a^6 +2)=(a-1)^4(aa+a+1)(aa+3a+3)≧ 0,
(a^4 -a^3 +1)^3 - (a^9 +2)/3 =(a-1)^4(aa+a+1)(a^6 +(2/3)a^3 + 2aa + a + 1/3)≧ 0,

168 :132人目の素数さん:2017/10/03(火) 04:09:48.40 ID:zw7D2ND3.net
左辺が2次式のときに、こんなのがあるナリ。
https://artofproblemsolving.com/community/c6h462983

x, y, z∈R、4b≧aに対して、
(x^2+ax+b)(y^2+ay+b)(z^2+az+b) ≧ {(4b-a^2)/3}^(3/2)*(x-y)(y-z)(z-x)

169 :132人目の素数さん:2017/10/03(火) 05:03:25.92 ID:zw7D2ND3.net
>>168
関連したもの
https://artofproblemsolving.com/community/c6h1484892p8689649
https://artofproblemsolving.com/community/c6h582856

a, b, c∈R、t≧0に対して、
(a^2+2)(b^2+2)(c^2+2) ≧ {(16√6)/9}*(a-b)(b-c)(c-a)
(a^2+t^2)(b^2+t^2)(c^2+t^2) ≧ {8t^3/(3√3)}*(a-b)(b-c)(c-a)

170 :132人目の素数さん:2017/10/03(火) 07:43:51.44 ID:zw7D2ND3.net
>>4
> for reals
> [1] (a^2+2)(b^2+2)(c^2+2) >= (1+a+b)(1+b+c)(1+c+a)
> for nonnegarives
> [3] (a^2+2)(b^2+2)(c^2+2) >= 3(a+b+c)^2+(abc-1)^2
> [4] (x^2+2)(y^2+2)(z^2+2) >= 4(x^2+y^2+z^2)+5(xy+yz+zx)+(xyz-1)^2
> [5] (a^2+2)(b^2+2)(c^2+2) >= 4(a^2+b^2+c^2)+5(ab+bc+ca)+(abc(a-1)^2(b-1)^2(c-1)^2)^(1/3)

>>70
> [1] (a^2+2)(b^2+2)(c^2+2) ≧ (1+a+b)(1+b+c)(1+c+a) ≧ 9(ab+bc+ca)
> [2] (a^2+2)(b^2+2)(c^2+2) ≧ 3(a+b+c)^2        ≧ 9(ab+bc+ca)
> [3] (a^2+2)(b^2+2)(c^2+2) ≧ (2a+2b+2c-abc)^2

>>86
> (1) (a^2+2)(b^2+2)(c^2+2) ≧ (ab+2)(bc+2)(ca+2)
> (2) (a^2+2)(b^2+2)(c^2+2) ≧ (2√2)*(a+b)(b+c)(c+a)
> (3) (a^2+2)(b^2+2)(c^2+2) ≧ 8*√{(a+b)(b+c)(c+a)}

>>101
> (a^2+2)(b^2+2)(c^2+2) ≧ (1/2)*(a+√2)(b+√2)(c+√2)(abc+2√2) ≧ 16abc√2

>>169
> a, b, c∈R、t≧0に対して、
> (a^2+2)(b^2+2)(c^2+2) ≧ {(16√6)/9}*(a-b)(b-c)(c-a)
> (a^2+t^2)(b^2+t^2)(c^2+t^2) ≧ {8t^3/(3√3)}*(a-b)(b-c)(c-a)

------------------------------------------------------

(a^2+2)(b^2+2)(c^2+2)がらみ
http://artofproblemsolving.com/community/c6h76508p897772
a, b, c >0 かつ k≦4 に対して、
(a^2+2)(b^2+2)(c^2+2) ≧ k(a^2+b^2+c^2) + (9-k)(ab+bc+ca)

リンク先の証明がよく分かりませぬ…

171 :132人目の素数さん:2017/10/03(火) 09:02:24.33 ID:zw7D2ND3.net
>>163-164
【A generalization of Turkevici’s inequality】
Prove that for any x1, x2, . . . , xn > 0 with product 1,
Σ[i<j] (x_i - x_j)^2 ≧ Σ[i=1 to n] x_i^2 - n

(リンク先URKが長くて書込み拒否された)


>>166
> リ、リンクが無ゑ…

エスパーかよw

172 :132人目の素数さん:2017/10/03(火) 09:29:49.83 ID:zw7D2ND3.net
>>164
> リンク先の解答:
> aa,bb ≧ cc ≧ dd としても一般性を失わない。
> 2{(左辺)-(右辺)}
> =(aa-cc)^2 +(bb-cc)^2 +(aa-dd)^2 +(bb-dd)^2 - 2(ab-cd)^2
> ≧(aa-dd)^2 +(bb-dd)^2 - 2(ab-cd)^2 …(1)
> ≧(1/2)(aa+bb - 2dd)^2 - 2(ab-cd)^2  …(2)
> ≧ 2(ab-cd)^2 -2(ab-cd)^2        …(3)
> = 0,

(1) は (aa-cc)^2 +(bb-cc)^2を捨てて、
(2) は (aa-dd)^2 +(bb-dd)^2 - (1/2)(aa+bb-2dd)^2 = (1/2)(aa-bb)^2 ≧0
(3) は aa+bb ≧2ab、-dd ≧-cd

c≧dしか使っていないようなハロゲンガス…

173 :132人目の素数さん:2017/10/03(火) 09:33:53.73 ID:zw7D2ND3.net
>>172
ごめん、書いた後で気づいた。(焼き土下座AA略)
カッコの中 aa+bb-2dd が負になって、2乗したら大きくならないために必要なんですね。

174 :132人目の素数さん:2017/10/03(火) 12:20:07.14 ID:zw7D2ND3.net
[Old and New Inequalities, Q,74]
a, b, c >0 に対して、a^2 + b^2 + c^2 + 2abc + 3 ≧ (1+a)(1+b)(1+c)

175 :132人目の素数さん:2017/10/03(火) 14:09:57.63 ID:v6f7ZU4g.net
>>147
まずその不等式を使おうと思わないでござる

>>162
(a^4 -a+3)^2 ≧ 3(a^6 +2)
何乗かすれば必ずヘルダーが使えそうな形に持っていけるのだろうか
たまたまできただけなのか

176 :132人目の素数さん:2017/10/03(火) 14:20:56.35 ID:K9DRTZfC.net
>>169

(aa+2)(bb+2)(cc+2) ≧(8/3)^(3/2)(a-b)(b-c)(c-a)=(8/3)^(3/2)

a = √(8/3)(A + 1/2)とおくと
aa+2 =(8/3)(AA+A+1),(a-b)= √(8/3)(A-B),etc.
ゆえ、次式と等価(arqady,2017/July/27)

(AA+A+1)(BB+B+1)(CC+C+1)≧(A-B)(B-C)(C-A)= ,
(略証)
Lhs - Rhs =(1/3){(AA+A+1)xx +(BB+B+1)yy +(CC+C+1)zz}+(1/6){(x-y)^2+(y-z)^2+(z-x)^2}≧0,
ここに
 x = BC+B+1,y = CA+C+1,z = AB+A+1,
とおいた。(szl6208,2017/July/28)

177 :132人目の素数さん:2017/10/03(火) 18:13:23.62 ID:K9DRTZfC.net
>>170

(A-1)(B-1)≧1 としても一般性を失わない。(WithOut Loss of Generality)
ABC ≧(A+B-1)C を使うナリ。

k=4 で成り立てば 0≦k≦4 でも成り立つ。
(2+aa)(2+bb)(2+cc) - 4(aa+bb+cc) -5(ab+bc+ca)
= 8 -5(ab+bc+ca) +2(aabb+bbcc+ccaa) + (abc)^2  (← ab,bc,caの式)
= 8 -5(A+B+C) + 2(AA+BB+CC) + ABC
≧ 8 -5(A+B) + 2(AA+BB) - (6-A-B)C + 2CC  (→ Cで平方完成)
={7(A+B-2)^2 + 8(A-B)^2 + (6-A-B-4C)^2}/8
≧ 0,
ここに A=bc,B=ca,C=ab とおいた。(red3,2011/Apr/10)

178 :132人目の素数さん:2017/10/03(火) 18:38:16.19 ID:K9DRTZfC.net
>>174
Q.74
これも同様に
abc ≧ (a+b-1)c により2次式に sage て平方完成でござるな。

(aa + bb + cc + 2abc + 3) - (1+a)(1+b)(1+c)
≧aa + bb + cc -ab -a -b -2c +2
={(a+b-2)^2 + (a-b)^2 + 4(c-1)^2}/4
≧ 0,

>>177 訂正
(A-1)(B-1)≧0 でござった。(AA略)

179 :132人目の素数さん:2017/10/03(火) 19:05:30.70 ID:zw7D2ND3.net
>>177
> (A-1)(B-1)≧1 としても一般性を失わない。(WithOut Loss of Generality)

ありがたや!
A,B,Cのうちの少なくとも2つは1以上か1以下、鳩の巣原理でござるか?


>>178
発音の難しい不等式(Turkevici) + AM-GMでござる。

180 :132人目の素数さん:2017/10/03(火) 19:41:43.70 ID:K9DRTZfC.net
>>171

http://artofproblemsolving.com/community/c6h5285
_generalization_of_Turkevici_inequality

と同じでござるか。(manlio,2004/Apr/24)

f " > 0 のとき
(n-2)Σ[k=1,n]f(a_k)+ n f((a_1+…+a_n)/n)≧ 2Σ[1≦i<j≦n]f((a_i+a_j)/2),

を使うらしいが…(Imht,2016/Nov/11)


>>178 訂正

= {(a+b-2)^2 + 3(a-b)^2 + 4(c-1)^2}/4
≧ 0,

181 :132人目の素数さん:2017/10/04(水) 13:47:45.14 ID:qBSU59BI.net
>>174
Turkevici's Inequality (>>163) の a,b,c,d を √a, √b, √c, 1 に置き換えて、
T := a^2 + b^2 + c^2 + 1 + 2√(abc) - (ab+bc+ca + a+b+c) ≧ 0

a^2 + b^2 + c^2 + 2abc + 3 - (1+a)(1+b)(1+c)
= T + {1-√(abc)}^2
≧0

182 :132人目の素数さん:2017/10/04(水) 18:18:46.95 ID:lC7ztqKn.net
>>175

x=1 のまわりでティラー展開すると、

{x^(n+1/2+r)- x^(n+1/2-r)+ 2r}^n -(1/3)(2r)^n・{x^(3n)+ 2}
= (n/24)(2r)^n・{(2n-3)^2 + 4rr -10}(x-1)^3
+{n(n-1)/12}(2r)^n・(7nn-11n+4rr-1)(x-1)^4
+ O((x-1)^5)

∴(2n-3)^2 + 4rr -10 = 0,

n=2 のとき r=3/2,(a^4 -a +3)^2 ≧ 3(a^6 +2),

n=3 のとき r=1/2,(a^4 -a^3 +1)^3 ≧(1/3)(a^9 +2),

なお、4乗の係数は
7nn-11n+4rr-1 =(3n+1)n +{(2n-3)^2 + 4rr -10}=(3n+1)n > 0.

183 :132人目の素数さん:2017/10/05(木) 01:05:50.43 ID:bWqoEeDi.net
>>182

〔補題〕
1 < n <(3+√10)/2,2r =√{10 -(2n-3)^2}のとき
{x^(n+1/2+r)- x^(n+1/2-r)+ 2r}^n ≧(1/3)(2r)^n・{x^(3n)+ 2},

の略証でござる。

184 :132人目の素数さん:2017/10/05(木) 03:28:31.90 ID:bWqoEeDi.net
>>164 >>172

ab+cd = p,cc+dd = q とおくと、
Lhs - Rhs
= pp + qq -4ccdd +(aa-bb)^2 -(aa+bb)q
= pp + qq -2pq +2cd(q-2cd)+(aa-bb)^2 -(a-b)^2・q
=(p-q)^2 + 2cd(c-d)^2 +(a-b)^2・{(a+b)^2 -q}≧ 0,
(In-seok Seoの解,KMO winter program)

(a+b)^2 ≧ q = cc+dd しか使ってねゑ…

185 :132人目の素数さん:2017/10/05(木) 04:08:19.96 ID:bWqoEeDi.net
>>163
Turkeviciの改良版でござる。

a^4 + b^4 + c^4 + d^4 + 2abcd
≧ ab(aa+bb)/2 + ac(aa+cc)/2 +ad(aa+dd)/2 +bc(bb+cc)/2 +bd(bb+dd)/2 +cd(cc+dd)/2
≧ aabb + aacc + aadd + bbcc + bbdd + ccdd,

Lhs - Rhs ={2 F_2(a,b,c)+ d F_1(a,b,c)}/6 + cyclic ≧ 0,
(darij grinberg,2006/Feb/04)

http://artofproblemsolving.com/community/c6h73107
_stronger_than_Turkevici's_inequality

186 :132人目の素数さん:2017/10/06(金) 01:06:47.71 ID:Ftw4WKLG.net
>>171 >>180

nについての帰納法による。
a_n = x を最小の要素としてもよい。
s' =(a_1,…,a_{n-1}の AM)
t' =(a_1,…,a_{n-1}の GM)
とおくと
s' ≧ t' ≧ x,

Lhs - Rhs = f(a_1,…,a_{n-1},x)
= f(t',・・・,t',x)+ f(a_1,…,a_{n-1})+ Σ[k=1,n-1](a_k -x)^2 -(n-1)(t'-x)^2
≧ f(t',・・,t',x)+ f(a_1,…,a_{n-1})+(n-1)(s'-x)^2 -(n-1)(t'-x)^2
= f(t',・・,t',x)+ f(a_1,…,a_{n-1})+(n-1)(s'+t'-2x)(s'-t')
≧ f(t',・・,t',x)   (←帰納法の仮定、s'≧t'≧x)
=(n-1){(n-1)t't' + xx}+ n・{x・t'^(n-1)}^(2/n)-{(n-1)t' + x}^2
=(n-2)xx + n・[x・t'^(n-1)]^(2/n)- 2(n-1)t'x
≧ 0,   (← AM-GM)
(harazi,2004/Apr/29)

>>183
略証とまでは言えねゑ…

187 :132人目の素数さん:2017/10/09(月) 16:35:49.31 ID:NEWgEIva.net
[2005 Uzbekistan National Olympiad]
a,b,cを三角形の3辺,a+b+c=2のとき、

1+abc<ab+bc+ca≦28/27+abc

を示せ

188 :132人目の素数さん:2017/10/09(月) 18:48:59.21 ID:AAcQM4kG.net
>>186

n=3 のときは
Lhs - Rhs = 2(aa+bb+cc) +3GG -(a+b+c)^2
= aa+bb+cc -2ab -2bc -2ca +3GG
≧ A^3 + B^3 + C^3 -AB(A+B)-BC(B+C)-CA(C+A)+3ABC
= F_1(A,B,C)
≧ 0,
ここに、A=a^(2/3),B=b^(2/3),C=c^(2/3)とおいた。


>>187

8(Mhs - Lhs)
= 4(a+b+c)(ab+bc+ca) -(a+b+c)^3 + 8abc
=(a+b-c)(b+c-a)(c+a-b)
> 0,

abc ≦{(a+b+c)/3}^3 = 8/27,  (← AM-GM)
Mhs = ab+bc+ca ≦{(a+b+c)^3 + 9abc}/{4(a+b+c)}= 1 +(9/8)abc ≦ Rhs,

189 :132人目の素数さん:2017/10/10(火) 00:15:49.75 ID:h4u4sSCs.net
>>180

n=4 の場合の略証
 x1 ≧ x2 ≧ x3 ≧ x4 としてよい。
 m =(x1+x2+x3+x4)/4 とおく。

・ x1+x4 ≧ x2+x3 のとき
(x1,m,m)majorizes((x1+x2)/2,(x1+x3)/2,(x1+x4)/2)
(x2,x3,x4)majorizes((x2+x3)/2,(x2+x4)/2,(x3+x4)/2)
∴Karamata により
 f(x1)+ f(m)+ f(m)≧ f((x1+x2)/2)+ f((x1+x3)/2)+ f((x1+x4)/2),
 f(x2)+ f(x3)+ f(x4)≧ f((x2+x3)/2)+ f((x2+x4)/2)+ f((x3+x4)/2),
 辺々たす。

・ x1+x4 ≦ x2+x3 のとき
(x1,x2,x3)majorizes((x1+x2)/2,(x1+x3)/2,(x2+x3)/2)
(m,m,x4)majorizes((x1+x4)/2,(x2+x4)/2,(x3+x4)/2)
∴Karamata により
 f(x1)+ f(x2)+ f(x3)≧ f((x1+x2)/2)+ f((x1+x3)/2)+ f((x2+x3)/2),
 f(m)+ f(m)+ f(x4)≧ f((x1+x4)/2)+ f((x2+x4)/2)+ f((x3+x4)/2),
 辺々たす。

文献[3]大関,p.125-126
文献[8]安藤「不等式」,p.10-11

- - - - - - - - - - - - - - - - - -

〔一般化された Turkevici不等式〕

(n-1){(a_1)^2 + … +(a_k)^2}+ nGG ≧(a_1 + … + a_n)^2,
ここに G =(a_1・a_2 … a_n)^(1/n),

n=2 等号
n=3 >>188
n=4 >>164 下, >>184-185
n≧5 nについての帰納法 >>186

190 :132人目の素数さん:2017/10/10(火) 02:08:31.45 ID:7jpvSrE2.net
[エレ解 1991-11]
任意の x>0 に対して、a^x + a^(1/x) ≦ a^(x + 1/x) が成り立つための正の数 a の条件を求めよ。

191 :132人目の素数さん:2017/10/10(火) 02:18:14.50 ID:7jpvSrE2.net
[エレ解 2013-10]
自然数 n≧2、C[n, k] は二項係数とする。
(1) Σ[k=1 to n] (-1)^(k+1) C[n, k] {1/(n^2)}^k < 1/n
(2) Σ[k=1 to n] C[n, k] {1/(n^2-1)}^k > 1/n
(3) Σ[k=1 to 2n] C[2n, k] {1/(n^2-1)}^k > 2/(n-1)

192 :132人目の素数さん:2017/10/10(火) 08:01:04.60 ID:h4u4sSCs.net
>>190

F(x)= Rhs - Lhs = a^(x+1/x)- a^x - a^(1/x)
とおく。

0<a≦1 ならば
 F(x)=(1 - a^x){1 - a^(1/x)}- 1 < 0,
題意より a>1 に限られる。

F '(x)= log(a){(1 -1/xx)a^(x+1/x)-a^(x)+(1/xx)a^(1/x)}
= log(a)a^(x+1/x){1 -1/xx -a^(-1/x)+(1/xx)a^(-x)}
= log(a)a^(x+1/x){g(1/x)- g(x)}/x,

y = a^(-x)は下に凸ゆえ、g(x)={1 - a^(-x)}/x はxについて単調減少。

∴ F '(x)および g(1/x)- g(x)は、x-1 と同符号。

∴ F(x)は x=1 に極小値 F(1)= a(a-2)をもつ。
以下、F(1)だけ見れば十分。
題意を満足する aの下限は 2

往年の数学者「ビブンのことはビブンせよ。」
最近の数学者「ビブン・セキブン・いいキブン」

193 :132人目の素数さん:2017/10/10(火) 11:09:03.78 ID:7jpvSrE2.net
a,b,c,d>0に対して、
(a+b)^3 (b+c)^3 (c+d)^3 (d+a)^3 ≧ 16(abcd)^2 (a+b+c+d)^4

出題元は a,b,c>0 となっているが…
https://artofproblemsolving.com/community/c6h299899p1650989

不等式は代数幾何? 僕は大好き。

194 :132人目の素数さん:2017/10/10(火) 19:13:50.79 ID:h4u4sSCs.net
>>192
補足
 xx{h(x)/x} ' = x h '(x)- h(x)= ∫[0,x] t h"(t)dt - h(0),

>>193
Problem 116(Crux Mathematicorum)
リンク先の解答:

(a+b)(b+c)(c+d)(d+a)= su +(ac-bd)^2 ≧ su,
ここで基本対称式を
s = a+b+c+d,
u = abc+bcd+cda+dab,
v = abcd,
とおいた。

これらの間にはMcLaurinの不等式
 2t/3s ≧ 3u/2t ≧ 4v/u(=HM),
が成立つ。tを消すと
 u/s ≧(4v/u)^2,
∴(su)^3 ≧16 v^2 s^4.
(Pain rinnegan,2009/Oct/11)

195 :132人目の素数さん:2017/10/11(水) 23:37:21.40 ID:PBLxDg/9.net
>>193

>>107 (1)でござったか。

ac+bd = p,ad+bc = q,ab+cd = r とおく。
(a+b)(c+d)= p+q,
(b+c)(d+a)= p+r,
su-4v = pq+qr+rp,
よって
(a+b)(b+c)(c+d)(d+a)
=(p+q)(p+r)
= pp +(suー4v)
= su +(pp-4v)
= su +(ac-bd)^2
≧ su,     >>108 と同じだ...

196 :ル.ヌー:2017/10/12(木) 00:23:05.21 ID:Hbkmuqaq.net
a∈Cに対して、f(z)=e^(1/(z−a)),z∈C\{a}において、aに収束する点列an∈C\{a}で lim n→∞ f(an)=+∞ となるものを見出せ。
の解答をお願い致します。 👀
Rock54: Caution(BBR-MD5:ae2afb6cd11f3e92f5cd12f037b4c3ac)


197 :132人目の素数さん:2017/10/12(木) 13:47:20.72 ID:IVBPcmrA.net
>>196

a_n = a + (1/n)

とかでいいんじゃね?

【考え方】
Cの上では exp はふにょふにょしてる事に注意しよう.
exp がシュッとでかくなるのは引数が実のときだけ.
だから 1/(a_n -a) が n:large に対してでかい実数になるように
すればいい.1/xは右から近づくと+∞,左から近づくと-∞
なんで a_n - a が実数になるようにしつつ a に右から近づけばいい.

198 :197:2017/10/12(木) 17:30:14.91 ID:IVBPcmrA.net
(失礼しました.質問スレと間違えて返事してしまいました)

199 :ル.ヌー:2017/10/12(木) 19:20:14.61 ID:Hbkmuqaq.net
f(z)=z/sinz,z∈Cにおいて,
(1) z=0はf(z)の除去可能特異点であることを示せ。
(2) f(z)の極をすべて求めよ、また、極での留数を求めよ。
(3) z=0まで定義域を拡大したf(z)のz=0におけるマクローリン展開の2次の項までを求め  よ。
(1).(2).(3)の解答をお願い致します。 👀
Rock54: Caution(BBR-MD5:ae2afb6cd11f3e92f5cd12f037b4c3ac)


200 :132人目の素数さん:2017/10/12(木) 19:36:02.98 ID:Hbkmuqaq.net
f(z)=z/sinh z,z∈Cにおいて
(1) f(z)はC上正則であることを示せ。
(2) z=0はf(z)の除去可能特異点であることを示せ。
(3) z=0まで定義域を拡大したf(z)のz=0におけるマクローリン展開の2次の項までを求めよ。
(1).(2).(3)の解答をお願い致します。

201 :132人目の素数さん:2017/10/12(木) 19:44:24.75 ID:Cwow0r6+.net
>>199
ここは質問スレじゃないぞ。

202 :132人目の素数さん:2017/10/12(木) 19:46:08.43 ID:Cwow0r6+.net
第9章で初めての荒らしかな。

203 :132人目の素数さん:2017/10/12(木) 23:48:07.31 ID:saIb7jMi.net
>>191

(1)1-(1 - 1/nn)^n < 1/n,

(2){1 + 1/(nn-1)}^n > n/(nn-1)> 1/n,

(3){1 + 1/(nn-1)}^(2n)-1 ={nn/(nn-1)}^(2n)-1
 ={1/(1-xx)}^(2/x)-1 >(1+x)/(1-x)-1 = 2x/(1-x)= 2/(n-1),

*) 2log(1-xx)+ x・log{(1+x)/(1-x)}
=(2+x)log(1+x)+(2-x)log(1-x)
= -∬[0,x]{2t/(1-tt)}^2 dt < 0,
より (1-xx)^(2/x)<(1-x)/(1+x),

204 :132人目の素数さん:2017/10/13(金) 07:23:30.38 ID:4eFIOMr+.net
>>195
4変数に関する基本対称式 s, t, u, v をみると新鮮でござるな。 p, q, r の関係も面白い。

205 :132人目の素数さん:2017/10/13(金) 09:54:07.95 ID:NUqZtYG4.net
>>204

s,t,u,v ( >>108 ) と p,q,r ( >>195 ) の関係

p+q+r = t,
pq+qr+rp = su - 4v,
pqr =(ss-4t)v + uu,

206 :132人目の素数さん:2017/10/14(土) 04:12:27.77 ID:WYmPKYWn.net
>>203

〔補題〕
-1 < x < 1 のとき
 2・log(1-xx)+ x・log{(1+x)/(1-x)}≦(xx/3)log(1-xx)≦ -(1/3)x^4 ≦ 0,

(略証)
f(x)=(2-xx/3)log(1-xx)+ x・log{(1+x)/(1-x)}とおく。xの偶関数。

f '(x)=(-2x/3)log(1-xx)+ log{(1+x)/(1-x)}-(2x/3)+(2/3){1/(1+x)-1/(1-x)},

f "(x)=(-2/3)log(1-xx)-2 +(5/3){1/(1+x)+ 1/(1-x)}+(2/3){-1/(1+x)^2 -1/(1-x)^2},

f "'(x)=(2/3){-1/(1+x)+1/(1-x)}+(5/3){-1/(1+x)^2 +1/(1-x)^2}+(2/3){2/(1+x)^3 -2/(1-x)^3}
 = -4xxx(3-xx/3)/(1-xx)^3,

f ""(x)=(2/3){1/(1+x)^2 +1/(1-x)^2}+(5/3){2/(1+x)^3 +2/(1-x)^3}-4/(1+x)^4 -4/(1-x)^4
 = -4xx(27+22xx-x^4)/[3(1-xx)^4]
 ≦ 0.

207 :132人目の素数さん:2017/10/14(土) 18:06:10.35 ID:VmZscmTD.net
[エレ解 2017-08]
x,y,z≧0、x+y+z≦1のとき、3x^2 - 4x+ 5y^2 - 2y + z^2 - 1 の最大最小を求めよ。

208 :132人目の素数さん:2017/10/15(日) 03:22:38.43 ID:qbyQ3Mho.net
>>203

1/(1-tt)はt>0で単調増加ゆえ、
(1/y)∫[0,y] 2/(1-tt)dt も y>0で単調増加。

(2+x)log(1+x)+(2-x)log(1-x)
=(2+x)∫[0,x/(2+x)]2/(1-tt)dt -(2-x)∫[0,x/(2-x)]2/(1-tt)dt
< 0,

209 :132人目の素数さん:2017/10/17(火) 17:29:43.38 ID:fUPTP4tk.net
実定数 a, b, c>0 とする。
実数 x, y が a/x - b/y ≧c、x < a/c、y > b/c をみたしながら動くとき、
ax-byが最大値をもつための条件と、そのときの最大値を求めよ。

(出典不明、問題はうろ覚え、10年くらい前に立ち読みした記憶から再生したので係数が微妙に違うかもしれない)

210 :132人目の素数さん:2017/10/18(水) 21:36:26.48 ID:YHzUPFkO.net
実定数 a>0 と、4x^2 + y^2 ≦1 をみたす実数 x, y に対して、
2xy + 2ax + ay の最大値・最小値と、そのときの x, y の値を求めよ。

(昔ネットで見たもの、Z会か進研ゼミのサンプル問題だったような…)
---------------------------------------------------------

4x^2 + y^2 =1 じゃないんで、(x, y) = ((r/2)cosθ, r sinθ)とおいたら泥沼に嵌った。

211 :132人目の素数さん:2017/10/21(土) 11:52:27.62 ID:juihTYxZ.net
>>191 (3)

(左辺)={1 +1/(nn-1)}^(2n)-1
={nn/(nn-1)}^(2n)-1
>(n+1)/(n-1)-1  (←補題)
= 2/(n-1),

〔補題〕
g_n = (1 +1/n)^(n +1/2), (nは正の整数)
とおくとき、g_n は単調減少。

∴ {(n+1)/n}^(2n+1) < {n/(n-1)}^(2n-1),
∴ (n+1)/(n-1) < {nn/(nn-1)}^(2n),

[エレ解スレ(2011.2).68-69]

212 ::2017/10/23(月) 21:29:52.29 ID:Dl6USvMt.net


213 ::2017/10/23(月) 21:30:11.14 ID:Dl6USvMt.net


214 ::2017/10/23(月) 21:30:31.35 ID:Dl6USvMt.net


215 ::2017/10/23(月) 21:30:55.55 ID:Dl6USvMt.net


216 ::2017/10/23(月) 21:31:14.83 ID:Dl6USvMt.net


217 ::2017/10/23(月) 21:31:31.71 ID:Dl6USvMt.net


218 ::2017/10/23(月) 21:31:55.02 ID:Dl6USvMt.net


219 ::2017/10/23(月) 21:32:15.81 ID:Dl6USvMt.net


220 ::2017/10/23(月) 21:32:36.53 ID:Dl6USvMt.net


221 ::2017/10/23(月) 21:32:58.40 ID:Dl6USvMt.net


222 :132人目の素数さん:2017/10/29(日) 12:39:14.89 ID:eHt4EM8U.net
今は昔、高校時代のZ会の通信添削より。

a,b,c≧0に対して、
(1) x^(1/3) + y^(1/3) ≦ {4(x+y)}^(1/3)
(2) x^(1/3) + y^(1/3) + z^(1/3) ≦ {9(x+y+z)}^(1/3)

223 :132人目の素数さん:2017/10/29(日) 12:50:37.86 ID:eHt4EM8U.net
>>222
今見ると簡単すぎて泣ける。
当時は Power Mean とか知らなんだし…。

224 :132人目の素数さん:2017/10/29(日) 12:53:45.50 ID:eHt4EM8U.net
a,b,c≧0に対して、
(9/4)(a+b+c)^5 ≧ 10(a^3 + b^3 + c^3)(a+b+c)^2 - 9(a^5 + b^5 + c^5) ≧ (a+b+c)^5

右側は簡単だったけど、左側は挫折…。

225 :132人目の素数さん:2017/10/29(日) 17:12:42.84 ID:eHt4EM8U.net
a,b,c≧0に対して、
a^3 + b^3 + c^3 + (6/7)abc ≧ (1/7)(a+b+c)^3

苦手な非同次の不等式。

226 :132人目の素数さん:2017/10/30(月) 13:51:50.89 ID:Fh66Dt0j.net
>>224
s = a+b+c,t = ab+bc+ca,u = abc,
とおく。
a^3 +b^3 +c^3 = s(ss-3t)+3u,
a^5 +b^5 +c^5 = s^5 - 5(a+b)(b+c)(c+a)(ss-t)= s^5 - 5(st-u)(ss-t),

(左辺)-(中辺)=(5/4)s(ss-6t)^2 + 15(ss-3t)u ≧ 0,
(中辺)-(右辺)= 15(st-u)(ss-3t)≧ 0,

>>225
(左辺)-(右辺)=(3/7){s(ss-3t)+(s^3-4st+9u)}=(3/7)(s・F_0 + F_1)≧ 0,

F_0 = ss-3t ≧ 0,
F_1 = s^3 -4st +9u ≧ 0.

227 :132人目の素数さん:2017/10/30(月) 14:14:21.95 ID:Fh66Dt0j.net
>>224
 
等号成立は
左側:{a,b,c}={0,√3-1,√3+1}
右側:{a,b,c}={1,1,1}

228 :132人目の素数さん:2017/11/02(木) 13:44:48.75 ID:FM0nIjMB.net
〔Jordanの不等式〕
0<θ<π/2 のとき、
 sinθ >(2/π)θ,

文献[3]大関、p.38-39 例題2.
[分かスレ436.016](微分を使わない方法)

229 :132人目の素数さん:2017/11/03(金) 12:23:07.50 ID:d8bRV0BU.net
>>228

ABを直径とする円をcとする。(半径r)
A,Bを通るもう一つの円をCとする。(半径 R >r)

このとき
 2R sinθ= AB = 2r,

また横方向のズレ幅からみて、明らかに
弧AcB > 弧ACB,
 πr > 2R θ,

辺々掛けて
 sinθ >(2/π)θ,

[分かスレ436.016]

230 :132人目の素数さん:2017/11/04(土) 11:44:18.19 ID:b2rnI0sj.net
〔問題〕
0≦a,b,c≦1 かつ (1-a)(1-b)(1-c)= abc のとき

(a+b+c)(aa+bb+cc-ab-bc-ca)+8abc ≧1,

231 :132人目の素数さん:2017/11/04(土) 19:09:20.95 ID:ks/8FAV3.net
>>226
かたじけのうござる。
次数が高いと、非負の和に変形するのが大変でござるな。

232 :132人目の素数さん:2017/11/04(土) 19:15:07.16 ID:ks/8FAV3.net
a,b,c≧0に対して、

(1) (a^3 + b^3 + c^3 + 15abc)^3 ≧ 216abc(ab+bc+ca)^3
(2) (a+b+c)^5 ≧ 12{(a^4)(b+c) + (b^4)(c+a) + (c^4)(a+b)}

233 :132人目の素数さん:2017/11/04(土) 19:58:16.57 ID:ks/8FAV3.net
a,b,c≧0に対して、

(3) {a^2 + b^2 + c^2 + 9(ab+bc+ca)}^3 ≧ 1000abc(a+b+c)^3

234 :132人目の素数さん:2017/11/05(日) 02:07:38.46 ID:6KxvYVjk.net
>>231
 全くでござるよ。

>>232 (2)
(左辺)-(右辺)
= s^5 -12(s^3・t -3stt -ssu +5tu)
= s(ss -6t +5u/s)^2 +(2ss -25u/s)u
≧ ssu,   (← s^3≧27u)

235 :132人目の素数さん:2017/11/05(日) 14:01:12.34 ID:6KxvYVjk.net
>>231
 そうでもない?

>>232 (2)

(a+b+c)^6 ={(a^3+b^3+c^3)+ 3(a+b)(b+c)(c+a)}^2
≧12(a^3+b^3+c^3)(a+b)(b+c)(c+a)
> 12{(a^4)(b+c)+(b^4)(c+a)+(c^4)(a+b)}(a+b+c),


(a+b)(b+c)> b(a+b+c),
(b+c)(c+a)> c(a+b+c),
(c+a)(a+b)> a(a+b+c),

236 :132人目の素数さん:2017/11/06(月) 22:40:01.09 ID:056U4iI+.net
>>234
こういう変形はどうやって思いつくんでせうか?

>>235
これはすごい…

237 :132人目の素数さん:2017/11/07(火) 09:32:33.90 ID:p59pyQGE.net
a,b,c≧0 かつ a+b+c>0 かつ a^2+b^2+c^2-2abc=1 のとき、(a-1)(b-1)(c-1)≧0

238 :132人目の素数さん:2017/11/07(火) 23:53:08.96 ID:cR3XovZi.net
>>237
0 < |α|,|β|,|γ| < π/2,
γ = α±β,
(a,b,c)=(cosα,cosβ,cosγ)
のとき
(a-1)(b-1)(c-1)< 0 ?


(1-aa)(1-bb)=(c-ab)^2 ≧ 0 ゆえ
a-1,b-1,c-1 は同符号だが…

239 :132人目の素数さん:2017/11/08(水) 03:17:51.21 ID:X+T0MJpc.net
ごめんなさい。問題文を書き間違えていました。(切腹)

a,b,c≧0 かつ a+b+c>3 かつ a^2+b^2+c^2-2abc=1 のとき、(a-1)(b-1)(c-1)≧0
             ↑

240 :132人目の素数さん:2017/11/08(水) 03:20:27.40 ID:X+T0MJpc.net
自然数 n に対して、
n+3 < {(n+1)(n+2)(n+3)(n+4)(n+5)(n+6)}^(1/6) < n+(7/2)

どっかの入試問題だったと思うけど、メモしていない…。

241 :132人目の素数さん:2017/11/08(水) 12:31:35.96 ID:mblwdtt/.net
>>240

左側
(n+1)(n+2)(n+3)(n+4)(n+5)(n+6)-(n+3)^6 =(n+3)(3n^4 +31n^3 +102nn +103n -3)> 0,

右側は GM-AM で


蛇足だが…
{(n+1)(n+2)(n+3)(n+4)(n+5)(n+6)}^(1/6)= n +7/2 -35/(24n)+245/(48nn)-23597/(1152n^3)+69139/(768n^4)-34892549/(82944n^5)+340456375/(165888n^6)-…

242 :132人目の素数さん:2017/11/09(木) 00:22:49.65 ID:3X7VVSFu.net
>233 (3)

s = a+b+c,t = ab+bc+ca,u = abc,
とおく。
同次式なので u=1 としてもよい。s≧3,t≧3.

t(tt-4s+3)≧ t^3 -4stu +9uu ≧ 0,
∴ t ≧ √(4s-3)≧ s(10-s)/7,
∴ ss+7t ≧ 10s,
∴ aa+bb+cc + 9(ab+bc+ca)≧ 10(a+b+c),
かなあ。

243 :132人目の素数さん:2017/11/09(木) 11:55:17.23 ID:3X7VVSFu.net
>>232 (1)

F_2 = s^4 -5sst +4tt +6su ≧ 0, (Schur)
これをtについて解くと
t ≦[5ss -√{3s(3s^3 -32u)}]/8,

u=1 として
t ≦[5ss -√{3s(3s^3 -32u)}]/8 ≦(s^3 +18u)/{3(s+2)},
∴(s^3 -3st+3u)+ 15u ≧ 6t,
∴ a^3 +b^3 +c^3 + 15abc ≧ 6(ab+bc+ca),
かなあ。

>>233 (3) >>242

√(4s-3)≧ s(10-s)/7 のところ

s≧4 のとき
 √(4s-3)≧ 25/7 ≧ s(10-s)/7,
3≦s≦4 のとき
 (4s-3)-{s(10-s)/7}^2 =(s-3)(49-49s+17ss-s^3)/49 ≧0,

244 :132人目の素数さん:2017/11/15(水) 00:16:45.88 ID:09YU9CDB.net
a,b,c ∈R のとき
(1) (a^2 + b^2 + c^2)^2 ≧ 3(a^3b + b^3c + c^3a)
(2) a^4 + b^4 + c^4 ≧ 2(a^3b + b^3c + c^3a) - (ab^3 + bc^3 + ca^3)
(3) a^4 + b^4 + c^4 ≧ 6(a+b+c)(a^2b + b^2c + c^2a) - 17{(ab)^2 + (bc)^2 + (ca)^2}
(4) 3(a^4 + b^4 + c^4 - a^3b + b^3c + c^3a) ≧ a^2(b-c)^2 + b^2(c-a)^2 + c^2(a-b)^2


a,b,c≧0 のとき

(11) a^3 + b^3 + c^3 ≧ 3(ab^2 + bc^2 + ca^2) - 2(a^2b + b^2c + c^2a)
(12) a^3 + b^3 + c^3 ≧ (17/9)(a^2b + b^2c + c^2a) - (8/3)abc
(13) a^4 + b^4 + c^4 ≧ (9/4)(ab^3 + bc^3 + ca^3) - (5/4)(a^3b + b^3c + c^3a)
(14) a^4 + b^4 + c^4 ≧ 6{(ab)^2 + (bc)^2 + (ca)^2} - 5(a^3b + b^3c + c^3a)
(15) a^4 + b^4 + c^4 ≧ (ab)^2 + (bc)^2 + (ca)^2 + 2(a^3b + b^3c + c^3a - ab^3 - bc^3 - ca^3)
(16) a^4 + b^4 + c^4 ≧ abc(a+b+c) + (2√2)(a^3b + b^3c + c^3a - ab^3 - bc^3 - ca^3)

巡回式は嫌いでござる。

245 :132人目の素数さん:2017/11/15(水) 01:09:17.86 ID:09YU9CDB.net
>>242
> ∴ t ≧ √(4s-3)≧ s(10-s)/7,

√(4s-3)≧ s(10-s)/7 って、不等号の向きが一定でないような…

246 :132人目の素数さん:2017/11/15(水) 12:35:38.33 ID:bRQyF7b9.net
>>245

まづ AM-GMより s ≧ 3u^(1/3)= 3,

あとは >>243 (下)を参照。

247 :132人目の素数さん:2017/11/15(水) 16:27:06.63 ID:bRQyF7b9.net
>>244

〔4次巡回不等式の基本定理〕(定理2.3.3)

a^4 + b^4 + c^4 - p(a^3b+b^3c+c^3a)- q(ab^3+bc^3+ca^3)+{(pp+pq+qq)/3 -1}(aabb+bbcc+ccaa)+{p+q-(pp+pq+qq)/3}abc(a+b+c)
={(A-B)^2+(B-C)^2+(C-A)^2}/6,
ただし、A,B,Cは
A = aa-bb +pbc -qca,
B = bb-cc +pca -qab,
C = cc-aa +pab -qbc,

(1)(p,q)=(3,0)
(2)(p,q)=(2,-1) 例題2.3.10(8)*
(3)(p,q)=(6,0) 例題2.3.10(7)
(4)(p,q)=(1,0) 例題2.3.10(4)
(13)  例題2.3.11(2) (2)を精密化したもの。 {右辺の係数を α+1、-α として α≦1.379…}
(14)  例題2.3.12(3)次の(15)から出る。{右辺の係数をγ+1,-γ として γ≦5.0779…}
(15)  例題2.3.11(5)
(16)  例題2.3.12(4)

* a:b:c = sin(π/9):{sin(2π/9)-sin(π/3)}:sin(2π/9)
のとき等号が成立するらしい。

文献[8]安藤「不等式」数学書房(2012)
 §2.3(4次斉次不等式)

248 :132人目の素数さん:2017/11/15(水) 18:14:50.44 ID:bRQyF7b9.net
>>244

min{a,b,c}= m とし、{a,b,c}={m,m+x,m+y}とする。(x,y≧0)

(11)
(左辺)-(右辺)= m(xx-xy+yy)+(x^3 + 2xxy -3xyy + y^3)≧0,

∵ x^3 +2xx -3x +1 ≧ 3x^(7/3)-3x + 1 ≧ 1 - 4・(3/7)^(7/4)= 0.091969

(12)
(左辺)-(右辺)= 10m(xx-xy+yy)/9 +{x^3 -(17/9)xxy + y^3}≧0,

∵ x^3 -(17/9)xx + 1 ≧ 1 -2・(17/27)^(3/2)= 0.00078779,

249 ::2017/11/15(水) 21:31:33.05 ID:WZuPK5Ir.net


250 ::2017/11/15(水) 21:31:51.29 ID:WZuPK5Ir.net


251 ::2017/11/15(水) 21:32:08.69 ID:WZuPK5Ir.net


252 ::2017/11/15(水) 21:32:26.30 ID:WZuPK5Ir.net


253 ::2017/11/15(水) 21:32:44.23 ID:WZuPK5Ir.net


254 ::2017/11/15(水) 21:33:03.14 ID:WZuPK5Ir.net


255 ::2017/11/15(水) 21:33:21.26 ID:WZuPK5Ir.net


256 ::2017/11/15(水) 21:33:40.13 ID:WZuPK5Ir.net


257 ::2017/11/15(水) 21:33:57.60 ID:WZuPK5Ir.net


258 ::2017/11/15(水) 21:34:16.51 ID:WZuPK5Ir.net


259 :132人目の素数さん:2017/11/16(木) 01:18:28.78 ID:+0/ZGG+j.net
>>248

xx-xy+yy ≧ 0,
(左辺−右辺)はmについて単調増加。
∴{a,b,c}の差(x,y)を固定して一斉に増加すれば増加する。
∴ m=0 の場合を考えれば十分。

(sageるのを忘れてしまった…)

260 ::2017/11/16(木) 05:25:38.76 ID:6ldUKvsQ.net


261 ::2017/11/16(木) 05:25:54.25 ID:6ldUKvsQ.net


262 ::2017/11/16(木) 05:26:11.00 ID:6ldUKvsQ.net


263 ::2017/11/16(木) 05:26:26.16 ID:6ldUKvsQ.net


264 ::2017/11/16(木) 05:26:44.23 ID:6ldUKvsQ.net


265 ::2017/11/16(木) 05:27:05.53 ID:6ldUKvsQ.net


266 ::2017/11/16(木) 05:27:21.26 ID:6ldUKvsQ.net


267 ::2017/11/16(木) 05:27:37.13 ID:6ldUKvsQ.net


268 ::2017/11/16(木) 05:27:53.04 ID:6ldUKvsQ.net


269 ::2017/11/16(木) 05:28:12.82 ID:6ldUKvsQ.net


270 :132人目の素数さん:2017/11/16(木) 20:12:37.97 ID:aDSEDWj+.net
三角形の辺長 a,b,c、外接円の半径R、内接円の半径 r に対して

(1) a^2+ b^2 + c^2 ≦ 9R^2
(2) a^2+ b^2 + c^2 ≦ 8R^2 + 4r^2

271 :132人目の素数さん:2017/11/17(金) 00:27:20.94 ID:VLF4tQu0.net
>>270

正弦定理より
aa+bb+cc = 4RR{sin(A)^2 + sin(B)^2 + sin(C)^2},

sin(A)^2 + sin(B)^2 + sin(C)^2 = 2 + 2cos(A)cos(B)cos(C)- 4cos((A+B+C)/2)cos((-A+B+C)/2)cos((A-B+C)/2)cos((A+B-C)/2)
  = 2 + 2cos(A)cos(B)cos(C),   (← A+B+C=π)

(1)
・鈍角または直角 のときは 左辺 ≦ 2,

・鋭角△ のときは AM-GM と凸性より
 cos(A)cos(B)cos(C)≦[{cos(A)+cos(B)+cos(C)}/3 ]^3 ≦[ cos((A+B+C)/3)]^3 = [ cos(π/3)]^3 = 1/8,
 左辺 ≦ 2 + 1/4 = 9/4,

∴ aa+bb+cc ≦ 9RR,
等号成立は A=B=C(正△)のとき

272 ::2017/11/17(金) 02:10:06.73 ID:Y8c01QBt.net


273 ::2017/11/17(金) 02:10:23.30 ID:Y8c01QBt.net


274 ::2017/11/17(金) 02:10:41.11 ID:Y8c01QBt.net


275 ::2017/11/17(金) 02:10:59.54 ID:Y8c01QBt.net


276 ::2017/11/17(金) 02:11:19.10 ID:Y8c01QBt.net


277 ::2017/11/17(金) 02:11:41.31 ID:Y8c01QBt.net


278 ::2017/11/17(金) 02:12:01.05 ID:Y8c01QBt.net


279 ::2017/11/17(金) 02:12:19.15 ID:Y8c01QBt.net


280 ::2017/11/17(金) 02:12:36.98 ID:Y8c01QBt.net


281 ::2017/11/17(金) 02:12:57.07 ID:Y8c01QBt.net


282 :132人目の素数さん:2017/11/18(土) 15:08:01.65 ID:E4miXSjs.net
>>270

(2)
正弦定理より
(a+b-c)/c = 2sin(A/2)sin(B/2)/sin(C/2),etc.

r/R =(2S/(a+b+c))(4S/abc)
 =(a+b-c)(b+c-a)(c+a-b)/(2abc)
 = 4sin(A/2)sin(B/2)sin(C/2),

(r/R)^2 = 2[1-cos(A)][1-cos(B)][1-cos(C)],

したがって、補題より
 sin(A)^2 + sin(B)^2 + sin(C)^2
 = 2 + 2cos(A)cos(B)cos(C)
 ≦ 2 + 2[1-cos(A)][1-cos(B)][1-cos(C)]
 = 2 +{4sin(A/2)sin(B/2)sin(C/2)}^2
 = 2 +(r/R)^2,


【補題】A+B+C=π, 0<A,B,C のとき、
 cos(A)cos(B)cos(C) ≦ [1-cos(A)][1-cos(B)][1-cos(C)] ≦ 1/8.
 [初代スレ.580]

【系】
{(a+b-c)(b+c-a)(c+a-b)}^2 ≧ (aa+bb-cc)(bb+cc-aa)(cc+aa-bb),
 [1992 Poland]44th,1st round(1992 Sept-Dec)No.9
[初代スレ.538(3)]

283 ::2017/11/18(土) 17:11:23.66 ID:jL40qEXq.net


284 ::2017/11/18(土) 17:11:39.69 ID:jL40qEXq.net


285 ::2017/11/18(土) 17:12:05.66 ID:jL40qEXq.net


286 ::2017/11/18(土) 17:12:26.94 ID:jL40qEXq.net


287 ::2017/11/18(土) 17:12:43.02 ID:jL40qEXq.net


288 ::2017/11/18(土) 17:12:59.82 ID:jL40qEXq.net


289 ::2017/11/18(土) 17:13:16.12 ID:jL40qEXq.net


290 ::2017/11/18(土) 17:13:32.46 ID:jL40qEXq.net


291 ::2017/11/18(土) 17:13:49.49 ID:jL40qEXq.net


292 ::2017/11/18(土) 17:14:06.65 ID:jL40qEXq.net


293 :132人目の素数さん:2017/11/18(土) 22:41:53.11 ID:E4miXSjs.net
>>282 〔補題〕

・鈍角または直角凾フとき
 cos(A)cos(B)cos(C)≦ 0 で成立。

・鋭角△のとき
(右辺)-(左辺)
=[1-cos(A)][1-cos(B)][1-cos(C)]- cos(A)cos(B)cos(C)
=[1-cos(A)][1-cos(B)]-4sin(A/2)sin(B/2)cos((A-B)/2)cos(C)+[cos(C)]^2
={2sin(A/2)sin(B/2)- cos(C)}^2 + 4sin(A/2)sin(B/2)[1-cos((A-B)/2)]
≧0.

294 ::2017/11/19(日) 01:45:08.43 ID:1TUhKzn4.net


295 ::2017/11/19(日) 01:45:24.09 ID:1TUhKzn4.net


296 ::2017/11/19(日) 01:45:40.18 ID:1TUhKzn4.net


297 ::2017/11/19(日) 01:45:59.18 ID:1TUhKzn4.net


298 ::2017/11/19(日) 01:46:15.18 ID:1TUhKzn4.net


299 ::2017/11/19(日) 01:46:31.65 ID:1TUhKzn4.net


300 ::2017/11/19(日) 01:46:53.40 ID:1TUhKzn4.net


301 ::2017/11/19(日) 01:47:10.08 ID:1TUhKzn4.net


302 ::2017/11/19(日) 01:47:27.60 ID:1TUhKzn4.net


303 ::2017/11/19(日) 01:47:45.83 ID:1TUhKzn4.net


304 :132人目の素数さん:2017/11/19(日) 15:05:19.28 ID:PrPXB/k6.net
>>282 〔系〕

b+c-a = x,c+a-b = y,a+b-c = z,
とおく。(Ravi変換)

(左辺)-(右辺)=(xyz)^2 -(aa+bb-cc)(bb+cc-aa)(cc+aa-bb)
= 2abxyzz -2(a+b)xyz(aa+bb-cc)+ 2cc(aa+bb-cc)^2
= 2{z√(abxy)-c(aa+bb-cc)}^2 + 2(√xy)z(aa+bb-cc){2c√(ab)-(a+b)√(xy)}
≧ 0,

∵ 2c√(ab)= c√{(a+b)^2 -(a-b)^2}
 ≧(a+b)√{cc -(a-b)^2}
 =(a+b)√(xy),

305 ::2017/11/19(日) 15:17:28.55 ID:1TUhKzn4.net


306 ::2017/11/19(日) 15:17:46.55 ID:1TUhKzn4.net


307 ::2017/11/19(日) 15:18:04.06 ID:1TUhKzn4.net


308 ::2017/11/19(日) 15:18:20.73 ID:1TUhKzn4.net


309 ::2017/11/19(日) 15:18:38.37 ID:1TUhKzn4.net


310 ::2017/11/19(日) 15:18:56.59 ID:1TUhKzn4.net


311 ::2017/11/19(日) 15:19:15.03 ID:1TUhKzn4.net


312 ::2017/11/19(日) 15:19:32.88 ID:1TUhKzn4.net


313 ::2017/11/19(日) 15:19:49.36 ID:1TUhKzn4.net


314 ::2017/11/19(日) 15:20:08.30 ID:1TUhKzn4.net


315 :132人目の素数さん:2017/11/20(月) 09:46:52.20 ID:cP+zUA27.net
>>304 (補足)

右辺: a,b,c の符号によらない。
左辺: a,b,c が同符号のとき(|a|+|b|+|c|)の因子を含まず、最も小さい。
よって a,b,c ≧0 としてよい。
a+b < c のときは aa+bb-cc < 0、右辺 < 0 となり成立。

316 :132人目の素数さん:2017/11/20(月) 10:04:54.20 ID:B0AOtDd9.net
難しいな

317 :132人目の素数さん:2017/11/20(月) 10:16:19.70 ID:B0AOtDd9.net
>>282
> {(a+b-c)(b+c-a)(c+a-b)}^2 ≧ (aa+bb-cc)(bb+cc-aa)(cc+aa-bb)

>>270 (2) がこんな形になろうとは…

318 :132人目の素数さん:2017/11/20(月) 11:23:28.64 ID:cP+zUA27.net
>>282 (系)
>>317

Schur の拡張より
(左辺)-(右辺)= p(a-b)(a-c)+ q(b-c)(b-a)+ r(c-a)(c-b)≧ 0

p = 2aa(a+c-b)(a+b-c)≧0,
q = 2bb(b+a-c)(b+c-a)≧0,
r = 2cc(c+b-a)(c+a-b)≧0,
は a,b,c と同順序

319 :132人目の素数さん:2017/11/20(月) 11:48:06.47 ID:/yxWXQiH.net
任意の二つの非負実数列{a_n},{b_n}に対して、
納i,j=1,n]min{a_ia_j,b_ib_j}≦納i,j=1,n]min{a_ib_j,a_jb_i}

USAMO-2000

320 :132人目の素数さん:2017/11/20(月) 16:29:50.97 ID:cP+zUA27.net
>>248 (訂正)

(12)

∵ x^3 -(17/9)xx + 1 = 31/(27^3) +(x +17/27)(x -34/27)^2 ≧ 31/19683 = 0.0015750

321 :132人目の素数さん:2017/11/20(月) 23:19:47.28 ID:cP+zUA27.net
>>248
(11)AM-GM で
x^3 + a^3 + a^3 ≧ 3aax,
 2xx + 2aa ≧ 4ax,
辺々たすと
 x^3 + 2xx + 2aa(a+1)≧ a(3a+4),
ここで 2aa(a+1)= 1 すなわち a = 0.565198… とすれば
 x^3 + 2xx +1 ≧ a(3a+4)x = 3.21914 x,

3x^(7/3)+ 4(2/7)^(7/4)≧ 3x,

>>320
(12)AM-GM で
 x^3 + 4(17/27)^3 =(1/2)x^3 + (1/2)x^3 + 4(17/27)^3 ≧ 3(17/27)xx =(17/9)xx,

322 :132人目の素数さん:2017/11/21(火) 17:40:36.86 ID:hVVDnwMW.net
>>319

〔補題1〕
r_i ≧ 0 のとき、実対称行列 min{r_i,r_j}は半正値。
(略証)
0 = r_0 ≦ r_1 ≦ r_2 ≦ …… ≦ r_n としてよい。
 Σ[i,j]min{r_i,r_j}x_i x_j = Σ[i=1,n]r_i((x_i)^2 + 2Σ[j=i+1,n] x_i x_j)
 = Σ[i=1,n]r_i((Σ[j=i,n]x_j)^2 -(Σ[j=i+1,n]x_j)^2)
 = Σ[i=1,n](r_i - r_{i-1})(Σ[j=i,n]x_j)^2
 ≧ 0,

〔補題2〕
min{a_i b_j,a_j b_i}- min{a_i a_j,b_i b_j}= min{r_i,r_j}x_i x_j

ここに、
 r_i = max{a_i/b_i,b_i/a_i}
 x_i = sgn(a_i-b_i)min{a_i,b_i}
とおいた。(証明略)

これらを使うと解けるらしいよ。

USAMO-2000 Problem_6

http://artofproblemsolving.com/wiki/index.php?title=2000_USAMO

323 :132人目の素数さん:2017/11/22(水) 09:41:28.62 ID:Pyw3tUJ6.net
ここの人達は
ソボレフの不等式とかポアンカレの不等式とかの解析の不等式は興味ないの?

324 ::2017/11/22(水) 10:59:07.28 ID:j3iP9uSb.net


325 ::2017/11/22(水) 10:59:25.04 ID:j3iP9uSb.net


326 ::2017/11/22(水) 10:59:44.99 ID:j3iP9uSb.net


327 ::2017/11/22(水) 11:00:04.41 ID:j3iP9uSb.net


328 ::2017/11/22(水) 11:00:23.07 ID:j3iP9uSb.net


329 ::2017/11/22(水) 11:00:42.66 ID:j3iP9uSb.net


330 ::2017/11/22(水) 11:01:01.78 ID:j3iP9uSb.net


331 ::2017/11/22(水) 11:01:22.29 ID:j3iP9uSb.net


332 ::2017/11/22(水) 11:01:41.84 ID:j3iP9uSb.net


333 ::2017/11/22(水) 11:02:01.16 ID:j3iP9uSb.net


334 :132人目の素数さん:2017/11/22(水) 12:39:34.60 ID:XxzuuRWx.net
>>322

補題1は
 g_i(t)=(1/π)log|1-(r_i)/t|
 ∫(-∞,∞)g_i(t)g_j(t)dt= min{r_i,r_j}
からも出る。

 森口・宇田川・一松「数学公式I」岩波全書221(1956)p.242


>>323
 ageるな危険

335 ::2017/11/22(水) 14:29:14.18 ID:j3iP9uSb.net


336 ::2017/11/22(水) 14:29:31.05 ID:j3iP9uSb.net


337 ::2017/11/22(水) 14:29:49.85 ID:j3iP9uSb.net


338 ::2017/11/22(水) 14:30:08.11 ID:j3iP9uSb.net


339 ::2017/11/22(水) 14:30:27.03 ID:j3iP9uSb.net


340 ::2017/11/22(水) 14:30:45.57 ID:j3iP9uSb.net


341 ::2017/11/22(水) 14:31:03.98 ID:j3iP9uSb.net


342 ::2017/11/22(水) 14:31:21.22 ID:j3iP9uSb.net


343 ::2017/11/22(水) 14:31:39.48 ID:j3iP9uSb.net


344 ::2017/11/22(水) 14:31:57.68 ID:j3iP9uSb.net


345 :132人目の素数さん:2017/11/23(木) 23:04:17.76 ID:BEW5rGkW.net
〔Problem 6〕

次の不等式をみたす有界無限実数列: x_0,x_1,x_2,… を1つ与えよ。

 i≠j ⇒ |x_i - x_j||i - j| > 1,

Construct a bounded infinite swquence x_0,x_1,x_2,…… such that |x_i - x_j||i - j| > 1 for every pair of distinct i,j.

IMO-1991(32nd,Sweden) modified.

346 :132人目の素数さん:2017/11/23(木) 23:29:04.99 ID:BEW5rGkW.net
実数列 x_0,x_1,x_2,… が有界であるとは、ある定数Cが存在して、
 すべての非負整数i≧ 0 に対して |x_i| ≦ C,
が成り立つことである。

347 ::2017/11/24(金) 00:15:03.95 ID:7RwNGOaZ.net


348 ::2017/11/24(金) 00:15:24.99 ID:7RwNGOaZ.net


349 ::2017/11/24(金) 00:15:40.50 ID:7RwNGOaZ.net


350 ::2017/11/24(金) 00:15:57.99 ID:7RwNGOaZ.net


351 ::2017/11/24(金) 00:16:19.42 ID:7RwNGOaZ.net


352 ::2017/11/24(金) 00:16:40.29 ID:7RwNGOaZ.net


353 ::2017/11/24(金) 00:17:01.85 ID:7RwNGOaZ.net


354 ::2017/11/24(金) 00:17:24.52 ID:7RwNGOaZ.net


355 ::2017/11/24(金) 00:17:44.49 ID:7RwNGOaZ.net


356 ::2017/11/24(金) 00:18:01.06 ID:7RwNGOaZ.net


357 :132人目の素数さん:2017/11/26(日) 12:53:55.06 ID:VpOkBIAF.net
【三重大】
a,b,c>0 に対して、a+b+c ≧ √(3ab) + √(3bc) - √(ca)

358 :132人目の素数さん:2017/11/26(日) 14:07:57.08 ID:b02GGEMo.net
>>357

√a = A,√b = B,√c = C とおく。

(左辺)-(右辺)=[B -(√3)(A+C)/2]^2 +(1/4)(A-C)^2 ≧ 0,

等号は(a,b,c)=(1,3,1)

359 :132人目の素数さん:2017/11/26(日) 14:54:01.10 ID:VpOkBIAF.net
a,b,c>0、α+β+γ=πのとき、
a+b+c ≧ 2{√(ab)} cosα + 2{√(bc)} cosβ+ 2{√(ca)} cosγ

360 :132人目の素数さん:2017/11/26(日) 15:22:24.62 ID:kAfhFVfw.net
a,b,c>0,a+b+c=3のとき、

a/(a+bc)+b/(b+ca)+c/(c+ab)≧3/2

を示せ

361 :132人目の素数さん:2017/11/27(月) 01:36:28.10 ID:f62zTFKa.net
>>322 >>334

補題1は
 g_i(t)= √(2/π)sin(r_i・t)/t,
 ∫[0,∞)g_i(t)g_j(t)dt= min{r_i,r_j}
からも出る。

 森口・宇田川・一松「数学公式I」岩波全書221(1956)p.251

362 :132人目の素数さん:2017/11/27(月) 10:13:05.15 ID:f62zTFKa.net
>>361

積和公式
2 sin(at)sin(bt)= cos((a-b)t)- cos((a+b)t),
より
(1/π)∫[0,∞)2sin(at)sin(bt)/tt dt
=(1/π)∫[0,∞){1-cos((a+b)t)}/tt dt -(1/π)∫[0,∞){1-cos((a-b)t)}/tt dt
={(|a+b|-|a-b|)/π}∫[0,∞){1-cos(u)}/uu du
=(|a+b|-|a-b|)/2      (*)
= min{a,b}.

*)高木「解析概論」改訂第三版、岩波書店(1961) 
 §48,p.169

363 :132人目の素数さん:2017/11/27(月) 13:02:07.28 ID:f62zTFKa.net
>>359

実対称行列S=
|1, -cosα,-cosγ|
|-cosα,1, -cosβ|
|-cosγ,-cosβ,1 |
とおく。

題意よりα+β+γ = π,

(cosα)^2 +(cosβ)^2 +(cosγ)^2 + 2cosα cosβ cosγ = 1,

AM-GM と 凸性より
 -1 < cosα cosβ cosγ ≦{cos(π/3)}^3 = 1/8,

Sの固有多項式は

|1-x,-cosα,-cosγ|
|-cosα,1-x,-cosβ|
|-cosγ,-cosβ,1-x|

= x{xx -3x +2(1 + cosα cosβ cosγ)}
= x f(x),

f(0)> 0,
f(3/2)= -1/4 + 2 cosα cosβ cosγ < 0,
∴ f(x)は2つの正根をもつ。

Sの固有値は 0 と 2つの正値。

∴Sは半正値。

364 :132人目の素数さん:2017/11/27(月) 22:46:38.03 ID:f62zTFKa.net
>>363

正攻法なんだけど、面白くねぇ?

365 :132人目の素数さん:2017/11/27(月) 22:53:52.69 ID:sVsMvcLE.net
たぶん半正値が分からんのでは?

366 :132人目の素数さん:2017/11/28(火) 12:39:41.20 ID:8oBcz/rm.net
>>365

〔例題1〕
任意の実数 a,b,c に対して
 aa+bb+cc ≧ 2pab + 2qbc + 2rca
となるための、p,q,r についての条件を求めよ。

(解)
 |p|≦ 1,|q|≦ 1,|r|≦ 1,pp+qq+rr + 2pqr ≦ 1.

 文献[3](大関)の冒頭の例題

----------------------------------

本問では
p = cos(A),q = cos(B),r = cos(C)
pp + qq + rr + 2pqr = 1.
となるから、上記の条件を満たす。

367 :132人目の素数さん:2017/11/28(火) 13:07:38.16 ID:8oBcz/rm.net
>>345-346

 x_i = k{i・√m - 1/2}, k = 1 + 2√m,

ここに m は平方数でない自然数。{ a }はaの小数部分
 (富蘭平太氏)

368 :132人目の素数さん:2017/11/28(火) 16:17:28.24 ID:xZ2b6WdA.net
a_1^2+a_2^2+……+a_100^2=1を満たす、非負実数列a_1,a_2,……,a_100に対して、
a_1^2*a_2+a_2^2*a_3+……+a_100^2*a_1<12/35

369 :132人目の素数さん:2017/11/28(火) 20:21:05.98 ID:8oBcz/rm.net
>>368

(右辺)= 12/35 = 0.342857

(a_1,a_2,…,a_100)=(√(2/3),√(1/3),0,…,0)=(0.81650,0.57735,0,…,0)
のとき
(左辺)= 2/(3√3)= 0.38490

(a_1,a_2,…,a_100)=(2/3,2/3,1/3,0,…,0)=(0.66667,0.66667,0.33333,0,…,0)
のとき
(左辺)= 4/9 = 0.44444

370 :132人目の素数さん:2017/11/28(火) 23:25:41.02 ID:xZ2b6WdA.net
>>369
12/35は12/25の間違いでした。これなら不等式は成立するはずです。すみません

371 :132人目の素数さん:2017/12/01(金) 02:32:13.25 ID:jDGDIkRq.net
>>368 >>370

一例だけど...

a_1 = 0.000835
a_2 = 0.587293
a_3 = 0.677084 〜 65/96
a_4 = 0.422379
a_5 = 0.134393
a_6 = 0.0133427
a_7 = 0.000305
a_k = 0    (k≧8)

のとき

M = 0.451389 〜 65/144 < 12/25,

12/25 まで解けぬでござるか…

372 :132人目の素数さん:2017/12/01(金) 22:53:39.42 ID:jDGDIkRq.net
>>360

s = a+b+c,t = ab+bc+ca,u = abc,
とおくと、
(左辺)={3u +2(tt-2su)+(ss-2t)u}/{u +(tt-2su)+(ss-2t)u + uu},

2(分子)-3(分母)= 3u +(tt-2su)-(ss-2t)u -3uu
=(t^3 -4stu +9uu)/t + (tt-3su)u/t +(st-9u)u/3  (s=3)
≧ 0,

t^3 -4stu +9uu = uuF_{-2}(a,b,c)≧ 0,
tt -3su = uF_{-1}(a,b,c)≧ 0,
st -9u = a(b-c)^2 + b(c-a)^2 + c(a-b)^2 ≧ 0,

373 :132人目の素数さん:2017/12/08(金) 12:40:19.79 ID:VQt4XLp7.net
>>323
ありませんね。

ヒルベルト空間、バナッハ空間、シュワルツ空間、ハーディ空間、BMO空間、ベゾフ空間、トリーベル・リゾルキン空間
なんかに興味があるようでつ…

374 :132人目の素数さん:2017/12/18(月) 09:51:44.58 ID:Y1Xeg8V4.net
正値数列 {a_n} の初項から第 k 項までの和を s_k とおくとき、

Σ[k=1 to n] (s_k)*(a_k)^2 > {4*(s_n)^3}/(9n) を証明せよ。

375 ::2017/12/28(木) 03:17:10.89 ID:THlb34Rw.net


376 ::2017/12/28(木) 03:17:27.29 ID:THlb34Rw.net


377 ::2017/12/28(木) 03:17:43.14 ID:THlb34Rw.net


378 ::2017/12/28(木) 03:17:59.64 ID:THlb34Rw.net


379 ::2017/12/28(木) 03:18:16.69 ID:THlb34Rw.net


380 ::2017/12/28(木) 03:18:33.53 ID:THlb34Rw.net


381 ::2017/12/28(木) 03:18:51.42 ID:THlb34Rw.net


382 ::2017/12/28(木) 03:19:26.84 ID:THlb34Rw.net


383 ::2017/12/28(木) 03:19:43.74 ID:THlb34Rw.net


384 ::2017/12/28(木) 03:19:58.20 ID:THlb34Rw.net


385 :132人目の素数さん:2017/12/29(金) 19:39:20.58 ID:tFbo94hj.net
実数 x に対して、8^x + 27^x + 64^x + 125^x ≧ 24^x + 30^x + 40^x + 60^x

386 :132人目の素数さん:2018/01/01(月) 04:36:26.50 ID:YGqriAdp.net
>>385

2^x = A,3^x = B,4^x = C,5^x = D
とおくと
A^3 + B^3 + C^3 + D^3 ≧ ABC + ABD + ACD + BCD
以下、相加-相乗平均で…
等号成立は A=B=C=D より x=0

      ∧_∧
     ( ´Д` )  新年あけまして
     /     ヽ
     し、__X__,ノJ

      /´⌒⌒ヽ
    l⌒    ⌒l  おめでとうございます
   ⊂ (   ) ⊃
      V ̄V

387 :132人目の素数さん:2018/01/01(月) 07:23:34.92 ID:v3DaEB7f.net
正の数 a、b、c に対して、
(a^{2018}-a^{30}+3)(b^{2018}-b^{30}+3)(c^{2018}-c^{30}+3) ≧ 9(a^2+b^2+c^2)

       人
 /⌒\ (__)
 \●/(__)/⌒\
   ∩ (・∀・ )\●/  あけおめでござる
   Y  ̄ ||y||  ̄`''φ
    Lノ /ニ|| ! ソ >
    乂/ノ ハ ヽー´
    `ー-、__|

388 :132人目の素数さん:2018/01/01(月) 07:51:57.61 ID:v3DaEB7f.net
実数a,b,cでおkだね。

389 :132人目の素数さん:2018/01/01(月) 16:48:15.07 ID:YGqriAdp.net
>>387-388

0.9347299 < a = b = c < 1

うーむ

390 :132人目の素数さん:2018/01/05(金) 01:17:08.66 ID:TIUiz4a3.net
〔問題387〕
実数 a,b,c について
(a^2018 -a^30 +3)(b^2018 -b^30 +3)(c^2018 -c^30 +3)≧ 3(a^4 +b^4 +c^4),

391 :132人目の素数さん:2018/01/08(月) 09:12:48.34 ID:o2HjFYS0.net
>>390
(a^2018 -a^30 +3)(b^2018 -b^30 +3)(c^2018 -c^30 +3)
≧ (a^1988 +2)(b^1988 +2)(c^1988 +2)
≧ 3(a^1988 + b^1988 +1)(1+1+c^1988)
≧ 3(a^994 + b^994 + c^994)^2
≧ (1/3)*(a^497 + b^497 + c^497)^4

          r〜〜〜〜〜
   __    _ノ うっうっうっ・・・
  /__  `ヽ_ ⌒ヽ〜〜〜〜〜
  |〈___ノf レ1(
 ,L| しL.し'゙"
 "`  "′

392 :132人目の素数さん:2018/01/08(月) 23:30:24.00 ID:6yE9/RTC.net
>>391

1に関して同じ側にある2つをa,bとしたでござるな。つまり
(a-1)(b-1)≧ 0.

393 :132人目の素数さん:2018/01/09(火) 20:59:25.04 ID:KWEYIsTZ.net
( ゚∀゚)つ 0.9999^101 < 0.99 < 0.9999^100 を示せ。

394 :132人目の素数さん:2018/01/09(火) 21:00:34.56 ID:KWEYIsTZ.net
>>390
ヒントが欲しいでござる。

395 :132人目の素数さん:2018/01/10(水) 01:12:13.78 ID:bRNIXSXz.net
>>394
ヒントぢゃねぇが…

x^(1988+n)- x^n +3 ≧ 9^(1/3)x^(4/3)
は n≧30 で成立つらしい。

396 :132人目の素数さん:2018/01/10(水) 01:32:22.50 ID:bRNIXSXz.net
>>393

(1 - 1/nn)^(n+1)< 1 - 1/n <(1 - 1/nn)^n,

(右側)
 {1,1,…,1,1 - 1/n}の n個で GM-AM する。
   n-1 個

(左側)
 {1,1,…,1,1 + 1/(n-1)}の n+1個で AM-GM する。
   n個
 {1 - 1/(nn-1)}^(n+1)≧ 1 + 1/(n-1),
 逆数をとる。

397 :132人目の素数さん:2018/01/10(水) 06:16:58.25 ID:M8EQn1L+.net
>>395
それを使うと、示すべき右辺 3(a^4 +b^4 +c^4) にAM-MGを使った形になっちまう…

(a^2018 -a^30 +3)(b^2018 -b^30 +3)(c^2018 -c^30 +3)≧ 9(abc)^(4/3)

398 :132人目の素数さん:2018/01/11(木) 04:40:46.35 ID:zTW4b5dN.net
>>374

・n=2 のとき

a^3 +(a+b)bb -(1/3)(a+b)^3
={(2a^3 - 3aab + b^3) + b^3}/9
≧(b^3)/9
≧ 0,

・n=3 のとき

a^3 +(a+b)bb +(a+b+c)cc -(1/5)(a+b+c)^3
={(7/4)x^3 +(23/8)y^3 + 4z^3 - 6xyz + 2x(3x/4 -y)^2 + 2x(3x/4 -z)^2 + 2y(3y/4 -z)^2}/5
>{(7/4)x^3 +(20/7)y^3 + 4z^3 - 6xyz}/5
≧(3{20^(1/3)}xyz - 6xyz)/5
= 3xyz/7
≧ 0,

399 :132人目の素数さん:2018/01/11(木) 04:50:04.41 ID:zTW4b5dN.net
>>398 の訂正

・n=3 のとき

a^3 +(a+b)bb +(a+b+c)cc -(1/5)(a+b+c)^3
={(7/4)a^3 +(23/8)b^3 + 4c^3 - 6abc + 2a(3a/4 -b)^2 + 2a(3a/4 -c)^2 + 2b(3b/4 -c)^2}/5
>{(7/4)a^3 +(20/7)b^3 + 4c^3 - 6abc}/5   (23/8 > 20/7)
≧3({20^(1/3)}abc - 2abc)/5   (AM-GM)
≧ 3abc/7
≧ 0,

400 :132人目の素数さん:2018/01/12(金) 00:58:22.61 ID:Or3GsGEd.net
三角形の成立条件 a+b>c、b+c>a、c+a>b を当たり前のように使っているけど、
これの証明って、手元にある本には載っていないんだけど、どうやるんだっけ?

401 :132人目の素数さん:2018/01/12(金) 01:05:31.40 ID:43CyjHwT.net
まず長さaの線分を描いてから、半径b、cの円を描いてみれば分かる

402 :132人目の素数さん:2018/01/12(金) 01:19:24.94 ID:Or3GsGEd.net
>>401
なるほど、さんくす。分かりやすい。

403 :132人目の素数さん:2018/01/12(金) 03:39:31.00 ID:aNi1PHCw.net
>>370-371

もう2つ...

・例1 等比数列
 a_k = a_1・r^(k-1),
r = 0.636323 a_1 = √(1-rr)= 0.772413
のとき
 M = 0.393502 < 0.48

・例2
 a_(2k+1)= a_1・r^k,
 a_(2k+2)= a_2・s^k,
 a_1 = 0.7141094  r = 0.359890
 a_2 = 0.6408248  s = 0.0921138
のとき
 M = 0.436598 < 0.48

404 :132人目の素数さん:2018/01/12(金) 17:59:00.61 ID:aNi1PHCw.net
>>370-371

もう1つ...

・例3
 a_1 = 0.66763055
 a_2 = 0.66280835
 a_k = a_2・r^(k-2)  (k>2)
 r ={(7+4√3)^(1/3)+(7-4√3)^(1/3)- 1}/4 = 0.455410041101
   (4r^3 +3rr -1 = 0 の実根)
のとき
 M =(2/3)√{(1+r)(1-r^3)/3}= 0.4418722310911 < 0.48

405 :132人目の素数さん:2018/01/13(土) 01:56:27.50 ID:vxAtUgvr.net
>>403

・例1
 a_1 = 0.7714225971
  (a^8 -9a^6 +30a^4 -30a^2 +9 =0 の正根)
 r = 0.63632317
  ((1+r)(1-r^3)-3rr = 0 の実根)
 M = r(1-rr)^(3/2)/(1-r^3)= 0.393502193

>>404

・例3
 r ={(2+√3)^(2/3)+(2-√3)^(2/3)- 1}/4 = 0.45541
  (4r^3 +3rr -1 = 0 の実根)
 M ={(207+48√3)^(1/3)+(207-48√3)^(1/3)- 1}/24
  (M^3 +(1/8)M^2 -(1/6)M -(1/27)= 0 の実根)

406 ::2018/01/20(土) 07:20:04.89 ID:Vdmu6X2x.net


407 ::2018/01/20(土) 07:20:27.41 ID:Vdmu6X2x.net


408 ::2018/01/20(土) 07:20:50.76 ID:Vdmu6X2x.net


409 ::2018/01/20(土) 07:21:11.16 ID:Vdmu6X2x.net


410 ::2018/01/20(土) 07:21:33.24 ID:Vdmu6X2x.net


411 ::2018/01/20(土) 07:21:53.05 ID:Vdmu6X2x.net


412 ::2018/01/20(土) 07:22:14.30 ID:Vdmu6X2x.net


413 ::2018/01/20(土) 07:22:34.10 ID:Vdmu6X2x.net


414 ::2018/01/20(土) 07:22:55.28 ID:Vdmu6X2x.net


415 ::2018/01/20(土) 07:23:14.32 ID:Vdmu6X2x.net


416 :132人目の素数さん:2018/01/23(火) 23:58:40.42 ID:7yFnQM5h.net
>>374
これだったか…
http://www.toshin.com/concours/mondai/answer201702.pdf

417 :132人目の素数さん:2018/01/24(水) 03:52:39.89 ID:9ooAixL8.net
>>416

コーシーの不等式より
n{Σ[k=1,n]s_k (a_k)^2}≧(Σ[k=1,n]a_k √s_k)^2,

0≦x<y に対して、
(y-x)√y > ∫[x,y]√t dt =(2/3){y^(3/2)- x^(3/2)},
ゆえに、k = 1,2,…,n に対して、
 a_k √s_k >(2/3){(s_k)^(3/2)-(s_{k-1})^(3/2)}
(ただし、s_0 = 0 とする。)
k = 1,2,…,n について足し合わせると、
 Σ[k=1,n]a_k √s_k >(2/3)(s_n)^(3/2),

以上により
n{Σ[k=1,n]s_k(a_k)^2}>(4/9)(s_n)^3,

418 :132人目の素数さん:2018/01/24(水) 15:22:02.70 ID:9ooAixL8.net
>>417

0≦x,y に対して、 AM-GM で

y^(3/2)+ x^(3/2)+ x^(3/2)≧ 3x√y,

∴ (y-x)√y ≧(2/3){y^(3/2)- x^(3/2)},

419 :132人目の素数さん:2018/02/07(水) 05:17:19.34 ID:hJs29j8F.net
[3']
三変数の相加相乗平均の不等式
「a,b,c > 0 に対し、a^3+b^3+c^3 ≧ 3abc」
を証明せよ。
ただし、不等式評価には次の不等式のみを用いること。
「任意の実数 p,q,r,x,y,z に対して (pp+qq+rr)(xx+yy+zz)≧(px+qy+rz)^2」

[4]
xは実数とする。
2+√2 ≦ √{1+sin(x)}+√{1+cos(x)}+√{1-sin(x)}+√{1-cos(x)}≦ √{2(2+√2)}+√{2(2-√2)}
を示せ。
 最小は x=nπ/2,最大は x=nπ/2 + π/4 のとき。

http://twitter.com/perfect08641086/

420 :132人目の素数さん:2018/02/07(水) 20:51:01.52 ID:WpqZ2RSO.net
〔類題〕
 -1≦y≦1 ⇒ √(1+y)+√(1-y)≦2
 等号成立は y=0.

421 :132人目の素数さん:2018/02/10(土) 03:52:34.10 ID:JLwIa9Z8.net
〔問題〕
a,b,c >0 のとき、
(1) a/(b+c)+ b/(c+a)+ c/(a+b)≧ 3/2,  (Nesbitt,Shapiro-3)
(2) a/(a+b)+ b/(b+c)+ c/(c+a)≦{(a+b+c)^2 + 3(ab+bc+ca)}/{4(ab+bc+ca)},
(3) a/(a+b)+ b/(b+c)+ c/(c+a)≦(ab+bc+ca)^2/{2abc(a+b+c)},

422 :132人目の素数さん:2018/02/10(土) 04:46:19.92 ID:p8fUG64o.net
よい不等式デスネ! 実に勤勉デス!

423 :132人目の素数さん:2018/02/10(土) 05:28:06.69 ID:p8fUG64o.net
【オイラーのφ関数】
(1) 奇素数 p、自然数 n に対して、φ(p^n) > √(p^n) を示せ。
(2) 自然数 n (≠2、6) に対して、φ(n) > √(n) を示せ。

あぁぁ、脳が…震え…

424 :132人目の素数さん:2018/02/10(土) 15:05:18.55 ID:JLwIa9Z8.net
>>421

(4) a/(a+b)+ b/(b+c)+ c/(c+a)≦ 3{(a+b+c)^2 + (ab+bc+ca)}/{8(ab+bc+ca)} ≦ (a+b+c)^2 / {2(ab+bc+ca)},

(略証)
s = a+b+c,
t = ab+bc+ca,
u = abc,
とおくと
(a+b)(b+c)(c+a) = st-u ≧ 8st/9,
3(st+aab+bbc+cca) = s(ss+t) -a(a-b)^2 -b(b-c)^2 -c(c-a)^2 ≦ s(ss+t),
したがって
(左辺) = (st+aab+bbc+cca)/(st-u)
≦ (1/3)s(ss+t)/(8st/9)
= 3(ss+t)/(8t).

425 :132人目の素数さん:2018/02/10(土) 15:19:43.46 ID:JLwIa9Z8.net
>>423
(1)
・n=1 のとき
 p≧3 ゆえ、(p-1)^2 - (p+1) = p(p-3) ≧ 0,
φ(p) = p-1 ≧ √(p+1) > √p
・n≧2 のとき
 n-1 ≧ n/2,
 φ(p^n) = (p-1)・p^(n-1) ≧ (p-1)・p^(n/2)

426 :132人目の素数さん:2018/02/11(日) 04:50:21.13 ID:oyQM1khy.net
>>419
[3']
 (aa+bb+cc)^2 =(bb+cc+aa)(cc+aa+bb)≧(bc+ca+ab)^2,
 aa+bb+cc -ab -bc -ca ≧ 0,
よって
 a^3 + b^3 + c^3 - 3abc =(a+b+c)(aa+bb+cc-ab-bc-ca)≧ 0,

427 :132人目の素数さん:2018/02/11(日) 06:27:54.17 ID:oyQM1khy.net
>>423
(1)
 p≧3 のとき、(p-1)^2 -(p+1)= p(p-3)≧ 0,
 p-1 ≧ √(p+1)> √p,
 φ(p^e)=(p-1)p^(e-1)> p^(e-1/2)≧ √(p^e)   (e≧1)

(2)
・nが奇数のとき
 nの素因数は奇素数のみ。
 ∴nの素因数pごとに分ければ成立。

・nが4の倍数のとき
 e≧2 に対して
 φ(2^e)= 2^(e-1)≧ 2^(e/2), …成立。(e=2 のときは等号)
 ∴nの素因数pごとに分ければ成立。

・n=2・(奇数),n>6 のとき
  nは 4より大きい素因数(p>4)または平方因子(e≧2)をもつ。
  p>4 ならば(p-1)^2 -(2p+1)= p(p-4)> 0,
   p-1 > √(2p+1)> √(2p)
   φ(2・p^e)=φ(p^e)=(p-1)p^(e-1)>(√2)p^(e-1/2)≧ √(2・p^e),
  e≧2 ならば
   φ(2・p^e)=φ(p^e)=(p-1)p^(e-1)≧ 2p^(e-1)> √(2・p^e)

428 :132人目の素数さん:2018/02/11(日) 11:12:22.15 ID:T9JK+LpN.net
a、b、c >0 に対して、
9 > (4a+b)/(a+4b) + (4b+c)/(b+4c) + (4c+a)/(c+4a) > 3/2

これが中学生向けの問題だと! あぁぁ、脳が…震え…

429 :132人目の素数さん:2018/02/11(日) 18:02:59.31 ID:oyQM1khy.net
>>428
 p≧q ⇒ 4 >(4p+q)/(p+4q)≧1,
 p≦q ⇒ 1 ≧(4p+q)/(p+4q)> 1/4,
だから
 a≧b≧c ⇒ 4 + 4 + 1 >(中辺)> 1 + 1 + 1/4,
 a≦b≦c ⇒ 4 + 1 + 1 >(中辺)> 1 + 1/4 + 1/4,

実際には 8.25 > (中辺)≧ 3.0 らしい…

430 :132人目の素数さん:2018/02/12(月) 01:28:54.22 ID:MYy378Zb.net
>>429

k≧2,
p = aab+bbc+cca -3abc ≧ 0,
q = abb+bcc+caa -3abc ≧ 0,
とおくと、
(a+kb)(b+kc)(c+ka)= kp + kkq +(1+k)^3・abc,

(ka+b)/(a+kb)+(kb+c)/(b+kc)+(kc+a)/(c+ka)
= 3 +(k-1){(2k-1)p+k(k-2)q}/{kp+kkq+(1+k)^3・abc}
≧ 3,

上限は 2k + 1/k,

431 :132人目の素数さん:2018/02/13(火) 16:55:38.41 ID:L2ZMaNlF.net
>>428-430
JJMO(2008)でござった。

>>430
上限は 2k + 1/k。 これをどうやって示すのか分かりませぬ…。

432 :132人目の素数さん:2018/02/13(火) 19:30:46.26 ID:ZOqrSE8B.net
〔大関の不等式〕
 単調減少な正数列 x_1 > x_2 > …> x_j > … > x_n > 0 について
 Π[j=1,n](x_j)^x_{j-1}> Π[j=1,n](x_j)^x_{j+1},
 Σ[j=1,n] x_{j-1}log(x_j)> Σ[j=1,n] x_{j+1}log(x_j),
 ただし、x_{n+1}= x_1,x_0 = x_n とする。

数セミ、2018年3月 NOTE

433 :132人目の素数さん:2018/02/13(火) 19:49:22.56 ID:ZOqrSE8B.net
>>431

(ka+b)/(a+kb)+(kb+c)/(b+kc)+(kc+a)/(c+ka)
= 2k + 1/k -{[2(k-1)^2 +1]kq + (1/k)(2k-1)(k-1)(k+1)^3・abc}/{kp+kkq+(1+k)^3・abc}
≦ 2k + 1/k,

434 :132人目の素数さん:2018/02/13(火) 23:57:57.21 ID:L2ZMaNlF.net
>>433
ありがたや〜

>>432
『大関の不等式』の大関って、不等式の本書いてる大関親子のどっちか? それともNOTEに投稿した読者の名前かな?
地方では、雑誌の発売日は翌日以降になるから、まだ読めていないのでござる。

435 :132人目の素数さん:2018/02/14(水) 00:53:43.35 ID:ZgrnGGF4.net
>>433
> (ka+b)/(a+kb)+(kb+c)/(b+kc)+(kc+a)/(c+ka)
> = 2k + 1/k -{[2(k-1)^2 +1]kq + (1/k)(2k-1)(k-1)(k+1)^3・abc}/{kp+kkq+(1+k)^3・abc}

qの係数は、2(k-1)^2 +1]kq ぢゃなくて、k(k^2-1)q ではござらぬか?

436 :132人目の素数さん:2018/02/14(水) 05:49:33.31 ID:ZgrnGGF4.net
n^5 = 27^5 + 84^5 + 110^5 + 133^5 をみたす自然数 n を求めるときに、大雑把に n のとりうる値の範囲を絞りたい。

n^5 ≡ 24 (mod 30) から、n = 24+30k で、答えは144なんだが、nの範囲を上手に知りたいのでござる。

下限は、n^5 = 27^5 + 84^5 + 110^5 + 133^5 > 135^n だから、n>135
上限が 174 より小さいってのを一発でエレガントに出すような不等式ってないでござる蟹?

437 :132人目の素数さん:2018/02/14(水) 06:19:10.42 ID:ZgrnGGF4.net
4^(1/5) = 1.319…

n^5 = 27^5 + 84^5 + 110^5 + 133^5 < 4*133^5 < (7/5)^5*133^5 = 186.2^5
n^5 = 27^5 + 84^5 + 110^5 + 133^5 < 4*133^5 < (33/25)^5*133^5 = 177.56^5

この方法はイマイチですな。

438 :132人目の素数さん:2018/02/14(水) 06:57:21.49 ID:+eR545sM.net
金利の計算等でよく知られている72の法則から
2^(1/5)≒1.144≒8/7

27^5 + 84^5 + 110^5 + 133^5 < 111^5 + 110^5 + 133^5
≒(111*8/7)^5+133^5≒127^5+133^5 <2*133^5≒(133*8/7)^5=152^5 とか

439 :132人目の素数さん:2018/02/14(水) 15:42:22.47 ID:/bHsoXtp.net
>>434
 子の清太氏でござる。(今や宇都宮大学も退官されて古希でござるな)

〔補題〕
0 < y < x_1 で F(y)= log(y/x_1)/(x_1 - y)は単調増加
 F '(y)={x_1/y -1 -log(x_1/y)}/(x_1 - y)^2 > 0 から出る。

(左辺)−(右辺)= Σ[j=1,n]{x_j・log(x_{j+1})- x_{j+1}・log(x_j)}
= Σ[j=1,n]{(x_j-x_1)・log(x_{j+1}/x_1)-(x_{j+1}-x_1)・log(x_j/x_1)}
= Σ[j=2,n-1]{(x_j-x_1)・log(x_{j+1}/x_1)-(x_{j+1}-x_1)・log(x_j/x_1)}
= Σ[j=2,n-1](x_1-x_j)(x_1-x_{j+1}){F(x_j)-F(x_{j+1})}
> 0    (← x_1 > x_j > x_{j+1})

>>435
 仰るとおり。死んでお詫びを…(AA略

440 :132人目の素数さん:2018/02/14(水) 16:20:35.13 ID:/bHsoXtp.net
>>436

等比級数と比較する(ダランベールの判定法?)
r = 110/133 とおく。r < 5/6
110 = 133 r,
84 < 90.9774 = 133 r^2,
27 < 75.2445 = 133 r^3,

n^5 < 133^5 (1 + r^5 + r^10 + r^15)
< 133^5/(1-r^5)
= 133^5 /{1-(5/6)^5}
= 133^5 /0.59812
= (133 * 1.10826)^5,

n < 133 * 1.10826 < 147.4

n = 144 に近い(?)

441 :132人目の素数さん:2018/02/14(水) 22:52:06.37 ID:/bHsoXtp.net
>>436
r = 110/133 とおく。

r^5 < 2/5,
1/(1-r^5)< 5/3,
n < 133・(5/3)^(1/5)= 133・1.10757 = 147.306

r^5 < 12/31,
1/(1-r^5)< 31/19,
n < 133・(31/19)^(1/5)= 133・1.10286 = 146.68

n=144 に近いかも(?)

442 :132人目の素数さん:2018/02/14(水) 23:03:45.21 ID:ZgrnGGF4.net
>>438-441
ありがとうございます。いろいろありますね。

> 金利の計算等でよく知られている72の法則から

全く知らなかったでござる。

443 :132人目の素数さん:2018/02/15(木) 19:36:01.68 ID:BNcyv0HF.net
>>439
清太氏に不等式の本を書いてもらいたい。
まだ書いていないこと沢山あるだろう…。

444 :132人目の素数さん:2018/02/16(金) 02:03:16.59 ID:A0Z5LkE8.net
>>430
p, q を使うという発想が凄いな。鬼がかっている…

445 :132人目の素数さん:2018/02/16(金) 02:15:48.88 ID:LmIoBxc5.net
>>443

もし出たら、餃子を食いながら解こうかな("^ω^)・・・ (宇都宮)

446 :132人目の素数さん:2018/02/16(金) 02:37:16.13 ID:A0Z5LkE8.net
清太氏の最新刊は、数学のかんどころシリーズの不等式だったけど、あのシリーズは、
『OnePoint双書の精神を継承し、ページを押さえ、テーマを絞り、手軽に読めるように』なので、あんまり載ってないんよな。

やはりここは、大関(親)の絶版書の内容も含む3つの著書を含めて、
まだ書いてないこともたくさん入れて、ページ数制限なしで分厚いのを出してほしい。

喩えるなら、「素数全(朝倉書店、2010)」とか「数学の女王(共立出版、2013)」みたいな感じで。

447 :132人目の素数さん:2018/02/16(金) 10:22:33.88 ID:zQDvOiPX.net
ここって案外人いるのね

448 :132人目の素数さん:2018/02/18(日) 19:05:09.70 ID:eirA2Wob.net
n∈N、r∈R、r≧1 に対して、
{(n+1)^(r+1)*n^r}/{(n+1)^(r+1) - n^(r+1)} ≧ Σ[k=1 to n] k^r ≧{(n+1)^r*n^(r+1)}/{(n+1)^(r+1) - n^(r+1)}

油断、怠慢、即ち怠惰!

449 :132人目の素数さん:2018/02/24(土) 01:29:07.74 ID:j7T05hBb.net
(1) a>b>c> 0 のとき、ax^2+bx+c=0 の解αは、|α|<1 をみたすことを示せ。
(2) x^3 + 3x^2 + 5x + 7 = 0 の解αは、1<|α|≦3 をみたすことを示せ。

あぁぁ、脳が…震える…
                     -‐. . : ヘ三≧-_
                  <: : /: : : : : : : ̄<三≧
               /: : : : : /: : : : : : :/: : : : : : <≧
              /: : : : : : / : : : : : :./: : : : : : : : : : : :<=
             /l:冓: : : :/:/: : : : : /: : : : : : : : | : : i: : : \
           /´/ ´ 刃: :i :i :{ : : : : :/ : : : : : : : : :| : : | : : : : :.i
           / /イ´./: :l  ‐ 、 :i: :i : : : :i.: : : :.:. | : : |: : : : : :ヘ
          レ / .// ii|  ● i:::::i : : : .::: : : : : :i : : i: : :i : : : :i
         i´   ´ / .   ヾ、. _ ノ::::..::. : .::::: : : : : :i : : i: : :i : : : :|
三=- _____.|     |.\.    ̄   /.,::::::  ‐ 、: :/: : /: : :i.: : : : |
三=---- 三三/    .}  冫  ─i´ ,,,. ´  |  ● i::: : :, :_., : |: : : :.|
´   /  i /i.    ./ / ヘ   ヾ丶 i  ヾ、. _ ノ /4/‖:::| : : : |
   冫"~  ̄     / / i .  丶   ̄i    ̄  ./ i i .|}::/: : : :/
  /        イ ´ / `メ ノ \   \     |/ | |.|.: : : :./
 |    ==   /| i  ` <丿メノ ` う   ア ´  ///: : / \     ___
  ┐  __  −=三〈 `ー-\ \ ノメ//''"      .//  ̄ _ - = 三三三三
  i-'".|三三三三=丶- 、  \ `、ノ//     ,.-'''"  -=三三三三三三=
  /  /三三三三三=┐ 丶  - ´, /     / -=三三三三三三三=
. 〈  〈 三三三三三三 .| | └‐- ´i/     /=三三三三三三三三 〈

450 :132人目の素数さん:2018/02/24(土) 04:17:06.67 ID:xZc/HRib.net
>>449
(1)
・解が実数のとき(bb-4ac≧0)
 |x|≧1 ⇒ axx+bx+c ≧ a|x| -b|x| +c =(a-b)|x|+ c > c > 0
 ∴|α|< 1
・解が共軛複素数のとき(bb-4ac<0)
 |α| = √(αα~)= √(c/a)< 1,

(2)
x≧-2 ⇒ x^3 +3x^2 +5x +7 =(x+2)(xx+x+3)+ 1 ≧ 1,
x≦-3 ⇒ x^3 +3x^2 +5x +7 =(x+3)(xx+5)-8 ≦ -8
中間値の定理より(-3,-2)に実解rがある。

x^3 +3x^2 +5x +7 =(x-r){xx +(3+r)x - 7/r},
xx +(3+r)x - 7/r = 0 は複素数解αをもつ。
解と係数の関係から rαα~ = -7,
|α|= √(αα~) =√(-7/r),
∴ √(7/3)< |α| < 2,

・蛇足
r = -1 +(1/3){6(-9+√87)}^(1/3)-(1/3){6(9+√87)}^(1/3)
 = -2.1795090246
|α|= 1.79213072

451 :132人目の素数さん:2018/02/24(土) 06:30:05.58 ID:j7T05hBb.net
449-450
http://repository.osakafu-u.ac.jp/dspace/bitstream/10466/1911/1/KJ00000331372.pdf

ほらほら、脳が震えてきただろう? ん〜?

452 :132人目の素数さん:2018/02/24(土) 18:59:44.58 ID:xZc/HRib.net
>>436

s = 5/4 とおく。
133 > 131.25 = 84 ss,
110 > 105 = 84 s,
84 = 84,
n^5 > 84^5 *(s^10 + s^5 +1)
= 84^5 *{(5/4)^10 +(5/4)^5 + 1}
> 84^5 *{9 + 3 + 1}
= 84^5 * 13
>(84 * 5/3)^5
= 140^5,

∴ 141 ≦ n ≦ 147   (>>440-441)

453 :132人目の素数さん:2018/02/25(日) 02:17:24.53 ID:gGaVEUAO.net
>>449
(1)は新潟大学ですね。

>>451
〔掛谷の定理〕
F(x)= a_0 x^n + a_1 x^{n-1} + …… + a_{n-1}x + a_0,
 a_0 > a_1 > a_2 > …… > a_{n-1}> a_n > 0
ならば、F(x)= 0 の解の絶対値は1より小さい。

(略証)
F(0)= a_n >0,F(1)> 0,
(1 - 1/x)F(x)/ x^n = a_0 - Σ[j=1,n](a_{j-1} - a_j)/x^j - a_n / x^(n+1),
x=1 のときは
 0 = a_0 - Σ[j=1,n](a_{j-1} - a_j)- a_n,
辺々引いて
(1 - 1/x)F(x) / x^n = Σ[j=1,n+1](a_{j-1} - a_j)(1 - 1/x^j) + a_n(1 - 1/x^{n+1}),
ここで、|x|≧1, x≠1 ならば
Re{1/x^j}≦|1/x|^j ≦ 1,
であるが、等号成立は x=1 に限るので
Re{1 - 1/x^j}> 0,
∴ Re{右辺}> 0,
∴ F(x)≠ 0,
∴|α|<1
http://www004.upp.so-net.ne.jp/s_honma/solution/solution.htm

「經濟研究」の別証明は、あまりにも迂回的で逆行的でござるな。
市大とちゃんと統合成立するかなぁ?

454 :132人目の素数さん:2018/02/25(日) 02:38:57.60 ID:gGaVEUAO.net
>>453

などと嘯いてたら、間違えてしまった......orz

F(x)= a_0 x^n + a_1 x^{n-1} + …… + a_{n-1}x + a_n,

455 :132人目の素数さん:2018/02/25(日) 10:26:23.15 ID:utpborzn.net
>>453-454
あぁぁ…、あなたはなんと勤勉なる事か!
偶々ネットで見かけた掛谷のpdfを見て出題したのでござるが、入試問題まで探してくるとはとはとはとは…!

456 :132人目の素数さん:2018/02/25(日) 10:32:57.68 ID:utpborzn.net
>>451>>453
その『掛谷の定理』関連について詳しく書かれている本はないかなぁ?

457 :132人目の素数さん:2018/02/25(日) 12:37:13.03 ID:utpborzn.net
掛谷宗一
http://wp1.fuchu.jp/~sei-dou/jinmeiroku/kakeya-souichi/kakeya-souichi.htm

458 :132人目の素数さん:2018/02/25(日) 12:46:00.21 ID:utpborzn.net
根の大きさの限界
http://nalab.mind.meiji.ac.jp/~mk/lecture/ouyoukaiseki4/algebraic-equation/kadai-2003-1/node5.html

459 :132人目の素数さん:2018/02/25(日) 16:08:11.57 ID:gGaVEUAO.net
逆数バージョン

〔掛谷の定理〕 
正係数のn次多項式
 F(x)= a_0 x^n + a_1 x^{n-1} + …… + a_{n-1}x + a_n,
 0 < a_0 < a_1 < a_2 < …… < a_{n-1}< a_n
について、F(x)= 0 の解の絶対値は1より大きい。

(略証)
F(0)= a_n >0,F(1)> 0,
(x-1)F(x)= a_0 x^(n+1)+ Σ[j=1,n](a_{n-j+1}- a_{n-j}) x^j - a_n,
x=1 のときは
 0 = a_0 + Σ[j=1,n](a_{n-j+1}- a_{n-j})- a_n,
辺々引いて
(x-1)F(x)= a_0(x^{n+1} -1) + Σ[j=1,n](a_{n-j+1}- a_{n-j})(x^j -1),
ここで、|x|≦ 1, x≠1 ならば
Re{x^j}≦|x|^j ≦ 1,
であるが、等号成立は x=1 に限るので
Re{x^j -1}< 0,
∴ Re{右辺}< 0,
∴ F(x)≠ 0,
∴|α|> 1

(系) x → 1/x とすれば >>453
http://www004.upp.so-net.ne.jp/s_honma/solution/solution.htm

460 :132人目の素数さん:2018/02/26(月) 05:41:30.17 ID:+hLINPlt.net
>>453 >>459

(1)
n次方程式 F(x)= 0 の根{r_k}がすべて実数のとき、
 F(x)が極値・停留値をとる点b{F '(x)= 0 の実数解}は次をみたす。
 r_min ≦ b ≦ r_max

(2)
n次多項式 F(x)が停留値をとる点β{F '(x)= 0 の解}は、
 F(x)= 0 のすべての根を含む凸領域内にある。

例) すべて単根{α_k}のときは
 β = Σ[k=1,n]t_k α_k
 重み t_k = |β-α_k|^(-2)/{Σ[k=1,n]|β-α_k|^(-2)}

461 :132人目の素数さん:2018/02/27(火) 03:08:49.84 ID:M/Cc1/YM.net
>>460
 重み t_k = |β-α_k|^(-2)/{Σ[j=1,n]|β-α_j|^(-2)}

462 :132人目の素数さん:2018/02/28(水) 05:08:58.38 ID:W7HTMDJw.net
>>448

r=1 のときは等号になるので r>1 とする。中辺を

S(n)= Σ[k=1,n] k^r

とおく。問題の式は

1/n > S(n)/n^(r+1)- S(n)/(n+1)^(r+1)> 1/(n+1),

S(n)/S(n-1)>{(n+1)/n}^(r+1)> S(n+1)/S(n),

S(n)/(n+1)^(r+1)> S(n-1)/n^(r+1) …… 増加列
S(n)/n^(r+1)> S(n+1)/(n+1)^(r+1) …… 減少列

{1/(n+1)}Σ[k=1,n]{k/(n+1)}^r >(1/n)Σ[k=1,n-1](k/n)^r
(1/n)Σ[k=0,n](k/n)^r > {1/(n+1)}Σ[k=0,n+1]{k/(n+1)}^r

となる。

463 :132人目の素数さん:2018/02/28(水) 05:42:35.66 ID:W7HTMDJw.net
>>448 (続き)

f(x)= x^r (r≧1)は下に凸だから、下の補題より

S(n)/{n(n+1)^r}≧ S(n-1)/{(n-1)n^r},

S(n)/{(n+1)n^r}≧ S(n+1)/{(n+2)(n+1)^r},

これと n/(n-1)>(n+1)/n >(n+2)/(n+1) から >>462 が出る。

なお、n >> r では S(n)〜{1/(r+1)}(n + 1/2)^(r+1)


〔補題〕
f(x)が 0<x<1 で下に凸ならば

1)(1/n)Σ[k=1,n]f(k/(n+1))≧{1/(n-1)}Σ[k=1,n-1]f(k/n),

2){1/(n+1)}Σ[k=0,n]f(k/n)≧{1/(n+2)}Σ[k=0,n+1]f(k/(n+1)),

(略証)
1)
凸性からJensenにより
{(n-k)/n}f(k/(n+1))+(k/n)f((k+1)/(n+1))≧ f(k/n),
k=1 から k=n-1 まで加えて(n-1)で割る。
2)
凸性からJensenにより
{k/(n+1)}f((k-1)/n)+{(n+1-k)/(n+1)}f(k/n)≧ f(k/(n+1)),
k=0 から k=n+1 まで加えて(n+2)で割る。

464 :132人目の素数さん:2018/03/01(木) 01:39:41.63 ID:utcJHdiE.net
>>462

> r=1 のときは等号になる

は間違いでした。

>>463

2) f(x)が 0≦x≦1 で下に凸ならば…

http://suseum.jp/gq/question/2868

465 :132人目の素数さん:2018/03/01(木) 03:59:42.98 ID:utcJHdiE.net
>>253 , >>259
 掛谷の定理

http://www.casphy.com/bbs/highmath/472060/264

http://suseum.jp/gq/question/2869

466 :132人目の素数さん:2018/03/02(金) 05:19:49.90 ID:IyAEI4K5.net
〔Popoviciuの不等式〕
f(x) が下に凸ならば、 (a+b+c)/3 = m に対して
f(a) + f(b) + f(c) + 3f(m) ≧ 2f((a+b)/2) + 2f(a+c)/2) + 2f((b+c)/2),

(略証)
a≦b≦c としてよい。
(i) a,b ≦ m ≦ c のとき
f(a) + f(b) ≧ 2f((a+b)/2),
f(m) + f(c) ≧ 2f((m+c)/2),
2f(m) + 2f((m+c)/2) ≧ 2f((a+c)/2) + 2f((b+c)/2),
辺々たす。

(ii) a ≦ m ≦ b,c のとき
f(a) + f(m) ≧ 2f((a+m)/2)
2f((a+m)/2) + 2f(m) ≧ 2f((a+b)/2) + 2f((a+c)/2),
f(b) + f(c) ≧ 2f((b+c)/2),
辺々たす。

文献[9]佐藤淳郎(訳)p.41 演習問題1.89

467 :132人目の素数さん:2018/03/02(金) 05:34:31.97 ID:IyAEI4K5.net
>>449
 ムンク「叫び」(1895〜1910)

http://giphy.com/gifs/scream-12TN725elPIWkg

468 :132人目の素数さん:2018/03/02(金) 13:52:18.45 ID:Ub9nfASN.net
>>466
ぬるぽビッチか、何もかもが懐かしい…

469 :132人目の素数さん:2018/03/03(土) 02:38:06.57 ID:MMc1xkls.net
a、b、c ∈ (1、∞) または、a、b、c ∈ (0、1) のとき、
log_a(bc) + log_b(ca) + log_c(ab) ≧ 4{ log_(ab)c + log_(bc)a + log_(ca)b }

(参考)
過去スレに、a、b、c ∈ (1、∞) のとき、
 左辺 > 定数
 定数 > 右辺 > 定数
というのを収集して貼ったような希ガス、ハロゲンガス…。


詳細は…不明ですか? あなた、怠惰…ですね
           ____________
        >/////////::::::丶
      //////////:::::::::::::::::::::\
      ////////,/:::::::::\:::::::::::::::::::丶
     .//////, イ::::::::::::::::::::::::::ヽ::::::::::::::::::\
    ////::::::::::::\:::::\::::::::::::::::::::::::::::::::::::::`i
    //:::::::::::::::::::::::::\:::::\:::::::::::::::::::::::::::::::::::i      , __
   /::::i:::::::::ヾi:::::::::::::::::::::::::::::\:::::::::::: rヘ:::::::::ト    /  !
   |:::::|::i::::::::::リ:::::::::/:::::,:::f´⌒ヽ .V:::::',::ヾi:::i::::! /      !
   i:::::|:::|:::::::/:i:i::`:::,:イ≡i ゚゙● i .V::::::'::::y:ィリ ̄       !
   .|::::::::|::::::::::::j,:イ≡彡 `==彳 ,, V::::::':::/ ヽ///,i      /
    |::::::::ト::::イi´●゙    ー ´ 7 Y:::::::∧ 人///     /----y
    i::::::::::::::::::弋゙_丿',      / ;} i:::::i  ゞ  /     /ヽ  /
    i:::::::::::\:::∧   '- ´  /ヽイ;;;} !::/i  ',./i     /ソ  ノ
    \::::ヽ:::::::└-i- ム-<丶tt i };} レ丶  ./ /    /::/ /\
      \::::::::/   `\\,エィ ´ ;;  / ´/ ./    ‖/     .\
       ヽ::/     !/,`丶;   ; ;イ ノ /! /     ! j       ヾ
        `ト     ヽ///!ー イ ´//// ソ               i
          \   \//////////./                i
            \   .\///////                  |

470 :132人目の素数さん:2018/03/03(土) 07:13:06.83 ID:hUH2ieOC.net
>>469

A = ln(a),B = ln(b),C = ln(c) とおく。
題意により、A,B,Cは同符号。
正であるとしても一般性を失わない。

S=A+B+C,T=AB+BC+CA,U=ABC とおく。
(左辺) = (B+C)/A + (C+A)/B + (A+B)/C = (ST-3U)/U,
(右辺) = 4{C/(A+B) + A/(B+C) + B/(C+A)} = 4{S(SS-2T) + 3U}/(ST-U),
(左辺) - (右辺) = {(ST-9U)T + 3SU・F_{-1}) + SSU・F_{-2} }/{T(ST-U)} ≧ 0,
ここに
U・F_{-1} = TT -3SU ≧ 0,
UU・F_{-2} = T^3 -4STU +9UU ≧ 0,

なお、
(右辺) ≧ 6 (Nesbitt、Shapiro-3)

471 :132人目の素数さん:2018/03/03(土) 07:18:06.63 ID:hUH2ieOC.net
>>466

f(m) + f((m+c)/2) ≧ f((a+c)/2) + f((b+c)/2),
f(m) + f((a+m)/2) ≧ f((a+b)/2) + f((a+c)/2),
のところが分からん?

〔補題〕
f(x) は m,nを含む区間で下に凸
m+d,n-d が mとnの中間にあるとき
 f(m) + f(n) > f(m+d) + f(n-d)

(略証)
m≠n、0<λ<1 に対して
(1-λ)f(m) + λf(n) > f((1-λ)m + λn) = f(m+d)
 λf(m) +(1-λ)f(n) > f(λm + (1-λ)n) = f(n-d)
辺々たす。
ここに、d = λ(n-m) とおいた。

472 :132人目の素数さん:2018/03/03(土) 19:46:02.49 ID:hUH2ieOC.net
>>469

〔補題〕
A,B,C が同符号のとき
(B+C)/A + (C+A)/B + (A+B)/C ≧ 4{A/(B+C) + B/(C+A) + C/(A+B)},

(略証)
AM-HM より
 A(1/B + 1/C)≧ 4A/(B+C),
 B(1/C + 1/A) ≧ 4B/(C+A),
 C(1/A + 1/B) ≧ 4C/(A+B),
辺々たす。

473 :132人目の素数さん:2018/03/04(日) 03:52:33.46 ID:IhTCj0CK.net
>>470

〔Nesbitt、Shapiro-3〕

A/(B+C) + B/(C+A) + C/(A+B) ≧ 3/2,

(左辺) = (A+B+C) {1/(B+C) + 1/(C+A) + 1/(A+B)} - 3
≧ (A+B+C) * 9/ {2(A+B+C)} - 3  (← AM-HM)
= 9/2 - 3
= 3/2.

(左辺) = (A+B+C) {1/(B+C) + 1/(C+A) + 1/(A+B)} - 3
= (1/2) {(B+C)+(C+A)+(A+B)} {1/(B+C) + 1/(C+A) + 1/(A+B)} - 3
≧ (1/2)(1+1+1)^2 - 3  (← コーシー)
= 9/2 - 3
= 3/2.

474 :132人目の素数さん:2018/03/13(火) 11:09:49.42 ID:IdxYrbr8.net
C[n,r]は二項係数とする。

(1) n ∈N (n≧2) に対して、2^{2n-1}/\sqrt(n) < C[2n, n] < 2^{2n-1} を示せ。

(2) n+1 以上 2n-1 以下の素数の積は、2^{2n-2} より小さいことを示せ。
  ただし、該当する素数がないときは、積を1とする。

(3) n 以下の素数の積は、2^{2n-1} 以下であることを示せ。

475 :132人目の素数さん:2018/03/14(水) 01:57:39.77 ID:hdbbxtzk.net
>>453 >>459

正係数のn次方程式
Σ[j=0,k] a_j {x^(n-j) + x^j} = 0, (n>2k)
について、

a_0 > a_1 > a_2 > … > a_{n-1} > a_n > 0
ならば、解の絶対値はすべて1である。

http://repository.kulib.kyoto-u.ac.jp/dspace/bitstream/2433/141059/1/1665-03.pdf
http://suseum.jp/gq/question/2869 (アンドロメダ氏)
http://www.casphy.com/bbs/highmath/472060/269 (不等式2)

476 :132人目の素数さん:2018/03/14(水) 03:30:09.01 ID:hdbbxtzk.net
>>474
(1)
2^(2n-1) /√n < C[2n,n] < 2^(2n-1) √{2/(n+1)},

nについて帰納法による。
n=2 のとき、8/√2 < C[4,2] < 8√(2/3) ゆえ成立。
n-1 について成り立つならば
 2^(2n-3)/√(n-1) < C[2n-2,n-1] < 2^(2n-3)√(2/n),
 4√{(n-1)/n} < 4 (2n-1)/2n < 4√{n/(n+1)},
 辺々かけて
 2^(2n-1)/√n < C[2n,n] = 2^(2n-1)√{2/(n+1)},
∴ n についても成立。

477 :132人目の素数さん:2018/03/14(水) 03:46:47.54 ID:KXfS4lra.net
>>475
いつもながら実に実に実に〜ぃ、素晴らしいデス!
参考資料まで探して頂き、感謝の極みでござるぞ!

478 :132人目の素数さん:2018/03/15(木) 02:36:40.53 ID:HCGObM3W.net
>>463

〔補題〕
http://www.casphy.com/bbs/highmath/472060/265 (不等式2)

(2)(x=0,x=1 も含む方)
大関: 文献[3] p.130 例題6.

479 :132人目の素数さん:2018/03/15(木) 16:54:44.46 ID:HCGObM3W.net
>>476

Stirlingの近似
 n!≒ √(2π)n^(n+1/2)e^{-n + 1/(12n)},
から
 C[2n,n]=(2n)!/(n!・n!)≒(4^n)/√(πn)・e^{-1/(8n)},

480 :132人目の素数さん:2018/03/20(火) 07:24:26.19 ID:HDkQdBLp.net
〔問題〕
80.60 < Σ[k=1,24]√k < 80.65 を示せ。

面白スレ26-103

481 :132人目の素数さん:2018/03/20(火) 23:56:16.21 ID:nOd+tfdt.net
数学セミナーに不等式の問題あった

482 :132人目の素数さん:2018/03/21(水) 00:14:13.24 ID:NaAK8rgB.net
信ちゃんのエレ解かな?

483 :132人目の素数さん:2018/03/21(水) 04:54:09.91 ID:Y0EoMfqc.net
>>480

左側:
 y=√x は上に凸だから、接線が上。
 √k > ∫[k-1/2,k+1/2]√x dx =(2/3){(k+1/2)^(3/2)-(k-1/2)^(3/2)},
右側:
 y=√x は上に凸だから、割線が下。
 {√k + √(k+1)}/2 < ∫[k,k+1] √x dx =(2/3){(k+1)^(3/2)- k^(3/2)},
 ただし、1≦k≦4 は別途たす。

484 :132人目の素数さん:2018/03/21(水) 04:59:29.95 ID:Y0EoMfqc.net
〔応用問題〕
 {√k + √(k+1)}/2 < ∫[k,k+1] √x dx =(2/3){(k+1)^(3/2)- k^(3/2)},
を用いて次を示せ。

(2) √2 < 99/70 = 1.41428571…    (k=8)
   √2 > 1393/985 = 1.41421320… (k=49)
   √2 < (19601/6)/2310 = 1.4142135642… (k=288)

(3) √3 < (1351/6)/130 = 1.73205128… (k=48)

(5) √5 < 2889/1292 = 2.236068111…  (k=80)

(6) √6 < (485/6)/33 = 2.4494949…  (k=24)

(7) √7 < 2024/765 = 2.645751634…  (k=63)

(10) √10 > 117/37 = 3.16216216…   (k=9)
   √10 < (27379/6)/1443 = 3.1622776622… (k=360)

(11) √11 < 3970/1197 = 3.316624895… (k=99)

(17) √17 > 268/65 = 4.123076923…   (k=16)

(37) √37 > 882/145 = 6.08275862…  (k=36)

面白スレ26 - 109〜110,117

485 :132人目の素数さん:2018/03/21(水) 05:17:17.36 ID:Y0EoMfqc.net
>>484 に追加

(15) √15 < 244/63 = 3.87301587… (k=15)

(35) √35 < 846/143 = 5.9160839…  (k=35)

(101) √101 > 4030/401 = 10.049875311… (k=100)

486 :132人目の素数さん:2018/03/23(金) 01:31:09.69 ID:fa7L1pkR.net
>>428
分子を変えた場合に最大最小値はどうなるのか気になるでござる ( ゚∀゚) ウヒョッ!

a、b、c、p、q >0 に対して、
(1) a/(pa+qb) + b/(pb+qc) + c/(pc+qa)
(2) b/(pa+qb) + c/(pb+qc) + a/(pc+qa)
(3) c/(pa+qb) + a/(pb+qc) + b/(pc+qa) ≧ 3/(p+q)

487 :132人目の素数さん:2018/03/23(金) 07:43:00.89 ID:EuazrwzR.net
>>486
(3)
c(pa+qb)+ a(pb+qc)+ b(pc+qa)=(p+q)(ab+bc+ca),
コーシーにより
(左辺)≧(a+b+c)^2 /{(p+q)(ab+bc+ca)}≧ 3/(p+q),
でござるか。

488 :132人目の素数さん:2018/03/23(金) 07:49:46.24 ID:EuazrwzR.net
>>483
左側:
積分計算を避けるなら、
AM-GM より
(kk -1/4)^3 ≧(kk)(kk -3/8)(kk -3/8),

{(k+1/2)^(3/2) - (k-1/2)^(3/2)}^2 = 2k(kk +3/4) -2(kk -1/4)^(3/2)
 ≦ 2k(kk +3/4) -2k(kk -3/8)
 = 9k/4,

√k ≧ (2/3){(k+1/2)^(3/2) - (k-1/2)^(3/2)},
以下は同様。

489 :132人目の素数さん:2018/03/24(土) 07:45:51.61 ID:Q7DsXoRu.net
>>483
右側:
{√(k+1)- √k}^2 = 1/{√(k+1)+ √k}^2 ≧ 1/{2(k+1)+ 2k}= 1/{2(2k+1)},
より
(右辺)^2 -(左辺)^2 =(4/9){(k+1)^(3/2)- k^(3/2)}^2 -(1/4){√k + √(k+1)}^2
 =(1/36)[{4(2k+1)^2 +5}{√(k+1)- √k}^2 - 2(2k+1)]
 ≧(1/36)[{4(2k+1)^2 +5}/{2(2k+1)}- 2(2k+1)]
 =(5/36)/{2(2k+1)},

{√k + √(k+1)}/2 <(2/3){(k+1)^(3/2)- k^(3/2)},
以下は同様。

490 :132人目の素数さん:2018/03/24(土) 19:30:42.65 ID:UFxacGAG.net
>>486
(1) 3/p > a/(pa+qb) + b/(pb+qc) + c/(pc+qa) > 0
(2) 3/q > b/(pa+qb) + c/(pb+qc) + a/(pc+qa) > 0

( ゚∀゚) プゥ
ノヽノ) =3'A`)ノ ヒャー
  くく へヘノ

491 :132人目の素数さん:2018/03/26(月) 20:49:46.38 ID:IKnRwfdR.net
>>490
それが限界ですか?

492 :132人目の素数さん:2018/03/27(火) 07:17:22.34 ID:H3+XdNyv.net
>>486

(1) Max{2/p,3/(p+q)} > a/(pa+qb) + b/(pb+qc) + c/(pc+qa) > min{3/(p+q),1/p},

(2) Max{2/q,3/(p+q)} > b/(pa+qb) + c/(pb+qc) + a/(pc+qa) > min{3/(p+q),1/q},

(3) c/(pa+qb) + a/(pb+qc) + b/(pc+qa) ≧ 3/(p+q),   >>487

493 :132人目の素数さん:2018/03/28(水) 05:38:19.81 ID:R1v0Ka1q.net
>>428 >>429

ドイツ Team Selection Test 2010 Vaimo 8, 問2
twitter.com/Inequalitybot/ [129]

494 :132人目の素数さん:2018/03/29(木) 07:31:07.02 ID:9DT8+Pw9.net
>>492
ど、どう証明するのかな? ・・・・・・・・ゴクリ。

    ヽ|/
   / ̄ ̄ ̄\
   /      ヽ
  / \ /  |
  | (●)(●)|‖|
  | / ̄⌒ ̄ヽ U|
  ||i二二ヽ| |
  |U\___ノ |
  |       |

495 :132人目の素数さん:2018/03/30(金) 06:53:28.40 ID:yIR/xm3k.net
>>474
(1)
2^(2n-1) /√n < C[2n,n] < 2^(2n) /√(πn),

大関:参考書[3]、p.53 例題10 (1987)
W.F.Sierpinski: "Elementary theory of numbers",PWN-Polish Sci. Publ. (1964)

496 :132人目の素数さん:2018/04/02(月) 10:26:49.91 ID:qYYBXa/1.net
〔問題983〕
実数 0 < x < π/6 に対して、 不等式
 sin(x) < 2x/(x +π/2)
を示せ。

分かスレ441-983、分かスレ442-10,28,47

497 :132人目の素数さん:2018/04/02(月) 11:35:20.76 ID:QTiBZAaI.net
>>496
問題だけじゃなく、証明も貼っておこうぜ! ここは不等式のコレクターのためのスレなんだからな。

> y=π/2 で成り立てば、
>  2{1-sin(x)}/(π/2 -x) > sin(x)/x,    >>28
>  x/sin(x) > (π/2 +x)/2,
> ならば十分。そこで
>  g(x) = x/sin(x),
> とおく。
> |x|<π/2 で g(x) は下に凸。    … (*)
> g(π/6)=π/3 と g(π/2)=π/2 を通る割線を曳く。
>  z = (π/2 +x)/2,
> -π/2 < x < π/6 のとき g(x) > (π/2 +x)/2,
>
> (*)
> 1-cos(x) ≧ 0,
> x-sin(x) = ∫[0,x] {1-cos(t)} dt > 0 (x>0)
> sin(x)-x・cos(x) = ∫[0,x] t・sin(t) dt > 0 (0<x<4.4934094579)
> より
> g '(x) = {sin(x)-x・cos(x)}/sin(x)^2,
> g "(x) = {1-cos(x)}/sin(x)・g '(x) + {1+cos(x)}{x-sin(x)}/sin(x)^3 > 0,

498 :132人目の素数さん:2018/04/03(火) 14:36:03.43 ID:RjUb/qt3.net
〔問題〕
(1)
f(x)g(x) = 1ならば
 f '(x)g '(x) < 0,
さらに f(x)f "(x) < 0 のとき
 f "(x)g "(x) < 0,

(2)
 g(x) = x/sin(x) について、
 |x| < 2.081575977818 ⇒ g "(x) > 0,

分かスレ442-069

499 :132人目の素数さん:2018/04/03(火) 15:02:35.39 ID:Y/OrPMIk.net
>>498
(1)のf,gって任意の関数かね?

500 :132人目の素数さん:2018/04/03(火) 22:49:34.80 ID:RjUb/qt3.net
>>499

(1) そうです。(微分可能な…)

(2)
|x|< 2.0815759778181 ⇒ {sin(x)/x}" < 0
を使っていいらしい。

501 ::2018/04/06(金) 09:34:42.48 ID:I+Mybrk/.net


502 ::2018/04/06(金) 09:35:02.15 ID:I+Mybrk/.net


503 ::2018/04/06(金) 09:35:19.38 ID:I+Mybrk/.net


504 ::2018/04/06(金) 09:35:35.89 ID:I+Mybrk/.net


505 ::2018/04/06(金) 09:35:56.14 ID:I+Mybrk/.net


506 ::2018/04/06(金) 09:36:14.10 ID:I+Mybrk/.net


507 ::2018/04/06(金) 09:36:30.59 ID:I+Mybrk/.net


508 ::2018/04/06(金) 09:36:50.06 ID:I+Mybrk/.net


509 ::2018/04/06(金) 09:37:10.35 ID:I+Mybrk/.net


510 ::2018/04/06(金) 09:37:31.35 ID:I+Mybrk/.net


511 :132人目の素数さん:2018/04/11(水) 14:21:47.84 ID:BsknsPEs.net
ライヒ・シュトレーベルの不等式
レンゲルの不等式

512 :132人目の素数さん:2018/04/12(木) 10:53:44.62 ID:8y+syOJY.net
非負実数 a_1、…、a_n に対して、
(Σ[k=1 to n] a_k){Σ[k=1 to n] (a_k)^(n-1)} ≦ n*Π[k=1 to n]a_k + (n-1)*Σ[k=1 to n] (a_k)^n

昔の手書きメモから発掘、詳細不明 ( ゚∀゚) ウヒョッ

513 :132人目の素数さん:2018/04/14(土) 00:31:34.32 ID:Rl6BZiHz.net
>>512

兩n = (右辺) - (左辺)
= (n-1)Σ[k=1,n] (a_k)^n - {Σ[k=1,n] a_k}{Σ[k=1,n] (a_k)^(n-1)} + n・a_1・a_2…a_n,
a_1 = a,a_2 = b,a_3 = c,a_4 = d,a_5 = e,
とおいてシューア展開すると、

兩1(a) = 0,
兩2(a,b) = 0,
兩3(a,b,c) = F_1(a,b,c)
兩4(a,b,c,d) = (2/3){F_2(a,b,c) + F_2(b,c,d) + F_2(c,d,a) + F_2(d,a,b)} + (1/3){F_1(a,b,c)d + F_1(b,c,d)a + F_1(c,d,a)b + F_1(d,a,b)c},
兩5(a,b,c,d,e) = (1/2)Σ[a,b,c] F_3(a,b,c) + (1/6)Σ[a,b,c] F_2(a,b,c)(d+e) + (1/6)Σ[a,b,c] F_1(a,b,c)de,

ここに Σ[a,b,c] は C[5,3] = 10項の和

514 :132人目の素数さん:2018/04/14(土) 00:53:24.57 ID:Rl6BZiHz.net
>>513

〔Schurの不等式〕
F_m(x,y,z) = (x^m)(x-y)(x-z) + (y^m)(y-z)(y-x) + (z^m)(z-x)(z-y) ≧ 0,

文献[3] 大関(1987) p.28
文献[8] 安藤(2012) p.27〜28
文献[9] 佐藤(訳)(2013) p.40

515 :132人目の素数さん:2018/04/15(日) 23:04:58.95 ID:gKDKhRE9.net
a, b, c >0 かつ abc=1 のとき、 (a^10 + b^10 + c^10)^2 ≧ 3*(a^13 + b^13 + c^13).



   ┏  ━ゝヽ''人∧━∧从━〆A!゚━━┓。
 < ゝ\',冫。’  ,,,,  ∧,,∧ ' ゛△´ ' ゝ'┃
∇  ┠─Σ┼  ,ニ,◎、・ω・') 冫/ そ', .┨'゚,。
.。冫▽ < 冫 r'/ミ/〉⊂ノ  乙 ≧  ▽
 。 ┃   Σ. 〈/")、〉ノノ  、'’ ≦ │て く
   ┠─ム┼ (_/_iiiノ     、,,’.┼ ァ Ζ┨ ミo'’`
.。○.〆  `、,~´+ ! .!  √ ▽ ',! ヽ.◇ o.┃
   ┗〆━┷. Z,..`"┷━''o.ヾo┷+\━┛,゛;

516 :132人目の素数さん:2018/04/16(月) 12:01:31.88 ID:I9VNB52o.net
兩n = Σ[m=1,n-2] 2(n-m-2)! m!/(n-1)! Σ[a,b,c] F_m(a,b,c) S_{n-m-2}(A-{a,b,c}) ≧0,

 A = {a_1,a_2,…,a_n}
 納a,b,c] は C[n,3] = n(n-1)(n-2)/6 項の和。
 S_j(B) は集合Bの要素で作ったj次の基本対称式
 A = {a_1,a_2,…,a_n}

ついで乍ら
納a,b,c] F_m(a,b,c) = {m(m+1)2}Σ[k=1,n] (a_k)^(m+2) S_{n-m-2}(A-{a_k})
  - m(n-m-1)納k=1,n] (a_k)^(m+1) S_{n-m-1}(A-{a_k})
  + {(n-m)(n-m-1)/2}納k=1,n] (a_k)^m S_{n-m}(A-{a_k})

517 :132人目の素数さん:2018/04/16(月) 12:04:49.28 ID:I9VNB52o.net
>>513

兩n = Σ[m=1,n-2] 2(n-m-2)! m!/(n-1)! Σ[a,b,c] F_m(a,b,c) S_{n-m-2}(A-{a,b,c}) ≧0,

 A = {a_1,a_2,…,a_n}
 Σ[a,b,c] は C[n,3] = n(n-1)(n-2)/6 項の和。
 S_j(B) は集合Bの要素で作ったj次の基本対称式
 A = {a_1,a_2,…,a_n}

ついで乍ら
Σ[a,b,c] F_m(a,b,c) = {m(m+1)2}Σ[k=1,n] (a_k)^(m+2) S_{n-m-2}(A-{a_k})
  - m(n-m-1)Σ[k=1,n] (a_k)^(m+1) S_{n-m-1}(A-{a_k})
  + {(n-m)(n-m-1)/2}Σ[k=1,n] (a_k)^m S_{n-m}(A-{a_k})

518 :132人目の素数さん:2018/04/17(火) 03:17:29.93 ID:+pEnOXwO.net
(1/2)*(3/4)*…*(999999/1000000) < 1/1000 を示せ。


  ∧_∧ 
 ( ´・ω・)  先月の数蝉に不等式の問題があったような…
 (つ旦と) 
  と_)_)

519 :132人目の素数さん:2018/04/17(火) 22:54:13.31 ID:PQyFkARt.net
>>518

√((2k-1)(2k+1)) = √(4kk-1) < 2k,

(左辺) = {√(1・3)/2}{√(3・5)/4}…{√((2n-1)(2n+1))/(2n)} / √(2n+1)
  < 1/√(2n+1)
  < 1/√(2n)
  = 0.001
 
(別法)
 Stirling の公式から

(左辺) = (2n-1)!! / (2n)!!
 = (2n-1)!! / {(2^n) n!}
 = (1/4)^n・C(2n,n)
 = 1/√(nπ)・{1 - 1/(8n) + 1/(128n^2) + 5/(1024n^3) - …… }
 < 1/√(nπ)
 = 1/√(500000π)
 = 0.00079788456080
 
なお、(左辺) = 0.00079788436133

520 :132人目の素数さん:2018/04/18(水) 13:35:34.72 ID:aemp1B+Z.net
コレクションになかったのを拾い集めてきた。(A1)以外は、たぶん過去スレにもないと思ふ。


【絶対値絡み】

(A1) a, b, c >0 に対して、|(a-b)/(a+b)| + |(b-c)/(b+c)| ≧ |(a-c)/(a+c)|

(A2) [宜蘭 2007]
相異なる a, b, c >0 に対して、|(a+b)/(a-b) + (b+c)/(b-c) + (c+a)/(c-a)| > 1

(A3) [疑問]
a, b, c >0 に対して、|(a-b)/(a+b) + (b-c)/(b+c) + (c-a)/(c+a)| のとりうる値の範囲は?


【分数式とか】

(B1) [中国 2008]
a, b, c >0 に対して、ab/c + bc/a + ca/b ≧ 2*(a^3 + b^3 + c^3)^(1/3)

(B2) [宜蘭 2010]
a, b, c >0 に対して、1/(a^2) + 1/(b^2) + 1/(c^2) + 1/{(a+b+c)^2} ≧ (7/25)*{1/a + 1/b + 1/c + 1/(a+b+c)}^2

(B3) [IMO short list 2008]
a, b, c, d >0 に対して、(a-b)(a-c)/(a+b+c) + (b-c)(b-d)/(b+c+d) + (c-d)(c-a)/(c+d+a) + (d-a)(d-b)/(d+a+b) ≧ 0

(B4) [不等式bot]
a, b >0 に対して、
(ab)^3 + (bc)^3 + (ca)^3 + 9(abc)^2 + abc(a^3 + b^3 + c^3 + 9abc) ≧ 3abc(a+b)(b+c)(c+a)


不等式botってのを最近見つけたんだけど、何これ? botって何ぞや?
同じ問題を繰り返し吐き出してるから、自動なのか?
登録してある問題をダブりなしに全部見てみたい。

521 :132人目の素数さん:2018/04/18(水) 13:36:45.65 ID:aemp1B+Z.net
【√がらみ】

(C1) [宜蘭 2008]
a, b, c >0 に対して、次式をみたす実数 k の最小値を求めよ。
a√b + b√c + c√a ≦ k√{(a+b)(b+c)(c+a)}

(C2) [香佐富斯坦 2010]
a, b >0 に対して、
√{(a^2-a+1)(b^2-b+1)} + √{(a^2+a+1)(b^2+b+1)} ≧ 2(x+y)

(C3) [スポック 2012]
a, b, c >0 に対して、
(a+b)√{(b+c)(c+a)} + (b+c)√{(c+a)(a+b)} + (c+a)√{(a+b)(b+c)} ≧ 4(ab+bc+ca)

(C4) [中国 2012]
a, b, c∈[0,1] のとき、√|a-b| + √|b-c| + √|c-a| の最大値を求めよ。

(C5) [波蘭 2004]
a, b, c∈R に対して、
√(2a^2+2b^2) + √(2b^2+2c^2) + √(2c^2+2a^2) ≧ √{3(a+b)^2 + 3(b+c)^2 + 3(c+a)^2}

(C6) [墺太利 2008]
a, b, c >0、a+b+c=1 に対して、
√{a^(b+c)*b^(c+a)*c^(a+b)} ≦ 1/3

(C7) [土耳古 2005]
a, b, c, d ∈R に対して、
√(a^4 + c^4) + √(a^4 + d^4) + √(b^4 + c^4) + √(b^4 + d^4) ≧ (2√2)*(ad+bc)

522 :132人目の素数さん:2018/04/18(水) 13:37:59.14 ID:aemp1B+Z.net
【微積分絡み】

(D1) [Putnum 1999]
実関数 f がC^3級で、任意の x∈R に対して、
 0 < f'(x)、 0 < f''(x)、 0 < f'''(x) ≦ f(x)
をみたすとき、f'(x) < 2f(x) を示せ。

(D2) [AoPS]
f は [0,1] で単調増加な凸関数で、f(0)=0、f(1)=1 をみたす。
g を fの逆関数とするとき、x^2 ≧ f(x)g(x) を示せ。

(D3) [近大 2008]
実関数 f がC^2級で、任意の x∈R に対して f''(x)≧f(x) をみたすとき、
f(x) ≧ f(0)*{e^x + e^(-x)}/2 + f'(0)*{e^x - e^(-x)}/2

(D4) [山梨医改、不等式bot]
f(0) = f(1) = 0、f'は[0,1]で連続のとき、∫[0,1] {f'(x)}^2 dx ≧ (π^2)*∫[0,1] {f(x)}^2 dx

(D5) [京大院 2011]
実連続関数 f,φ は区間[a,b]上で狭義単調増加のとき、
∫[a,b] f(x)dx = 0 ならば、∫[a,b] f(x)φ(x)dx > 0 を示せ。

(D6) [羅馬尼亜 2004]
fが[0,1]で積分可能で、∫[0,1] f(x)dx = ∫[0,1] xf(x)dx = 1 のとき、∫[0,1] {f(x)}^2 dx ≧ 4

523 :132人目の素数さん:2018/04/18(水) 13:38:24.42 ID:aemp1B+Z.net
          ;ヾ ;ヾ ;";ヾ;"  ;ヾ ;ヾ ;
        ;ヾ ;ヾ ;ヾ";ヾ;ヾ ';ヾ ;ヾ ;ヾ ;ヾ ;ヾ
       ;ヾ.;ヾ ';ヾ ;ヾ ;ヾ;ヾ ';ヾ ;ヾ ;ヾ ;ヾ ;ヾ ;ヾ"゙
    ""ヾ゙;ヾ〃;ヾ ;ヾ゙;ヾ ;ヾ ;ヾ"〃ヾ ;ヾ ;ヾ ;ヾ;ヾ
  "';ヾ;ヾ ;ヾ;ヾ ;ヾ ;ヾ ;ヾ ;ヾ ;ヾ ;//;ヾ;ヾ〃゙;ヾ ;ヾ;ヾ"
   """;ヾ ;ヾ ;ヾ ;ヾ ;ヾ ;ヾ ;ヾ" ;ヾ ;ヾ ;ヾ ;ヾ ;ヾ ;ヾ;ヾ"
  ""ヾ;ヾ ;ヾ ;ヾ ゙;ヾ〃ミヾ ;ヾ゙;ヾ ;ヾ 〃;ヾ ;ヾ ;ヾ ;ヾ;ヾ
     ;ヾ ;ヾ 〃;ヾ ;ヾ ;ヾ ;ヾ ;ヾ ;ヾ ;ヾ ;ヾ ;ヾ ;ヾ ;ヾ"
     "" ;ヾ ;ヾ゙ヾ ;ヾ ;ヾ;ヾ"゙ "iヾ;ヾ;ヾ" ゙ヾ;ヾ;ヾ"
        ""   'ヾ;ヾ" || l | ゙|/;ヾ"     "
             "   |l i  l゙l|
,,,,   ",,,," ,,, " ∧ ∧ ,,, |l | ゙ || '' ,, "  " ,,    春は不等式!
         ( ゚∀゚)∬ ノノ 从ヾ ヽ、   ,,, ''  やうやう白くなりゆく山際
 '' ` ` /   (_)旦.   /              少し明かりて、
     /         /    ''''     "" 紫だちたる雲の細くたなびきたる
      ̄ ̄ ̄ ̄ ̄ ̄ ̄

524 :132人目の素数さん:2018/04/19(木) 02:39:28.61 ID:jDhUrl+i.net
>>520

(A2) [155]
 (左辺) = |(p+q)/(p-q)|,
 ここに p = aab+bbc+cca -3abc ≧ 0,q = abb+bcc+caa -3abc ≧ 0,
 q-p = (a-b)(b-c)(c-a) = ,
 佐藤(訳) 問題3.103

(A3)
 絶対値の中身 = (a-b)(b-c)(c+a)/{(a+b)(b+c)(c+a)},
 -1 〜 +1

(B1) [96]
  (ab/c + bc/a + ca/b)^3 ≧ 8(aaa+bbb+ccc) + 3abc,
 bc/a=x,ca/b=y,ab/c=z とおく。

(B2) [198]
 a+b+c = s,1/a+1/b+1/c = 3/h とおく。
 s-3h ≧ 0,
 (左辺) ≧ 3/hh + 1/ss,
 (右辺) = (7/25)(3/h+1/s)^2,
 (左辺) - (右辺) ≧ 6(2s-h)(s-3h)/(5hs)^2 ≧ 0,
 等号成立は s-3h = 0,a=b=c.

(B3) [100]
 a-c,b-d の2次形式として正定値。

(B4) [107]
 (左辺) - (右辺) = (sssu+ttt+27uu) - 9stu ≧0 (←AM-GM)
 ここに、s = a+b+c、t = ab+bc+ca、u = abc.

525 :132人目の素数さん:2018/04/19(木) 02:45:19.23 ID:jDhUrl+i.net
>>521

(C1) [70]
 コーシーにより、
 (左辺)^2 ≦ (a+b+c)(ab+bc+ca) = (a+b)(b+c)(c+a) + abc ≦ (9/8)(a+b)(b+c)(c+a),
 K = 3/√8.
 佐藤(訳) 問題3.113

(C2) [114]
(左辺)^2 = 2{(aa+1)(bb+1) + ab + √(a^4+aa+1)√(1+bb+b^4)}
 = 2{(aa+3ab+bb) + (ab-1)^2 + √(a^4+aa+1)√(1+bb+b^4)}
 ≧ 2{(aa+3ab+bb) + (aa+ab+bb)}
 = 4(a+b)^2   (←コーシー)
 等号成立は xy=1.

(C3) [16]
 (a+b)√{(b+c)(c+a)} ≧ (a+b){c+√(ab)} ≧ (a+b)c + 2ab,
 循環的にたす。

(C4) [62]
 bはaとcの中間にあるとする。
 √|a-b| + √|b-c| + √|c-a| ≦ (1+√2)|c-a| ≦ 1 + √2、
 等号は(a,b,c)=(0,1/2,1)

(C5) [86]
 2√{3(aa+bb+cc)} ≧ √(2aa+2bb) + √(2bb+2cc) + √(2cc+2aa) ≧ √{3(a+b)^2+3(b+c)^2+3(c+a)^2} ≧ 2(a+b+c),
 (左辺)^2 = 4(aa+bb+cc) + 4√(aa+bb)√(bb+cc) + … + …
 ≧ 4(aa+bb+cc) + 2(a+b)(b+c) + 2(b+c)(c+a) + 2(c+a)(a+b)
 = 3(a+b)^2 + 3(b+c)^2 + 3(c+a)^2
 = (中辺)^2.

(C6) [49]
 f(x) = (1-x)log(x) ≦ -(1-x)^2 は 0<x<1 で上に凸。
 f(a) + f(b) + f(c) ≦ 3f((a+b+c)/3) = 2log(1/3)

(C7) [71]
 √(xx+yy) ≧ (x+y)/√2 を使う。

526 :132人目の素数さん:2018/04/19(木) 02:51:51.34 ID:jDhUrl+i.net
>>522

(D1) [143]
 未だ解けぬ〜

(D2) [164]
{f(x)/x} ' = {xf '(x) - f(x)}/xx =∫[0→x] {f '(x) - f '(t)}dt/xx > 0(←fは凸)
 f(x)/x は単調増加,
 x < g(x) < 1,
 f(x)/x ≦ f(g(x))/g(x) = x/g(x),

(D3) [144]
 0 ≦∫[0,x] {f ''(t) - f(t)}sinh(x-t)dt = [ f(t)cosh(x-t)+f '(t)sinh(x-t) ](t=0,x) = f(x) - f(0)cosh(x) - f '(0)sinh(x).

(D4) [121] (Wirtingerの不等式)
 g(x) = cot(x)とおく。
 g '(x) + g(x)g(x) = -1,
 [f(x)f(x)g(x)](x=0-π) = 0,
 ∴ 0 ≦∫{f '(x)−f(x)g(x)}^2 dx
 = ∫f '(x)f '(x)dx - [f(x)f(x)g(x)](x=0-π) + ∫f(x)f(x){g '(x) + g(x)g(x)}dx
 = ∫f '(x)f '(x)dx - ∫f(x)f(x)dx,
 大関・青柳「不等式」槇書店 p.204

(D5) [211]
 中間値の定理から、a<c<b なるcがあって f(c)=0,
 単調性から、(x-c)f(x)≧0、(x-c){φ(x)-φ(c)}≧0,
 これを入れる。

(D6) [54]
 0 ≦∫{f(x)+2-6x}^2 dx
 = ∫f(x)^2 dx + 4∫f(x)dx -12∫f(x)・x dx +4∫(1-3x)^2 dx
 = ∫f(x)^2 dx + 4 -12 +4.

[ ]内は Inequalitybot の番号ですぅ。

527 :132人目の素数さん:2018/04/19(木) 05:10:55.02 ID:gkRveId7.net
(C8) [月即別 2013] [187]
a≧b≧0 のとき、(a^2+b^2)^(1/2) + (a^3+b^3)^(1/3) + (a^4+b^4)^(1/4) ≦ 3a+b

528 :132人目の素数さん:2018/04/19(木) 13:00:30.09 ID:jDhUrl+i.net
>>527

(C8) [187]
 (a^2 + b^2)^(1/2) ≦ a + (√2−1)b,
 (a^3 + b^3)^(1/3) ≦ a + {2^(1/3)−1}b,
 (a^4 + b^4)^(1/4) ≦ a + {2^(1/4)−1}b,
辺々たす。
 (左辺) ≦ 3a + 0.8633417b

529 :132人目の素数さん:2018/04/21(土) 09:22:11.44 ID:I5oMZRza.net
>>87

 a^(2/3) = A,b^(2/3) = B,c^(2/3) = C とおくと、

 aa+bb+cc - 2(ab+bc+ca) + 2abc+1
 ≧ A^3 + B^3 + C^3 - 2{AB(A+B) +BC(B+C) +CA(C+A)} + 3ABC
 = 兩3(A,B,C)     >>513
 = F_1(A,B,C)
 ≧ 0,

530 :132人目の素数さん:2018/04/22(日) 13:39:47.36 ID:7rjXNdwL.net
>>163

〔Turkevici の不等式〕 - 改

a^4 + a^4 + a^4 + a^4 + 2abcd - {ab(aa+bb) +ac(aa+cc) +ad(aa+dd) +bc(bb+cc) +bd(bb+dd) +cd(cc+dd)}/2
= a^4 + a^4 + a^4 + a^4 + 2abcd - {(a^3)(b+c+d) -(b^3)(c+d+a) -(c^3)(d+a+b) -(d^3)(a+b+c)}/2
= (1/2)兩4
≧ 0,     >>513

531 :132人目の素数さん:2018/04/22(日) 13:55:51.43 ID:7rjXNdwL.net
>>163

〔Turkevici の不等式〕- 改

a^4 + b^4 + c^4 + d^4 + 2abcd - {ab(aa+bb) +ac(aa+cc) +ad(aa+dd) +bc(bb+cc) +bd(bb+dd) +cd(cc+dd)}/2
= {3(a^4 + b^4 + c^4 + d^4) -(a+b+c+d)(a^3+b^3+c^3+d^3) + 4abcd}/2
= (1/2)兩4
≧ 0     >>513

532 :132人目の素数さん:2018/04/22(日) 16:33:05.00 ID:/laO1mG2.net
( ゚∀゚) ワクワクが止まらんね。

533 :132人目の素数さん:2018/04/24(火) 00:18:06.70 ID:43b+grE0.net
>>515
 a^(10/3) = A,b^(10/3) = B,c^(10/3) = C とおくと本題は
 (A^3+B^3+C^3)^2 ≧ 3(A^4+B^4+C^4)

〔補題〕
 (A^3+B^3+C^3)^2 ≧ (AB+BC+CA)(A^4+B^4+C^4)

m = min{A,B,C} とおき、
{A,B,C} = {m,m+x,m+y} (x≧0,y≧0)
とする。
(左辺) - (右辺) = (A^3+B^3+C^3)^2 - (AB+BC+CA)(A^4+B^4+C^4) 
 = (m^4)(xx-xy+yy) + (2m^3)xy(x+y) + (2m^2){2xx(x-y)^2 +5xxyy +2yy(x-y)^2} + m(x+y){xx(2x-2.5y)^2 +(7/2)xxyy +yy(2.5x-2y)^2} + (x-y)(x^5-y^5) + 2(xy)^3
 ≧ 0,

534 :132人目の素数さん:2018/04/24(火) 11:04:06.96 ID:43b+grE0.net
>>515 >>533

〔補題〕
 (A^3+B^3+C^3)^2 ≧ (AB+BC+CA)(A^4+B^4+C^4)

(左辺) - (右辺) = F_0(A,B,C) F_0(AA,BB,CC) + (ABC)^2 F_{-2}(A,B,C) ≧ 0,

F_0(A,B,C) = (A-B)(A-C) + (B-C)(B-A) + (C-A)(C-B) = {(A-B)^2 + (B-C)^2 + (C-A)^2}/2 ≧ 0,

F_{-2}(A,B,C) = (A-B)(A-C)/AA + (B-C)(B-A)/BB + (C-A)(C-B)/CC = ABC F_1(1/A,1/B,1/C) ≧ 0,

535 :132人目の素数さん:2018/04/24(火) 23:16:08.34 ID:43b+grE0.net
>>533 >>534

〔補題〕
1≦n≦3,A〜C≧0 のとき
 (A^n + B^n + C^n)^2 ≧ (AB+BC+CA) {A^(2n-2)+B^(2n-2)+C^(2n-2)} ≧ 3ABC {A^(2n-3)+B^(2n-3)+C^(2n-3)},

右側はチェビシェフなど。

536 :132人目の素数さん:2018/04/25(水) 00:55:21.59 ID:i3CGBkWM.net
>>533 >>534 >>535

〔補題〕
1≦n≦5,A〜C≧0 のとき
 (A^n + B^n + C^n)^2 ≧ 3ABC {A^(2n-3) + B^(2n-3) + C^(2n-3)},

(例)
n=3 のとき
(左辺) - (右辺) = (A^3 +B^3 +C^3) (A^3 +B^3 +C^3 -3ABC) ≧ 0,

n=4 のとき
(左辺) - (3/2) {(a^3)(b+c) + (b^3)(c+a) + (c^3)(a+b)}
 = (1/2) {(aa-ab+bb)(a-b)^2 + (bb-bc+cc)(b-c)^2 + (cc-ca+aa)(c-a)^2}
 ≧ 0,
ここに、a=AA,b=BB,c=CC.

(3/2) {(a^3)(b+c) + (b^3)(c+a) + (c^3)(a+b)} - (右辺)
 = (3/2) {(A^6)(B-C)^2 + (B^6)(C-A)^2 + (C^6)(A-B)^2}
 ≧ 0,

537 :132人目の素数さん:2018/04/25(水) 22:02:16.65 ID:CPKgHcHK.net
三角形の辺長 a、b、c に対して、
  {√(a+b-c)}/(√a + √b - √c) + {√(b+c-a)}/(√b + √c - √a) + {√(c+a-b)}/(√c + √a - √b) ≦ 3

538 :132人目の素数さん:2018/04/27(金) 02:42:51.45 ID:MuDbcQQ+.net
>>537 [6]

A = √b+√c-√a > 0,
B = √c+√a-√b > 0,
C = √a+√b-√c > 0,
とおく。
b+c-a = AA - (A-B)(A-C)/2,
√(b+c-a) ≦ A - (A-B)(A-C)/4A,
(左辺) = √(b+c-a) /A + √(c+a-b) /B + √(a+b-c) /C
 ≦ 3 - (A-B)(A-C)/(4AA) - (B-C)(B-A)/(4BB) - (C-A)(C-B)/(4CC)
 = 3 - (1/4) F_{-2}(A,B,C)
 = 3 - (ABC/4) F_1(1/A,1/B,1/C)
 ≦ 3.

 IMOSL-2006 予選 A.6、JMO春合宿
 文献[8] 安藤 (2012),p.147 例題3.2.3(9),
 http://www.casphy.com/bbs/highmath/ 不等式1-307、434、437

539 :132人目の素数さん:2018/04/27(金) 02:55:00.93 ID:WAhBLXIV.net
( ゚∀゚) いつも素晴らしいデスネ。GWに精読させていただきます。

540 :132人目の素数さん:2018/04/27(金) 03:03:26.10 ID:WAhBLXIV.net
実数 a_k、b_k (1≦k≦n)) に対して、
1 + Σ[k=1 to n] (a_k + b_k)^2 ≦ (4/3)*{1 + Σ[k=1 to n] (a_k)^2}*{1 + Σ[k=1 to n] (b_k)^2}

541 :132人目の素数さん:2018/04/27(金) 07:15:04.11 ID:WAhBLXIV.net
任意の m、n∈N (m > n) に対して、
lcm(m, n) + lcm(m+1, n+1) > 2mn/{√(m-n)}

542 :132人目の素数さん:2018/04/27(金) 11:37:15.92 ID:MuDbcQQ+.net
>>540

A = Σ[k=1,n] (a_k)^2,
B = Σ[k=1,n] (b_k)^2,
C = Σ[k=1,n] a_k b_k,
とおく。
A+B-2C = Σ[k=1,n] (a_k - b_k)^2 ≧ 0,
AB-CC = Σ[1≦j<k≦n] (a_j b_k - a_k b_j)^2 ≧ 0  (←コーシー)

(右辺) - (左辺) = (4/3)(1+A)(1+B) - (1+A+B+2C)
= (1/3) (1+A+B+4AB-6C)
= (1/3) {(A+B-2C) + 4(AB-CC) + (1-2C)^2}
≧ 0,

等号成立は a_k = b_k,A = B = C = 1/2.

543 :132人目の素数さん:2018/04/28(土) 00:44:27.52 ID:9CKS2DSq.net
>>541

gcd(m,n) | (m-n)
gcd(m+1,n+1) | (m-n)
左辺は互いに素ゆえ、 (←背理法で)
gcd(m,n)gcd(m+1,n+1) | (m-n)

lcm(m,n) + lcm(m+1,n+1)
= mn/gcd(m,n) + (m+1)(n+1)/gcd(m+1,n+1)
> mn{1/gcd(m,n) + 1/gcd(m+1,n+1)}
> 2mn/√{gcd(m,n)gcd(m+1,n+1)}    (←AM-GM)
≧ 2mn/√(m-n),

544 :132人目の素数さん:2018/04/29(日) 13:40:34.24 ID:LZWvDOTX.net
>>374 >>398 >>399 >>416 >>417

 nΣ[k=1,n] s_k (a_k)^2 ≧ M_n (s_n)^3,
とおく。

M_2 = 0.7377393811182 = 2(47-14√7)/27
   (a,b) =(√7 -1,4-√7)(3+√7,2+√7)

M_3 = 0.6481616033162
   (a,b,c) = (1.38436,1.13916,1)

M_4 = 0.60233351875
   (a,b,c,d) = (1.52472,1.25465,1.10139,1)

M_5 = 0.574255

M_6 = 0.5551782

M = 0.444444 = 4/9   (n→∞),

545 :132人目の素数さん:2018/04/29(日) 22:17:02.33 ID:LZWvDOTX.net
>>544
 Memo.

漸化式は
 a_{n+1} = (1/2) {√(2x-1) - 1} s_n,
 s_{n+1} = s_n + a_{n+1},
 M_n = (n/3) (x-1),
ここに
 x = (1 + a_n/s_n)^2.

(例)

M_1 = 1
 a_1 = s_1 = 1

M_2 = 2(47-14√7)/27 = 0.7377393811182
 a_2 = (√7 -1)/2 = 0.8228756555323
 s_2 = (√7 +1)/2 = 1.8228756555323

M_3 = 0.64816160331616
 a_3 = 0.72235563718495
 s_3 = 2.54523129271725

M_4 = 0.60233351872589
 a_4 = 0.65585825517001
 s_4 = 3.20108954788726

M_5 = 0.57425545264547
 a_5 = 0.60768519695068
 s_5 = 3.80877474483794

M_6 = 0.55517800140267
 a_6 = 0.57066170678793
 s_6 = 4.37943645162587

本題から逸れてしまった…

546 :132人目の素数さん:2018/04/29(日) 22:33:49.66 ID:LZWvDOTX.net
>>374 (改)

Σ[k=1,n] (s_{k-1} + s_k)/2 ・ (a_k)^2 > (4/9n) (s_n)^3,

便宜上 s_0 = 0 とおいた。

* 中点 (s_{k-1} + s_k)/2 で接線を曳く。

547 :132人目の素数さん:2018/04/30(月) 07:31:33.28 ID:iiZ/CJ+E.net
>>544
>>545 Memo. の続き

M_10 = 0.51565443182467
 a_10 = 0.47804498656917
 s_10 = 6.41086198943751

M_100 = 0.45433807243808
 a_100 = 0.21749813721698
 s_100 = 32.0226683930223

M_1000 = 0.44575956171259
 a_1000 = 0.10051892239154
 s_1000 = 150.383787216053

M_10000 = 0.44460977509949
 a_10000 = 0.04662595061307
 s_10000 = 699.152499550131

548 :132人目の素数さん:2018/05/02(水) 00:00:30.49 ID:mJGWQQsb.net
>>545
Memo.

(略証)
nについての帰納法による。
 a_{n+1} = A と略す。
まず s_n を固定して a_1 〜 a_n を動かしたときの最小値は、
 Σ[k=1,n+1] s_k (a_k)^2 - μ(s_{n+1})^3
 = Σ[k=1,n] s_k (a_k)^2 + (s_n + A)A^2 - μ(s_n + A)^3
 ≧ (M_n /n)(s_n)^3 + (s_n + A)A^2 - μ(s_n + A)^3 = f(A)
つまり a_1 〜 a_n の比はnの場合と同じでよい。

次に f(A) = 0 が重根をもつようにμを決めるのだが、言い換えれば
f(A) = 0 と f '(A) = 0 が共通根をもつことである。

f(A) = (M_n /n)(s_n)^3 + (s_n + A)A^2 - μ(s_n + A)^3 = 0,
f '(A) = 2(s_n)A + 3A^2 - 3μ(s_n + A)^2 = 0,
から A とμを決める。
まずμを消去すれば
 A(s_n + A) - 3M_n /(2n・s_n) = 0,
∴ A = (1/2){√[1 + 6M_n /(n・(s_n)^3)] -1}s_n,
これを使うとμが求まり
 M_{n+1} = (n+1)μ = {(n+1)/3}([1 + A/s_{n+1}]^2 - 1),
 s_{n+1} = s_n + A,
と表わせる。

549 :132人目の素数さん:2018/05/02(水) 03:13:46.81 ID:mJGWQQsb.net
〔問題8〕
閉区間 [0,1] で定義された連続関数f(x)は、次の条件を満たすとする。
ある正の実数Lが存在して、[0,1] 上のすべての実数xにおいて
 0 ≦ f(x) ≦ L∫[0,x] f(t)dt
が成り立つ。
このとき、[0,1] 上のすべての実数xにおいてf(x)=0であることを示せ。
http://www.toshin.com/concours/mondai/mondai8.php

〔問題12〕
p_1,p_2,…,p_k を m 以下のすべての素数とする。
この時、以下の不等式が成り立つことを示せ。
 log(m) - 1 ≦ (1/m)log(m!) < Σ[i=1,k] log(p_i)/(p_i - 1)
http://www.toshin.com/concours/mondai/mondai12.php

〔問題18〕
正の実数 a,b,c が ab+bc+ca=1 を満たすとき
 (b+c) {√(aa+1) +a} ≧ 2,
 (c+a) {√(bb+1) +b} ≧ 2,
 (a+b) {√(cc+1) +c} ≧ 2,
が成り立つことを示せ。
http://www.toshin.com/concours/mondai/mondai18.php

〔問題32〕
nを2以上の整数とする。正の実数 a_1,a_2,…,a_n に対して不等式
 Σ[k=1,n] (kk-2k+2)a_k + Σ[k=1,n-1] (1/a_k)(a_{k+1})^2 ≧ (n^2)a_n
が成り立つことを示せ。また、等号が成立する条件を求めよ。
http://www.toshin.com/concours/mondai/mondai32.php

550 :132人目の素数さん:2018/05/02(水) 06:04:11.72 ID:G30sYyfK.net
( ゚∀゚) ウヒョッ! スバラスィ!

551 :132人目の素数さん:2018/05/02(水) 14:19:59.58 ID:mJGWQQsb.net
>>548
 訂正スマソ

 A (s_n + A) - (3M_n /2n) (s_n)^2 = 0,
∴ A = (1/2) {√(1 + 6M_n /n) - 1} s_n,

552 :132人目の素数さん:2018/05/02(水) 22:45:46.08 ID:G30sYyfK.net
[bot 5]
a, b, c≧0 のとき、a(a-b)(a-2b) + b(b-c)(b-2c) + c(c-a)(c-2a) ≧0

(1) この証明は?
(2) a(a-mb)(a-nb) + … とイパーン化できるでござるか?

553 :132人目の素数さん:2018/05/03(木) 00:16:14.95 ID:CZ0Fa01r.net
>>552 [111]

(1)
min{a, b, c} = M, {a, b, c} = {M, M+x, M+y} とすると、
(左辺) = 2M(xx-xy+yy) + x(x-2y)^2 + (x-y)^2・y ≧0.

USA.ELMO-2009 day1-Q.3

554 :132人目の素数さん:2018/05/05(土) 12:58:01.27 ID:cos8i+vX.net
>>552 [111]

(2)
m(m+1) ≦ 3 + 4√2 のとき
 a(a-b)(a-mb) + b(b-c)(b-mc) + c(c-a)(c-ma) ≧ 0

m(m+1) = 3 + 4√2 の根は
 m_1 = -{√(13+16√2) +1}/2 = -3.4844353317658568752
 m_2 = {√(13+16√2) -1}/2 = 2.4844353317658568752

等号成立条件
・m_1 < m < m_2 のとき (a,b,c) = (1,1,1)
・m = m_1,m_2 のとき
 (a,b,c) = (1,1,1)、(0,t1,1)、(0,1,t2) とそのrotation
 t_1,t_2 は tt - (1+√2)t + 1 = 0 の根
 t_1 = {1+√2 -√(2√2-1)}/2 = 0.531010056459569184633
 t_2 = {1+√2 +√(2√2-1)}/2 = 1.883203505913525864169

555 :132人目の素数さん:2018/05/06(日) 01:42:47.04 ID:KhrVKVJy.net
>>549 
〔問題18〕
 (a+b)√{(c+b)(c+a)} ≧ (a+b)(c+√ab) ≧ (a+b)c + 2ab,

>>521 >>525  (C3) [16] と同じ。

556 :132人目の素数さん:2018/05/07(月) 23:55:33.94 ID:LPTyY7qu.net
〔問題〕
自然数nに対して

(1) C[2n,n] = (2n)! / (n!)^2 ≧ 4^n / (2√n),

(2) C[3n,n] = (3n)! / {n!・(2n)!} ≧ (27/4)^n ・4/(9√n),

等号成立は n=1

>>512 >>513
Janos Suranyi の不等式と云うらしい…

557 :132人目の素数さん:2018/05/08(火) 00:30:20.27 ID:rSTdfkqz.net
>>556

(1) は >>474 >>476 >>495 と同様

(2) もnについての帰納法で

C[3n+3,n+1] / C[3n,n] = {(3n+1)(3n+2)(3n+3)}/{(2n+1)(2n+2)(n+1)}
= (27/4) {(n+1/3)(n+2/3)} / {(n+1/2)(n+1)}
= (27/4) (N + 2/9) / {(n+1/2)(n+1)}    ← N=n(n+1) とおいた。
> (27/4) (N + 1/8) / {(n+1/2)(n+1)}
≧ (27/4) √{N(N+1/4)} / {(n+1/2)(n+1)}
= (27/4) (n+1/2)√{n(n+1)} / {(n+1/2)(n+1)}
= (27/4) √{n/(n+1)},
により成立

558 ::2018/05/08(火) 12:23:26.09 ID:FpEjvdxJ.net


559 ::2018/05/08(火) 12:23:45.98 ID:FpEjvdxJ.net


560 ::2018/05/08(火) 12:24:05.04 ID:FpEjvdxJ.net


561 ::2018/05/08(火) 12:24:26.08 ID:FpEjvdxJ.net


562 ::2018/05/08(火) 12:24:45.97 ID:FpEjvdxJ.net


563 ::2018/05/08(火) 12:25:05.95 ID:FpEjvdxJ.net


564 ::2018/05/08(火) 12:25:25.76 ID:FpEjvdxJ.net


565 ::2018/05/08(火) 12:25:45.88 ID:FpEjvdxJ.net


566 ::2018/05/08(火) 12:26:08.46 ID:FpEjvdxJ.net


567 ::2018/05/08(火) 12:26:32.09 ID:FpEjvdxJ.net


568 :132人目の素数さん:2018/05/08(火) 18:14:59.57 ID:rSTdfkqz.net
>>556 の系

C[3n,2n] ≧ 4/(9√n)・(27/4)^n,

C[3n-1,2n-1] = (2/3)C[3n,2n] ≧ (2/√n)・(27/4)^(n-1),

569 :132人目の素数さん:2018/05/12(土) 19:38:44.22 ID:a2jXMLKH.net
x^2 + y^2 + z^2 ≧ xy + yz + zx + (3/4)*(x-y)^2

これって既出だっけ?

570 :132人目の素数さん:2018/05/12(土) 22:48:16.65 ID:ERQyPxVg.net
>>569 [42]

(xx+yy+zz) - (xy+yz+zx) = (3/4)(x-y)^2+(1/4)(x+y-2z)^2,

でござる。

571 :132人目の素数さん:2018/05/19(土) 05:31:14.93 ID:xhdfIuy0.net
>>549

〔問題8〕の解答

h(x) = e^(-Lx) ∫[0,x] f(t)dt とおくと題意により
 h(x) ≧ 0 = h(0)   …… (1)
また h(x) は(0,1) 上で微分可能で
 h '(x) ≦ 0,
∴ h(x) = h(0) + ∫[0,x] h '(t)dt ≦ h(0)  …… (2)
(1) (2) により [0,1] 上で h(x) = h(0) = 0 が成り立つ。
したがって、[0,1] 上のすべての実数xにおいて
 0 ≦ f(x) ≦ L ∫[0,x] f(t)dt = 0,
より、f(x) = 0 である。   ■

http://www.toshin.com/concours/mondai/mondai8.php

572 :132人目の素数さん:2018/05/19(土) 07:16:34.16 ID:xhdfIuy0.net
>>549

〔問題12〕の解答

(左側)
 任意の正の整数mに対し、
 log(m!) = Σ[L=1,m-1] log(L+1) ≧ Σ[L=1,m-1] ∫[L,L+1] log(t)dt = ∫[1,m] log(t)dt = m{log(m) -1} +1,
∴ log(m) - 1 ≦ (1/m)log(m!)

(右側)
実数xに対し、x以下の最大の整数を [x] で表わす。
また、0でない整数nと素数pに対し、v_p(n) で、nの素因数分解に現れるpの回数を表わすものとする。
ここで、m! はm以下の素数しか素因数に持たないので、
 log(m!) = Σ[i=1,k] v_pi(m!) log(p_i)
と表わされる。ここで、
 v_p(m!) < m/(p-1)
が分かるのでこれを上の式と組み合わせて
 (1/m)log(m!) < Σ[i=1,k] log(p_i)/(p_i -1)
が示された。(終)

http://www.toshin.com/concours/mondai/mondai12.php

573 :132人目の素数さん:2018/05/19(土) 07:59:43.93 ID:xhdfIuy0.net
>>549

〔問題32〕の解答

(a_{k+1})^2 / a_k ≧ 2k・a_{k+1} - kk・a_k,
辺々たして
Σ[k=1,n-1] (a_{k+1})^2 / a_k ≧ Σ[k=2,n] 2(k-1) a_k - Σ[k=1,n-1] kk・a_k
 = nn・a_n - Σ[k=1,n] a_k - Σ[k=1,n] (k-1)^2・a_k
 = nn・a_n - 1 - Σ[k=1,n] (k-1)^2・a_k
を導く。等号成立条件は、各 k=1,2,…,n-1 で a_{k+1} = k・a_k である場合だから、すべての i=1,2,…,n に対し
 a_i = (i-1)! /{Σ[k=1,n] (k-1)!}
が成立することである。

http://www.toshin.com/concours/mondai/mondai32.php

574 :132人目の素数さん:2018/05/20(日) 11:07:45.50 ID:TfngVWLR.net
(a^2 + b^2 + c^2)^2 - (ab+bc+ca)^2 ≧ (√6)(a-b)(b-c)(c-a)(a+b+c)

575 :132人目の素数さん:2018/05/20(日) 11:34:45.57 ID:TfngVWLR.net
botの192って画像がないな

576 :132人目の素数さん:2018/05/20(日) 18:26:26.97 ID:1IiDnvUy.net
>>575 [192]

任意の実数a,b,cに対し、
(a-b)(a-c)(aa-bc)^2 + (b-c)(b-a)(bb-ca)^2 + (c-a)(c-b)(cc-ab)^2 ≧ 0,
を示せ。

 //www.casphy.com/bbs/highmath/不等式2-188 (じゅー)

577 :132人目の素数さん:2018/05/20(日) 18:37:09.27 ID:1IiDnvUy.net
>>574 [104]

s = a+b+c,t = ab+bc+ca,u = abc, = (a-b)(b-c)(c-a) とおく。
ss-3t≧0,
(左辺) = (ss-2t)^2 -tt
 = (ss-t)(ss-3t)
 = (1/3){2ss + (ss-3t)}(ss-3t)
 ≧ {(2√2)/3}|s|(ss-3t)^(3/2),
 ≧ (√6)|s處,
∵ 4(ss-3t)^3 = 27刧 + {(a+b-2c)(b+c-2a)(c+a-2b)}^2 ≧ 27刧,
等号成立は等間隔かつ ss+3t = 0 より{1-√6,1,1+√6}

http://www.casphy.com/bbs/highmath/ 不等式2-197

578 :132人目の素数さん:2018/05/20(日) 19:27:29.09 ID:1IiDnvUy.net
>>572 (右側) 補足

自然数nと素数pに対し、v_p(n) で、nの素因数分解に現れるpの回数を表わすものとする。
 v_pi(m!) = [ m/p ]+ [ m/p^2 ] + [ m/p^3 ] + … + [ m/p^d ]
ここに、d = [ log(n)/log(p) ].

これもルジャンドルの定理と云うらしい。
http://mathtrain.jp/legendretheorem

〔補題12〕
 v_p(m!) < m/(p-1)

(略証)
d = [ log(n)/log(p) ] とおくと
 v_pi(m!) ≦ m/p + m/p^2 + m/p^3 + … + m/p^d < m/(p-1),

579 :132人目の素数さん:2018/05/20(日) 20:56:05.37 ID:TfngVWLR.net
a, b, c > 0 に対して、a/(b+c) + 20b/(c+a) + 17c/(a+b) > 8

best possible かどうか分からん

580 :132人目の素数さん:2018/05/20(日) 20:57:26.68 ID:TfngVWLR.net
√がらみ
http://artofproblemsolving.com/community/c6h1375433p7600195

581 :132人目の素数さん:2018/05/21(月) 00:35:23.92 ID:2xKr+/2q.net
B. 4931.
Prove that if a, b, c are the sides of a triangle then
{a^2(b+c) + b^2(a+c)} /(abc) > 3.
http://www.komal.hu/feladat?a=honap&h=201802&t=mat&l=en

B. 4925.
http://www.komal.hu/feladat?a=honap&h=201801&t=mat&l=en

B. 4953.
http://www.komal.hu/feladat?a=honap&h=201804&t=mat&l=en

P.1, Problem 1.
https://archives.ust.hk/dspace/bitstream/9999/46212/1/math-02a-a109.pdf

582 :132人目の素数さん:2018/05/21(月) 04:03:40.39 ID:9YF4F+CN.net
>>581

B.4925 (改) (KoMaL,h=201801)
 0<a<n のとき
 a/{a^(n+1) + (n-a)} ≦ 1/n

(略解)
 a^(n+1) -(n+1)a + n
 = (a-1){a^n + a^(n-1) + … + a -n}
 = Σ[k=1,n] (a-1)(a^k -1)
 ≧ 0,


B.4931 (KoMaL,h=201802)
 {aa(b+c) + bb(a+c)}/abc > 3,

(略解)
 aa(b+c) + bb(a+c) = ab(a+b-c) + (a-b)^2・c + 3abc ≧ 3abc,


B.4953 (KoMaL,h=201804)
 log(n) + Σ[k=2,n] √{(k-1)/k} < Σ[k=2,n] √{k/(k-1)},

(略解)
 x>0 ⇒ x < sinh(x),
 a>1 ⇒ 2log(a) < a - 1/a,
 a = √{k/(k-1)} とおく。
 log(k) - log(k-1) < √{k/(k-1)} - √{(k-1)/k},
k=2 から k=n までたす。


Math. Excalibur,Vol.21,No.4,p.1 (2018)
Problem 1.
 a,b,c >0,a+b+c=1 のとき
 a√(2b-1) + b√(2c+1) + c√(2a+1) ≦ √{2-(aa+bb+cc)},

(略解)
関数f(x) = √x は上に凸ゆえ、Jensenで
(左辺) ≦ √{a(2b+1) + b(2c+1) + c(2a+1)}
 = √{(a+b+c) + 2(ab+bc+ca)} / (a+b+c)
 = √{1 +2(ab+bc+ca)}
 = (右辺)
等号成立は (a,b,c) = (1/3,1/3,1/3) および (1,0,0) など。

583 :132人目の素数さん:2018/05/21(月) 15:35:20.78 ID:9YF4F+CN.net
>>579

左辺が最小になる点では
(b+c)^2 : (c+a)^2 : (a+b)^2 = 1 : 20 : 17,
(b+c) : (c+a) : (a+b) = √1 : √20 : √17,
b+c = √1,
c+a = √20,
a+b = √17,
a = (-√1 +√20 +√17)/2,
b = (+√1 -√20 +√17)/2,
c = (+√1 +√20 -√17)/2,

(左辺) ≧ a√1 + b√20 + c√17
 = √(1・20) +√(20・17) +√(17・1) -19
 = 8.0343304952

584 :132人目の素数さん:2018/05/21(月) 18:58:12.63 ID:9YF4F+CN.net
>>579 >>583

b+c = A,c+a = B,a+b = C とおくと

(左辺) = 1・(B+C-A)/(2A) + 20(C+A-B)/(2B) + 17(A+B-C)/(2C)
 = (1/2)(1・B/A + 20A/B) + (1/2)(20C/B + 17B/C) +(1/2)(1・C/A + 17A/C) - (1+20+17)/2
 ≧ √(1・20) + √(20・17) + √(17・1) - 19  (← AM-GM)
等号成立は A:B:C = √1:√20:√17

585 :132人目の素数さん:2018/05/22(火) 01:36:29.77 ID:+yEhb6+c.net
>>583-584
す、すげぇ…

586 :132人目の素数さん:2018/05/22(火) 05:22:53.47 ID:RuE2vaj6.net
>>486 >>487 (3)

 文献[9] 佐藤(訳) (2013) p.48 演習問題 1.101

・p=1,q=2 の例
 文献[9] 佐藤(訳) (2013) p.48 例 1.6.7 及び p.131 問題 3.30
 チェコ-スロバキアMO-1999

587 :132人目の素数さん:2018/05/22(火) 05:40:43.12 ID:RuE2vaj6.net
>>575 >>576 [192]

一次式:φ(x) = (a+b+c)x−(ab+bc+ca)により、
A = φ(a) = aa-bc,
B = φ(b) = bb-ca,
C = φ(c) = cc-ab.
A - B = (a+b+c)(a-b)、etc.
i)a+b+c≠0 のとき、
 (左辺) = {AA(A-B)(A-C) + BB(B-C)(B-A) + CC(C-A)(C-B)}/(a+b+c)^2 = F_2(A、B、C)/(a+b+c)^2 ≧0、
ii)a+b+c=0 のとき、A=B=C.

 http://www.casphy.com/bbs/highmath/ 不等式2-188

588 :132人目の素数さん:2018/05/22(火) 21:32:13.53 ID:+yEhb6+c.net
一松のじっちゃんが「大学への数学2018年6月号」に不等式の記事を書いておられる。
エルデシュの不等式とか

589 :132人目の素数さん:2018/05/27(日) 01:54:57.05 ID:4AQyIVUB.net
不等式と聞ゐちゃあ捨て置けねゑ…。このためだけに買ってきた。

タイトル 「三角形に関する不等式のいくつか」、4ページ

レムスの不等式と、求角不等式。
内角のcosの等式から、a^2+b^2+c^2 と8R^2の大小関係。
 (中略)
エルデシュの不等式。
過去スレで見たことある不等式。

あと、「老人のグチだが、(中略)近年お数学検定で、不等式の証明問題は成績が悪い傾向が見られる。」
とあるが、検定問題で出題されている不等式を全て公開してほしい。

590 :132人目の素数さん:2018/05/27(日) 05:51:38.11 ID:pjauAWRB.net
ここで不等式解いてる人って50後半の会社員だったりする?

591 :132人目の素数さん:2018/05/27(日) 07:16:00.59 ID:i5aSKt1a.net
>>589

数検専用スレ
http://rio2016.5ch.net/test/read.cgi/math/1512773695/
http://rio2016.5ch.net/test/read.cgi/math/1479394767/
へどうぞ。


〔問題〕
a_n = (1 + 1/n)^n, b_n = (1 + 1/n)^(n+1)   (nは正の整数)
とおくとき、nが増加すると a_n は増加し、b_n は減少することを証明せよ。
 (数検2011-秋-1級-2次-Q2)

592 :132人目の素数さん:2018/05/27(日) 16:46:19.71 ID:i5aSKt1a.net
>>374 >>416 >>417 >>544 >>545 >>547 >>548

[第7章.939,942-944]

593 :132人目の素数さん:2018/05/28(月) 09:56:02.44 ID:TTo2rnUU.net
>>589

(aa+bb+cc) - 8RR = 4RR {sin(A)^2 + sin(B)^2 + sin(C)^2 - 2}
= 4RR {1 - cos(A)^2 - cos(B)^2 - cos(C)^2}
= 8RR cos(A) cos(B) cos(C),

〔補題〕
A+B+C = π のとき
 cos(A)^2 + cos(B)^2 + cos(C)^2 + 2cos(A)cos(B)cos(C) = 1,

〔ライプニッツの不等式〕
 9RR - (aa+bb+cc) = 9(OG)^2 ≧ 0,
 O:外心 G:重心

・文献[9] 佐藤(訳) 朝倉書店 (2013) p.87-89 定理2.4.4 定理2.4.5

594 :132人目の素数さん:2018/05/29(火) 02:45:55.24 ID:n11ck1yy.net
>>591 は相加-相乗平均(AM-GM)で出るらしい。 (出題者・談)

{1,1,…,(n-1)/n} ⇒ a_n > a_{n-1} > … > a_1 = 2

{1,1,…,n/(n-1)} ⇒ b_n < b_{n-1} < … < b_1 = 4

しかし c_n = (1 + 1/n)^(n +1/2) が減少するのを出すのは難しい。
2項定理を使うか?

595 :132人目の素数さん:2018/05/29(火) 03:01:10.20 ID:n11ck1yy.net
>>594

2項定理により、

(1 - 1/nn)^(2n+1) = 1 -2/n + 1/nn -1/(3n^3) +2/(3n^4) -3/(5n^5) +53/(90n^6) -…

 < (1 - 1/n)^2

596 :132人目の素数さん:2018/05/30(水) 08:12:41.15 ID:FdL02lRD.net
>>589
Erdos-Mordell inequality
https://en.wikipedia.org/wiki/Erd%C5%91s%E2%80%93Mordell_inequality

597 :132人目の素数さん:2018/05/30(水) 08:18:45.80 ID:FdL02lRD.net
Crux PROBLEMS
(2012年のが公開された。5年以内のはパスワードがないと見れない)

3690, 3703, 3706, 3709
https://cms.math.ca/crux/v38/n1/Problems_38_1.pdf

3712, 3715, 3719(←破棄)
https://cms.math.ca/crux/v38/n2/Problems_38_2.pdf

3719(←Replacement), 3723, 3726, 3729
https://cms.math.ca/crux/v38/n3/Problems_38_3.pdf

3731, 3735, 3737, 3740
https://cms.math.ca/crux/v38/n4/Problems_38_4.pdf

3741, 3744, 3747, 3749
https://cms.math.ca/crux/v38/n5/Problems_38_5.pdf

3752, 3754, 3757, 3759
https://cms.math.ca/crux/v38/n6/Problems_38_6.pdf

3763, 3767, 3769
https://cms.math.ca/crux/v38/n7/Problems_38_7.pdf

3773, 3774, 3776, 3779
https://cms.math.ca/crux/v38/n8/Problems_38_8.pdf

(3781), 3783, (3784), (3786), 3788, 3789
https://cms.math.ca/crux/v38/n9/Problems_38_9.pdf

3793, 3795, 3797,
https://cms.math.ca/crux/v38/n10/Problems_38_10.pdf

(*゚∀゚)=3ハァハァ

598 :132人目の素数さん:2018/06/01(金) 08:31:09.16 ID:PJfeGZ2B.net
>>597 から

3690.(v38_n1)
 Let a, b, and c be three distinct positive real numbers with a+b+c=s. Show that
  (5xx-6xy+5yy)(a^3+b^3+c^3) + 12(xx-3xy+yy)abc > (x-y)^2・s^3,

3709.(v38_n1)
 Let a, b, and c be non-negative real numbers, k and L≧0 and define
  (a+b)/2 - √ab = k^2,  (a+b+c)/3 - (abc)^(1/3) = L^2.
 Prove that
  max(a,b,c) - min(a,b,c) ≧ (3/2)(k-L)^2.

3712.(v38_n2)
 Prove that for any positive numbers a,b,c
  √{a(aa+bc)/(b+c)} + √{b(bb+ca)/(c+a)} + √{c(cc+ab)/(a+b)} ≧ a+b+c.

3719.(v38_n3,Replacement)
 Prove that if a,b,c>0, then
 a/√{bb+(1/4)bc+cc} + b/√{cc+(1/4)ca+aa} + c/√{aa+(1/4)ab+bb} ≧ 2.

3723.(v38_n3)
 Let a,b,c be positive real numbers such that a+b+c=s. If n is a positive integer, prove that
 (3a)^n /{(b+s)(c+s)} + (3b)^n /{(c+s)(a+s)} + (3c)^n /{(a+s)(b+s)} ≧ (27/16)s^(n-2).

3731.(v38_n4)
 Let a,b,c be positive real numbers such that a+b+c=s. Prove that
  a^(n+1) + b^(n+1) + c^(n+1) ≧ (aa+bb+cc)^n / s^(n-1),
 for all non-negative integers n.

3737.(v38_n4)
 Four non-negative real numbers a,b,c,d are given. Show that
  1/(a^3+b^3) + 1/(b^3+c^3) + 1/(c^3+d^3) + 1/(a^3+c^3) + 1/(b^3+d^3) + 1/(a^3+d^3) ≧ 243/{2(a+b+c+d)^3},
 Equality: {a,b,c,d} = {0,1,1,1}

3741.(v38_n5)
 Find the largest value of a and the smallest value of b for which the inequalties
  ax/(a+xx) < sin(x) < bx/(b+xx)
 hold for all 0<x<π/2.

3744.(v38_n5)
 Let a,b,c be positive real numbers with sum s. Prove that
  (a^8+b^8)/(aa+bb)^2 + (b^8+c^8)/(bb+cc)^2 + (c^8+a^8)/(cc+aa)^2 ≧ (a^3+b^3+c^3-abc)s/4.

3752.(v38_n6)
 Show that if n≧2 is a positive integer then
 (1/2)(1 +1/n -1/nn)^2 < (1 - 1/2^3)(1 - 1/3^3) … (1 - 1/n^3).

Crux mathematicorum, Vol.38 (2012)、一部改作

599 :132人目の素数さん:2018/06/01(金) 08:45:52.63 ID:PJfeGZ2B.net
>>597 から

3763.(v38_n7)
 Let a,b,c be positive real numbers. Prove that
  a/(2a+b+c) + b/(2b+c+a) + c/(2c+a+b) ≦ a/(2b+2c) + b/(2c+2a) + c/(2a+2b).

3793.(v38_n10)
 Let a, b, and c be positive real numbers such that
  √a + √b + √c = 2014/√2.
 Show that
  2014 ≦ √(a+b) + √(b+c) + √(c+a) ≦ 2014√2,
 Equality:(LHS) √a = √b = √c = 2014/(3√2),(RHS) √a = 2014/√2,b=c=0,

・三角形関係

3726.(v38_n3)
 Let a,b,c,s,r,R represent the angles (measured in radians),the semi-perimeter,the in-radius and the circum-radius of a triangle,respectively.
 Prove that
  (A/B + B/C + C/A)^3 ≧ 2ss/(Rr).

3729.(v38_n3)
 If a,b,c are the side lengths of a triangle,prove that
  (b+c)/(aa+bc) + (c+a)/(bb+ca) + (a+b)/(cc+ab) ≦ 3(a+b+c)/(ab+bc+ca).

3757.(v38_n7)
 Let A, B, C be the angles (measured in radians),R the circum-radius and r the in-radius of a triangle.
 Prove that
 1/A + 1/B + 1/C ≦ (9/2π)(R/r).

3767.(v38_n7)
 Let R,r be the circum-radius and in-radius of a right-angled triangle.
 Prove that
  R/r + r/R ≧ 2√2.

3776.(v38_n8) 別名「富士山」
 In △ABC prove that
 tan(A/2) + tan(B/2) + tan(C/2) ≧ (1/2){1/cos(A/2) + 1/cos(B/2) + 1/cos(C/2)}.

Crux mathematicorum, Vol.38 (2012)、一部改作

600 :132人目の素数さん:2018/06/01(金) 11:22:47.44 ID:PJfeGZ2B.net
>>598

3690.
軸を45°回して (x+y)/√2 = u,(x-y)/√2 = v とおく。
5xx-6xy+5yy = 2uu +8vv,
12(xx-3xy+yy) = -6uu +30vv,
(x-y)^2 = 2vv,
これを入れて
(左辺) - (右辺) = (2uu+8vv)(a^3+b^3+c^3) + (-6uu+30vv)abc -2vv(a+b+c)^3
 = 2(a^3+b^3+c^3 -3abc)uu + 6F_1(a,b,c)vv   (←シューア)
 ≧ 0,

>>3723.
通分すると
(分子) = (a+s)(3a)^n + (b+s)(3b)^n + (c+s)(3c)^n
 ≧ (4s/3){(3a)^n + (3b)^n + (3c)^n}  (←チェビシェフ)
 = (4s)(3^n)(a^n + b^n + c^n)/3
 ≧ (4s)s^n,
(分母) = (a+s)(b+s)(c+s) ≦ (4s/3)^3,   (← GM-AM)
(左辺) ≧ (27/16)s^(n-2),

>>3731.
コーシーの拡張より
 (a+b+c)(a+b+c) … (a+b+c){a^(n+1) + b^(n+1) + c^(n+1)} ≧ (aa+bb+cc)^n,
    (n-1)個

601 :132人目の素数さん:2018/06/02(土) 06:34:19.54 ID:qc99k5Fr.net
>>598

3741.
a = ππ/{2(π-2)} = 4.322734721
 b = 6

 cos(t) < 1 を [0,x] で逐次積分すると、
 sin(x) < x,  (x>0)
-cos(x) < -1 + xx/2!,
-sin(x) < -x + (x^3)/3!,  (x>0)
 cos(x) < 1 - xx/2! + (x^4)/4!,
 sin(x) < x - (x^3)/3! + (x^5)/5!
  = {x - ((14-xx)/720)x^5}/(1+xx/6)
  < x/(1+xx/6),   (0<x<π/2)

3752.
 a_n = 1 +1/n -1/nn = (nn+n-1)/nn,
とおく。
 a_n / a_{n-1} = (n-1)^2・(nn+n-1)/{nn(nn-n-1)}
 = 1 - (nn-3n+1)/{nn(nn-n-1)}
 ≦ 1 - 1/(2nn)     (n≧5)

∵ 2(nn-3n+1) - (nn-n-1) = n(n-5) + 3 ≧ 3  (n≧5)

 (a_n / a_{n-1})^2 ≦ {1 - (1/2nn)}^2
 = 1 - 1/nn + 1/(4n^4)
 < 1 - 1/n^3,

602 :132人目の素数さん:2018/06/02(土) 13:03:43.11 ID:bqWw3wOw.net
正整数nと1より大きい正の実数xに対し、
Σ[k=1,n]{kx}/[kx]<Σ[k=1,n]1/(2k-1)
{kx}はkxの小数部分を表し、[kx]はkxの整数部分を表すものとする

603 :132人目の素数さん:2018/06/05(火) 10:01:49.78 ID:RI7aB28L.net
>>599

3763.
(左) HM-AM より
 a/(2a+b+c) ≦ (1/4){a/(a+b) + a/(a+c)},
 b/(2b+c+a) ≦ (1/4){b/(b+c) + b/(b+a)},
 c/(2c+a+b) ≦ (1/4){c/(c+a) + c/(c+b)},
辺々たすと
 (左辺) ≦ 3/4,

(右)
 a/(2b+2c) = (a+b+c)/(2b+2c) - 1/2
 b/(2c+2a) = (a+b+c)/(2c+2a) - 1/2
 c/(2a+2b) = (a+b+c)/(2a+2b) - 1/2
辺々たすと
 (右辺) = (a+b+c){1/(2b+2c) + 1/(2c+2a) + 1/(2a+2b)} - 3/2
 ≧ (a+b+c)・9/{4(a+b+c)} - 3/2 (← AM-HM)
 = 9/4 - 3/2
 = 3/4,      (Nesbitt,Shapiro-3)

604 :132人目の素数さん:2018/06/06(水) 08:56:29.19 ID:xxwxn7ab.net
>>593

〔Chapple - Euler の不等式〕
外接円の半径をR、内接円の半径をrとするとき
 R(R-2r) = OI^2 ≧ 0
 O:外心 I:内心

605 :132人目の素数さん:2018/06/06(水) 17:21:28.63 ID:xxwxn7ab.net
>>602

 x → x+1 とすれば分母が k 増えるので左辺は減少する。1≦x≦2 で考える。
 (m-1)/n ≦ {x} < m/n となるmをとる。
 m = [nx] - n[x] +1, (1≦m≦n)

〔補題〕
 Σ[k=1,n] {kx}/[kx] < Σ[k=1,m-1] 1/(2k-1) + ({x} - (m-1)/n)/(2m-1),

右辺は、(1,0) - (1+1/n,1) - (1+2/n,1+1/3) - …… - (1+m/n,Σ[k=1,m] 1/(2k-1)) - …… (2,Σ[k=1,n] 1/(2k-1)) を結んだ折れ線を表わす。

606 :132人目の素数さん:2018/06/09(土) 21:53:12.92 ID:TZRRIZyQ.net
bot[195]
6(x^3 + y^3 + z^3)^2 ≦ (x^2 + y^2 + z^2)^3

これはシュワちゃんと関係あるん?

607 :132人目の素数さん:2018/06/10(日) 17:09:15.57 ID:KetZUwRK.net
>>606 [195]

x+y+z = 0 より
 x^3+y^3+z^3 = 3xyz,
 xx+yy+zz = [(x-y)^2 + 3zz]/2,
xとyは同符号とすれば
0 ≦ 4xy ≦(x+y)^2 = zz,

(左辺) = 6(3xyz)^2 = 54(xy)(xy)(zz) ≦ (3zz/2)^3 ≦ {[(x-y)^2 +3zz]/2}^3 = (xx+yy+zz)^3.

 蕪湖市数学競技会

608 :132人目の素数さん:2018/06/11(月) 23:56:07.74 ID:1c3kALJq.net
>>607
相加、相乗か! ピコーン!

609 :132人目の素数さん:2018/06/12(火) 00:18:29.69 ID:BRfgTgz+.net
以下、x、y、z∈R とする。

(1) (x^2 + y^2 + z^2)^3 ≧ 6(x^3 + y^3 + z^3)^2
(2) (x^2 + y^2 + z^2)^3 ≧(x^3 + y^3 + z^3 - 3xyz)^2 + (ab+bc+ca)^3
(3) (x^2 + y^2 + z^2)^3 ≧ 2{(x-y)(y-z)(z-x)}^2
(4) 2(x^2 + y^2)(y^2 + z^2)(z^2 + x^2) ≧ {(x-y)(y-z)(z-x)}^2
(5) 合体 or 改造できるかな?

出典
(1) >>606、bot195、蕪湖市数学競技会
(2)(3)(4)は過去に扱ったと思うが、元ネタを記録していないので詳細不明

  ∧_∧
  ( ;´∀`) < むむむ…、我慢できないでござる!
  人 Y /   
 ( ヽ し
 (_)_)

610 :132人目の素数さん:2018/06/12(火) 00:20:38.29 ID:BRfgTgz+.net
詳細不明…、不明デスか? あなた…、怠惰デスネ!

611 :132人目の素数さん:2018/06/12(火) 00:44:08.66 ID:BRfgTgz+.net
>>609 の続き

(6) (x^2 + y^2 + z^2)^3 ≧ 8(x^2 - yz)(y^2 - zx)(z^2 - xy)

612 :132人目の素数さん:2018/06/12(火) 16:00:37.51 ID:YFJLrlqV.net
>>609

(1) x+y+z=0 のとき、…

(2)
 xx+yy+zz = S2,xy+yz+zx = t,
とおく。
 S2 - t = {(x-y)^2 + (y-z)^2 + (z-x)^2}/2 ≧ 0,
(S2)^3 - t^3 = {(S2)^2 + S2・t +tt}(S2-t)
 ≧ {(S2)^2 + S2・t - 2tt}(S2-t)
 = (S2+2t)(S2-t)^2
 = (x+y+z)^2・{(xx+yy+zz) -(xy-yz-zx)}^2
 = (x^3+y^3+z^3 -3xyz)^2,

(3)
 yはxとzの中間にあるとしてよい。
 0 ≦ (x-y)(y-z) ≦ (1/4)(x-z)^2,
 xx+yy+zz = (1/2)(x+z)^2 + (1/2)(x-z)^2 + yy ≧ (1/2)(x-z)^2,
 (左辺) ≧ (1/8)(x-z)^6 ≧ 2(x-z)^2 {(x-y)(y-z)}^2 = (右辺),

613 :132人目の素数さん:2018/06/14(木) 22:38:12.04 ID:6IyvuHHw.net
Asia Pacific Mathematical Olympiad APMO 2004
でググって5番目あたりに出てくるPDFの Problem 5。

模範解答がワケワカメ…。
これより強い不等式を、前スレでやったような排気ガス…

614 :132人目の素数さん:2018/06/15(金) 00:29:19.87 ID:UYSEHwOg.net
数学セミナーエレガントな解法2月号にある不等式の問題の正解率が異様に低かったらしい
そもそも問題すら理解してない回答が多かったって講評だった

615 :132人目の素数さん:2018/06/15(金) 02:05:18.57 ID:mm39PC7P.net
>>609
(4)
(1-i)(x+iy)(y+iz)(z+ix) = (1-i){-(xyy+yzz+zxx-xyz) +i(xxy+yyz+zzx-xyz)}
= -(x-y)(y-z)(z-x) +i{(x+y)(y+z)(z+x)-4xyz},
絶対値の2乗をとって
 2(xx+yy)(yy+zz)(zz+xx) = {(x-y)(y-z)(z-x)}^2 + {(x+y)(y+z)(z+x) -4xyz}^2,

>>613
 [前スレ.456]
(abc)^2 +aa +bb +cc +2 -2(ab+bc+ca)≧ 0 を使う?
文献[9] 佐藤(訳)、問題3.85改、練習問題1.90(i)

616 :132人目の素数さん:2018/06/15(金) 02:30:14.62 ID:d1fXPxbR.net
なるほど!

>>613
x、y、z∈R のとき、(aa+2)(bb+2)(cc+2) ≧ 9(ab+bc+ca)

[前スレ.456]
x、y、z∈R のとき、(aa+2)(bb+2)(cc+2) ≧ 3(a+b+c)^2

合体!
(aa+2)(bb+2)(cc+2) ≧ 3(a+b+c)^2 ≧ 9(ab+bc+ca)

617 :132人目の素数さん:2018/06/15(金) 02:32:47.15 ID:mm39PC7P.net
>>609 (4) >>615
 s = x+y+z,
 t = xy+yz+zx,
 u = xyz,
  = (x-y)(y-z)(z-x),
で表わせば
 2(ss-2t)(tt-2su) -2uu = 刧 + (st-5u)^2,

618 :132人目の素数さん:2018/06/15(金) 02:38:30.76 ID:d1fXPxbR.net
左辺を見て、昨夏の不等式三昧の夜を思い出す ( ゚∀゚) ウヒョッ!

[前スレ.469前後]
x、y、z∈R 、k≧0 のとき、(aa+k)(bb+k)(cc+k) ≧ (3kk/4)*(a+b+c)^2 などなど…

619 :132人目の素数さん:2018/06/15(金) 02:40:06.93 ID:d1fXPxbR.net
>>614
もう最新号出る時期か。よし明日読みに行こう。

620 :132人目の素数さん:2018/06/17(日) 01:22:10.06 ID:8Ln3gkjC.net
立ち読みで疎覚えだが、数蝉NOTE。

a、b、c >0、a+b+c=1 のとき、Σ[cyc] a/(b^2+bc+c^2) ≧3.

621 :132人目の素数さん:2018/06/17(日) 01:33:38.57 ID:lI+JiKnS.net
>>619

〔Igarashi の不等式〕
 a,b,c>0 のとき、
 a/(bb+bc+cc) + b/(cc+ca+aa) + c/(aa+ab+bb) ≧ (a+b+c)/(ab+bc+ca) ≧ 3/(a+b+c),
 2018年7月号NOTE

(略証)
 a ' = bb + bc + cc,
 b ' = cc + ca + aa,
 c ' = aa + ab + bb,
とおくと
 aa ' + bb ' + cc ' = (a+b+c) (ab+bc+ca),  … これがミソ(?)
コーシーにより
 (左辺) = a/a ' + b/b' + c/c' ≧ (a+b+c)^2 /(aa ' + bb ' + cc ') = (a+b+c)/(ab+bc+ca),

622 :132人目の素数さん:2018/06/17(日) 01:44:34.71 ID:8Ln3gkjC.net
>>621
おお、これだ。さんくす。
解説でZZZが一般化してたけど、なんかよく分からなかった…。

623 :132人目の素数さん:2018/06/17(日) 01:45:36.54 ID:lI+JiKnS.net
>>620 >>621
 被りました。

 f(x) = 1/x は下に凸だから、Jensenにより
 (左辺) = a f(a ') + b f(b ') + c f(c ')
  ≧ (a+b+c) f((aa'+bb'+cc')/(a+b+c))
  = (a+b+c) f(ab+bc+ca)
  = (a+b+c)/(ab+bc+ca),

624 :132人目の素数さん:2018/06/17(日) 01:46:16.03 ID:8Ln3gkjC.net
>>621
>  a ' = bb + bc + cc,
>  b ' = cc + ca + aa,
>  c ' = aa + ab + bb,
> とおくと
>  aa ' + bb ' + cc ' = (a+b+c) (ab+bc+ca),  … これがミソ(?)

この変形は初めて見た。コレクションに入れておこう。

625 :132人目の素数さん:2018/06/17(日) 01:48:52.60 ID:8Ln3gkjC.net
あと一松じっちゃんの不等式の解説で、s(2(s^2-2t)-5t)+27u の因数分解があったような。
立ち読みだったんで s、t、u で覚えて帰ったから怪しいが…。
手計算で因数分解しようとして挫折した。 手計算でできるのか?

626 :132人目の素数さん:2018/06/17(日) 02:42:52.98 ID:lI+JiKnS.net
>>625

(b+c-2a)(c+a-2b)(a+b-2c) = (s-3a)(s-3b)(s-3c) = -2s^3 +9st -27u,
を使うでござる。

エレ解スレ【2016.11】
http://rio2016.5ch.net/test/read.cgi/math/1476702312/785-786

627 :132人目の素数さん:2018/06/18(月) 22:44:29.14 ID:wEh7fB1P.net
>>622
Nesbittと合体したでござるか…

〔Nesbitt-Igarashi の不等式〕
 a,b,c>0 のとき、
 (a+b+c) {a/(bb+bc+cc) + b/(cc+ca+aa) + c/(aa+ab+bb)}
 ≧ 2 {a/(b+c) + b/(c+a) + c/(a+b)}
 ≧ (a+b+c)^2 /(ab+bc+ca)
 ≧ 3,
 数セミ、2018年7月号NOTE-改

628 :132人目の素数さん:2018/06/19(火) 02:28:33.39 ID:8eLVrD8z.net
>>614
よく分からぬ難しげな不等式で、攻めづらかったかも。
この式が出てきた背景は、解説で触れていたけれど。

629 :132人目の素数さん:2018/06/19(火) 02:47:35.19 ID:/rZEmPAN.net
うむ、よう分からなんだ。

630 :132人目の素数さん:2018/06/19(火) 05:03:35.48 ID:/rZEmPAN.net
botの101の問題、分かりますか?

631 :132人目の素数さん:2018/06/19(火) 13:57:17.17 ID:8eLVrD8z.net
>>630 [101]

a〜d>0、a+b+c+d-1=0 のとき
 6(a^3 + b^3 + c^3 + d^3) ≧ aa+bb+cc+dd + 1/8.
 フランス TeamSelectionTest-2007 Q.2

(略解)
 f(x) = 6x^3 - (xx + 1/32)
 = (5/8)(x-1/4) + 2(3x+1)(x-1/4)^2
 ≧ (5/8)(x-1/4),
より
 f(a) + f(b) + f(c) + f(d) ≧ (5/8)(a+b+c+d-1) = 0.

{x = 1/4 で接線を曳く。f '(1/4) = 5/8}

632 :132人目の素数さん:2018/06/19(火) 17:30:30.95 ID:/rZEmPAN.net
>>631
さんくす。4月から見てるけど、101だけ出てこないのだ。
画像のない192は頻繁に出てくるのにな。偏りすぎている。

633 :132人目の素数さん:2018/06/20(水) 02:26:05.78 ID:ZoYl55O4.net
>>632 [192]
 
任意の実数a,b,cに対し、
 (a-b)(a-c)(aa-bc)^2 + (b-c)(b-a)(bb-ca)^2 + (c-a)(c-b)(cc-ab)^2 ≧ 0,
を示せ。
 casphy! - highmath(高校数学) - 不等式2-188
 じゅー君が高校生のとき作ったヤツ(?)

(略証)
i)a+b+c≠0 のとき、
 A = aa-bc,B = bb-ca,C = cc-ab,
とおくと
 A-B = (a+b+c)(a-b),etc.
(左辺) = {AA(A-B)(A-C)+BB(B-C)(B-A)+CC(C-A)(C-B)}/(a+b+c)^2
 = F_2(A,B,C)/(a+b+c)^2  (←シューア)
 ≧0,
ii)a+b+c=0 のとき、
 A = B = C,
 (左辺) = AA F_0(a、b、c) ≧ 0.
これで ☆9 だって。

634 :132人目の素数さん:2018/06/20(水) 23:16:58.87 ID:ZoYl55O4.net
>>613 >>615

〔補題〕
a,b,c≧0 のとき
(abc)^2 +aa +bb +cc +2 -2(ab+bc+ca)≧ 0,

(略証)
a = A^(3/2),b = B^(3/2),c = C^(3/2) とおくと
 (abc)^2 + 2 -3ABC = (ABC)^3 +1 +1 -3ABC ≧ 0,  (←AM-GM)
 A(A-B)(A-C) + B(B-C)(B-A) + C(C-A)(C-B) = F1(A,B,C) ≧ 0,
 AB(A+B) -2ab = AB(√A - √B)^2 ≧ 0,etc.
辺々たす。

635 :132人目の素数さん:2018/06/22(金) 04:52:05.21 ID:5dKvywCX.net
>>634
 >>529 ( Suranyi-3, >>512 >>513  を使った) からも出る…

>>555
 >>549 〔問題18〕は [204] でござった ...orz

636 :132人目の素数さん:2018/06/22(金) 11:12:57.72 ID:mDZvFtTn.net
不等式に関する研究
https://repository.kulib.kyoto-u.ac.jp/dspace/handle/2433/100782

ちと古いがな。他にないかな?

637 :132人目の素数さん:2018/06/23(土) 07:31:00.50 ID:NONJ6Zo1.net
>>611
これはどう証明するのですか?

638 :132人目の素数さん:2018/06/23(土) 18:12:52.09 ID:BnO9HX6O.net
〔問題677〕

Pを凸多面体とし、Pの辺を L_1,L_2,…,L_n とする。
各 1≦i≦n について L_i を辺にもつPの2つの面を考え、
その2つの面のなす角を外側から測ったものを θ_i とする。
(2面の外向き法線のなす角。2面角)

このとき、Σ[i=1,n] θ_i ≧ 3π であることを示せ。

JMO夏季セミナー
http://jmoss.jp/mon/old.php → 第9回 (G,入江)

面白スレ26-677

639 :132人目の素数さん:2018/06/23(土) 22:49:56.64 ID:BnO9HX6O.net
[213]
正の実数列 {a_k} が各自然数kに対して
a_{k+1} ≧ k・a_k / {(a_k)^2 + (k-1)}
を満たすとする。すべての n≧2 に対して
a_1 + a_2 + … + a_n ≧ n,
を示せ。
 IMO Shortlist 2015 A.2 ☆2

640 :132人目の素数さん:2018/06/23(土) 23:13:45.08 ID:BnO9HX6O.net
>>639 [213]
nについての帰納法による。

・n=2 のとき
 a_1 + a_2 ≧ a_1 + 1/a_1 ≧ 2  (← AM-GM)

・n>2 のとき
 a_n ≧1 のときは明らかに成立つ。
 a_n ≦1 のとき 題意より
 k/a_{k+1} ≦ (k-1)/a_k + a_k,
 a_k ≧ k/a_{k+1} - (k-1)/a_k,
 k=1,…,n-1 でたす。
 a_1 + a_2 + … + a_{n-1} ≧ (n-1)/a_n,
 a_1 + a_2 + … + a_n ≧ (n-1)/a_n + a_n
  = n + (n-1 - a_n)(1 - a_n)/a_n
  ≧ (n-2) + 1/a_n + a_n
  ≧ n,   (← 0 < a_n ≦1)

641 :132人目の素数さん:2018/06/23(土) 23:13:45.10 ID:BnO9HX6O.net
>>639 [213]
nについての帰納法による。

・n=2 のとき
 a_1 + a_2 ≧ a_1 + 1/a_1 ≧ 2  (← AM-GM)

・n>2 のとき
 a_n ≧1 のときは明らかに成立つ。
 a_n ≦1 のとき 題意より
 k/a_{k+1} ≦ (k-1)/a_k + a_k,
 a_k ≧ k/a_{k+1} - (k-1)/a_k,
 k=1,…,n-1 でたす。
 a_1 + a_2 + … + a_{n-1} ≧ (n-1)/a_n,
 a_1 + a_2 + … + a_n ≧ (n-1)/a_n + a_n
  = n + (n-1 - a_n)(1 - a_n)/a_n
  ≧ (n-2) + 1/a_n + a_n
  ≧ n,   (← 0 < a_n ≦1)

642 :132人目の素数さん:2018/06/24(日) 04:02:09.24 ID:BW6lbwPs.net
>>632 [101]

25分前に出ますた。

643 :132人目の素数さん:2018/06/24(日) 05:10:52.64 ID:dz2BpZ6O.net
>>642
きたか…!!

  ( ゚д゚ ) ガタッ
  .r   ヾ
__|_| / ̄ ̄ ̄/_
  \/    /

644 :132人目の素数さん:2018/06/24(日) 05:15:05.23 ID:dz2BpZ6O.net
あとは消失した192を作り直してもらうことと、224問目以降を作ってもらうことだな

645 :132人目の素数さん:2018/06/25(月) 23:38:45.92 ID:qOAzU6BU.net
>>611 >>637

基本対称式を x+y+z = s,xy+yz+zx = t,xyz = u とおく。
 xx-yz = xs-t,yy-zx = ys-t,zz-xy = zs-t,
より
 (左辺) - (右辺) = (ss-2t)^3 - 8(xs-t)(ys-t)(zs-t)
 = (ss-2t)^3 - 8(us^3 - t^3)
 = ss{(ss-3t)^2 + (8/3)(tt-3su) + (1/3)tt}
 ≧ 0,
等号成立は x+y+z = 0.

646 :132人目の素数さん:2018/06/25(月) 23:48:14.02 ID:qOAzU6BU.net
>>611 の〔類題〕

x,y,z ∈ R のとき
-(35+13√13)/486 ≦ (xx-yz)(yy-zx)(zz-xy)/(xx+yy+zz)^3 ≦ 1/8,
 -0.1684612481

 左側等号は (x,y,z) = ((3-√13)/2,1,1) など。  -0.302775637732

647 :132人目の素数さん:2018/06/29(金) 11:42:15.93 ID:kgKL/5Ht.net
正の実数a,b,cはa+b+c=3を満たす。このとき、
1/(2+aa+bb)+1/(2+bb+cc)+1/(2+cc+aa)≦3/4

2009 イランTST

648 :132人目の素数さん:2018/06/30(土) 11:28:21.01 ID:ApWhDcRo.net
>>609
(1) は x+y+z=0 の条件があるから、一緒にまとめるべきではなかったね。

649 :132人目の素数さん:2018/07/01(日) 11:44:56.53 ID:o+nodY1/.net
>>647

左辺を f(a,b,c) とおく。
1≦c とし、(a+b)/2 = (3-c)/2 = m とおく。
 f(a,b,c) ≦ f(m,m,c) ≦ 3/4
を示す。

(左)
aa+bb ≧ 2mm より
1/(2+aa+bb) = 1/{2 +2mm +(1/2)(a-b)^2} ≦ 1/(2+2mm),
1/(2+cc+bb) + 1/(2+cc+aa) = 2{2+cc+(aa+bb)/2}/{(2+cc+bb)(2+cc+aa)}
 ≦ 2/(2+cc+mm),
∵ (2+cc+bb)(2+cc+aa) -(2+cc+mm){2+cc+(aa+bb)/2}
 = (1/4)(a-b)^2 (2+cc-3mm) + (1/16)(a-b)^4
 = (1/4)(a-b)^2 {2+cc-(3/4)(3-c)^2} + (1/16)(a-b)^4
 = (1/32)(a-b)^2 (19+c)(c-1) + (1/16)(a-b)^4
 ≧ 0,   (← c≧1)

(右)
 f(m,m,c) = 1/(2+2mm) + 2/(2+cc+mm)
 = (3/4){1 - (c-1)^2・(5cc-26c+37)/[8(2+2mm)(2+cc+mm)] }
 ≦ 3/4.

650 :132人目の素数さん:2018/07/01(日) 14:00:30.20 ID:AVymxtb0.net
実数x_1,x_2,…,x_nに対して次の不等式が成立することを示せ
Σ[i,j=1,n]|x_i+x_j|≧nΣ[i=1,n]|x_i|

2006 イランTST

651 :132人目の素数さん:2018/07/02(月) 01:07:30.65 ID:dZnBLmxp.net
>>649
 m ≦ 1 ≦ c より
 2+cc-3mm ≧ 0,

652 :132人目の素数さん:2018/07/02(月) 07:40:46.47 ID:dZnBLmxp.net
>>650

x_1, x_2, …, x_p > 0,
x_{p+1}, …, x_n ≦ 0, とする。(0≦p≦n)

(左辺) = Σ[i,j=1,p] |x_i+x_j| + Σ[i,j=p+1,n] |x_i+x_j| + Σ[i=1,p][j=p+1,n] |x_i+x_j|
= Σ[i,j=1,p] (|x_i|+|x_j|) + Σ[i,j=p+1,n] (|x_i|+|x_j|) + Σ[i=1,p][j=p+1,n] |x_i+x_j|
= 2p S_p + 2(n-p) S_n + 2S~,
ここに
 S_p = Σ[i=1,p] |x_i|, S_n = Σ[j=p+1,n] |x_j|, S~ = Σ[i=1,p][j=p+1,n] |x_i+x_j|,
とおいた。

・p = n/2 のときは成立する。(S~≧0)

・0 ≦ p < n/2 のとき
 S~ ≧ Σ[i=1,p][j=p+1,n] (|x_i|-|x_j|) = (n-p) S_p - p S_n,
 0 < (n-2p)/(n-p) ≦ 1 を掛けて
 S~ ≧ {(n-2p)/(n-p)}S~ ≧ (n-2p){S_p - [p/(n-p]S_n},
 (左辺) ≧ n S_p + {n + (n-2p)^2 /(n-p)}S_n ≧ n(S_p + S_n),

・n/2 < p ≦ n のとき
 S~ ≧ Σ[i=1,p][j=p+1,n] (|x_j|-|x_i|) = -(n-p) S_p + p S_n,
 0 < (2p-n)/p ≦ 1 を掛けて
 S~ ≧ {(2p-n)/p}S~ ≧ (2p-n){-[(n-p)/p]S_p + S_n},
 (左辺) ≧ {n + (2p-n)^2 /p}S_p + n S_n ≧ n(S_p + S_n),

653 :132人目の素数さん:2018/07/02(月) 16:23:14.90 ID:dZnBLmxp.net
>>652 訂正

はじめの方で
(左辺) = … + … + 2Σ[i=1,p][j=p+1,n] |x_i+x_j| 
の係数2が抜けてました。(後の論証に影響ないと思いますが…)

654 :132人目の素数さん:2018/07/03(火) 11:38:56.42 ID:F6g7HQZx.net
>>652
混乱しているので修正

(左辺) = 2p S(+) + 2(n-p) S(-) + 2S~,
ここに
 S(+) = Σ[i=1,p] |x_i|, S(-) = Σ[j=p+1,n] |x_j|, S~ = ……
とおいた。

結論は
 (左辺) ≧ …… ≧ n{S(+) + S(-)},

655 :132人目の素数さん:2018/07/05(木) 18:50:30.31 ID:da/jl28d.net
非負実数a,b,c,dと1≦p≦2なる実数pに対して、次の不等式が成立することを示せ
(a+b)^p+(c+d)^p+(a+c)^p+(b+d)^p≦a^p+b^p+c^p+d^p+(a+b+c+d)^p

656 :132人目の素数さん:2018/07/05(木) 18:54:49.03 ID:da/jl28d.net
>>655
修正
p≧2

657 :132人目の素数さん:2018/07/05(木) 21:25:46.65 ID:lZ8d7PQt.net
デジャヴを感じる

658 :132人目の素数さん:2018/07/06(金) 08:14:08.28 ID:Fbh8MKIz.net
>>37(1) >>40 >>41 >>44

〔Redhefferの不等式〕
a_1 〜 a_n >0 のとき
G_k = (a_1・a_2…a_k)^(1/k) とおくと
G_1 + G_2 + …… + G_n ≦ Σ[k=1,n] (1+1/k)^k・a_k - n・G_n,

 和書[3] (大関, 1987) p.114-115 例題3
 文献 Ray Redheffer: Proc. London Math. Soc., Vol. s3-17, Iss. 4, p.683-699 (1967/Oct)
    "Recurrent inequalities"

659 :132人目の素数さん:2018/07/13(金) 09:38:28.60 ID:/EP6VcDe.net
>>658

(G_{k-1},G_{k-1},…,G_{k-1},(1+1/k)^k・a_k)のk個ででAM-GM する。
  (k-1)個

 (k+1)G_k - (k-1)G_{k-1} ≦ (1+1/k)^k・a_k,

k=1〜n でたす。(便宜上、G_0=0)

660 :132人目の素数さん:2018/07/14(土) 06:57:00.45 ID:bqEthaWH.net
Math Lovers 数学を愛する会 @Math_Beautiful_ (2017/12/20 00:26:49)
半径rの円に外接する円の面積をSとしたとき、以下が成立。
https://pbs.twimg.com/media/DRa1WNUVoAAgNXE.jpg
http://twitter.com/Math_Beautiful_/status/943140572276985856

661 :132人目の素数さん:2018/07/14(土) 22:00:02.78 ID:fIrZynJm.net
>>660

「円に外接する三角形の面積だろ!」
とかツッコミたくないが。
 
その場合は
 a = {cot(B/2)+cot(C/2)} r/2 などより、
S = {cot(A/2)+cot(B/2)+cot(C/2)} rr
 ≧ 3cot((A+B+C)/6) rr  (←下に凸)
 = 3cot(π/6) rr
 = (3√3) rr,

662 :132人目の素数さん:2018/07/15(日) 22:13:34.38 ID:8ME/vsb7.net
>>609 (2)
>>612 (2)

 [x,y,z] [x,z,y] [S2,t,t]
 |z,x,y| |y,x,z| = |t,S2,t|
 [y,z,x] [z,y,x] [t,t,S2]
の行列式は
 D(x,y,z)^2 = D(S2,t,t).
ここに
 D(x,y,z) = x^3 + y^3 + z^3 -3xyz
 = (x+y+z)(xx+yy+zz-xy-yz-zx)
 = (x+y+z)(S2-t).

663 :132人目の素数さん:2018/07/16(月) 02:00:56.06 ID:Dv9n2PFO.net
>>609 (3) [182](1)

 大数宿題 - 2013 Q.5
[第7章].114[2](1)、116
Casphy! - higmath - 不等式2 - 170

664 :132人目の素数さん:2018/07/17(火) 14:27:57.12 ID:s0zcFt1B.net
( ゚∀゚)つ https://tohoku.repo.nii.ac.jp/?action=pages_view_main&active_action=repository_view_main_item_detail&item_id=41283&item_no=1&page_id=33&block_id=38

665 :132人目の素数さん:2018/07/17(火) 23:51:33.85 ID:GyPvcBOe.net
>>662
F(x,y,z) は既約かつ対称な多項式で
 F(x,y,z)^2 = F(xx+yy+zz,xy+yz+zx,xy+yz+zx)
を満たすとする。

F(x,y,z) = x+y+z,
F(x,y,z) = xx+yy+zz -xy-yz=zx,
以外にも解があるかな。

666 :132人目の素数さん:2018/07/18(水) 22:49:25.94 ID:QRQo+1y+.net
>>655 >>656

f(x) = x^(p-1) とおくと、
x>0 で f '(x) = (p-1)x^(p-2) > 0, f "(x) = (p-2)(p-1)x^(p-3) ≧0.

f"(x) ≧ 0(下に凸)だから、(*)
 f(a+b) + f(a+c) ≦ f(a) + f(a+b+c),
 f(a+b) + f(b+d) ≦ f(b) + f(a+b+d),
 f(a+c) + f(c+d) ≦ f(c) + f(a+c+d),
f(b+d) + f(c+d) ≦ f(d) + f(b+c+d),
各式に a,b,c,d を掛けて足す。
f '(x) >0(単調増加)を使うと
 g(a+b) + g(c+d) + g(a+c) + g(b+d) ≦ g(a)+g(b)+g(c)+g(d) + g(a+b+c+d),
ここに g(x) = x・f(x)

(略証)
 0 < ∫[0,b]∫[0,c] f "(a+u+v) du dv
  = f(a+b+c) + f(a) - f(a+b) - f(a+c),

667 :132人目の素数さん:2018/07/24(火) 00:24:54.43 ID:jKOHqerG.net
一辺の長さが1である辺を奇数個もつ任意の多角形の面積をSとすると次の不等式が成立
S≧√3/4

668 :132人目の素数さん:2018/07/24(火) 03:36:50.43 ID:6NVSU4Ku.net
>>667
すべての辺の長さが1である、奇数角形?

669 :132人目の素数さん:2018/07/24(火) 03:44:42.26 ID:6NVSU4Ku.net
〔問題2018〕
a>0,b>0,c>0,a+b+c=3 のとき次を示せ。

 a^(1/2018) + b^(1/2018) + c^(1/2018) + (2/√3) √{a(1-b)+b(1-c)+c(1-a)} ≧ 3,

 (K. Chikaya, 2018/June/19)
すうじあむ //suseum.jp/gq/question/2884 を改良
 casphy.com/bbs/highmath/472060/ 不等式2-304

670 :132人目の素数さん:2018/07/24(火) 06:57:15.83 ID:jKOHqerG.net
>>668
多角形のすべての辺のうち一辺の長さ1である辺の総数は奇数個であるもの

671 :132人目の素数さん:2018/07/24(火) 07:01:13.36 ID:jKOHqerG.net
>>668
偶数角形でもいい
例としては四角形のうち三つの辺が長さ1で他は長さ1ではないものだったり、一つの辺のみが1でほかは長さ1でないようなもの

672 :132人目の素数さん:2018/07/24(火) 07:44:00.98 ID:bmjGlIcJ.net
凸とは限らない3角形または4角形または5角形または……
であって辺の長さはすべて1であるもの
ですね。凸とは限ってないので内角が180°も桶なので>>697でもいいけど “凸とは限らない” がある方が良かったかも。

673 :132人目の素数さん:2018/07/25(水) 00:53:28.01 ID:9UVBQKrl.net
>>671

> 偶数角形でもいい

辺長1の正N角形の、連続する2m個の頂点を結んでできる凸2m角形を考える。(N ≫ m^3)

外接円の半径は R = 1/{2sin(π/N)},

S < (弓形の面積)
 = (扇形の面積) - (三角形の面積)
 = (1/2)RR{(4mπ/N) - sin(4mπ/N)}
 < (1/12)RR(4mπ/N)^3    (*)
 = 1/{48sin(π/N)^2}(4mπ/N)^3
 〜 (4/3) m^3 (π/N)
 → 0  (N→∞)

*) x>0 のとき x - (1/6)x^3 < sin(x) < x,

674 :132人目の素数さん:2018/07/25(水) 01:35:01.94 ID:9UVBQKrl.net
>>673

訂正スマソ
 (4mπ/N) → (2(2m-1)π/N)

或いは
(弦の長さ) < (2m-1)
(幅) = R {1-cos((2m-1)π/N)}
 < R (1/2) {(2m-1)π/N}^2   (**)
 = 1/{4sin(π/N)} {(2m-1)π/N}^2
 〜 (1/4)(2m-1)^2 (π/N)
 → 0   (N→∞)

**) 1 - (1/2)xx < cos(x) ≦ 1

675 :132人目の素数さん:2018/07/25(水) 12:53:08.48 ID:aSG76bcm.net
問題の見栄え良くするために、問題文はしょりすぎなんだよ。
偶数角形でもいいといってるのは例えば四角形ABCDで
AB=BC=CD=1、DA=2でもいいって意味だろ?
あくまで辺の長さの和は奇数。
この場合は五角形ABCDEでAE=DE=1、角Eは180°とみなして
1辺の長さ1の5角形とみなす。
そういう場合、面積は√3/4より大きくなる。

偶数角形で辺の長さ1で反例出したいなら平たいひし形で終わり。

676 :132人目の素数さん:2018/07/25(水) 13:10:46.86 ID:ia9tQLFJ.net
で結局問題は>>667でいいの?
真偽は別としてこれだけで問題の条件は十分伝わるよね
勝手に凸がどうたらって条件を加えてる>>672は別の問題ってことでいいの?

677 :132人目の素数さん:2018/07/25(水) 14:13:22.87 ID:h4vuL6tT.net
>>675 辺の長さの和が奇数とは書いてない。そもそも辺の長さは整数とは限らないし

[667(元問題)] 多角形Pは次の条件を満たすとき S >= sqrt(3) / 4
【条件】Pの辺のうち長さが1であるものは奇数個
[672] すべての辺が1である多角形Pの面積は S >= sqrt(3)/4
(凸の条件が何を言ってるのかよくわからない)
[673-4] 「辺が1の正多角形Pの面積は S >= sqrt(3)/4」を否定する証明(たぶん)

なんかごちゃごちゃしたけど問題は667でいいんだよね

678 :132人目の素数さん:2018/07/25(水) 15:17:43.00 ID:9UVBQKrl.net
>>675 >>677
 すべての辺の長さを自然数に限定?
 >>673 >>674 の例で、辺長1を固定しつつ端の2点を持って引っぱると、
 自然数にならぬか…

>>675
 菱形だと長さ1の辺が4つになる。 3辺長を1に固定して1点をずらす。

679 :132人目の素数さん:2018/07/25(水) 17:44:43.59 ID:LF80ki5t.net
点列P0.‥Pnは以下を満たす。
・nは奇数、P0=Pn
・隣接2点間の距離は1
・点列を順に結んで得られる曲線は単純閉曲線C
この時、Cで囲まれる領域の面積は√3/4以上であることを示せ。
ですな

680 :132人目の素数さん:2018/07/25(水) 18:45:08.16 ID:ia9tQLFJ.net
>>679
それは667と別問題だよね
それも成り立つの?

681 :132人目の素数さん:2018/07/25(水) 19:07:44.57 ID:LF80ki5t.net
>>667
これは成り立つ。
私は>>667もこの意味だと思う。
長さ1の辺が奇数個でそうでない辺がいくらあっても桶
みたいな設定で何かいえると思えない。

682 :132人目の素数さん:2018/07/25(水) 20:08:15.16 ID:ia9tQLFJ.net
>>681
問題が間違えてるってことね
679っぽい状況は数オリ辞典かなんかで見た記憶あるけど思い出せない

683 :132人目の素数さん:2018/07/25(水) 21:16:48.71 ID:bivqXzWq.net
続けたまえ

684 :132人目の素数さん:2018/07/26(木) 00:22:16.92 ID:ZQZlAKvh.net
>>679

すべての辺の長さが1である、奇数角形  >>668
ですね。

685 :132人目の素数さん:2018/08/03(金) 04:53:25.06 ID:tRRMlHHD.net
>>520 (B3) [100]
 49th IMO spain 2008, SL-A7

s = a+b+c+d,
p = s+a+c,
q = s+b+d,
M = (s-d)(s-b) = (s+a+c)s + bd,
N = (s-a)(s-c) = (s+b+d)s + ac,
W = (b+d)M-(a+c)N = bd(b+d) - ac(a+c),   …(3)
とおく。
2(左辺) = p(a-c)^2 /M + 3(a-c)(b-d)W/MN + q(b-d)^2 /N,
これは a-c,b-d の斉2次式なので、判別式(Hessian)を調べる。
pq = 2ss + (a+c)(b+d) > 2ss,
MN = {(s+a+c)s+bd} {(s+b+d)s+ac}
 = (s+a+c)(s+b+d)ss + ac(s+a+c)s + bd(s+b+d)s + abcd
 > 2s^4 + 2ac(a+c)s + 2bd(b+d)s,  (← s>a+c,s>b+d)
辺々掛けて
4pqMN > 8ssMN
 > 16(s^3){s^3 + ac(a+c)+bd(b+d)}
 > 192{ac(a+c)+bd(b+d)}^2   {← s^3 > 3ac(a+c)+3bd(b+d)}
 > 192{bd(b+d)-ac(a+c)}^2
 = 192WW
 ≧ 9WW.
∴ 判別式(Hessian) < 0
∴ 正定値。

http://www.imo-official.org/problems/IMO2008SL.pdf
 IMO-2008, SL-A7, Solution-2

686 :132人目の素数さん:2018/08/03(金) 05:18:52.07 ID:tRRMlHHD.net
>>685 訂正

M = (s-d)(s-b) = (a+c)s + bd,
N = (s-a)(s-c) = (b+d)s + ac,

MN = {(a+c)s+bd} {(b+d)s+ac}
 > {ac(a+c) + bd(b+d)}s,

4pqMN > 8ssMN
 > 8(s^3){ac(a+c) + bd(b+d)}
 > 8ac(a+c)^4 + 8bd(b+d)^4
 ≧ 32{ac(a+c)}^2 + 32{bd(b+d)}^2
 > 32{bd(b+d) - ac(a+c)}^2
 = 32WW
 ≧ 9WW,

687 :132人目の素数さん:2018/08/12(日) 09:29:55.12 ID:QnRFj99l.net
〔問題670〕
nを自然数、xを実数とするとき
 [nx] ≧ Σ(k=1,n) [kx]/k
を示せ。ただし [x] はガウス記号である。

[面白スレ26-670,同27-144]

688 :132人目の素数さん:2018/08/19(日) 09:59:40.08 ID:nbZP1PKM.net
すうじあむの解答見てきたけど、h(t)の最大値を求めるところまでは分かった。
で、h(t)の最大値がf(x,y)の最大値になるのは明らかなん?
バラバラに動く変数を1変数に置き換えたものを調べて間違いないん?

689 :132人目の素数さん:2018/08/19(日) 10:10:32.25 ID:FG4Iu3mB.net
何の問題

690 :132人目の素数さん:2018/08/19(日) 17:14:39.44 ID:oIedIwUK.net
>>688
f(x,y) の最大値 ≦ h(t) の最大値 = e^{-2} で
f(1,1) = e^{-2}.
からそう結論しました。
もしも反例が見つかったら晒してください。遠慮はいりません。

>>689
たぶんこれ。
Find the maximum value of the following function for all positive real numbers x,y.
f(x,y) = e^(-x-y) {ln(x)+ln(y)+1}.

http://suseum.jp/gq/question/2901

691 :132人目の素数さん:2018/08/29(水) 09:51:11.01 ID:4VhTCH8s.net
これ誰か教えて
http://suseum.jp/gq/question/2876

692 :132人目の素数さん:2018/08/29(水) 17:19:12.01 ID:7MjvDqGy.net
>>691
そのサイトFlashがないと読めないみたい。
もうこのご時世Flashないと読めないサイトわざわざ見る気になれん。

693 :132人目の素数さん:2018/08/29(水) 20:45:48.67 ID:2P4d/CfT.net
>>692
つ Chrome

694 :132人目の素数さん:2018/08/29(水) 23:01:07.06 ID:gzTBhfMR.net
>>692

「三次方程式の解の素朴な性質」 Q.2876
a, b, c を任意の複素数とする。 3次方程式 z^3 + az^2 + bz + c = 0 の解αで
 | 2bα + 3c | ≦ | 3α^3 |
をみたすものが存在することを示してください。
 (2018/04/01 アンドロメダ)

695 :132人目の素数さん:2018/09/07(金) 00:33:08.04 ID:KiGUc8ne.net
>>520 (B3)
>>524

まづ 左辺を a-c, b-d の斉2次式で表わす。
 2 (左辺) = F (a-c)^2 + H (a-c)(b-d) + G (b-d)^2,
ここに
 F = 1/(c+d+a) + 1/(a+b+c),
 G = 1/(b+c+d) + 1/(d+a+b),
 H = 3{ -(a+c)/(c+d+a)(a+b+c) + (b+d)/(d+a+b)(b+c+d)},
とおいた。この斉2次式が正定値となる条件は,
 (判別式) = HH - 4FG < 0,
そこで F, G, H を評価する。
AM-HM より
 F = 1/(c+d+a) + 1/(a+b+c) ≧ 4/{2(a+c)+(b+d)},
 G = 1/(b+c+d) + 1/(d+a+b) ≧ 4/{(a+c)+2(b+d)},
∴ FG ≧ 16/{2(a+c)^2+5(a+c)(b+d)+2(b+d)^2} ≧ 16/{(9/4)(a+b+c+d)^2} > 7/(a+b+c+d)^2,

0 < (a+c)/(c+d+a)(a+b+c) = (a+c)/{(a+c)(a+b+c+d)+bd} < 1/(a+b+c+d),
0 < (b+d)/(b+c+d)(c+d+a) = (b+d)/{(b+d)(a+b+c+d)+ac} < 1/(a+b+c+d),
∴|H| < 3/(a+b+c+d)、

以上により (判別式) = HH - 4FG < 0 したがって左辺は正定値。

 IMO-2008 Short list A.7
 不等式bot(@inequalitybot) [100]  ☆12
 面白スレ26-535,961  面白スレ27-354,356
//www.casphy.com/bbs/highmath/472060/126 (7), 311

696 :132人目の素数さん:2018/09/10(月) 16:31:37.54 ID:EoRvG7WQ.net
正の数 a, b が a+b=1をみたすとき、任意の正の数 x, y に対して ax+by ≧ x^a y^b.

これはAM-GMの一般化でござるかな?

697 :132人目の素数さん:2018/09/10(月) 23:33:36.12 ID:3cPf5e0b.net
>>696

a,bが有理数のときは AM-GM そのものでござるな。

698 :132人目の素数さん:2018/09/11(火) 02:13:16.34 ID:J1j+ih4S.net
重みつきAM-GM の限定版でござるかな?

699 :132人目の素数さん:2018/09/11(火) 02:13:37.80 ID:J1j+ih4S.net
正の数 a, b, x, y が ax+by = x+y をみたすとき、a^(ax) * b^(by) ≧ 1.

700 :132人目の素数さん:2018/09/11(火) 07:28:10.88 ID:cF4T1n2w.net
>>699

(a-1)x + (b-1)y +(c-1)z = 0 をみたすとき

a log(a) = -a log(1/a) ≧ -a(1/a -1) = a-1,
b log(b) ≧ b-1,
c log(c) ≧ c-1,
ax log(a) + by log(b) + cz log(c) ≧ (a-1)x + (b-1)y + (c-1)z = 0,

701 :132人目の素数さん:2018/09/11(火) 11:30:17.92 ID:J1j+ih4S.net
>>700
むむむ…、さすがでござるな。

>>696
正の数 a, b が a+b=1をみたすとき、任意の実数 x, y に対して (ax+by)(ay+bx) ≧ xy.

702 :132人目の素数さん:2018/09/11(火) 12:34:01.15 ID:cF4T1n2w.net
>>701

(ax+by)(ay+bx) = (a+b)^2・xy + 2ab(x-y)^2 ≧ (a+b)^2・xy,

またはコーシーで
(ax+by)(ay+bx) ≧ (a√xy + b√xy)^2 = (a+b)^2・xy,

703 :132人目の素数さん:2018/09/11(火) 13:50:37.80 ID:J1j+ih4S.net
>>702
むむむ…、さすがでござるな。
一つ目の解法の式変形は思いつかぬ…。
ただ、係数の2は不要ですな。

704 :132人目の素数さん:2018/09/11(火) 18:22:10.58 ID:J1j+ih4S.net
非負実数 a,b,c に対して、
(a-b)(a-c)a^4 + (b-c)(b-a)b^4 + (c-a)(c-b)c^4 ≧ 5{(a-b)(b-c)(c-a)}^2.

不等式の秋でござるな。(AA略)

705 :132人目の素数さん:2018/09/13(木) 00:43:44.97 ID:RZVt7y9v.net
>>704
Schurより強い不等式ってことになるのかな?

706 :132人目の素数さん:2018/09/13(木) 12:38:23.72 ID:FJXqJvMw.net
>>705
 非負実数に限れば。

 偶数次の Schur はすべての実数で成立つが、>>704 は (a, b, c) = (a, 0, -a) で不成立。

707 :132人目の素数さん:2018/09/13(木) 13:32:48.84 ID:RZVt7y9v.net
なるほど。 s,t,uで置き換えて証明できるのかな?

708 :132人目の素数さん:2018/09/14(金) 01:54:32.08 ID:E4BddXGB.net
>>704
(左辺)-(右辺)
= F_4 - 5Δ^2
= s^6 - 7s^4t + 28s^3u + 8s^2t^2 - 112stu + 16t^3 + 63u^2

苦しいでござる。
別の方法を考えた方がいいか…。

709 :132人目の素数さん:2018/09/14(金) 04:17:45.76 ID:E4BddXGB.net
去年の秋にやっていたΔがらみの不等式が、このスレを (c-a) で検索するとたくさん出てくる。
それらの中にない(と思う)ものを見つけたのでメモ。

a, b, c > 0に対して、
(a^2 + 3b^2)(b^2 + 3c^2)(c^2 + 3a^2) ≧ {(a-b)(b-c)(c-a)}^2.

https://artofproblemsolving.com/community/c6h1595851p9908733

リンク先の模範解答を見る限りでは、任意の実数で成り立っているんじゃないの?

710 :132人目の素数さん:2018/09/14(金) 05:11:17.74 ID:0c+5G0AL.net
>>704

bはaとcの中間にあるとする。
(左辺) = (a^n)(a-b)^2 + (a^n-b^n+c^n)(a-b)(b-c) + (b^n)(b-c)^2,

(a,b,c) を(間隔を変えず一斉に)d減らしたとき、a^n, c^n は減少する。(0<d≦a,b,c)
では a^n -b^n +c^n はどうか?

Max{a,c} = M, min{a,c} = m とおくと
M^n - b^n = (M-d)^n - (b-d)^b + ∫[0,d] n{(M-d+t)^(n-1) - (b-d+t)^(n-1)} dt
   ≧ (M-d)^n - (b-d)^b,
m^n ≧ (m-d)^n,       (0<d≦m)
辺々たして
 M^n -b^n +m^n ≧ (M-d)^n -(b-d)^n +(m-d)^n,
 a^b -b^n +c^n ≧ (a-d)^n -(b-d)^n +(c-d)^n,
すなわち a^n -b^n +c^n も減少する。
よって(左辺)は減少するから、c=0 の場合に成立てば十分である。

(左辺) ≧ (a-b)a^(n+1) - (a-b)b^(n+1)
   = (a-b)^2・{a^n+a^(n-1)・b+……+a・b^(n-1)+b^n}
   ≧ (n+1)(a-b)^2・(ab)^(n/2)       (AM-GM)
   = (右辺),

711 :132人目の素数さん:2018/09/14(金) 06:40:30.61 ID:E4BddXGB.net
おお! なるほど! かたじけない!

       |
   \  __  /
   _ (m) _ピコーン
      |ミ|
    /  `´  \
     (゚∀゚ )
     ノヽノヽ
       くく

712 :132人目の素数さん:2018/09/14(金) 13:35:31.86 ID:0c+5G0AL.net
>>709

 (aa+3bb)(bb+3cc)(cc+3aa) ≧ {(a+b)(b+c)(c+a)}^2,

左辺は a,b,c の符号によらない。
a,b,c の符号だけを変えたとき、右辺が最も大きいのは a,b,c が同符号のもの。
∴ a,b,c >0 に対して成立てば十分。

(左辺)/(右辺) = (aa+3bb)/(a+b)^2・(bb+3cc)/(b+c)^2・(cc+3aa)/(c+a)^2 = f(a/b) f(b/c) f(c/a),
ここに f(x) = (xx+3)/(x+1)^2,

a,b,c >0 ⇒ f(a/b) f(b/c) f(c/a) ≧ 1 を示す。

(1) a/b, b/c, c/a の1つが 0 < x ≦ (-16+√333)/7 = 0.321184 にあるとき。

 [4f(x)-3](x+1)^2 = 4(xx+3) -3(x+1)^2 = (x-3)^2 ≧ 0,
∴ f(x) の最小値は f(3) = 3/4
 f(x) ≧ (4/3)^2 となるものが1つでもあれば 成立する。
 その条件は [16f(x)-9](x+1)^2 = 16(x+1)^2 -9(xx+3) = 7xx +32x -11 ≦ 0,
 -4.8926125 = (-16-√333)/7 ≦ x ≦ (-16+√333)/7 = 0.321184

(2) a/b, b/c, c/a ≧ (-16+√333)/7 = 0.321184 のとき。
 x ≧ (-16+√333)/7 = 0.321184 のとき
 x(x+1)^4 - (xx+3)^2 = (x^3 +x^2 +3x-1)(x-1)^2 ≧ 0,
∴ f(x) ≧ 1/√x,
∴ f(a/b) f(b/c) f(c/a) ≧ √(b/a) √(c/b) √(a/c) = 1,

以上により成立つ。

>>710 訂正
(左辺) = (a^n)(a-b)^2 + (a^n-b^n+c^n)(a-b)(b-c) + (c^n)(b-c)^2,

713 :132人目の素数さん:2018/09/14(金) 13:51:02.18 ID:0c+5G0AL.net
>>712 訂正スマソ

その条件は [16-9f(x)](x+1)^2 = …

x(xx+3)^2 - (x+1)^4 = (x^3 +x^2 +3x-1)(x-1)^2 ≧ 0,

714 :132人目の素数さん:2018/09/15(土) 17:18:49.08 ID:PF1WMC9C.net
[出典]
>>704 https://artofproblemsolving.com/community/c6t148441f6h1111827_schur_degree_6
>>709 https://artofproblemsolving.com/community/c6h1595851p9908733

704の出典の2つ目のレスに 「It's obvious by EMV」 とあるけど、EMVってなんじゃらほい?

>>712
すごいな。
仮に同じ解法で解ける問題が出てきても、自分では気づきそうにないなあ。

715 :132人目の素数さん:2018/09/16(日) 05:37:44.55 ID:dbi3zUMB.net
>>714
「EMV inequality」でググると、一番上に
A large sieve inequality of Elliott-Montgomery-Vaughan type for automorphic forms and two applications
というのがヒットするんだけど、さっぱり分からん…。

716 :132人目の素数さん:2018/09/16(日) 06:51:57.72 ID:liVS5BiP.net
>>714

http://artofproblemsolving.com/community/c6h205183p1130901
の Theorem 1 でござるな。n=3 のときは

〔EMV定理〕
f(x, y, z): R^3 → R は連続で C^1 級函数とする。次の2つの条件
 (i) xyz = 0 ⇒ f(x, y, z) ≧ 0,
 (ii) x, y, z ≧ 0 ⇒ ∂f/∂x + ∂f/∂y + ∂f/∂z ≧ 0,
を同時に満たすならば
 x, y, z ≧0 ⇒ f(x, y, z) ≧ 0.

Example 3. (Suranyi) >>512-513
Problem 1. (Schur, n=1) >>514
Problem 2. (Turkevici) >>163-164, 185, 530-531 (一般化 >>189)
Problem 6. >>486-487, 492

717 :132人目の素数さん:2018/09/16(日) 09:55:39.99 ID:liVS5BiP.net
>>709 >>712

{a+b√(-3)}{b+c√(-3)}{c+a√(-3)} = -(3q +8abc) + p√(-3),

ここに p = aab+bbc+cca -3abc,q = abb+bcc+caa -3abc,

(左辺) - (右辺) = (aa+3bb)(bb+3cc)(cc+3aa) - {(a+b)(b+c)(c+a)}^2

 = (3q+8abc)^2 + 3pp - (p+q+8abc)^2

 = 2pp -2pq +8qq +16abc(2q-p)

 = 2pp -2pq +(8/3)qq + (16/3) {(q+3abc)^2 -3abc(p+3abc)}

 = 2pp -2pq +(8/3)qq + (16/3) {(abb+bcc+caa)^2 -3caa・abb -3abb・bcc -3bcc・caa}

 = 2pp -2pq +(8/3)qq + (8/3) {[a(ca-bb)]^2 + [b(ab-cc)]^2 + [c(bc-aa)]^2}

 ≧ 0,

∵ (x+y+z)^2 - 3(xy+yz+zx) = (xx+yy+zz) - (xy+yz+zx) = {(x-y)^2+(y-z)^2+(z-x)^2}/2,

718 :132人目の素数さん:2018/09/16(日) 10:50:45.59 ID:dbi3zUMB.net
むむむ…。

719 :132人目の素数さん:2018/09/16(日) 11:16:21.07 ID:dbi3zUMB.net
[三角形の辺長 a,b,c に関するアレ]

(1) abc ≧ (a+b-c)(b+c-a)(c+a-b).
(2) (a^a)(b^b)(c^c) ≧ (a+b-c)^a (b+c-a)^b (c+a-b)^c.
(3) (a+b-c)(b+c-a)(c+a-b) + (a+b)(b+c)(c+a) ≧ 9abc.

(1)は、不等式好きなら誰でも知っているレムスの不等式。
(2)は、上の上の不等式ヲタなら やはり常識である不等式。
(3)が、今回ご紹介する商品。

この他に a+b-c、b+c-a、c+a-b がらみの不等式があれば紹介してクリリン。

720 :132人目の素数さん:2018/09/16(日) 18:53:35.06 ID:HwxbsDio.net
三角形は多いのに四角形の辺に関する不等式なかなか見かけない

721 :132人目の素数さん:2018/09/17(月) 01:23:24.03 ID:iDwWzM3i.net
>>719
△なのでRavi変換する。
 x = b+c-a,
 y = c+a-b,
 z = a+b-c,
とおくと
 x+y+z = a+b+c,

(1)
AM-GM で
 a = (y+z)/2 ≧ √(yz),
 b = (z+x)/2 ≧ √(zx),
 c = (x+y)/2 ≧ √(xy),
より
 abc = (y+z)(z+x)(x+y)/8 ≧ xyz,

a,b,c ≧ 0 のとき
 abc - (a+b-c)(b+c-a)(c+a-b) = F_1(a,b,c) ≧ 0,

(2)
 log(左辺) = a log(a) + b log(b) + c log(c)
  ≧ y log(a) + z log(b) + x log(c)   (←チェビシェフ)
  ≧ (y/2)log(yz) + (z/2)log(zx) + (x/2)log(xy)
  = (y+z)/2 log(z) + (z+x)/2 log(x) + (x+y)/2 log(y)
  = a log(z) + b log(x) + c log(y)
  = log(右辺),

(3)
 (左辺) = (2x)(2y)(2z) + (2x+y+z)(x+2y+z)(x+y+2z) = F1(x,y,z) ≧ 0,

722 :132人目の素数さん:2018/09/17(月) 20:35:16.00 ID:iDwWzM3i.net
>>721 (2)
 チェビシェフは不成立でした。スマソ

 log(a+b-c) = log(a) + log{1 +(b-c)/a} ≦ log(a) + (b-c)/a,
 a log(a+b-c) ≦ a log(a) +b -c,
巡回的にたす。

723 :132人目の素数さん:2018/09/28(金) 16:36:49.64 ID:gDbOCyp+.net
自然数 k,n (k<n)に対して、(n/k)^k ≦ nCk ≦ (en/k)^k を示せ。
ここで e はネイピア数。

724 :132人目の素数さん:2018/09/30(日) 05:30:25.69 ID:60e7kxgM.net
>>723
左側:
C[n, k] = Π[j=0, k-1] (n-j)/(k-j) > Π[j=0, k-1] (n/k) = (n/k)^k,

右側: 補題より
C[n, k] = n(n-1)…(n-k+1)/k! < (n^k)/k! < e^(k-1)・(n/k)^k,

〔補題〕
k≧2 のとき (k^k)/k! < e^(k-1),

(略証)
(1 +1/j)^j = Σ[L=1, j] C[j, L](1/j)^L = Σ[L=1, j] (1-1/j)(1-2/j)…(1-(L-1)/j)/L!
はjについて単調増加。
∴ {(j+1)/j}^j = (1 + 1/j)^j < e,
j=1,…,k-1 を入れて掛けると
 (k^k)/k! < e^(k-1),

(別法)
マクローリン展開から
 e^x > x^{k-1} /(k-1)! + (x^k)/k! + x^{k+1} /(k+1)!
   = (x^k)/k! {(k/x) + 1 + x/(k+1)},
 e^k > (k^k)/k! {2 + k/(k+1)} > (k^k)/k! e,   (k≧3)
∴ e^{k-1} > (k^k)/k!,
k=2 は直接確かめる。   (終)

725 :132人目の素数さん:2018/09/30(日) 07:21:11.36 ID:9qTOp1VD.net
R^n上の対称行列Tが任意のx∈R^nに対して(x,Tx)≧0を満たす時、T≧0と定義する
又、対称行列U,Vに対してU-V≧0の時、U≧Vと定義する
この時、以下について答えよ

(1)R^n上の任意の対称行列T≧0に対し、T=U^2となる対称行列U≧0が一意に存在する事を示せ(尚、この時、U=√Tと定義する)

(2)R^n上の任意の対称行列A,B≧0に対し、A+B≧2√(AB)の真偽を答え、真ならば証明を、偽ならば反例を挙げよ

726 :学術:2018/09/30(日) 12:30:06.76 ID:L25jHE+s.net
未踏士気 ☯

727 :学術:2018/09/30(日) 12:56:48.99 ID:L25jHE+s.net
ゴルフ行こうよ。永遠の−0テンプルバンカーショット。ナイトゴルフ。SWVPW。

728 :学術:2018/09/30(日) 13:19:08.26 ID:L25jHE+s.net
https://www.youtube.com/watch?time_continue=36&v=R3nQFePLx1w

729 :学術:2018/09/30(日) 13:20:48.37 ID:L25jHE+s.net
https://www.youtube.com/watch?v=L1eCH8rxMwM

730 :学術:2018/09/30(日) 13:24:07.75 ID:L25jHE+s.net
https://www.youtube.com/watch?v=U28krJWG-To

新宿神戸三(宮)高に数学でも余に降るかな。 

731 :132人目の素数さん:2018/09/30(日) 14:21:36.32 ID:TIqo4Krx.net
>>725 (2)
A,Bが対称行列でもABが対称行列になるとは限らないぞ。

732 :132人目の素数さん:2018/09/30(日) 20:51:20.91 ID:60e7kxgM.net
>>724 の〔補題〕

分かスレ447 - 82, 438

733 :132人目の素数さん:2018/10/01(月) 00:50:29.88 ID:eM2YcEDk.net
>>724 の補題を改良

〔補題'〕
k≧2 のとき (k^k)/k! < e^(k-1) < (k^k)/(k-1)!

(略証)
(1 -1/jj)^j > 1 -1/j,   … AM-GM
(1 +1/j)^j = (1 -1/jj)^j /(1 -1/j)^j > 1/(1 -1/j)^(j-1) = {1 +1/(j-1)}^(j-1),
∴ (1 +1/j)^j = {(j+1)/j}^j はjについて単調増加
∴ {(j+1)/j}^j < e,
j=1,2,…,k-1 を入れて掛けると
 (k^k)/k! < e^(k-1),

{jj/(jj-1)}^j > (1 +1/jj)^j > (1 +1/j),   … AM-GM
∴ {j/(j-1)}^j = {jj/(jj-1)}^j・(1 +1/j)^j > (1+1/j)^(j+1)
∴ (1 +1/j)^(j+1) = {(j+1)/j}^(j+1) はjについて単調減少
∴ {(j+1)/j}^(j+1) > e,
j=1,2,…,k-1 を入れて掛けると
 (k^k)/(k-1)! > e^(k-1),

分かスレ447-448

734 :132人目の素数さん:2018/10/01(月) 08:28:02.86 ID:eM2YcEDk.net
>>725 (1)

Tのn個の固有値d_j を主対角線に並べた実対角行列を D とし、
対応する固有ベクトルw_j を各列に並べた行列をWとする。
 T w_j = w_j d_j,
 T W = W D,
n個の固有ベクトルw_jが1次独立のとき |W|≠0 で Tは対角化可能。
 T = W D W^(-1),
T≧0 すなわち Tの固有値がすべて非負のとき、Dの対角要素が非負で、√Dも実対角行列。
 T = W D W^(-1) = {W √D W^(-1)}^2 = U^2,

Tが実対称行列のときは、固有ベクトルを適当に選んでWを実直交行列にとれる。
 W^(-1) = W~

735 :132人目の素数さん:2018/10/01(月) 08:34:53.76 ID:jOqQMyBJ.net
>>732
訂正
A+B≧√2(AB+BA)は成り立つかでした

736 :132人目の素数さん:2018/10/01(月) 08:35:39.23 ID:jOqQMyBJ.net
√{2(AB+BA)}っすな

737 :132人目の素数さん:2018/10/02(火) 21:43:18.74 ID:6WqYSECx.net
a, b, c >0 に対して、
a/{b(b+c)^2} + b/{c(c+a)^2} + c/{a(a+b)^2} ≧ 9/{4(ab+bc+ca)}

今年も不等式の秋が来ましたな。
9/{4(ab+bc+ca)} の出てくる不等式は過去スレで扱ったな。

738 :132人目の素数さん:2018/10/02(火) 22:15:34.85 ID:6WqYSECx.net
x, y ∈ R に対して、

(1) 1/(x+1)^2 + 1/(y+1)^2 ≧ 1/(xy+1)

(2) a^3 + b^3 + c^3 - 3abc ≦ (a^2 + b^2 + c^2)^(3/2)

739 :132人目の素数さん:2018/10/02(火) 22:20:09.91 ID:6WqYSECx.net
a, b, c > 0 に対して、

(1) 3 + √{(a^2 + b^2 + c^2)(1/a^2 + 1/b^2 + 1/c^2)} ≧ (2/3)(a+b+c)(1/a + 1/b + 1/c)

(2) √{(a^4 + b^4 + c^4)(1/a^4 + 1/b^4 + 1/c^4)} ≧ 1 + √[1 + √{(a^5 + b^5 + c^5)(1/a^5 + 1/b^5 + 1/c^5)}]

(3) a^4/(a^3 + b^3) + b^4/(b^3 + c^3) + c^4/(c^3 + a^3) ≧ (a+b+c)/3

(4) {(a-b)/c}^2 + {(b-c)/a}^2 + {(c-a)/b}^2 ≧ (2√2)*{(a-b)/c + (b-c)/a + (c-a)/b}

(5) a/{√(2b^2+2c^2)} + b/(c+a) + c/(a+b) ≧ 3/2

(6) a+b+c=3 のとき、44 ≧ (a^2+2)(b^2+2)(c^2+2) ≧ 27



参考 (2) https://artofproblemsolving.com/community/q1h1328831p7152622

740 :132人目の素数さん:2018/10/02(火) 22:39:23.54 ID:6WqYSECx.net
むかし立ち読みした本に、不等式の証明を行列を使ってやっていたんだけど、どんな本を検索したら見つかりますかね?

741 :132人目の素数さん:2018/10/03(水) 03:42:16.06 ID:7h2ip4rW.net
>>737
9/{4(ab+bc+ca)} の出てくる不等式…

〔問題〕
a,b,c > 0 に対して
 1/(b+c)^2 + 1/(c+a)^2 + 1/(a+b)^2 ≧ 9/{4(ab+bc+ca)},

イランMO-1996
Inequalitybot [148]

>>738
(1)
 (x, y) = (2 -1/n, -1/2),
 1/(xy+1) = 2n,

(2)
 a+b+c = s, ab+bc+ca = t とおく。
 |a^3+b^3+c^3-3abc| = |a+b+c| (aa+bb+cc-ab-bc-ca)
 = |s| (ss-3t)
 ≦ (ss-2t)^(3/2)      (← GM-AM)
 = (aa+bb+cc)^(3/2),

*) ss≧0, ss-3t≧0 より、AM-GM で
 (ss-2t)^3 - ss(ss-3t)^2 = (3ss -8t)tt = (1/3){8(ss-3t) +ss}tt ≧ 0,

742 :132人目の素数さん:2018/10/03(水) 16:46:30.48 ID:7h2ip4rW.net
>>738 (2) を改造^^

a,b,c∈R に対して
 | a^3+b^3+c^3 - 3abc |^2 ≦ (aa+bb+cc)^3 - (ab+bc+ca)^3,

(略証)
 s = a+b+c, t = ab+bc+ca とおく。
 (ss-2t)^3 - t^3 - ss(ss-3t)^2 = 3(ss-3t)tt ≧ 0,
 (左辺) = ss(ss-3t)^2 ≦ (ss-2t)^3 - t^3 = (右辺),

743 :132人目の素数さん:2018/10/03(水) 16:56:34.73 ID:7h2ip4rW.net
>>738 (2) を改造^^

a,b,c∈R に対して
 | a^3+b^3+c^3 - 3abc |^2 ≦ (aa+bb+cc)^3 + 8(ab+bc+ca)^3,

(略証)
 s = a+b+c, t = ab+bc+ca とおく。
 (ss-2t)^3 + (2t)^3 - ss(ss-3t)^2 = 3sstt ≧ 0,
 (左辺) = ss(ss-3t)^2 ≦ (ss-2t)^3 + (2t)^3 = (右辺),

744 :132人目の素数さん:2018/10/03(水) 16:58:43.50 ID:M4sIaMdv.net
>>742>>743
乙でござるな。 この2つは どこか修正が入ったの?

745 :132人目の素数さん:2018/10/03(水) 21:33:33.92 ID:M4sIaMdv.net
ああ、-1と8の違いか。

746 :132人目の素数さん:2018/10/04(木) 02:15:28.34 ID:wFWA09/F.net
>>739
(3)
a^4 - (a^3+b^3)(a-kb) = {k(a^3+b^3) -abb} b
 = {k[a^3 +(1/2)b^3 +(1/2)b^3] -abb} b
 ≧ {3k/(2^(2/3)) -1} ab^3,    (AM-GM)
(係数) ≧0 より
 k = (1/3)・2^(2/3) = 0.529133684

 a^4/(a^3 + b^3) ≧ a - kb,
循環的にたす。
 (左辺) ≧ (1-k)(a+b+c) = 0.470866316 (a+b+c).

747 :132人目の素数さん:2018/10/04(木) 05:59:28.01 ID:wFWA09/F.net
>>742>>609 (2), >>612 にござる。

>>739 (6) 右側 は >>616 >>618

 (aa+2)(bb+2)(cc+2) = uu + 2(tt-2su) + 4(ss-2t) + 8
 = (uu+1+1) + (2/3)(t-3)^2 + (4/3)(tt-3su) + (ss-4t) + 3ss
 ≧ 3ss,

※ (uu+1+1) + (ss-4t) ≧ 3u^(2/3) + {F1(a,b,c)-9u}/s
 = 3{u^(2/3) -3u/s} + F1(a,b,c)/s
 ≧ 0,

748 :132人目の素数さん:2018/10/05(金) 00:10:36.73 ID:7iOX1iCn.net
>>618 >>739 (6) >>747

a,b,c ≧ 0, a+b+c ≦ √(8k) のとき
 kk{(a+b+c)^2 +k} ≧ (aa+k)(bb+k)(cc+k) ≧ (3kk/4)(a+b+c)^2,

左側は
 a+b+c ≦ √(8k) より
 ab ≦ (1/4)(a+b)^2 ≦ 2k,
 (a+b)c ≦ (1/4)(a+b+c)^2 ≦ 2k,

 (aa+k)(bb+k) = k{(a+b)^2 +k} - ab(2k-ab) ≦ k{(a+b)^2 +k},

∴ (aa+k)(bb+k)(cc+k) ≦ k{(a+b)^2 +k}(cc+k) = kk(ss+k) -k(a+b)c{2k-(a+b)c} ≦ kk(ss+k),

749 :132人目の素数さん:2018/10/05(金) 06:07:13.55 ID:7iOX1iCn.net
>>741 (上)

4(ab+bc+ca){1/(b+c)^2 + 1/(c+a)^2 + 1/(a+b)^2} - 9
= {ab(4aa+7ab+4bb)(a-b)^2 + bc(4bb+7bc+4cc)(b-c)^2 + ca(4cc+7ca+4aa)(c-a)^2 + (2abc)F_1(a,b,c)}/{(a+b)(b+c)(c+a)}^2
= {4t・F_2+(3tt/s)F_1+(9tu/s)F_0+(st-9u)u} / (st-u)^2
≧ 0,

F_n (a,b,c) = (a^n)(a-b)(a-c) + (b^n)(b-c)(b-a) + (c^n)(c-a)(c-b) ≧ 0,

750 :132人目の素数さん:2018/10/07(日) 07:11:47.76 ID:0iw9KjaT.net
>>724
なるへそ。右辺のeは1個少なくても成り立つんですな。

751 :132人目の素数さん:2018/10/07(日) 09:38:24.95 ID:0iw9KjaT.net
さあ、はじめようか?
>>737の左辺は、どこに挟まるのでござるかな?

{a/(2bc)}^2 + {b/(2ca)}^2 + {c/(2ab)}^2
≧ 1/(4a^2) + 1/(4b^2) + 1/(4c^2)
≧ 1/(4ab) + 1/(4bc) + 1/(4ca)
≧ 1/(b+c)^2 + 1/(c+a)^2 + 1/(a+b)^2 ← (>>741, >>749)
≧ 9/{4(ab+bc+ca)}
≧ 1/{(a+b)(b+c)} + 1/{(b+c)(c+a)} + 1/{(c+a)(a+b)}
≧ 9/{(a+b)(b+c) + (b+c)(c+a) + (c+a)(a+b)}
≧ 27/{4(a+b+c)^2}
≧ 9/{(a+b)^2 + (b+c)^2 + (c+a)(a+b)^2}
≧ 9/{4(a^2 + b^2 + c^2)}

 " ;ヾ ; ;";ヾ; ;"/" ; ;ヾ ;ヾ;ヾ ; ;ヾ ; ; ヾ ;ゞ  " ;ヾ ; ;";ヾゝゝ" ;ヾゞ           ヽ            /
,." ;ヾ ; ;";ヾ; ;"/" ; ;ヾ ;ヾ;ヾ ; ;ヾ ; ; ヾ ;ゞ  " ;ヾ ; ;";ヾゝゝ" ;ヾ ; ; ヾ ;ゞ;        \        /
 ゞヾ ; ;" ; ; ;; ;"iiiiii;;;;;::::: :)_/ヽ,.ゞ:,,ヾゞヾゞ__;::/        `      `        `   ー ─ ' `
   ゞヾゞ;\\iiiiii;;;;::::: :|;:/ヾ; ;ゞ "ゝゞ ; ;`
 " ;゛ ; ;" ; ;ゞ "|iiiiii;;;;::: : |:/ ヾゞ        `         `      ` `
  `      ,|i;iiiiiii;;;;;;::: :| `    `         `     `      ` `   `
        ,|iiii;iiii;;;;:;_ _: :| ___  不等式の秋    `        `        `,
   `    |iiiiiii;;;;;;((,,,):::|/  ≧ \                    ヾ从//"
    `   |iiiiiiii;;;;ii;;;;;;;;::|::::: (● (● |           `  ゙  `    ヾ'./"
         |iiiiii;iii;;;;i;;:: ::::|ヽ::::......ワ...ノ                 ○     .||.       ,
    `   |iii;;iiiii;::;:;;;;::::::| ( つ且 ~      `              ○○   | |
  , , .,.. ,..M|M|iMii;;ii:i;;i:i;:; ゝ つつ.,.. ,...... ,.... ,,,.,.. ,.... ,,,.,.. ,..,,,,.,...,..,.,| ̄ ̄|,.,..(  ).. ,,,..,,.. ,.... ,,,.,...,.. .. ,.... ,,,.,.. ,.... ,,,

752 :132人目の素数さん:2018/10/07(日) 10:33:46.31 ID:0iw9KjaT.net
a, b, c > 0に対して、
(a^2 + 3b^2)(b^2 + 3c^2)(c^2 + 3a^2) ≧ {(a-b)(b-c)(c-a)}^2 ←(>>709-710)

a, b, c > 0に対して、
(a^2 + ab + b^2)(b^2 + bc + c^2)(c^2 + ca + a^2) ≧ {(a-b)(b-c)(c-a)}^2.

ところで
(a^2 + 3b^2)(b^2 + 3c^2)(c^2 + 3a^2) と (a^2 + ab + b^2)(b^2 + bc + c^2)(c^2 + ca + a^2)
の大小は定まりそうにないですが、どうですか?

753 :132人目の素数さん:2018/10/07(日) 17:27:07.24 ID:0iw9KjaT.net
>>748
神掛かってる!
大量投下したやつを今ごろ確認しているところでござるが、関連する昨夏の不等式を再掲。
(自分のmemoから抜き出したので、未紹介のものもあるかもしれない。)

a、b、c∈R、k≧0、4≧λ≧0 に対して、
(1) (aa+k)(bb+k)(cc+k) ≧ (3kk/4)*(a+b+c)^2
(2) (aa+k)(bb+k)(cc+k) ≧ {(4k/3)^(3/2)}*(a-b)(b-c)(c-a)
(3) (aa+k)(bb+k)(cc+k) ≧ (kk/4)*{λ(aa+bb+cc) + (9-λ)(ab+bc+ca)}
(4) {aa+ (k+1)/3}{bb+ (k+1)/3}{cc+ (k+1)/3} ≧ {(k+4)/3}^2*{ab+bc+ca+ (k-5)/3}

a、b、c∈R、k≧1 に対して、
(5) (aa+k)(bb+k)(cc+k) ≧ (k+1)^2*(ab+bc+ca+k-2) + (abc-1)^2

a、b、c∈R、k≧2 に対して、
(6) (aa+k)(bb+k)(cc+k) ≧ (k+1)(ab+bc+ca+k-2)^2

a、b、c∈R、k≧(√2)-1 に対して、
(7) (aa+k)(bb+k)(cc+k) ≧ (k+1)^2*{(a+b+c)^2/3 + k-2}

754 :132人目の素数さん:2018/10/07(日) 17:28:57.36 ID:0iw9KjaT.net
>>753
訂正。(3)(5)は a,b,c≧0.

755 :132人目の素数さん:2018/10/07(日) 18:16:09.56 ID:0iw9KjaT.net
>>748
> a,b,c ≧ 0, a+b+c ≦ √(8k) のとき
>  kk{(a+b+c)^2 +k} ≧ (aa+k)(bb+k)(cc+k) ≧ (3kk/4)(a+b+c)^2,

左側の等号成立条件は a=b=c=k=0 以外にありますか?

756 :132人目の素数さん:2018/10/07(日) 18:38:14.85 ID:0iw9KjaT.net
>>755
kは要らんね、a=b=c=0以外に等号が成立することあるかな?

757 :132人目の素数さん:2018/10/07(日) 18:42:14.42 ID:0iw9KjaT.net
連投すまぬ。
a,b,cのうちの2つが0なら成り立ちますね。他にないかな?

758 :132人目の素数さん:2018/10/07(日) 19:18:13.82 ID:0iw9KjaT.net
>>755
a,b,cのうちの少なくとも2つが0、
a,b,cのうちの一つが0で、2つが√(2k)のとき

これだけかな?

759 :132人目の素数さん:2018/10/08(月) 02:39:14.27 ID:OXto1iGE.net
>>738 (1)
x, y >0 として証明。
lhs - rhs = {xy(x-y)^2 + (xy-1)^2}/{(x+1)^2 (y+1)^2 (xy+1)} ≧0.

一般化できるかな?つまり、
x,y,z>0 のときに、1/(x+1)^2 + 1/(y+1)^2 + 1/(z+1)^2 ≧ 1/(xyz+1) は成り立つ?

760 :132人目の素数さん:2018/10/08(月) 03:25:37.55 ID:OXto1iGE.net
4文字なら、a,b,c,d>0に対して、

1/(1+a)^2 + 1/(1+b)^2 + 1/(1+c)^2 + 1/(1+d)^2
≧ 1/(1+ab) + 1/(1+cd)
> 1/(1+abcd).

761 :132人目の素数さん:2018/10/08(月) 03:40:58.25 ID:moWJj/Va.net
〔補題〕
(1) 4(2-√3) > (√6 -√2),
(2) 12(2-√3) > 4(2-√3) + 2(√6 -√2) > 3(√6 -√2),
(3) (√2 +√3) > 2(√6 -√2) + 4(2-√3),
(4) 22/7 > 2(√6 -√2) + 4(2-√3),
(5) 6 + (√6 -√2) > (√5)(√2 +√3),

762 :132人目の素数さん:2018/10/08(月) 04:06:45.56 ID:moWJj/Va.net
>>761
(1)
 √3 -1 ≒ 0.7320508   1/√2 ≒ 0.70710678
 (左辺) - (右辺) = 2(√3 -1)(√3 -1 -1/√2) > 0,
(2)
 (1) から直ちに出る。
(3)
 (左辺) - (右辺) = (1/4)(√2 -1)^2・(√3 -1)^4・(√3 -√2) > 0,
(4)
 (左辺) - (右辺) = (1/14)(√2 -1)^3・(√3 -1)^4・(3√6 -7) > 0,
(5)
 さてどうするか…

なお、Snellius-Huygens から、2(√6 -√2) + 4(2-√3) > π が分かる。

763 :132人目の素数さん:2018/10/08(月) 06:50:50.12 ID:moWJj/Va.net
>>761

(1)別解
 4tan(π/12) > π/3 > 4sin(π/12),
 4(2-√3) > π/3 > (√6-√2),

http://d.hatena.ne.jp/haruya12/20120314/1331712378

764 :132人目の素数さん:2018/10/08(月) 17:08:48.13 ID:moWJj/Va.net
>>759
s = x+y+z, t = xy+yz+zx, u = xyz とおく。

lhs - rhs = {3+4s+2ss+2(st-3u)+(tt-2su)}/(u+t+s+1)^2 - 1/(u+1)
 = {2+2s+(ss-2t)-5u+2(ss-t)u+2(st-9u)u+11uu+(tt-2su)u}/{(u+t+s+1)^2・(u+1)},
 ≧0.        (← x,y,z≧0)

* 2 -5u +11uu = 63/44 + 11(5/22 -u)^2 ≧ 63/44,

765 :132人目の素数さん:2018/10/09(火) 04:04:05.31 ID:CC0qY+GR.net
>>764
キタ━(゚∀゚)━!!!
なるほど、対称式とSchurすごいな。 stu method でも呼ぶかな

766 :132人目の素数さん:2018/10/09(火) 04:15:31.03 ID:CC0qY+GR.net
n変数にして証明できますかね?
a_k >0 (k=1,2,…n) に対して、Σ1/(1+a_k)^2 ≧ 1/(1+Πa_k).

767 :132人目の素数さん:2018/10/09(火) 05:55:14.05 ID:CC0qY+GR.net
x>0に対して、9x^{10} + 2 ≧ 9x^8 + 2x^9 をAM-GMで示せ。
(蛇足だが、この不等式は任意の実数で成り立つ)

768 :132人目の素数さん:2018/10/09(火) 06:15:35.95 ID:jtiWu+AA.net
>>766
nについての帰納法でやってみた。

n=2 は >>759 より成立。

n≧3 のとき
(1) x_j ≧ 1 があるとき、帰納法の仮定により
 Σ[k=1,n] 1/(1+a_k)^2 > Σ[k≠j] 1/(1+a_k)^2
 ≧ 1/(1+Π[k≠j] a_k)
 ≧ 1/(1+Π[k=1,n] a_k),

(2) x_1〜x_n がすべて1以下のとき、右辺は増加する。
・n=3 の場合がチョト面倒。
 (右辺) = 1/(xyz+1) - 1/(xy+1)
 = xy(1-z)/{(xyz+1)(xy+1)}
 ≦ xy(1-z)/{xy(z+1)}       (← xy(1-z)≧0)
 = (1-z)/(z+1),
 (左辺) - (右辺) ≧ 1/(xy+1) + 1/(z+1)^2 -1/(xyz+1)  (←帰納法の仮定)
 ≧ 1/(z+1)^2 - (1-z)/(z+1)
 = {z/(z+1)}^2
 ≧ 0,

・n≧4 ならば
 Σ[k=1,n] 1/(1+a_k)^2 ≧ Σ[k=1,n] 1/4   (← a_k≦1)
 = n/4
 ≧ 1
 > 1/(1+Π[k=1,n] a_k),

769 :132人目の素数さん:2018/10/09(火) 06:56:44.02 ID:jtiWu+AA.net
>>767

AM-GM より
9x^10 -10x^9 + 1
 = (x-1) (9x^9 -x^8 -x^7 -x^6 -x^5 -x^4 -x^3 -x^2 -x -1)
 = (x-1)^2 (9x^8 +8x^7 +7x^6 +6x^5 +5x^4 +4x^3 +3x^2 +2x +1)
 = (x-1)^2 {5x^8 + (x+1)^2 (4x^6 +3x^4 +2x^2 +1)}
 ≧ 0,

AM-GMより
4x^10 -5x^8 + 1
 = (x^2 -1) (4x^8 -x^6 -x^4 -x^2 -1)
 = (x^2 -1)^2 (4x^6 +3x^4 +2x^2 +1)
≧ 0,

(与式) = {(上) + (下)・9}/5

770 :132人目の素数さん:2018/10/09(火) 11:09:20.09 ID:CC0qY+GR.net
>>769
ごめん、どこでAM-GMを使っているのか分からない。

771 :132人目の素数さん:2018/10/09(火) 14:38:57.52 ID:jtiWu+AA.net
>>767

(左辺) - (右辺) = 2(4x^10 -5x^8 +1) + {(x-1)x^4}^2

 ≧ 2(4x^10 -5x^8 +1)

 = 2{(X^5 + X^5 + X^5 + X^5 + 1) - 5 X^4}   (← X=x^2≧0)

 ≧ 0,

最後のところで AM-GM を使いました。

772 :132人目の素数さん:2018/10/09(火) 16:36:47.68 ID:CC0qY+GR.net
>>767
AM-GMより、
x^{10} + x^9 ≧ 2x^9,
8x^{10} + 2 ≧ 10x^8. (x^8 が8個と 1が2個)

辺々加えて、
9x^{10} + 2 + x^8 ≧ 10x^8 + 2x^9.

( ゚∀゚) ウヒョッ!

773 :132人目の素数さん:2018/10/09(火) 18:11:28.36 ID:CC0qY+GR.net
>>738(1) >>759 >>764 >>768
> x,y,z>0 のとき、1/(x+1)^2 + 1/(y+1)^2 + 1/(z+1)^2 ≧ 1/(xyz+1).

右辺を見て次の不等式を思い出したが、繋がるかな?

x,y,z>0 のとき、1/{x(1+y)} + 1/{y(1+z)} + 1/{z(1+x)} ≧ 3/(1+xyz).

774 :132人目の素数さん:2018/10/09(火) 18:46:04.69 ID:jtiWu+AA.net
>>767

p_0 = 9,
p_1(x) = 6.19544630295     + (x-0.03352960039751934)^2 p_0 > 0,
p_2(x) = 3.8953637526451576 + (x-0.003121543171869486)^2 p_1(x) > 0,
p_3(x) = 2.0721715662084579 + (x+0.08618793580133872)^2 p_2(x) > 0,
p_4(x) = x^8 + 2(x+1)^2 (4x^6 +3x^4 +2x^2 +1),
    = 0.5197441948878409 + (x+0.8393520966569508138)^2 p_3(x) > 0,
p_5(x) = 9x^10 -2x^9 -9x^8 +2 = (x-1)^2 p_4(x) > 0,

( ゚∀゚) ウヒョッ!

775 :132人目の素数さん:2018/10/09(火) 18:49:56.43 ID:CC0qY+GR.net
>>774
細かい数字が出てよく分からんけど、p_k(x) の定義は何ですか?

776 :132人目の素数さん:2018/10/09(火) 20:03:46.82 ID:CC0qY+GR.net
>>768
> n≧3 のとき
> (1) x_j ≧ 1 があるとき、帰納法の仮定により
>  Σ[k=1,n] 1/(1+a_k)^2 > Σ[k≠j] 1/(1+a_k)^2

不等号が逆向きになりませんか?

  Σ[k=1,n] 1/(1+a_k)^2 < Σ[k≠j] 1/(1+a_k)^2

777 :132人目の素数さん:2018/10/09(火) 20:31:33.55 ID:CC0qY+GR.net
ごめん勘違いしていた。忘れてください。

778 :132人目の素数さん:2018/10/09(火) 21:22:11.48 ID:CC0qY+GR.net
>>737
(問題再掲)
> a, b, c >0 に対して、
> a/{b(b+c)^2} + b/{c(c+a)^2} + c/{a(a+b)^2} ≧ 9/{4(ab+bc+ca)}

(証明)
(ab+bc+ca)*[a/{b(b+c)^2} + b/{c(c+a)^2} + c/{a(a+b)^2}]
≧ [ √(ab*a/{b(b+c)^2}) + √(bc*b/{c(c+a)^2}) + √(ca*c/{a(a+b)^2}) ]^2
= [ a/(b+c) + b/(c+a) + c/(a+b) ]^2
≧ (3/2)^2.

  ∧_∧
  ( ;´∀`) < シコシコ、ネビットの順に使うナリ。
  人 Y /
 ( ヽ し
 (_)_)

779 :132人目の素数さん:2018/10/09(火) 23:28:00.13 ID:jtiWu+AA.net
>>759 >>766 >>768

n≧3 のとき
p = Π[k=1,n-1] a_k, z = a_n とおく。

(右辺) = 1/(p・z+1) - 1/(p+1)
 = p(1-z)/{(p・z+1)(p+1)}
 = Max{ p(1-z)/{(p・z+1)(p+1)}, 0}
 ≦ Max{ (1-z)/(z+1), 0}
 ≦ 1/(z+1)^2,
∴ (左辺) - (右辺) ≧ 0,

>>775
 p_k(x) は 2k次の多項式。
 p_5(x) = (左辺) - (右辺) = 9x^10 -2x^9 -9x^8 +2,
 p_k(x) の最小値を b_k とし、そのときのxを a_k とする。
 p_{k-1}(x) = {p_k(x) - b_k}/(x-a_k)^2,

780 :132人目の素数さん:2018/10/10(水) 05:28:35.94 ID:hapOoDe1.net
>>746
するってぇと、こういうことかい?

k = (1/n)*(n-1)^{(n-1)/n} とおくとき、a,b,c>0 に対して、
a^{n+1}/(a^n + b^n) + b^{n+1}/(b^n + c^n) + c^{n+1}/(c^n + a^n) ≧ (1-k)(a+b+c).

781 :132人目の素数さん:2018/10/10(水) 10:06:50.14 ID:hapOoDe1.net
>>710
一般の自然数nの場合に右辺はどうなるのでせうか? 次式は成り立ちますか?

a,b,c>0に対して、
(a-b)(a-c)a^n + (b-c)(b-a)b^n + (c-a)(c-b)c^n ≧ (n+1){(a-b)(b-c)(c-a)}^2.

782 :132人目の素数さん:2018/10/10(水) 19:24:52.49 ID:hapOoDe1.net
>>781
両辺の次数が合ってないから、考えるだけ無駄ですな。

783 :132人目の素数さん:2018/10/10(水) 19:47:02.29 ID:hapOoDe1.net
a,b,c>0とし、Δ= (a-b)(b-c)(c-a)とおく。昨夏にやった不等式について。

(1) (27/8)*(a^2+b^2)(b^2+c^2)(c^2+a^2) ≧ (a^2+ab+b^2)(b^2+bc+c^2)(c^2+ca+a^2) ≧ (a^2+b^2)(b^2+c^2)(c^2+a^2) ≧ Δ^2
(2) k*Δ^2 ≧ (a^2-ab+b^2)(b^2-bc+c^2)(c^2-ca+a^2) ≧ Δ^2
(3) m*Δ^2 ≧ (a-b)(a-c)a^4 + (b-c)(b-a)b^4 + (c-a)(c-b)c^4 ≧ 5Δ^2

(疑問1) k、mの値を知りたい。
(疑問2) (1)もΔ^2の定数倍で挟みたい。

784 :132人目の素数さん:2018/10/11(木) 03:47:19.96 ID:XBFA4KXK.net
>>760
 1/(1+ab) + 1/(1+cd) > 1/(1+ab/2)^2 + 1/(1+cd/2)^2 > 1/(1+abcd/4),
 >>738(1) >>759

>>773 (下)
 1/{x(1+y)} + 1/{y(1+z)} + 1/{z(1+x)} ≧ 3/{G(1+G)} ≧ 3/(1+xyz),
 G = (xyz)^(1/3),

 バルカンMO-2006
 [8] 安藤哲哉 (2014) 例題3.1.7(4)
 [9] 佐藤淳郎[訳] (2013) 問題3.93
 Inequalitybot [77]

>>783
 例えば a=b≠c ⇒ =0

785 :132人目の素数さん:2018/10/11(木) 17:32:09.43 ID:XBFA4KXK.net
・n=2
 1/(1+a)^2 + 1/(1+b)^2 > 1/(1+ab),   >>759(上)

・n=3
 1/(1+a)^2 + 1/(1+b)^2 + 1/(1+c)^2 > 1/(1+abc/2),   >>759(下) >>773(上)

・n=4
 1/(1+a)^2 + 1/(1+b)^2 + 1/(1+c)^2 + 1/(1+d)^2 > 1/(1 + abcd/4),   >>760 >>784

・nについての帰納法で >>784
 Σ1/(1+a_k)^2 ≧ 1/{1 + 4Π(a_k /2)},

786 :132人目の素数さん:2018/10/12(金) 02:23:37.06 ID:UbVFOF8C.net
>>785 念のため…

〔補題〕
 n≧2, a_k≧0 (k=1〜n) のとき
 Σ[k=1, n] 1/(1+a_k)^2 ≧ 1/{ 1 + (Π[k=1, n] a_k) /2^(n-2) },

(略証)
nについての帰納法による。
・n=2 のとき
 >>759 (上)
・n≧3 のとき
(左辺) = Σ[k=1, n] 1/(1+a_k)^2
 ≧ 1/{1 + (Π[k=1, n-1] a_k) /2^(n-3) } + 1/(1+a_n)^2   (←帰納法の仮定)
 ≧ 1/{1 + (Π[k=1, n-1] a_k) /2^(n-2) }^2 + 1/(1+a_n)^2
 ≧ 1/{1 + (Π[k=1, n] a_k) /2^(n-2) }           ( >>759 上)
 = (右辺).

787 :132人目の素数さん:2018/10/12(金) 04:07:57.14 ID:f607XAs3.net
(a^2-ab+b^2)(b^2-bc+c^2)(c^2-ca+a^2) を同じ式で挟むとしたら、こんなもん?

(a^2+b^2)(b^2+c^2)(c^2+a^2)
≧ (a^2-ab+b^2)(b^2-bc+c^2)(c^2-ca+a^2)
≧ (8/27)*(a^2+b^2)(b^2+c^2)(c^2+a^2)


>>784
成程、a=bのときを考えれば凾ナ挟めないのは明らかですね。

>>786
ちょうど悩んでいたところで助かりますた。
直近でやった不等式が使えるとは、偶然以上の何かを感じる…

788 :132人目の素数さん:2018/10/12(金) 05:35:02.53 ID:UbVFOF8C.net
>>753

(1)  (aa+k)(bb+k)(cc+k) ≧ (3kk/4)ss + (u-K)^2,   ただし K = (k/2)^(3/2),
   [前スレ.456] [前スレ.469] >>4 [3]

(略証)
 (aa+k)(bb+k)(cc+k) = uu + k(tt-2su) + kk(ss-2t) + k^3
 = {uu + 2(k/2)^3} + (2k/3)(tt-3su) + (k/3)(t-3k/2)^2 + kk(ss-t) + (3kk/4)ss
 ≧ (u-K)^2 + (k/3)(t-3k/2)^2 + (2k/3)(tt-3su) + (kk/4){ss-4t+3u^(2/3)} + (3kk/4)ss
 = (u-K)^2 + (k/3)(t-3k/2)^2 + (2k/3)(tt-3su) + (kk/4s)F1(a,b,c) + (3kk/4)ss,

※ uu + 2(k/2)^3 = uu + 2KK = (u-K)^2 + K(u+u+K) ≧ (u-K)^2 + (3kk/4)u^(2/3),
  ただし K = (k/2)^(3/2),
  ss -4t +3u^(2/3) ≧ ss -4t +9u/s = F1(a,b,c)/s,

(3) はλ=4 が最良で、
  (aa+k)(bb+k)(cc+k) ≧ (kk/4)(4ss-3t) + (u-K)^2,   但し K = (k/2)^(3/2),
  [前スレ.469] >>4 [4] >>36

789 :132人目の素数さん:2018/10/12(金) 06:10:22.02 ID:f607XAs3.net
去年、アイゼンシュタイン整数を使って、a,b,c>0に対して、

(1) (a^2+ab+b^2)(b^2+bc+c^2)(c^2+ca+a^2) ≧ (3√3/8)*|(a^2-bc)(b^2-ca)(c^2-ab)|,
(2) (a^2+ab+b^2)(b^2+bc+c^2)(c^2+ca+a^2) ≧ (3√3/8)*(ab+bc+ca)^3

が出て、でも(2)は次より弱いから無視。

(3) (a^2+ab+b^2)(b^2+bc+c^2)(c^2+ca+a^2) ≧ (ab+bc+ca)^3

もっと細かく書くと、

(a^2+ab+b^2)(b^2+bc+c^2)(c^2+ca+a^2)
≧ (27/64)*(a+b)^2 (b+c)^2 (c+a)^2
≧ (1/3)*(a+b+c)^2 (ab+bc+ca)^2
≧ (ab+bc+ca)^3.

------------------------------------------------
(疑問1) 同様にやったら、次が成り立つと思うんですが、計算合ってます蟹?

(1)’ (a^2-ab+b^2)(b^2-bc+c^2)(c^2-ca+a^2) ≧ (3√3/8)*|(a^2-bc)(b^2-ca)(c^2-ab)|,
(2)’ (a^2-ab+b^2)(b^2-bc+c^2)(c^2-ca+a^2) ≧ (3√3/8)*|ab-bc+ca|^3

------------------------------------------------
(疑問2) (2)より強い(3)があったように、(2)’より強い次式って成り立ちますか?
2乗の差をとって計算していたのですが、挫折しますた。

(3)’(a^2-ab+b^2)(b^2-bc+c^2)(c^2-ca+a^2) ≧|ab-bc+ca|^3

790 :132人目の素数さん:2018/10/12(金) 07:15:19.21 ID:f607XAs3.net
>>787
8/27 じゃなくて 1/8 だよな。

791 :132人目の素数さん:2018/10/12(金) 09:29:09.68 ID:f607XAs3.net
------------------------------------------------
(疑問3) a,b,c>0 に対して、
4(a^2 + b^2 + c^2)^3 ≧ (a^2-ab+b^2)(b^2-bc+c^2)(c^2-ca+a^2)

が成り立つけど、左辺の係数の4をもっと小さくできないだろうか?

(左-右 = 12t(F_0)^2 + 12t^2 F_0 + 4t^3 + (2F_1 - st + 9u)^2 ≧0)

------------------------------------------------
(疑問4) 以前やった2つの不等式
a,b,c>0 に対して、(a^2+ab+b^2)(b^2+bc+c^2)(c^2+ca+a^2) ≧ {(a-b)(b-c)(c-a)}^2,
a,b,c∈Rに対して、(a^2+b^2+c^2)^3 ≧ 2{(a-b)(b-c)(c-a)}^2

の左辺について、a,b,c>0 に対して何か不等式は作れないだろうか? 👀
Rock54: Caution(BBR-MD5:1341adc37120578f18dba9451e6c8c3b)


792 :132人目の素数さん:2018/10/12(金) 09:35:22.26 ID:f607XAs3.net
>>791
疑問3は計算間違っていました。すみません。

793 :132人目の素数さん:2018/10/12(金) 21:08:11.69 ID:f607XAs3.net
(1) a,b,c∈R に対して、8(a^2+b^2)(b^2+c^2)(c^2+a^2) ≧ (a+b)^2 (b+c)^2 (c+a)^2.
(2) a,b,c∈R に対して、2(a^2+b^2)(b^2+c^2)(c^2+a^2) ≧ (a-b)^2 (b-c)^2 (c-a)^2.
(3) a,b,c>0 に対して、(a^2+b^2)(b^2+c^2)(c^2+a^2) ≧ (a-b)^2 (b-c)^2 (c-a)^2.

(2),(3) に比べて (1)の左辺の係数8が大きいですが、これが限界?
(1)の条件を a,b,c>0 に変えたら、係数は小さくできるかな?
最良値かどうかを判断する考え方がイマイチ分かりませぬ… ('A`)

794 :132人目の素数さん:2018/10/13(土) 03:26:19.86 ID:OSa6VDkO.net
>>789

(3) (aa+ab+bb)(bb+bc+cc)(cc+ca+aa) ≧ (1/3)sstt ≧ |t|^3,

(略証)
 ss±3t = {(a±b)^2 + (b±c)^2 + (c±a)^2}/2 ≧ 0,  (複号同順)
∴ |t| ≦ ss/3,

(疑問1)
(1)' … 1
 (aa-ab+bb)(bb-bc+cc)(cc-ca+aa) - (aa-bc)(bb-ca)(cc-ab)
= {(a-b)cc}^2 + {(b-c)aa}^2 + {(c-a)bb}^2 + (abc)^2
 ≧ 0,
 (aa-ab+bb)(bb-bc+cc)(cc-ca+aa) + (aa-bc)(bb-ca)(cc-ab)
 = {(a-b)ab}^2 + {(b-c)bc}^2 + {(c-a)ca}^2 + (abc)^2
 ≧ 0,

(2)' … 1/27
(疑問2) … 1/27
(3)’(aa-ab+bb)(bb-bc+cc)(cc-ca+aa) ≧ (1/81)sstt ≧ (1/27)|t|^3,
  (3) と同様に出ます。(*) 右辺はtのままです。

795 :132人目の素数さん:2018/10/13(土) 03:29:35.03 ID:OSa6VDkO.net
>>791 >>792

(疑問3) … 3/8
 (aa-ab+bb)(bb-bc+cc)(cc-ca+aa) = (ss-3t)(tt-3su) + stu -8uu,

(左辺) - (右辺)
 = (3/8)(aa+bb+cc)^3 - (aa-ab+bb)(bb-bc+cc)(cc-ca+aa)
 = (3/8)(ss-2t)^3 - (ss-3t)(tt-3su) -stu +8uu
 = (1/32)(3s^3 -10st +16u)^2 + (3/32){s(ss-2t)}^2  (←uで平方完成)
 ≧ 0,
 等号成立は (a, 0, -a) etc.

796 :132人目の素数さん:2018/10/13(土) 03:48:21.26 ID:OSa6VDkO.net
>>794
 絶対値は間違いです...orz

(3) (aa+ab+bb)(bb+bc+cc)(cc+ca+aa) ≧ (1/3)sstt,

(3)’(aa-ab+bb)(bb-bc+cc)(cc-ca+aa) ≧ (1/81)sstt,

797 :132人目の素数さん:2018/10/13(土) 07:40:05.27 ID:OSa6VDkO.net
>>794 >>795 >>796

aa+ab+bb = (3/4)(a+b)^2 + (1/4)(a-b)^2,
aa-ab+bb = (1/4)(a+b)^2 + (3/4)(a-b)^2,
(a+b)(b+c)(c+a) = st-u ≧ 8st/9,

(3)
 (aa+bb+cc)^3 ≧ (27/8)(aa+bb)(bb+cc)(cc+aa)     AM-GM
 ≧ (aa+ab+bb)(bb+bc+cc)(cc+ca+aa)
 ≧ (1/3){9(a+b)(b+c)(c+a)/8}^2
 ≧ (1/3)sstt,

(3')
 (3/8)(aa+bb+cc)^3 ≧ (aa-ab+bb)(bb-bc+cc)(cc-ca+aa)
 ≧ {(a+b)(b+c)(c+a)/8}^2
 ≧ (1/81)sstt,

>>794 (1)' から >>611 (6)

798 :132人目の素数さん:2018/10/13(土) 07:46:23.21 ID:OSa6VDkO.net
>>791
(疑問4)
 (上) >>752 (中)
 (下) >>609 (3)

>>793

(1) これが限界。 a=b=c で等号が成立するなら、a,b,c>0 に変えても同じぢゃね?

(2) (1-i)(a+ib)(b+ic)(c+ia) = -(a-b)(b-c)(c-a) + i{(a+b)(b+c)(c+a) - 4abc},
   >>609 (4) >>615

799 :132人目の素数さん:2018/10/13(土) 12:39:06.10 ID:po5VPR1l.net
>>794-798
非常に詳しくありがとうございます。一つ一つ確認しているところです。

800 :132人目の素数さん:2018/10/14(日) 05:52:44.82 ID:Rg/i5zok.net
>>741
> |s| (ss-3t) ≦ (ss-2t)^(3/2)      (← GM-AM)

どのように相加相乗を使っているのですか?

801 :132人目の素数さん:2018/10/14(日) 06:14:49.66 ID:Rg/i5zok.net
742-743を見て、探してみたが、意外と少なかった… ('A`)ヴォエァ!

------------------------------------------
不等式スレ内を検索して

a^3+b^3+c^3-3abc : >>29>>738>>742-743
a^3+b^3+c^3+3abc : 第5章>>269、第2章>>372

------------------------------------------
My Collections から (出典不明)

(1) a,b,c∈Rに対して、(a^2+b^2+c^2)^3 ≧ (a^3+b^3+c^3-3abc)^2 + (ab+bc+ca)^3
(2) a,b,c≧に対して、a^3+b^3+c^3-3abc ≧ (1/4)*(a+b-2c)^3

802 :132人目の素数さん:2018/10/14(日) 19:32:20.99 ID:0CPQSloM.net
>>28 (2), >>29 (1), [前スレ.262], [初代スレ.836-869]
「楠瀬の不等式」
出典: 数学セミナー、出題:1992年4月、解説:1992年7月

a,b,c ∈ R に対して
 aa+bb+cc ≧ |ab| + |bc| + |ca| ≧ |ab+bc+ca|,
>>742 >>743 から
 | a^3+b^3+c^3 - 3abc |^2 ≦ (aa+bb+cc)^3 - |ab+bc+ca|^3.

>>800
 {ss, ss-3t, ss-3t} はいずれも非負。
 AM = ss-2t,  GM = {s(ss-3t)}^(2/3).

803 :132人目の素数さん:2018/10/14(日) 19:47:58.72 ID:Q7SdFY73.net
(x_1+…+x_n)/n=xとするとき、
(Σ(x_k-x)^3)^2 と (Σ(x_k-x)^2)^3 の大小について何か言えますか?
Σはk=1からnまでの和です。

804 :132人目の素数さん:2018/10/15(月) 01:00:34.63 ID:TBaDGY4B.net
>>803

(x_1 + x_2 + … + x_n) /n = A とおくとき、

( Σ[j=1,n] (x_j - A)^3 )^2 / ( Σ[k=1,n] (x_k - A)^2 )^3 ≦ (n-2)^2 /n(n-1) < 1,

等号成立は {a,…,a, b} など。

805 :132人目の素数さん:2018/10/15(月) 08:43:04.04 ID:TBaDGY4B.net
>>801 (1) は >>609 (2), >>742 と同じでつね。
>>802 (中) の方がチョト強い。

>>803
n=3 のとき
{(x-A)^2 + (y-A)^2 + (z-A)^2}^3 - 6{(x-A)^3 + (y-A)^3 + (z-A)^3}
= 2{(x-y)(y-z)(z-x)}^2 + (x+y+z-3A){……}

(略証)
(x-A)^3 + (y-A)^3 + (z-A)^3 = 3(x-A)(y-A)(z-A) + (x+y+z-3A){……} = 3(x-A)(y-A)(z-A),
より
{(x-A)^2 + (y-A)^2 + (z-A)^2}^3 - 6{(x-A)^3 + (y-A)^3 + (z-A)^3}
= 2{(x-y)(y-z)(z-x)}^2 + (x+y+z-3A){ …… }
≧ 0,

806 :132人目の素数さん:2018/10/15(月) 09:33:15.98 ID:r7ZX+DeY.net
>>805
むむむ…


ところで、ちょっと作ったんだけど、係数はこれが最善かな?
a,b,c∈R に対して、(a^2+b^2+c^2)^3 ≧ (27/16)*{(a-b)(b-c)(c-a)}^2.

807 :132人目の素数さん:2018/10/15(月) 11:27:35.52 ID:TBaDGY4B.net
>>806 … 2 ぢゃね?
 >>791 (疑問4・下) >>609 (3)


>>802
〔楠瀬の不等式〕
x,y,z ≧ 0 のとき
 x^3 + y^3 + z^3 -3xyz = (x+y+z)(xx+yy+zz-xy-yz-zx) ≧ A|(x-y)(y-z)(z-x)|,
ここに A = √(9+6√3) = √{(3/2)√3}(1+√3) = 4.403669475

(略証)
 (左辺) - (右辺) = (x^3 +y^3 +z^3 -3xyz) - A|(x-y)(y-z)(z-x)|
 = (1/2)(x+y+z){(x-y)^2 +(y-z)^2 +(z-x)^2} - A|(x-y)(y-z)(z-x)|,

 x,y,z の間隔を固定して一斉に動かしても、{ … } 内と右辺は変わらない。
 最小元が 0 のときに成り立てばよい。 以下 z=0 とする。

 (左辺) - (右辺) = x^3 -A xy|x-y| +y^3,

・0≦x≦y のとき
 x^3 + A xy(x-y) + y^3 = (x + y/αα)(x-αy)^2,
  α = {(1+√3) - √(2√3)}/2 = 0.43542054468234
  1/αα = (1+√3) + A/√3 = 5.27451056440629

・0≦y≦x のとき
 x^3 - Axy(x-y) + y^3 = (x + y/ββ)(x-βy)^2
  β = {(1+√3) + √(2√3)}/2 = 2.29663026289
  1/ββ = (1+√3) - A/√3 = 0.18959105073

 αβ = 1,

808 :132人目の素数さん:2018/10/15(月) 11:58:50.55 ID:r7ZX+DeY.net
>>807
ホントだ…、ありがとうございます。

809 :132人目の素数さん:2018/10/15(月) 12:36:41.62 ID:r7ZX+DeY.net
a,b,c∈R に対して、(a^2+b^2+c^2)^3 ≧ k(a^2+b^2)(b^2+c^2)(c^2+a^2)

k = 27/8 が限界かと思うけど、2になりますかね?

810 :132人目の素数さん:2018/10/15(月) 12:53:20.17 ID:TBaDGY4B.net
>>806 >>807

(略証)
 bはaとcの中間にあるとしてよい。
 0 ≦ (a-b)(b-c) ≦ (1/4)(a-c)^2,
∴ aa+cc = (1/2)(a+c)^2 + (1/2)(a-c)^2 ≧ (1/2)(a-c)^2,
∴ (aa+cc)^3 ≧ (1/8)(a-c)^6 ≧ 2(a-c)^2 {(a-b)(b-c)}^2 = 2刧,
 >>609 (3), >>612 より再録

811 :132人目の素数さん:2018/10/16(火) 00:51:28.34 ID:5DYkLdwz.net
>>805

 (x-A) + (y-A) + (z-A) = 0,
x-A と y-A が同符号のとき
 (z-A)^2 = {(x-A) + (y-A)}^2 ≧ 4|(x-A)(y-A)|,
より
 (x-A)^2 + (y-A)^2 + (z-A)^2 = (1/2)(x+y-2A)^2 + (1/2)(x-y)^2 + (z-A)^2
 = (3/2)(z-A)^2 + (1/2)(x-y)^2
 ≧ (3/2)(z-A)^2,

{(x-A)^2 + (y-A)^2 + (z-A)^2}^3 ≧ (27/8)(z-A)^6 ≧ 6{3(x-A)(y-A)(z-A)}^2


>>809
 k = 27/8 ですね。 A,B,C≧0 より
(左辺) - (右辺) = (A+B+C)^3 -(27/8)(A+B)(B+C)(C+A)
 = S^3 - (27/8)(ST-U)
 = (S^3 -4ST +9U) + (5/8)(ST-9U)
 ≧ 0,

812 :132人目の素数さん:2018/10/16(火) 03:14:15.05 ID:K3TfA7ci.net
>>802
> a,b,c ∈ R に対して
>  aa+bb+cc ≧ |ab| + |bc| + |ca| ≧ |ab+bc+ca|,
> >>742 >>743 から
>  | a^3+b^3+c^3 - 3abc |^2 ≦ (aa+bb+cc)^3 - |ab+bc+ca|^3.

下の2行が分かりませぬ…。

813 :132人目の素数さん:2018/10/16(火) 03:32:04.94 ID:5DYkLdwz.net
>>>812

ab+bc+ca ≧ 0 のときは >>742 から明らか。

ab+bc+ca ≦ 0 のときは

(7/9) >>742 + (2/9) >>743 より

 (a^3+b^3+c^3 -3abc)^2 ≦ (aa+bb+cc)^3 + (ab+bc+ca)^3.

814 :132人目の素数さん:2018/10/16(火) 04:56:11.27 ID:K3TfA7ci.net
>>813
なるほど! ありがとございます。

815 :132人目の素数さん:2018/10/16(火) 14:07:17.11 ID:K3TfA7ci.net
適当に検索して見つけたんだけど、解答が見当たらない…
http://www.ssmrmh.ro/wp-content/uploads/2018/09/12-RMM-SPRING-EDITION-2019-2.pdf

SP172、SP174、SP179、UP177など、見たことないのがある。

816 :132人目の素数さん:2018/10/17(水) 02:05:16.29 ID:CNsWZSmr.net
>>815

SP.172
Prove that for any real numbers x,y,z:
(x+y+z)(y+z-x)(z+x-y)(x+y-z) ≦ (2yz)^2.

SP.173
 Prove that for any positive real numbers x,y,z:
 {xx√(yy+zz) + yy√(zz+xx) + zz√(xx+yy)} / (x^3+y^3+z^3) ≦ √2.

SP.174
Prove that for any positive real numbers a,b,c,x,y,z:
 (a^3+x^3+x^3+x^3)(y^3+b^3+y^3+y^3)(z^3+z^3+c^3+z^3) ≧ (ayz+bzx+cxy+xyz)^3.

SP.179 (改)
 If x ∈ [0,1) then:
1/2 < cos(x) ≦ 1 ≦ arcsin(x) + e^(-x).

UP.177
 If x,y,z,t >1 then:
 {log(x)/log(ztx)} {log(y)/log(txy)} {log(z)/log(xyz)} {log(t)/log(yzt)} < 1/16.

817 :132人目の素数さん:2018/10/17(水) 03:59:20.74 ID:CNsWZSmr.net
>>815
解答作りますた。

SP.172
 (x+y+z)(y+z-x) = (y+z)^2 -xx = 2yz - (xx-yy-zz),
 (z+x-y)(x+y-z) = xx - (y-z)^2 = 2yz + (xx-yy-zz),
辺々掛ける。
 (左辺) = (2yz)^2 - (xx-yy-zz)^2 ≦ (2yz)^2,

(*) x,y,z がΔの3辺の場合は、Δの面積が2辺の積の半分以下であることを表わす。

SP.173
 (左辺)^2 ≦ 3x^4・(yy+zz) + 3y^4・(zz+xx) + 3z^4・(xx+yy)
  = x^3・{3(xyy + xzz)} + y^3・{3(yzz + yxx)} + z^3・{3(zxx + zyy)}
  ≦ x^3・{(x^3+y^3+y^3) + (x^3+z^3+z^3)} + y^3・{(y^3+z^3+z^3) + (y^3+x^3+x^3)} + z^3・{(z^3+x^3+x^3) + (z^3+y^3+y^3)}
  = 2(x^3+y^3+z^3)^2,
  
SP.174
 コーシーそのもの。

SP.179
 arcsin(x) ≧ x, (0≦x<1)
 e^(-x) ≧ 1 - x,
辺々たす。

UP.177
 X=log(x), Y=log(y), Z=log(z), T=log(t) はすべて正だから AM-GM で
 Z+T+X ≧ 3(ZTX)^(1/3),
 T+X+Y ≧ 3(TXY)^(1/3),
 X+Y+Z ≧ 3(XYZ)^(1/3),
 Y+Z+T ≧ 3(YZT)^(1/3),
辺々掛けて
 (Z+T+X)(T+X+Y)(X+Y+Z)(Y+Z+T) ≧ 81 XYZT,

(左辺) = X/(Z+T+X)・Y/(T+X+Y)・Z/(X+Y+Z)・T/(Y+Z+T) ≦ 1/81,

818 :132人目の素数さん:2018/10/17(水) 04:23:31.00 ID:TTcOhpLB.net
>>816-817
おおおーありがとうございます。蒐集が捗る!

819 :132人目の素数さん:2018/10/17(水) 06:46:49.96 ID:TTcOhpLB.net
ROMANIAN MATHEMATICAL MAGAZINE (RMM 11)、解答なし
http://www.ssmrmh.ro/wp-content/uploads/2018/09/11-RMM-WINTER-EDITION-2018-1.pdf
JP158、JP165、SP164、SP165など、いかがでござるか?

JP165の右辺を見て、毒電波を受信した。

a,b,c∈R に対して、
√{6(a^2+b^2+c^2)} ≧ √(a^2+b^2) + √(b^2+c^2) + √(c^2+a^2) ≧ 2√(a^2+b^2+c^2)


---------------------------------------------
ROMANIAN MATHEMATICAL MAGAZINE って、不等式専門雑誌なん?
最新2回分には解答が公開されないっぽい。
http://www.ssmrmh.ro/category/current-issue/
---------------------------------------------

820 :132人目の素数さん:2018/10/17(水) 06:48:59.18 ID:TTcOhpLB.net
ROMANIAN MATHEMATICAL MAGAZINE (RMM 10)
http://www.ssmrmh.ro/wp-content/uploads/2018/07/10-RMM-AUTUMN-EDITION-2018-1.pdf
解答
http://www.ssmrmh.ro/wp-content/uploads/2018/07/10-RMM-AUTUMN-EDITION-2018-SOLUTIONS-compressed.pdf

SP140 がなんとなく気に入った。
公式の模範解答がイマイチに感じるのは、このスレの猛者の解答になれているせいなのか…

821 :132人目の素数さん:2018/10/17(水) 06:51:20.00 ID:TTcOhpLB.net
解答のない号で、三角形がらみ(a,b,c,R,r,S,A,B,Cのみ)、シンプル、既出でないものを抽出。

ROMANIAN MATHEMATICAL MAGAZINE (RMM 12)
http://www.ssmrmh.ro/wp-content/uploads/2018/09/12-RMM-SPRING-EDITION-2019-2.pdf
JP173、JP179、UP171、UP175

ROMANIAN MATHEMATICAL MAGAZINE (RMM 11)
http://www.ssmrmh.ro/wp-content/uploads/2018/09/11-RMM-WINTER-EDITION-2018-1.pdf
JP157、UP155


ところで、JP171、JP174、JP153 などで説明なしに使われている h_a、m_a、l_a などは何を意味するのだろう? 垂線、中線、二等分線かな?
定義が分からないので、見た目がシンプルでも上のリストから外してしまったが…

822 :132人目の素数さん:2018/10/17(水) 08:05:48.87 ID:TTcOhpLB.net
>>815-817
SP.173の分母を払った式

(√2)(x^3+y^3+z^3) ≧ x^2√(y^2+z^2) + y^2√(z^2+x^2) +z^2√(x^2+y^2)

を見て、右辺にCSを使えば片付きそうな気がしたが、大きくなり過ぎた。

√{2(x^4+y^4+z^4)(x^2+y^2+z^2)}
≧ (√2)(x^3+y^3+z^3)
≧ x^2√(y^2+z^2) + y^2√(z^2+x^2) +z^2√(x^2+y^2)

823 :132人目の素数さん:2018/10/17(水) 08:47:58.61 ID:CNsWZSmr.net
>>819

JP.158
 Let a,b,c>0. Prove that:
  (1/a + 1/b + 1/c) + a/(bb+cc) + b/(cc+aa) + c/(aa+bb) ≧ 1/(a+b) + 1/(b+c) + 1/(c+a),

JP.165 (改)
 If a,b,c≧0 then:
 4(a+b+c) ≦ (2√2){√(aa+bb) + √(bb+cc) + √(cc+aa)},

SP.164 (改)
 If a,b,c > 0 then:
 (a+b)√(aa-ab+bb) + (b+c)√(bb-bc+cc) + (c+a)√(cc-ca+aa) ≧ 2(aa+bb+cc),

SP.165 (改)
 If a,b,c ≧0 then:
 (a+b)√(aa+bb) + (b+c)√(bb+cc) + (c+a)√(cc+aa) ≧ (1/√2){(aa+bb+cc) + (a+b+c)^2},

824 :132人目の素数さん:2018/10/17(水) 09:14:42.86 ID:CNsWZSmr.net
>>819 >>823

JP.165 (改)
 x+y ≦ √{2(xx+yy)} より
 4(a+b+c) ≦ (2√2){√(aa+bb) + √(bb+cc) + √(cc+aa)},

SP.164 (改)
コーシーより
 (x+y)√(xx-xy+yy) = √{(x+y)(x^3+y^3)} ≧ xx + yy,
(略証)
 (x+y)^2・(xx-xy+yy) - (xx+yy)^2 = (x+y)(x^3+y^3) - (xx+yy)^2 = xy(x-y)^2 ≧0,

SP.165 (改)
 √(xx+yy) ≧ (x+y)/√2, etc.
 (左辺) ≧ {(a+b)^2 + (b+c)^2 + (c+a)^2}/(√2) ≧ (√2)(aa+bb+cc+t),
 t = ab+bc+ca,

>>819
a,b,c∈R に対して、
√{6(aa+bb+cc)} ≧ √(aa+bb) + √(bb+cc) + √(cc+aa)
 ≧ √{4(aa+bb+cc) + 2t}
 ≧ (√2)s,
 s = a+b+c, t = ab+bc+ca,

(略証)
左側はコーシー
中は √(xx+yy)√(xx+zz) ≧ xx+xy, etc.
 ∵ (xx+yy)(xx+zz) - (xx+yz)^2 = {x(y-z)}^2 ≧ 0, (コーシー)
右側は aa+bb+cc ≧ t.

825 :132人目の素数さん:2018/10/17(水) 10:31:10.77 ID:CNsWZSmr.net
>>820

SP.140
 Let a,b,c be positive real numbers. Prove that:
 (b+c)/a + (c+a)/b + (a+b)/c ≧ 4(aa+bb+cc)/(ab+bc+ca) + 2(ab+bc+ca)/(aa+bb+cc) ≧ 6,

826 :132人目の素数さん:2018/10/18(木) 03:45:39.48 ID:Dw4OfxmO.net
>>820 >>825

SP.140 (改)
 (b+c)/a + (c+a)/b + (a+b)/c ≧ 4(aa+bb+cc)/(ab+bc+ca) + 2
 ≧ 3(aa+bb+cc)/(ab+bc+ca) + 3(ab+bc+ca)/(aa+bb+cc) ≧ 6,

(略証)
 s = a+b+c, t = ab+bc+ca, u = abc とおく。
(左辺) - 4(ss-2t)/t - 2
 = (st-3u)/u - 4(ss-2t)/t - 2
 = st/u - 4ss/t + 3
 = (s/ttu)(t^3 -4stu +9uu) + (3/tt)(tt-3su)
 ≧ 0,


>>819 >>823 >>824
SP.164 (改) より
 √(xx-xy+yy) > {(xx+yy)/(x+y), M_4} > M_3 > √{(xx+yy)/2} > (x+y)/2 > √(xy) > 2xy/(x+y),

ここに M_r = {(x^r+y^r)/2}^(1/r) はr乗平均, M_1 = (x+y)/2, M_2 = √{(xx+yy)/2},

827 :132人目の素数さん:2018/10/18(木) 06:51:05.46 ID:Dw4OfxmO.net
>>821

RMM 12 (Spring2019)

JP.173
 Prove that in any triangle ABC,
  1/a + 1/b + 1/c ≧ √{3/(2Rr)} ≧ (√3)/R.

JP.179
 In acute triangle ABC the following relationship hplds:
 3 ≦ sin(2A)/sin(2B) + sin(2B)/sin(2C) + sin(2C)/sin(2A) ≦ 3/{8cos(A)cos(B)cos(C)},

UP.171
 Find that in any acute-angled triangle ABC the following inequality holds:
 min{a/(b+c), b/(c+a), c/(a+b)} ≦ {cos(A) + cos(B) + cos(C)}/3 ≦ Max{a/(b+c), b/(c+a), c/(a+b)},

UP.175 (改)
 In acute triangle ABC the following relationship holds:
 (b+c)^2/(bb+cc-aa) + (c+a)^2/(cc+aa-bb) + (a+b)^2/(aa+bb-cc) ≧ 12,

等号成立は正△のとき、だろうな…

828 :132人目の素数さん:2018/10/18(木) 07:52:45.35 ID:Dw4OfxmO.net
>>819 >>823

RMM 11 (Winter2018)

JP.158 (訂正)
 Let a,b,c>0. Prove that:
  (1/a + 1/b + 1/c) + a/(bb+cc) + b/(cc+aa) + c/(aa+bb) ≧ 3/(a+b) + 3/(b+c) + 3/(c+a),

(略証) チェビシェフしたあと、
 (1/x + 1/y)/2 + (x+y)/{2(xx+yy)} - 3/(x+y)
 = (x+y)/(2xy) + (x+y)/{2(xx+yy)} - 3/(x+y)
 = (x+y)(xx+xy+yy)/{2xy(xx+yy)} - 3/(x+y)
 = (x-y)^2 (xx-xy+yy)/{2xy(xx+yy)(x+y)}
 ≧ 0,

829 :132人目の素数さん:2018/10/18(木) 15:11:32.69 ID:k/D5nzuI.net
>>822
√{2(x^4+y^4+z^4)(x^2+y^2+z^2)} にCSを使うと、使い方次第で
≧ (√2)(x^3+y^3+z^3) にも
≧ x^2√(y^2+z^2) + y^2√(z^2+x^2) +z^2√(x^2+y^2) にもなるんだな。

830 :132人目の素数さん:2018/10/19(金) 11:50:11.00 ID:UmCMoNsS.net
>>822 >>829
 コーシーとチェビシェフの合わせ技(?)

〔補題〕
 (a,b,c) と (p,q,r) が同順序のとき
 √(aa+bb+cc) √(pp+qq+rr) ≧ (ap+bq+cr) ≧ (a+b+c)(p+q+r)/3 ≧ (aq+ar+bp+bq+cp+cq)/2,

 (a,b,c) と (p,q,r) が逆順序のとき
 √(aa+bb+cc) √(pp+qq+rr) ≧ (aq+qr+bp+br+cp+cq)/2 ≧ (a+b+c)(p+q+r)/3 ≧ (ap+bq+cr),

831 :132人目の素数さん:2018/10/23(火) 15:23:49.80 ID:QCR0wRAh.net
z∈C が |z + 1/2| < 1/2 をみたすとき、
任意の n∈N に対して |1 + z + z^2 + … + z^n|^2 < 1.

832 :132人目の素数さん:2018/10/23(火) 16:45:18.22 ID:foOj88Cn.net
>>831

題意より、
|z| ≦ |z+1/2| + (1/2) < 1,
∴ |1-z|^2 = (1-z)(1-z~)
 = (3/2) - 2|z+1/2|^2 + 3|z|^2
 > 1 + 3|zz|
 > 1 + 2|zz| + |zz|^2
 = (1+|zz|)^2,
∴ |1-z| > 1 + |zz| > 1 + |z|^(n+1) ≧ |1 - z^(n+1)|.

東工大-2000 前期 Q.2
[第7章.114,116,160]
Inequalitybot [183]

833 :132人目の素数さん:2018/10/23(火) 18:52:36.49 ID:QCR0wRAh.net
>>825-826
botの106に似てない?

834 :132人目の素数さん:2018/10/24(水) 04:35:15.90 ID:RiX0WTIF.net
>>831
この問題の結論の不等式って、|1 + z + z^2 + … + z^n| < 1 と書かずに、
あえて2乗にしているのは、何か意味があるのかな?

835 :132人目の素数さん:2018/10/24(水) 22:16:00.49 ID:LB37fX3V.net
>>627 (Nesbitt-Igarashi)

(略証)
各辺に ab+bc+ca を掛けると コーシー型になる:

{a(bb+bc+cc) + b(cc+ca+aa) + c(aa+ab+bb)} {a/(bb+bc+cc) + b/(cc+ca+aa) + c/(aa+ab+bb)}
 ≧ {a(b+c) + b(c+a) + c(a+b)} {a/(b+c) + b/(c+a) + c/(a+b)}
 ≧ (a+b+c)^2,

そこで ラグランジュの恒等式
 (ax + by + cz)(a/x + b/y + c/z) - (a+b+c)^2 = (ab/xy)(x-y)^2 + (bc/yz)(y-z)^2 + (ca/zx)(z-x)^2,
を使う。

・左辺は
 x = bb + bc + cc,
 y = cc + ca + aa,
 z = aa + ab + bb,
 ax + by + cz = (a+b+c)(ab+bc+ca),     >>621
 (左辺) - (a+b+c)^2 = {a(a+b+c)/(bb+bc+cc)}{b(a+b+c)/(cc+ca+aa)}(a-b)^2 + …

・中辺は
 x = b + c,
 y = c + a,
 z = a + b,
 ax + by + cz = 2(ab+bc+ca),
 (中辺) - (a+b+c)^2 = {a/(b+c)}{b/(c+a)}(a-b)^2 +{b/(c+a)}{c/(a+b)}(b-c)^2 + {c/(a+b)}{a/(b+c)}(c-a)^2,

ここで、
 (a+b+c)/(bb+bc+cc) > (b+c)/(bb+bc+cc) > 1/(b+c),
 (a+b+c)/(cc+ca+aa) > (c+a)/(cc+ca+aa) > 1/(c+a),
 (a+b+c)/(aa+ab+bb) > (a+b)/(aa+ab+bb) > 1/(a+b),
だから
 (左辺) ≧ (中辺).

* (x,y,z) はもっと改良できるかも…

836 :132人目の素数さん:2018/10/25(木) 12:43:20.62 ID:0sa6guuR.net
>>835 *

 x = (b^n - c^n)/(b-c),
 y = (c^n - a^n)/(c-a),
 z = (a^n - b^n)/(a-b),
とすると
 x-y = -(a-b) D_n /,
 y-z = -(b-c) D_n /,
 z-x = -(c-a) D_n /,
ここに
 D_n = det{ [1,1,1] [a,b,c] [a^n,b^n,c^n] }
  = (a-b)(b-c)(c-a) = D_2,   … Vandermonde の行列式

837 :132人目の素数さん:2018/10/26(金) 11:32:25.02 ID:QcEpehDd.net
>>836

3文字のとき
 D_n = det{ [1,1,1] [a,b,c] [a^n,b^n,c^n] }
   = (c-b)a^n + (a-c)b^n + (b-a)c^n,

特性多項式
 (λ-a)(λ-b)(λ-c) = λ^3 -s・λ^2 + tλ -u,
 ただし s = a+b+c,t = ab+bc+ca,u = abc,

漸化式
 D_n = s・D_{n-1} - t・D_{n-2} + u・D_{n-3},

D_n/ = Σ {すべての(n-2)次積}
  … (n-2)個の重複組み合わせに対応

D_0 / = 0,
D_1 / = 0,
D_2 / = 1,
D_3 / = a+b+c = s,
D_4 / = aa+ab+ac+bb+bc+cc = ss-t,
D_5 / = s^3 -2st +u,
D_6 / = s^4 -3sst +tt +2su,
D_7 / = s^5 -4s^3・t +3stt +3ssu -2tu,

838 :132人目の素数さん:2018/10/26(金) 17:18:14.29 ID:QcEpehDd.net
>>837

まづ
 x_1 = y_1 = z_1 = 1,
 x_2 = b+c,y_2 = c+a,z_2 = a+b,
 x_3 = bb+bc+cc,y_3 = cc+ca+aa,z_3 = aa+ab+bb,
 ……
 x_n = b^(n-1) + b^(n-2)c + …… + c^(n-1),
とおく。

ラグランジュの恒等式から
 (ax+by+cz)(a/x + b/y + c/z) - (a+b+c)^2
 = (a/x)(b/y)(x-y)^2 + (b/y)(c/z)(y-z)^2 + (c/z)(a/x)(z-x)^2
 = (D_n/)^2 {(a/x_n)(b/y_n)(a-b)^2 + (b/y_n)(c/z_n)(b-c)^2 + (c/z_n)(a/x_n)(c-a)^2},  >>835

そこで
 (D_n/)/x_n,(D_n/)/y_n,(D_n/)/z_n
がnについて単調増加であることを示そう。

F_n = x_n (D_{n+1}/) - x_{n+1} (D_n/)
= {(b-a)(ab)^2 + (c-b)(bc)^2 + (a-c)(ca)^2} /
= (D_{-n}/)u^n
= Σ {ab,bc,ca の (n-1)次積}
≧ 0,
∴ nについて単調増加。
 (D_{n+1}/) / x_{n+1} ≧ (D_n/) / x_n ≧ …… ≧ (D_2/) / x_2 = 1/(b+c),

これを Nesbitt-Igarashi 列とか呼ぼう。

 F_0 = 0,
 F_1 = 1,
 F_2 = t,
 F_3 = tt -su,
 F_4 = t^3 -2stu +uu,
漸化式
 F_n = t F_{n-1} - su F_{n-2} + uu F_{n-3},

839 :132人目の素数さん:2018/10/26(金) 19:23:36.31 ID:irAy8QvC.net
>>835-836
やっと>>627の証明が分かった!

840 :132人目の素数さん:2018/10/27(土) 00:14:06.23 ID:4A6u4AJ8.net
Nesbitt ってネビットだよな? まさかネスビットって発音するん?

841 :132人目の素数さん:2018/10/27(土) 01:26:02.22 ID:rzBY84ap.net
国にもよるだろうけど、アメリカ(米語)では [nεzbIt]
http://www.clear-english.com/db/nesbitt.html

842 :132人目の素数さん:2018/10/27(土) 02:35:22.24 ID:4A6u4AJ8.net
>>841
うーむ。
Nesbitt's inequality の英語のwikiを見てきたが、どこの国の人か分からんなあ。

ところで Nesbitt's inequality の一般化について、このスレでやったことあったっけ?

843 :132人目の素数さん:2018/10/29(月) 23:18:40.58 ID:YnG/8gLh.net
不等式ぢゃないが、次の等式を手計算で証明するのはキツそうでござるかな?

(6a^2 - 4ab + 4b^2)^3 + (3b^2 + 5ab - 5a^2)^3
= (6b^2 - 4ab + 4a^2)^3 + (3a^2 + 5ab - 5b^2)^3

844 :132人目の素数さん:2018/10/30(火) 03:45:36.60 ID:bXAGzjkG.net
>>843
(6aa-4ab+4bb)^3 - (6bb-4ab+4aa)^3 = (3aa+5ab-5bb)^3 - (3bb+5ab-5aa)^3,

(略証)
x^3 - y^3 = (x-y)(xx+xy+yy),
から
(maa-nab+nbb)^3 - (naa-nab+mbb)^3
= (m-n)(a-b)(a^3+b^3) {(mm+mn+nn)(aa+ab+bb) -3(m+n)n・ab}
= (a-b)(a^3+b^3) {(m^3 - n^3)(aa+ab+bb) - 3(m-n)(m+n)n・ab},

(m,n) = (6,4) (3,-5) のときは
 m^3 - n^3 = 152,
 (m-n)(m+n)n = 80,
となり、相等しい。

845 :132人目の素数さん:2018/10/30(火) 04:35:23.11 ID:XhFYWByL.net
>>844
おぉ有難い。上手にやりましたね。
それにしても、この等式を見つけ出したラマヌジャンは変態ジャン。

846 :132人目の素数さん:2018/10/30(火) 06:59:54.63 ID:XhFYWByL.net
>>844
> = (a-b)(a^3+b^3) {(m^3 - n^3)(aa+ab+bb) - 3(m-n)(m+n)n・ab},

ここは
= (a-b)(a^3+b^3) {(m^3 - n^3)(aa+ab+bb)}
じゃないですか?

847 :132人目の素数さん:2018/11/02(金) 01:22:59.67 ID:im1SI6w9.net
>>843

 6^3 + (-4)^3 + (-3)^3 + (-5)^3 = 0,
 4^3 + (-6)^3 + 5^3 + 3^3 = 0,
から推して
 (6aa+pab+4bb)^3 + (-4aa-pab-6bb)^3 + (-3aa+qab+5bb)^3 + (-5aa-qab+3bb)^3 = 0,
と予想する。(p,q は或る定数)
ab=0 のときは明らか。

6ab(aa-bb){2(5p-4q)(aa+bb) + (84+pp-4qq)ab} = 0,

5p -4q = 0, 84 +pp -4qq = 0,

p = ±4, q=±5  (複号同順)

(例)
a = ±1,b = ±2,p=±4,q=±5 (複号同順)のとき
 ±{30,-36,27,-3} = ±3{10,-12,9,-1}

848 :132人目の素数さん:2018/11/02(金) 02:51:41.47 ID:im1SI6w9.net
>>847

12^3 - 10^3 = 9^3 - 1^3 = 8^3 - (-6)^3 = 728,
のような珍例を「ナニワ数」と云う。…っちゅうのは冗談やけどな。

・系列解は他にもある。
 {7aa-16ab-3bb,14aa+4ab+6bb,-14aa+4ab-6bb,-7aa-16ab+3bb} (Dickson)
 (maa-pab-nbb)^3 + (-maa-pab+nbb)^3 = -6pab(maa-nbb)^2 -2ppp(ab)^3
 m → km,n' → -kn,p' → -p/kk とすれば 6pab(maa+nbb)^2 + 2(p/kk)^3 (ab)^3
 辺々たすと 2p{12mn - (1 - 1/k^6)pp}(ab)^3,
 12mn - (1 - 1/k^6)pp = 0 ならば成立。

 {aa-7ab+63bb,8aa-20ab-42bb,6aa+20ab-56bb,-9aa+7ab-7bb}

http://www.maroon.dti.ne.jp/fermat/dioph1.html

・Fermat cubic surface とか云うらしい。
http://www.math.harvard.edu/~elkies/4cubes.html

849 :132人目の素数さん:2018/11/03(土) 05:17:44.99 ID:/E6xXixt.net
m^3 - n^3 = m’^3 - n’^3 のとき、 ラマヌジャン系列

(maa+pab+nbb)^3 - (naa+pab+mbb)^3
= (m-n)(a^2-b^2){(mm+mn+nn)(a^4+aabb+b^4) + 3(m+n)p ab(a^2+b^2) + 3(pp+mn) aabb}
= (m^3 - n^3) (a^6 - b^6) + 3(m^2-n^2)p ab(a^4 - b^4) + 3(m-n)(pp+mn) aabb(a^2-b^2),

→ m^3 - n^3,(m^2-n^2)p,(m-n)(pp+mn) が等しいとき、相等しい。

(maa+qab-nbb)^3 - (naa+qab-mbb)^3
= (m-n) (a^2+b^2){(mm+mn+nn)(a^4-aabb+b^4) + 3(m+n)q b(a^2-b^2) + 3(qq-mn) aabb}
= (m^3 - n^3) (a^6 + b^6) + 3(m^2-n^2)q ab(a^4 - b^4) + 3(m-n)(qq-mn) aabb(a^2 + b^2),

→ m^3 - n^3,(m^2-n^2)q,(m-n)(qq-mn) が等しいとき、相等しい。

850 :132人目の素数さん:2018/11/04(日) 04:17:06.32 ID:b4HN2Vl2.net
>>848
むむむ、難しいな…

851 :132人目の素数さん:2018/11/05(月) 04:13:53.63 ID:B1F8UTQM.net
〔問題2943〕
a,b,c,d,e は正の実数とする。次を示せ。
 (aa+bb+cc+dd+ee)^5 ≧ 3 abcde (a+b+c+d)(b+c+d+e)(c+d+e+a)(d+e+a+b)(e+a+b+c),

http://suseum.jp/gq/question/2943 (K.Chikaya)

852 :132人目の素数さん:2018/11/05(月) 04:57:42.95 ID:mrtyuQkn.net
>>851
この出題者が出していた大量の不等式の問題は、もう削除されて見れないんだよな。
実に惜しいことをした。

853 :132人目の素数さん:2018/11/08(木) 02:11:08.99 ID:PxlSRNgU.net
bot.62
x,y,z∈[0,1] のとき、sqrt|x-y| + sqrt|y-z| + sqrt|z-x| の最大値

どぉやるんでせうか?

854 :132人目の素数さん:2018/11/09(金) 03:38:15.13 ID:UXVKU4RE.net
(3/4)*(1 + A/H)^2 ≧ (A/G)^3 + (G/H)^3

この手の不等式が胸やけ起こしそうなくらい沢山載ってる本ないかな?

855 :132人目の素数さん:2018/11/09(金) 06:52:20.28 ID:pvdoV3Z4.net
>>853 [62]

yはxとzの中間にあるとする。コーシーで
 (√|x-y| + √|y-z|)^2 ≦ (1+1) (|x-y|+|y-z|) = 2|x-z|,
(左辺) ≦ (1+√2)|z-x| ≦ 1+√2,
等号は(0,1/2,1) etc.

中国MO-2012 Round2-A.3

856 :132人目の素数さん:2018/11/11(日) 22:10:14.90 ID:+dEG2DHU.net
三角形の辺長 a,b,c に対して、
Σ[cyc] (a+b-c)(b+c-a)/(c+a-b) ≧ 3(aa+bb+cc)/(a+b+c).

857 :132人目の素数さん:2018/11/12(月) 08:15:22.35 ID:TKDy5P8X.net
>>856

b+c-a = x, c+a-b = y, a+b-c = z,
とおく。(Ravi変換)
2a = y+z, 2b = z+x, 2c = x+y, a+b+c = x+y+z,

(左辺) = xy/z + yz/x + zx/y = (xxyy+yyzz+zzxx)/xyz,

(右辺) = 3(aa+bb+cc)/(a+b+c)^2 = 6(xx+yy+zz+xy+yz+zx)/{4(x+y+z)},

4(x+y+z)(xxyy+yyzz+zzxx) - 6xyz(xx+yy+zz+xy+yz+zx)
= (3x+y+z)[x(y-z)]^2 + (x+3y+z)[y(z-x)]^2 + (x+y+3z)[z(x-y)]^2 ≧ 0,
かな。

858 :132人目の素数さん:2018/11/12(月) 09:10:27.03 ID:pzDvdx7K.net
Heronを使ったら、問題文はもっと見やすくなりましたな

859 :132人目の素数さん:2018/11/12(月) 19:53:45.29 ID:TKDy5P8X.net
>>854

(3/4)(1 + A/H)^2 - (A/G)^3 - (G/H)^3 -1 = (1/108){(a-b)(b-c)(c-a)/abc}^2 ≧ 0,

(略証)
s = a+b+c, t = ab+bc+ca, u = abc,  = (a-b)(b-c)(c-a),
とおくと
A = s/3, G = u^(1/3), H = 3u/t,
A/H = st/9u, A/G = s/{3u^(1/3)}, G/H = t/{3u^(2/3)},
ゆえ
(左辺) = (3/4)(1+st/9u)^2 - s^3/27u - t^3/27uu -1
 = (1/108uu){(st+9u)^2 -4s^3u -4t^3 -108uu}
 = (1/108uu)竸2,

860 :132人目の素数さん:2018/11/13(火) 09:14:18.15 ID:btM1CEFR.net
>>859
(3/4)*(1 + A/H)^2 ≧ (A/G)^3 + (G/H)^3 + 1

3変数の場合に上式を証明しているけど、これは一般の場合にも成り立つのかな?
>>854では、右辺に +1がないのには意味があるのかな?

861 :132人目の素数さん:2018/11/13(火) 11:28:53.25 ID:UigxEbMv.net
>>860
n≧4 では不成立かも。
(1,1,1,10^4) とか (1,1,1,10^(-4)) とか…

862 :132人目の素数さん:2018/11/13(火) 12:55:32.39 ID:btM1CEFR.net
>>854
> (3/4)*(1 + A/H)^2 ≧ (A/G)^3 + (G/H)^3

これじゃの
Crux Mathematicorum, P.74, 1834.
http://www.imomath.com/othercomp/Journ/ineq.pdf

863 :132人目の素数さん:2018/11/13(火) 22:03:59.25 ID:btM1CEFR.net
[2] かんどころ P.121定理6.7 は、証明ついてないようだけど、どうやればいいか分かりますか?

864 :132人目の素数さん:2018/11/14(水) 00:54:00.60 ID:uakH23jG.net
>>860
右辺に +1 が無いと緩くなります。

2変数の場合は
 (3/4)(1 + A/H)^2 - (A/G)^2 - (G/H)^2 -1 = (3a+b)(a+3b){(a-b)/8ab}^2 ≧ 0,

∵ (A/G)^2 = (G/H)^2 = A/H = (a+b)^2 /4ab,

865 :132人目の素数さん:2018/11/14(水) 03:43:20.66 ID:uakH23jG.net
余談ですが、n変数の (A-G)/(G-H) の下限は
 n=2  1.0
 n=3  0.90096030150908885
 n=4  0.7761577683742073233
 n=5  0.67617485
 n=6  0.59845640
 n=7  0.53716474
 n=8  0.48781223
 n=9  0.44727765
 n=10  0.41339822
ぐらいかな。

http://suseum.jp/gq/question/2646, 2948

866 :132人目の素数さん:2018/11/14(水) 04:26:30.36 ID:fnsD9k3Y.net
>>863, >>865
A-Gと言えば、Jacobsthalくらいしか思いつかないなあ

867 :132人目の素数さん:2018/11/14(水) 14:30:21.97 ID:fnsD9k3Y.net
a≧b≧0, x≧y≧0 に対して、
(ax+y+c)(x+by+c)≧{(a+1)x+c}{(b+1)y+c}.

868 :132人目の素数さん:2018/11/14(水) 14:58:31.03 ID:fnsD9k3Y.net
>>867
あまりにもショボすぎるので、改造してみた。

a,b,c,d,x,y,z∈R, a≧d≧0, b≧c≧0, x≧y≧0 に対して、
(ax+cy+z)(bx+dy+z)≧{(a+b)x+z}{(c+d)y+z}.

後ろのzも pz+qw, rz+sw にできぬか?

                l三`ー 、_;:;:;:;:;:;:j;:;:;:;:;:;:_;:;:;_;:-三三三三三l
               l三  r=ミ''‐--‐';二,_ ̄    ,三三三彡彡l_   この感じ・・・・
              lミ′   ̄    ー-'"    '=ミニ彡彡/‐、ヽ
                  l;l  ,_-‐ 、    __,,.. - 、       彡彡彳、.//  
_______∧,、_‖ `之ヽ、, i l´ _,ィ辷ァ-、、   彡彡'r ノ/_ ______
 ̄ ̄ ̄ ̄ ̄ ̄ ̄'`'` ̄ 1     ̄フ/l l::. ヽこ~ ̄     彡彳~´/  ̄ ̄ ̄ ̄ ̄ ̄
                 ヽ   ´ :l .l:::.         彡ィ-‐'′
                ゝ、  / :.  :r-、        彡′
              / ィ:ヘ  `ヽ:__,ィ='´        彡;ヽ、
          _,,..-‐'7 /:::::::ヽ   _: :_    ヽ      ィ´.}::ヽ ヽ、
      _,-‐'´    {  ヽ:::::::::ヘ `'ー===ー-- '   /ノ /::::::ヘ, ヽー、

869 :132人目の素数さん:2018/11/14(水) 16:19:40.82 ID:fnsD9k3Y.net
できた ( ゚∀゚) ウヒョッ

a,b,c,d,p,q,r,s,x,y,z.w∈R,
a≧d≧0, b≧c≧0, p≧s≧0, q≧r≧0, x≧y≧0, z≧w≧0 に対して、
(ax+cy+pz+rw)(bx+dy+qz+sw)≧{(a+b)x+(p+q)z}{(c+d)y+(r+s)w}.

870 :132人目の素数さん:2018/11/14(水) 16:53:56.03 ID:fnsD9k3Y.net
話を元に戻すと、>>867 を使ったAM-GMの証明 ([2] かんどころP.118)で、
1回目に>>867を使うところは分かる。

(a_1 + a_2 + (a_3+…+a_n))(a_1 + a_2 + (a_3+…+a_n))
≧(2a_1 + (a_3+…+a_n))(2a_2 + (a_3+…+a_n))

2回目に>>867を使うところ、どこが対応しているのか分からんのですが、どうなってるのですか?

(2a_1 + (a_3+…+a_n))(a_1 + a_2 + (a_3+…+a_n))
≧(3a_1 + (a_4+…+a_n))(2a_3 + (a_2+a_4+…+a_n))

以下続けて (k*a_1+ a_{k+1}+…+a_n) と (a_1 + a_2 + (a_3+…+a_n)) に>>867を使って
最終的に n*a_1 と S-a_1+a_k (k=2.3.…,n) になるまで続けるんだけど、そこが分かりませぬ。

871 :132人目の素数さん:2018/11/15(木) 01:35:19.26 ID:BIkI04V5.net
>>870

>>867 を使わなくても出せるでござる。
A-S≧0,d≧0 のとき
 (A-d)S - A(S-d) = d(A-S) ≧ 0,

ここで
 S = a_1 + a_2 + … + a_n,
 A = k・a_1 + a_{k+1} + … + a_n, (k=n のとき A=n・a_1)
 d = a_1 - a_k ≧ 0,
とおいて
{(k-1)a_1 +(a_k + … +a_n)}S - (k・a_1 +a_{k+1} + … +a_n)(S -a_1 +a_k) = (a_1 -a_k)(k・a_1 -S) ≧ 0,    (k=2,3,…,n)

872 :132人目の素数さん:2018/11/15(木) 01:39:19.56 ID:BIkI04V5.net
>>866
〔Jacobsthalの不等式〕
(n-1)個の正の実数 x_1, x_2, …, x_(n-1) の相加平均をA '、相乗平均をG ' とする。
それに x_n (>0) を追加した n個組の相加平均をA_n、相乗平均をG_n とする。このとき
 n(A_n - G_n) ≧ (n-1)(A '-G '),  …[1]
 (A_n/G_n)^n ≧ (A '/G ')^(n-1),  …[2]

(略証)
A_n, G_n, x_n を A, G, x と略記する。
[1]
 n A - (n-1)A '= x,
 n G - (n-1)G '= G '{n(G/G ') - (n-1)} ≦ G '(G/G ')^n = x, (← Bernoulli)
辺々引く。
[2]
 A '(A/A ')^n ≧ A '{n(A/A ') - (n-1)} = n - (n-1)A '= x, (← Bernoulli)
 G '(G/G ')^n = x,
辺々割る。

[1] または [2] を n=1 まで繰り返すと A ≧ G が出る。

ニコニコ大百科
http://dic.nicovideo.jp/a/jacobsthalの不等式

873 :132人目の素数さん:2018/11/15(木) 05:32:47.02 ID:Kjq0ut8v.net
>>871
ありがとうございます。
なるほど、>>867を使わずにできますね。
>>871の不等式を使って、残りも同様にしていけばいいんですね。
つまり prime132氏が新証明(?)をしたわけですな。

Guha が1967年に>>867を繰り返し使ってAM-GMを証明した方法も知りたい。
「Guha 1967 AM-GM」をgoogleで検索して一番上に出る
When Less is More: Visualizing Basic Inequalities
のPP.31-32に n=4のときに、Guha's inequality を繰り返し使った例があり、
それを見ても、2回目以降にどう使っているのか分かりません。

Guha's inequality
a≧0, p≧q≧0, x≧y≧0, then
(px+y+a)(x+qy+a)≧((p+1)x+a)((q+1)y+a).

(4A_4)^4
= (a+b+c+d)(a+b+c+d)(a+b+c+d)(a+b+c+d)
≧ (2a+c+d)(2b+c+d)(a+b+c+d)(a+b+c+d) …(1)
≧ (3a+d)(2b+c+d)(b+2c+d)(a+b+c+d)   …(2)
≧ 4a(2b+c+d)(b+2c+d)(b+c+2d)      …(3)
≧ 4a(3b+d)(3c+d)(b+c+2d)         …(4)
≧ 4a(4b)(3c+d)(c+3d)            …(5)
≧ 4a(4b)(4c)(4d)               …(6)
= (4G_4)^4

と書かれているんですが、(1)は(p,q,x,y,a) = (1,1,a,b,c+d)で理解できる。
(2)〜(6)はどう適用したのか謎。
たとえば(2)で (2a+c+d)(a+b+c+d)≧(3a+d)(b+2c+d) となるには
(p,q,x,y,a)に何を対応させているのか?x=a, y=c 以外が謎。
左辺第1因子の形からp=2,q=1でないといけないけど、第2因子に2がない。

874 :132人目の素数さん:2018/11/15(木) 06:40:30.77 ID:Kjq0ut8v.net
(3)→(4)は (p,q,x,y,a) = (2,2,b,c,d)で
(5)→(6)は (p,q,x,y,a) = (3,3,c,d,0)か。

じゃあ、残り3ヵ所は、どう適用したんだろう?
(1)→(2) (2a+c+d)(a+b+c+d)≧(3a+d)(b+2c+d)
(2)→(3) (3a+d)(a+b+c+d)≧(4a)(b+c+2d)
(4)→(5) (3b+d)(b+c+2d)≧(4b)(c+3d)

実は使ってないってオチなのか?

875 :132人目の素数さん:2018/11/15(木) 08:38:51.30 ID:Kjq0ut8v.net
本人の論文を探すしかないな。
U.C.Guha, arithmetric mean-geometric mean inequality, Mathematical Gazette, 51(1967),pp.14-146

というのは分かったけど、ネットに転がってないかな

876 :132人目の素数さん:2018/11/15(木) 09:20:07.76 ID:Kjq0ut8v.net
Handbook of Means and Their Inequalities, pp.101-102 を見たら、
Guhaの不等式を使った証明の数式部分が、省略している部分も含めて
[2] かんどころ P.118と全く同じだった。

877 :132人目の素数さん:2018/11/17(土) 07:54:31.62 ID:js5kwOKA.net
(1)
|x|≦1, |y|≦1 (x,y∈R) に対して、
0≦ xx + yy - 2xxyy + 2xy*√{(1-xx)(1-yy)} ≦1.

(2)
m>n>1 (m,n∈Z) に対して、
(m+n+1)!/(m!*n!) > {(m+n)^{m+n}}/{(m^m)(n^n)} > 2^{2n-1}.

878 :132人目の素数さん:2018/11/17(土) 11:12:15.05 ID:js5kwOKA.net
(3)
a,b∈C に対して、
|a+b|/(1+|a+b|) < (|a|+|b|)/(1+|a|+|b|) < |a|/(1+|a|) + |b|/(1+|b|)

879 :132人目の素数さん:2018/11/18(日) 02:10:29.27 ID:ENzLbcND.net
>>877
(1)
 x = sinθ, y = sinφ (-π/2≦θ,φ≦π/2) とおく。
 √(1-xx) = cosθ, √(1-yy) = cosφ,
 x√(1-yy) + y√(1-xx) = sinθcosφ + sinφcosθ = sin(θ+φ),
両辺を2乗する。

(2)(左)
 log{(m+n+1)!} -(m+n)log(m+n) > (3/2)log(m+n) -(m+n) +0.8918
 log(m!) - m・log(m) < (1/2)log(m) -m +1,
 log(n!) - n・log(n) < (1/2)log(n) -n +1,
辺々引くと
 log{(m+n+1)!} -log(m!) -log(n!) -(m+n)log(m+n) +m・log(m) +n・log(n)
 > (3/2)log(m+n) - (1/2)log(mn) - 1.1082
 > (1/2)log(m+n) + (1/2)log{(m+n)^2 /4mn} + log(2) - 1.1082
 ≧ (1/2)log(m+n) - 0.41505
 ≧ (1/2)log(3) - 0.41505 (m+n≧3)
 = 0.549306

(2)(右)
 (m+n)^{m+n} = (m+n)^{m-n} (m+n)^{2n}
 ≧ m^{m-n} (4mn)^n
 = m^m (4n)^n,

∴ (m+n)^(m+n)/(m^m・n^n) ≧ 4^n,

>>878
(3)
 x/(1+x) は x≧0 で単調増加 (x∈R)
 |a+b| ≦ |a| + |b|
∴ φ(|a+b|) ≦ φ(|a|+|b|)
 = |a|/(1+|a|+|b|) + |b|/(1+|a|+|b|)
 ≦ φ(|a|) + φ(|b|),

880 :132人目の素数さん:2018/11/18(日) 02:17:09.96 ID:ENzLbcND.net
>>879 (2) (左)

〔補題〕
log(m!) < (m+1/2) log(m) -m+1,
(略証)
{log(k-1) + log(k)}/2 < ∫[k-1,k] log(x)dx より
log(m!) = Σ[k=2,m] log(k)
< ∫[1,m] log(x)dx + (1/2)log(m)
= [ x・log(x) -x ](x=1,m) + (1/2)log(m)
= (m+1/2)log(m) -m +1,

log(n!) < (n +1/2) log(n) -n+1,
(略証)
{log(k-1) + log(k)}/2 < ∫[k-1,k] log(x)dx より
log(n!) = Σ[k=2,n] log(k)
< ∫[1,n] log(x)dx + (1/2)log(n)
= [ x・log(x) -x ](x=1,n) + (1/2)log(n)
= (n+1/2)log(n) -n +1,

log{(m+n+1)!} > (m+n+3/2) log{(m+n+1)!}
= Σ[k=2,m+n+1] log(k)
> ∫[3/2,m+n+3/2] log(x)dx
= [ x・log(x) -x ](x=3/2,m+n+3/2)
= (m+n+3/2) log(m+n+3/2) -(m+n) -(3/2)log(3/2)
> (m+n+3/2) log(m+n) +(3/2) - (m+n) - (3/2)log(3/2)
= (m+n+3/2) log(m+n) - (m+n) + 0.8918023378

881 :132人目の素数さん:2018/11/18(日) 02:23:36.94 ID:ENzLbcND.net
>>879 (2)(左)

log {(m+n+1)!} > (m+n+3/2) log(m+n) - (m+n) + 0.8918
(略証)
log(k) > ∫[k-1/2,k+1/2] log(x)dx より
log{(m+n+1)!} = Σ[k=2,m+n+1] log(k)
> ∫[3/2,m+n+3/2] log(x)dx
= [ x・log(x) -x ](x=3/2,m+n+3/2)
= (m+n+3/2) log(m+n+3/2) -(m+n) -(3/2)log(3/2)
> (m+n+3/2) log(m+n) +(3/2) - (m+n) - (3/2)log(3/2)
= (m+n+3/2) log(m+n) - (m+n) + 0.8918023378

882 :132人目の素数さん:2018/11/18(日) 20:34:10.43 ID:Fcj0HO3Z.net
>>877
(1)
右辺-中辺 = [xy - √{(1-xx)(1-yy)}]^2 ≧0,
中辺-左辺 = {x√(1-yy) + y√(1-xx)}^2 ≧0.

 (゚∀゚ )
  ノヽノ) =3 プゥ
  くく

883 :132人目の素数さん:2018/11/18(日) 20:43:09.25 ID:Fcj0HO3Z.net
>>882
|x|≦1, |y|≦1の条件なんて要らなかったのでは?

884 :132人目の素数さん:2018/11/18(日) 20:45:54.55 ID:Fcj0HO3Z.net
根号内の問題で必要だったか。

885 :132人目の素数さん:2018/11/18(日) 23:09:08.22 ID:Fcj0HO3Z.net
[2016東大]
正の実数 x に対して (1 + 1/x)^x < e < (1 + 1/x)^{x + 1/2}.

886 :132人目の素数さん:2018/11/18(日) 23:25:53.36 ID:Fcj0HO3Z.net
a,b,c>0, a+b+c=1に対して、(1+ 1/a)(1+ 1/b)(1- 1/c) の取りうる値の範囲を求めよ。

887 :132人目の素数さん:2018/11/19(月) 00:32:28.39 ID:eL1RQpps.net
>>865
 n=4 のとき、(A-G)/(G-H) ≧ 9/16
 CGMO-2011 A.4
 inequalitybot [35]

888 :132人目の素数さん:2018/11/19(月) 01:54:17.81 ID:eL1RQpps.net
〔問題168〕
a,b,c>0 のとき
 (aa-bc)(b+c)^r + (bb-ca)(c+a)^r + (cc-ab)(a+b)^r ≧ 0, (0<r<1)
                          ≦ 0, (r>1, r<0)
  V.Cirtoaje:"Algeblaic inequalities"、1-1-7
 inequalitybot [168]

889 :132人目の素数さん:2018/11/19(月) 02:03:20.14 ID:eL1RQpps.net
>>888

 x = (b+c)^r,
 y = (c+a)^r,
 z = (a+b)^r,
とおくと
 a = (y^{1/r} + z^{1/r} - x^{1/r})/2,
 b = (z^{1/r} + x^{1/r} - y^{1/r})/2,
 c = (x^{1/r} + y^{1/r} - z^{1/r})/2,

 aa-bc = {y^(2/r) +z^(2/r) -x^(1/r)[y^(1/r) + z^(1/r)]}/2,
 bb-ca = {z^(2/r) +x^(2/r) -y^(1/r)[z^(1/r) + x^(1/r)]}/2,
 cc-ab = {x^(2/r) +y^(2/r) -z^(1/r)[x^(1/r) + y^(1/r)]}/2,

(左辺) = (aa-bc)x + (bb-ca)y + (cc-ab)z
 = {x^(2/r)y +xy^(2/r) -(x+y)(xy)^(1/r)}/2 + ……
 = (x^{1/r} - y^{1/r})(x^{1/r -1} - y^{1/r -1})xy + ……

890 :132人目の素数さん:2018/11/19(月) 09:27:44.83 ID:eL1RQpps.net
>888 訂正

〔問題168〕
a,b,c>0 のとき
 (aa-bc)(b+c)^r + (bb-ca)(c+a)^r + (cc-ab)(a+b)^r > 0, (r<1)
                          < 0, (r>1)
                          = 0, (r=1)

891 :132人目の素数さん:2018/11/19(月) 09:57:43.41 ID:eL1RQpps.net
>>885
y>0 とする。
 (1 + y/2)^2 > 1+y > 1,
∴ 1/(1+y/2)^2 < 1/(1+y) < 1,
0〜y で積分すると
 y/(1+y/2) < log(1+y) < y,
∴ (1+y)^(1/y) < e < (1+y)^(1/y + 1/2),
y=1/x とおく。

892 :132人目の素数さん:2018/11/19(月) 10:04:46.71 ID:Merao6vt.net
>>879
右辺は甘かったか…。

>>881
> log(k) > ∫[k-1/2,k+1/2] log(x)dx

これは明らかなんですか?

893 :132人目の素数さん:2018/11/19(月) 10:21:14.09 ID:eL1RQpps.net
>>877 (1) >>882

(x+iz)(y+iw) = (xy-zw) + i(xw+yz),

(xx+zz)(yy+ww) = (xy-zw)^2 + (xw+yz)^2,

z=√(1-xx),w=√(1-yy) とおく。

894 :132人目の素数さん:2018/11/19(月) 10:34:04.63 ID:eL1RQpps.net
>>892

log(k) > (1/2)log(kk-dd) = {log(k+d) + log(k-d)}/2,

y=log(x) は上に凸だから、x=kでの接線より下側にある。
k-d<x<k+d かつ接線より下の台形の面積は(接線の傾きによらず)2d log(k)
∴ 2d log(k) > ∫[k-d,k+d] log(x)dx

895 :132人目の素数さん:2018/11/19(月) 10:45:40.04 ID:Merao6vt.net
>>894
なるほど、ありがとうございます。

>>893
そんなカラクリがあったとは…。

896 :132人目の素数さん:2018/11/19(月) 12:18:26.26 ID:Merao6vt.net
>>880-881
log{(m+n+1)!} の評価は、log k > ∫[k-1,k] log(x)dx でもいけそうな排気ガス…

897 :132人目の素数さん:2018/11/19(月) 13:19:03.28 ID:Merao6vt.net
>>896
無理だった。

>>881
> = (m+n+3/2) log(m+n+3/2) -(m+n) -(3/2)log(3/2)
> > (m+n+3/2) log(m+n) +(3/2) - (m+n) - (3/2)log(3/2)

の部分で、以下はどうやって分かるのですか?
(m+n+3/2) log(m+n+3/2) > (m+n+3/2) log(m+n) + (3/2)

898 :132人目の素数さん:2018/11/19(月) 14:36:05.79 ID:Merao6vt.net
>>887
少し違うような気がするが、これかな?
http://www.imojp.org/challenge/old/cgmo10q.html

899 :132人目の素数さん:2018/11/19(月) 23:39:54.57 ID:eL1RQpps.net
>>897

(N+d) log(N+d) - (N+d) log(N)
= -(N+d)log{N/(N+d)}
= -(N+d) log{1 - d/(N+d)}
≧ -(N+d) {-d/(N+d)}
= d,

900 :132人目の素数さん:2018/11/20(火) 00:02:55.19 ID:5+1z6eBQ.net
>>899
さんくす。

x=a(>0) における log x の接線を考えて、
 (x-a)/a + log a ≧ log x.
x=1, a = (N+d)/N を代入すればいいのかな。

901 :132人目の素数さん:2018/11/20(火) 00:11:33.44 ID:vn8Rd3zq.net
>>898
3.
{1/a,1/b,1/c,1/d} について 16A + 9H ≧ 25G,
>>887 と同じですね。

902 :132人目の素数さん:2018/11/20(火) 02:59:32.68 ID:5+1z6eBQ.net
>>877(2)左側

0≦x≦1 において f(x) = x^m (1-x)^n は x = m/(m+n) で最大値をとる.

I(m,n) = ∫[0,1] f(x)dx とおくと, I(m,n) ≦ f(m/(m+n)) より
(m!*n!)/{(m+n+1)!} ≦ {(m^m)(n^n)}/{(m+n)^{m+n}}

[東京医科歯科大学2013数学第3問]

903 :132人目の素数さん:2018/11/21(水) 16:55:51.41 ID:LdWYnCJ+.net
>>856-857
大昔のPutnumに、これより弱い不等式があったよね。

>>885
Moreau's inequality が思い浮かぶと同時に、一松先生を思い出す。(謎掛け)

904 :132人目の素数さん:2018/11/21(水) 21:37:39.12 ID:LdWYnCJ+.net
三角形の辺長 a,b,c に対して、
(1) Σ[cyc] aa(b+c-a) ≦3abc.
(2) Σ[cyc] aab(a-b) ≧0.

そもそも(1)は辺長でなくても非負実数で成り立つでおじゃるな。

905 :132人目の素数さん:2018/11/22(木) 00:31:23.80 ID:x/Au2Ugh.net
>>904
(1)
(右辺) - (左辺) = a(a-b)(a-c) + b(b-c)(b-a) + c(c-a)(c-b) = F1(a,b,c) ≧ 0,
 △である必要はない。

(2)
a = y+z、b = z+x、c = x+y とおく。(Ravi変換)
(左辺) = aab(a-b) + bbc(b-c) + cca(c-a)
= 2(xyyy+yzzz+zxxx) - 2xyz(x+y+z)
= (2/7)(2xyyy +yzzz +4zxxx -7xxyz) + cyc.
≧ 0,
 IMO-1983, A.6
 文献[9] 佐藤(訳) (2013) 問題2.24
 Inequalitybot [24]

906 :132人目の素数さん:2018/11/22(木) 01:53:17.42 ID:dl5e5cbL.net
>>856-857, >>903
Putnum 1966.A2
http://webee.technion.ac.il/people/aditya/www.kalva.demon.co.uk/putnam/putn66.html

907 :132人目の素数さん:2018/11/22(木) 01:56:17.94 ID:dl5e5cbL.net
>>886
ヒント:AM-GM

908 :132人目の素数さん:2018/11/22(木) 02:23:23.48 ID:x/Au2Ugh.net
>>856 >>903 >>906

27th Putnum-1966
A2.
A triangle has sides a, b, c. The radius of the inscribed circle is r. Show that
 1/(b+c-a)^2 + 1/(c+a-b)^2 + 1/(a+b-c)^2 ≧ 1/(2r)^2,

909 :132人目の素数さん:2018/11/22(木) 02:58:32.21 ID:dl5e5cbL.net
>>855
中国MOの問題が載っているリンクがあれば教えてください。

910 :132人目の素数さん:2018/11/22(木) 03:48:05.20 ID:dl5e5cbL.net
x, y≧1 に対して、x√(y-1) + y√(x-1) ≦xy.

911 :132人目の素数さん:2018/11/22(木) 07:37:55.95 ID:LiApVatG.net
>>909
https://artofproblemsolving.com/community/c3167_china_contests

912 :132人目の素数さん:2018/11/22(木) 13:09:54.80 ID:dl5e5cbL.net
>>911
かたじけのうございます。

913 :132人目の素数さん:2018/11/22(木) 16:30:26.57 ID:x/Au2Ugh.net
>>910

√(x-1) = X,
√(y-1) = Y,
とおく。
XX-X+1 ≧ X,
YY-Y+1 ≧ Y,
(右辺) - (左辺) = xy - x√(y-1) - y√(x-1)
 = (XX+1)(YY+1) - (XX+1)Y - (YY+1)X
 = {(XX+1)-X} {(YY+1)-Y} - XY
 ≧ XY - XY
 = 0,

あるいは
x = (cosh u)^2, y = (cosh v)^2 とおく。

914 :132人目の素数さん:2018/11/22(木) 17:07:41.43 ID:x/Au2Ugh.net
>>910 >>913
x-2X ≧ 0,
y-2Y ≧ 0,
(右辺) - (左辺) = xy - xY - yX
 = x(y-2Y)/2 + y(x-2X)/2
 ≧ 0,
でもいいけど…

915 :132人目の素数さん:2018/11/23(金) 00:28:05.76 ID:hLVWs+G2.net
a, b >0 に対して、aabb(aa+bb-2)≧(ab-1)(a+b).

非同次は苦手でおじゃる。

916 :132人目の素数さん:2018/11/23(金) 10:42:23.96 ID:hLVWs+G2.net
三角形の辺長a,b,cに対して、a(b+c-a)<2bc.

917 :132人目の素数さん:2018/11/23(金) 11:36:47.43 ID:hLVWs+G2.net
>>916 は「美しい不等式pp.69-70」にあるが、証明が美しくないよな。
普通に差をとったら綺麗にできるのになあ。

918 :132人目の素数さん:2018/11/23(金) 15:05:23.08 ID:eRIDJVQi.net
>>915

a, b, c >0 に対して、aabb (aa+bb-2cc) ≧ (ab-cc) (a+b) ccc,

これで どうじゃ(同次ゃ)

919 :132人目の素数さん:2018/11/23(金) 15:20:30.99 ID:eRIDJVQi.net
>>915 >>918

(左辺) - (右辺)
≧ (a+b)(ab)^(5/2) - (a+b)(ab)^(3/2)・cc - (ab-cc)(a+b)c^3
= (a+b)(ab - cc) [(ab)^{3/2} - c^3]
≧ 0,

920 :132人目の素数さん:2018/11/23(金) 15:59:31.52 ID:eRIDJVQi.net
>>916 >>917
 x = (b+c-a)/2, y = (c+a-b)/2, z = (a+b-c)/2,
とおく。(Ravi変換)
普通に差をとったら出来ますね^^
 2bc - a(b+c-a) = 2(x+z)(x+y) - 2x(y+z) = 2xx + 2yz ≧ 0,

文献[9] 佐藤(訳) §2.2 例2.2.1 p.69 (2013)

921 :132人目の素数さん:2018/11/23(金) 16:55:45.35 ID:hLVWs+G2.net
>>917 >>920
なるほど、Ravi変換は無敵でござるな。

この問題を問題集で見て感じたのは、三角形の成立条件を使った例なのに、
三角形の成立条件が一目で分かりにくい小汚い計算をしていた点。

b,cについて対称だから b≧cとする所まではいい。次のようにした方が美しいと思わん?

aが最大または最小のとき、
 2bc - a(b+c-a) = bc + (a-b)(a-c) > 0.

aがbとcの間の数のとき、b≧a≧cだから、
 2bc - a(b+c-a) = (a-c)(c+a-b) + c(b+c-a) > 0.

922 :132人目の素数さん:2018/11/23(金) 21:00:03.01 ID:hLVWs+G2.net
>>915 >>918-919
難しすぎる。が、改造してみた。

a, b >0 に対して、ab(aa+bb-2)≧(ab-1)(a+b).

923 :132人目の素数さん:2018/11/24(土) 01:13:59.36 ID:OpCiwKZy.net
>>922
a, b, c, r > 0 に対して (ab)^{r+1/2} (aa+bb-2cc) ≧ (ab-cc) (a+b) (cc)^r,

(略証)
(左辺) - (右辺)
≧ (a+b)(ab)^{r+1} - (a+b)(ab)^r・cc - (ab-cc)(a+b)(cc)^r
= (a+b)(ab-cc) [(ab)^r - (cc)^r]
≧ 0,

924 :132人目の素数さん:2018/11/24(土) 01:30:44.11 ID:R0eGczxp.net
>>915 + >>922
a, b >0 に対して、aabb(aa+bb-2) ≧ ab(ab-1)(a+b) ≧ (ab-1)(a+b).

つまり改造後の不等式は、より強い式でござる ( ゚∀゚) ウヒョッ!
調子に乗って、さらに改造すると、

a, b >0 に対して、{√(ab)}*(aa+bb-2) ≧ (ab-1)(a+b).

925 :132人目の素数さん:2018/11/24(土) 01:31:26.25 ID:R0eGczxp.net
>>923
すでに改造済みだったか…

926 :132人目の素数さん:2018/11/24(土) 04:44:14.38 ID:OpCiwKZy.net
>>923
(左辺) ≧ (a+b)(ab)^{r+1} - (a+b)(ab)^r・cc
= (a+b)(ab-cc)(ab)^r
≧ (a+b)(ab-cc)(ab)^(r-1) cc
≧ ……
≧ (a+b)(ab-cc)(ab)(cc)^(r-1)
≧ (a+b)(ab-cc)(cc)^r
= (右辺),

927 :132人目の素数さん:2018/11/24(土) 22:03:32.05 ID:R0eGczxp.net
(1) a,b,c∈Rに対して、
  (a^5+b^5+c^5)^2 ≧ 3abc(a^7+b^7+c^7).
(2) x,y,z>0, xyz=1に対して、
  (x^10+y^10+z^10)^2 ≧ 3(x^13+y^13+z^13).

928 :132人目の素数さん:2018/11/24(土) 22:24:35.03 ID:R0eGczxp.net
>>522 (D1)
http://kskedlaya.org/putnam-archive/1999s.pdf

929 :132人目の素数さん:2018/11/25(日) 00:37:10.99 ID:AuW29Ma5.net
>>522 (D1)

 f '(x) < (3/2)^(1/3) f(x) = 1.14471424 f(x),

60th Putnam (1999/Dec/04) B-4

〔補題〕
 lim(x→-∞) F(x) ≧0,
 F '(x) > 0 for all x∈R
ならば
 F(x) > 0 for all x∈R
(背理法で示せる。)

 g(x) = (3/2)f(x)^3 - {f '(x)}^3,
とおくと
 g '(x) = 3f '(x) {(3/2)f(x)^2 - f '(x)f "(x)} ≡ 3f '(x) h(x),
 h '(x) = 3f(x) f '(x) - f '(x) f '''(x) - {f "(x)}^2
  = f '(x) {f(x) - f '''(x)} + {2f(x) f '(x) - [f "(x)]^2}
  ≡ f '(x) {f(x) - f '''(x)} + L(x),
 L '(x) = 2f '(x){f(x) - f '''(x)} + {f '(x)}^2 > 0,
補題により
 L(x) = 2f(x) f '(x) - [f "(x)]^2 > 0,
 h '(x) > 0,
補題により
 h(x) = (3/2)f(x)^2 - f '(x)f "(x) > 0,
 g '(x) > 0,
補題により
 g(x) = (3/2)f(x)^3 - {f '(x)}^3 > 0,
 f '(x) < (3/2)^(1/3) f(x),

930 :132人目の素数さん:2018/11/25(日) 01:05:38.13 ID:AuW29Ma5.net
>>522 (D1) >>928 >>929

Inequalitybot [143]

931 :132人目の素数さん:2018/11/25(日) 01:38:03.75 ID:AuW29Ma5.net
>>927 (2)
 >>515 を参照ウオ。

〔補題〕  >>533 >>534
 (A^3+B^3+C^3)^2 ≧ (AB+BC+CA)(A^4+B^4+C^4),

932 :132人目の素数さん:2018/11/25(日) 16:13:30.17 ID:AuW29Ma5.net
>>929 訂正

 L '(x) = 2f "(x) {f(x) - f '''(x)} + 2{f '(x)}^2 > 0,
でした、

933 :132人目の素数さん:2018/11/25(日) 22:54:06.18 ID:Lv92uXqz.net
(1)
a,b,c∈R, r>0 に対して、
a(b+c)^r + a(b+c)^r + a(b+c)^r ≧0.

(2)
a,b,c>0 に対して
√(aa+ab+bb) + √(bb+bc+cc)√(cc+ca+aa) ≧ 3√(ab+bc+ca).

(3)
a,b,c∈R に対して、次式をみたすkの最大値を求めよ.
abc(a+b+c)^2 ≧ k(ab+bc+ca)(a+b-c)(b+c-a)(c+a-b).

------------------------------------------
http://www.imomath.com/othercomp/Journ/ineq.pdf
1831, 1394, 1120

934 :132人目の素数さん:2018/11/25(日) 23:17:46.77 ID:AuW29Ma5.net
>>931

>>533 >>534 より

(A^3 + B^3 + C^3)^2 - (AB+BC+CA)(A^4 + B^4 + C^4) = F_0(A, B, C) F_0(AA, BB, CC) + F_1(BC, CA, AB) ≧ 0,

F_n(x,y,z) = (x^n)(x-y)(x-z) + (y^n)(y-z)(y-x) + (z^n)(z-x)(z-y) ≧ 0,

935 :132人目の素数さん:2018/11/26(月) 04:45:07.58 ID:JAq6ovHt.net
>>933

1831. (p.74)
 a,b,c ∈ R, r>0 は奇数 のとき
 a(a+b)^r + b(b+c)^r + c(c+a)^r ≧ 0,

1394. (p.51)
(略解)
AM-GM で
(左辺) ≧ 3{(ab+bb+aa)(bb+bc+cc)(aa+cc+ca)}^(1/6)
 ≧ 3√(ab+bc+ca),   (←コーシー)

1120. (p.34)
(略解)
a,b,c ≧ 0 とする。
{b+c-a, c+a-b, a+b-c} の中の2つの和は非負だから、負であるものは高々1つ。
いずれかが負の場合は、任意のk>0 について
(左辺) ≧ 0 ≧ (右辺).
以下では b+c-a≧0, c+a-b≧0, a+b-c≧0, k=3 とする。
(左辺) - (右辺) = abc(a+b+c)^2 - 3(ab+bc+ca)(a+b-c)(b+c-a)(c+a-b)
= c(a+b-c)(3a+3b-c)(a-b)^2 + a(b+c-a)(3b+3c-a)(b-c)^2 + b(c+a-b)(3c+3a-b)(c-a)^2 ≧ 0,
等号成立は a=b=c のとき。

936 :132人目の素数さん:2018/11/26(月) 05:26:09.09 ID:JAq6ovHt.net
>>933

1126. (p.34)
 0 < x ≦ 1 に対して次を示せ。
 x < sinh(x) < 3x/{1+1+√(1-xx)} < tan(x),

1270. (p.44)
 x>0 に対して次を示せ。
 x/√(1+xx) < tanh(x) < √{1-exp(-xx)} < x,

937 :132人目の素数さん:2018/11/26(月) 06:28:31.22 ID:P3VGSuRj.net
>>933 >>935
> (2)
> a,b,c>0 に対して
> √(aa+ab+bb) + √(bb+bc+cc)√(cc+ca+aa) ≧ 3√(ab+bc+ca).

my collection に次式を発見、しかし詳細不明。
a,b,c∈R に対して、
√(aa+ab+bb) + √(bb+bc+cc)√(cc+ca+aa) ≧ (3/2)*(a+b+c).

938 :132人目の素数さん:2018/11/26(月) 09:39:29.12 ID:P3VGSuRj.net
>>859-861
n≧4では、逆向きが成り立つという仮説を立ててみた。
n=2,3のとき、 (3/4)*(1 + A/H)^2 ≧ (A/G)^n + (G/H)^n + 1
n≧4のとき、 (3/4)*(1 + A/H)^2 ≦ (A/G)^n + (G/H)^n + 1

939 :132人目の素数さん:2018/11/26(月) 11:29:31.39 ID:P3VGSuRj.net
>>887
4文字だと証明が大変そうだけど、いい方法あるんですか?

940 :132人目の素数さん:2018/11/26(月) 15:57:48.91 ID:P3VGSuRj.net
a,b,c,d>0 に対して、
 9*(1/a^2 + 1/b^2 + 1/c^2 + 1/d^2) + 48/(aa+bb+cc+dd)
 ≧ 8{1/(ab) + 1/(ac) + 1/(ad) + 1/(bc) + 1/(bd) + 1/(cd)}.
等号は a = 3b = 3c = 3d.

検索中に見つけた。ちゃんと成り立つのかな?
http://www.mathoe.com/dispbbs.asp?boardID=107&ID=44556

941 :132人目の素数さん:2018/11/27(火) 02:28:27.59 ID:QMhuYErk.net
>>938
n≧4 のとき
(右辺) = (A/G)^n + (G/H)^n + 1^n
≧ 2(A/H)^(n/2) + 1   (AM-GM)
≧ (3/4){1 + (A/H)^(n/4)}^2,

2xx +1 - (3/4)(x+1)^2 = (x-1)(5x-1)/4 ≧ 0, (x≧1)

>>939
That's what I wanna know. (それは こっちが訊きたい...)

>>940
5点で等号成立ですね…
(a,b,c,d) = (1,1,1,1) (3,1,1,1) (1,3,1,1) (1,1,3,1) (1,1,1,3)

942 :132人目の素数さん:2018/11/27(火) 03:19:22.59 ID:QMhuYErk.net
>>937
 xx+xy+yy = (3/4)(x+y)^2 + (1/4)(x-y)^2 ≧ (3/4)|x+y|^2,
より
 (左辺) ≧ (√3)/2・(|a+b|+|b+c|+|c+a|)
 ≧ (√3)/2・|2a+2b+2c|
 = (√3)|a+b+c|,

ab+bc+ca ≧ 0 ならば 1394. が成立。 >>933 (2)

943 :132人目の素数さん:2018/11/27(火) 03:24:43.69 ID:oixSVMNZ.net
>>941
なんと、あっさり証明できるとは!

944 :132人目の素数さん:2018/11/27(火) 03:51:33.88 ID:oixSVMNZ.net
>>937
a,b,c∈R に対して、
√(aa+ab+bb) + √(bb+bc+cc) + √(cc+ca+aa) ≧ (3/2)*|a+b+c|.

[証]
xx+xy+yy - (x+ y/2)^2 = (3/4)*y^2 ≧0
∴ √(xx+xy+yy) ≧ |x+ y/2|

√(aa+ab+bb) + √(bb+bc+cc) + √(cc+ca+aa)
≧ |a+ b/2| + |b+ c/2| + |c+ a/2|
≧ |(a+ b/2) + (b+ c/2) + (c+ a/2)|
= (3/2)*|a+b+c|.

等号成立条件は a=b=c=0.
---------------------

>>942の等号成立条件は a=b=c だから、上式は緩くて次式が良いってことですかね?
√(aa+ab+bb) + √(bb+bc+cc) + √(cc+ca+aa) ≧ (√3)|a+b+c|.

945 :132人目の素数さん:2018/11/27(火) 05:43:13.00 ID:QMhuYErk.net
>>944
うん。(等号成立条件は a=b=c)

946 :132人目の素数さん:2018/11/27(火) 17:31:28.13 ID:oixSVMNZ.net
>>911で紹介して頂いたサイトから検索してみた。

>>887
>  n=4 のとき、(A-G)/(G-H) ≧ 9/16
>  CGMO-2011 A.4
>  inequalitybot [35]

a,b,c,d>0, abcd=1に対して、
1/a + 1/b + 1/c + 1/d + 9/(a+b+c+d) ≧(25/4).

(3数) https://artofproblemsolving.com/community/c6h497071
(3数) https://artofproblemsolving.com/community/c6h506861p2847126
(4数) https://artofproblemsolving.com/community/c6h422665p2389389
(n数) https://artofproblemsolving.com/community/c6h354052
(類題)http://www.mathoe.com/dispbbs.asp?boardID=107&ID=44556>>940

うーむ、分からん…。

947 :132人目の素数さん:2018/11/27(火) 23:19:46.82 ID:QMhuYErk.net
>>938
 n≧4 のとき
 (A/G)^n + (G/H)^n + 1 ≧ 3([(A/G)^2 + (G/H)^2 + 1]/3)^{n/2} ≧ 3([(A/H) + (A/H) + 1]/3)^{n/2},

 (A/G)^n + (G/H)^n + 1 ≧ (A/H)^{n/2} + (A/H)^{n/2} + 1≧ 3([(A/H) + (A/H) + 1]/3)^{n/2},

もある…

948 :132人目の素数さん:2018/11/28(水) 01:00:16.79 ID:gsXNT+S8.net
>>865

n=3 は 16x^3 +63x^2 -42x -25 = 0 の根
n=4 は 729x^5 +3923x^4 +9002x^3 -42x^2 -5107x -1849 = 0 の根。

min{(A-G)/(G-H)} ≧ 4(n-1)/nn,  …… Sqing の評価 (2018/June/03)
http://artofproblemsolving.com/community/c6h422665p2389389

949 :132人目の素数さん:2018/12/02(日) 03:31:03.51 ID:2QZ/NwgQ.net
〔問題2969〕
aa+bb+cc = 2 のとき
 {3 - (ab+bc+ca)}^3 /{(a+b-c)(b+c-a)(c+a-b)}^2 ≧ 27/8,
  (by K. Chikaya)

http://suseum.jp/gq/question/2969
http://www.casphy.com/bbs/highmath/472060/ (不等式2-324)

950 :132人目の素数さん:2018/12/04(火) 00:15:50.97 ID:OyfgKqsW.net
a,b,c>0 に対して、
(a+b+c)(1/a + 1/b + 1/c) ≧ 1 + 24(aa+bb+cc)/{(a+b+c)^2}.

951 :132人目の素数さん:2018/12/04(火) 00:24:16.17 ID:OyfgKqsW.net
最近の不等式botの、食べ物の写真のノイズが邪魔過ぎる。

952 :132人目の素数さん:2018/12/04(火) 03:55:41.62 ID:MMKOwiX9.net
>>951
京都市左京区一乗寺 「つけ麺 惠那く」
http://kyotopi.jp/articles/7dhfJ

大盛り450gです。(プラス100円)
多麺体だ〜

953 :132人目の素数さん:2018/12/04(火) 04:30:49.15 ID:MMKOwiX9.net
>>950

s = a+b+c,u = abc とおく。

(左辺) - (右辺)
= {c(a-b)^2 + a(b-c)^2 + b(c-a)^2}/u - (8/ss){(a-b)^2 + (b-c)^2 + (c-a)^2}
= (c/u -8/ss)(a-b)^2 + (a/u -8/ss)(b-c)^2 + (b/u -8/ss)(c-a)^2
= p(a-b)(a-c) + q(b-c)(b-a) + r(c-a)(c-b),

(b+c)ss ≧ 4a(b+c)^2 ≧ 4a(4bc) = 16u より p = (b+c)/u -16/ss ≧ 0,
同様にして
 q = (c+a)/u - 16/ss ≧ 0,
 r = (a+b)/u - 16/ss ≧ 0,
また、(a,b,c) と (p,q,r) は逆順序だから、Schur の拡張により
 p(a-b)(a-c) + q(b-c)(b-a) + r(c-a)(c-b) ≧ 0,

954 :132人目の素数さん:2018/12/04(火) 04:47:43.20 ID:OyfgKqsW.net
>>953
瞬殺ですな、乙でござる。

さて、>>950を書き込む際に出典を探したが見つけらず。
おそらく Vasile Cirtoaje だろうが、検索したが閲覧できず。
2年前には未完成のpdfが閲覧できたが、2018.07以降の書籍化が原因だろう。

7.Cirtoaje V. - Mathematical Inequalities, Volumes 1-5 (p. 344, 400, 486, 522, 544), Lambert Academic Publishing, 2018.
http://ac.upg-ploiesti.ro/vcirtoaje/vcirtoaje.php

955 :132人目の素数さん:2018/12/04(火) 05:04:22.35 ID:OyfgKqsW.net
>>950 >>953
Cauchyより強ければよかったのに。

956 :132人目の素数さん:2018/12/04(火) 15:04:24.55 ID:MMKOwiX9.net
>>950
4点で等号が成立
(a,b,c) = (1,1,1) (2,1,1) (1,2,1) (1,1,2)
          p=0  q=0  r=0
 
…てことは、これで最良でござる。  >>955

957 :132人目の素数さん:2018/12/04(火) 23:43:56.06 ID:OyfgKqsW.net
>>956
ほんとだ、不等号の向きを勘違いしていた。つまりこういうことですな。

a,b,c>0 に対して、
(a+b+c)(1/a + 1/b + 1/c) ≧ 1 + 24(aa+bb+cc)/{(a+b+c)^2}.≧9.

958 :132人目の素数さん:2018/12/05(水) 00:21:40.27 ID:FGF1AG7S.net
(1) a,b,c∈R に対して、(aa+bb+cc)^2 ≧ 3(a^3b+b^3c+c^3a).

(2) a,b,c,d>0, abcd=1 に対して、a^4b+b^4c+c^4d+d^4a ≧ a+b+c+d.

(3) a,b,c>0 に対して、3/4 ≦ Σ[cyc] ab/{(b+c)(c+a)} < 1.

959 :132人目の素数さん:2018/12/05(水) 18:21:09.37 ID:fLK6/i8S.net
>>958
(1)
 A = aa -bb +3bc,
 B = bb -cc +3ca,
 C = cc -aa +3ab,
とおくと
(左辺) - (右辺) = {(A-B)^2 + (B-C)^2 + (C-A)^2}/6 ≧ 0,
 >>244 (1) >>247

(2) AM-GMで
 {23(a^4)b +7(b^4)c +11(c^4)d +10(d^4)a}/51 ≧ a(abcd) = a,
巡回的にたす。

(3)
右)
 (中辺) = 1 - 2abc/{(a+b)(b+c)(c+a)} < 1,
左)
 (中辺) - 3/4 = {(a+b)(b+c)(c+a)-8abc}/(a+b)(b+c)(c+a) ≧ 0, (AM-GM)

960 :132人目の素数さん:2018/12/06(木) 01:12:38.36 ID:9xq/7gl/.net
>>958 (2)
ついでだけど、n文字の場合も AM-GM で
 Σ[j=1,n] k_j (a_j)^n a_{j+1} ≧ a_1(a_1・a_2…a_n) = a_1,
巡回的にたす。
 k_j = 1/(n+1) - (-1)^j・n^{n-j}/[n^n - (-1)^n] > 0,
 a_{n+1} = a_1,

961 :132人目の素数さん:2018/12/09(日) 13:27:41.69 ID:GNoL1vgk.net
アダマールの不等式

962 :132人目の素数さん:2018/12/09(日) 16:29:22.32 ID:GNoL1vgk.net
>>958-959
(1)は Vasc's Inequality というらしい。

Vascって人名かな?
https://i.imgur.com/FSQSdQY.jpg

963 :132人目の素数さん:2018/12/09(日) 16:38:35.94 ID:GNoL1vgk.net
a,b,c>0, aa+bb+cc=3 に対して、
(a^5)/(b^3+c) + (b^5)/(c^3+a) + (c^5)/(a^3+b) ≧ (3/2)*(abc)^2.

バスク大佐の不等式を使って証明できるらしい…

964 :132人目の素数さん:2018/12/10(月) 04:00:08.91 ID:IsN+FPDR.net
>>963
aa+bb+cc = S とおく。
a(b^3 +c) + b(c^3 +a) + c(a^3 +b) = (ab^3 +bc^3 +ca^3) + (ab+bc+ca) ≦ SS/3 + S,  (>>958 (1))

コーシーで
{a(b^3 +c) + b(c^3 +a) + c(a^3 +b)}(左辺) ≧ (a^3 +b^3 +c^3)^2 ≧ (aa+bb+cc)^3 /(1+1+1) = (1/3) S^3,

∴ (左辺) ≧ (S^3)/(SS+3S) = SS/(S+3),

965 :132人目の素数さん:2018/12/10(月) 14:28:08.88 ID:IsN+FPDR.net
>>959 (1)
4点で等号成立。
A=B=C より
(a, b, c) = (1, 1, 1) (1, 1+t, 3+1/t) = (1, 1+t, tt-3t-1)
 t は t^3 -3t^2 -4t -1 = 0 の根
 t = -0.69202147163 -0.3568958679 4.0489173395

966 :132人目の素数さん:2018/12/13(木) 13:13:26.57 ID:7ZQJsx3/.net
n次元ベクトル a_1、…、a_m の相加平均を A = (a_1+…+a_m)/m とおく。
任意のn次元ベクトル x に対して、
Σ[k=1 to m] |a_k - x|^2 = Σ[k=1 to m] |a_k - A|^2 + m*|x - A|^2.

むむっ、不等式じゃないな…

967 :132人目の素数さん:2018/12/13(木) 17:55:55.30 ID:hPiKmmhx.net
どれも-1以上である実数a,b,c,d,eはa+b+c+d+e=5であるという
このときの(a+b)(b+c)(c+d)(d+e)(e+a)の最大値と最小値を求めよ

2019年度中国数学オリンピック第一問

968 :132人目の素数さん:2018/12/14(金) 06:52:26.17 ID:DbCBFUHo.net
>>966

n次元ベクトル y = (y_1, y_2, …, y_n) に対して
|y|^2 = (y_1)^2 + (y_2)^2 + …… + (y_n)^2
∴ n個の成分に分けて考え、和をとればよい。
∴ n=1 (スカラー) の場合に帰着する。

a_k - x = (a_k - A) - (x - A),
(a_k -x)^2 = (a_k - A)^2 - 2(a_k - A)(x - A) + (x - A)^2,
となるが、右辺第2項は 和をとれば
納k=1,m] (a_k - A) = (a_1 + a_2 + … + a_m) - mA = 0,
となって消える。

969 :132人目の素数さん:2018/12/15(土) 02:39:44.86 ID:xhjRoR3J.net
>>967
 最大値 32  a=b=c=d=e=1 のとき。
 最小値 -512 {a,b,c,d,e} = {9,-1,-1,-1,-1} のとき。
かな

970 :132人目の素数さん:2018/12/15(土) 03:42:27.66 ID:xhjRoR3J.net
>>951
大阪の「三豊麺」もある。
http://sanpomen.jp/
 
重量(茹で上がり)
 並盛 1玉  350g
 大盛 1.5玉 550g
 特盛 2玉  750g
 山盛 2.5玉 900g (+100円)

(ミツトヨの秤で量ったんぢゃないけど)

大阪のみなみ周辺では、千日前、体育館前、日本橋に店舗があります。

971 :132人目の素数さん:2018/12/15(土) 10:03:02.54 ID:iiLDF3is.net
>>970
もしかして面白いと思って書いているのだとしたら、反省した方がいいでござるよ。
荒らすのは止めましょう。

972 :132人目の素数さん:2018/12/15(土) 10:08:48.90 ID:G/Zv1nOc.net
>>969
最大値が間違ってるみたいだね

973 :132人目の素数さん:2018/12/15(土) 10:20:56.92 ID:iiLDF3is.net
>>968-969>>972
たとえば、(a,b,c,d,e) = (-1,-1,-1,4,4)のとき、288.

この手の不等式を見たときに、方針がぱっと出てこない悲しさ。

974 :132人目の素数さん:2018/12/15(土) 10:43:58.04 ID:iiLDF3is.net
最大値や最小値をとるときの変数に、限界値の-1が入るのは理由があるのかな?

975 :132人目の素数さん:2018/12/16(日) 13:25:07.42 ID:u7Il/9pT.net
>>967
(1) a,b,c>-1, a+b+c=3 のとき、-32≦(a+b)(b+c)(c+a)≦8.
(2) a,b,c,d>-1, a+b+c=4 のとき、-48≦(a+b)(b+c)(c+a)≦144.
(3) a,b,c,d,e>-1, a+b+c+d+e=5 のとき、-512≦(a+b)(b+c)(c+d)(d+e)(e+a)≦288.

976 :132人目の素数さん:2018/12/16(日) 13:54:54.32 ID:u7Il/9pT.net
書き直し
>>967
(1) a,b,c>-1, a+b+c=3 のとき、-32≦(a+b)(b+c)(c+a)≦8.
(2) a,b,c,d>-1, a+b+c=4 のとき、-48≦(a+b)(b+c)(c+d)(d+a)≦144.
(3) a,b,c,d,e>-1, a+b+c+d+e=5 のとき、-512≦(a+b)(b+c)(c+d)(d+e)(e+a)≦288.

977 :132人目の素数さん:2018/12/16(日) 14:36:59.67 ID:6WIjEwcK.net
>>976
これは一般化できそう

978 :132人目の素数さん:2018/12/16(日) 15:54:23.07 ID:u7Il/9pT.net
>>976
また書き間違ってるな (2)の条件のところ

979 :132人目の素数さん:2018/12/16(日) 15:54:47.11 ID:u7Il/9pT.net
もう一度書き直し
>>967
(1) a,b,c>-1, a+b+c=3 のとき、-32≦(a+b)(b+c)(c+a)≦8.
(2) a,b,c,d>-1, a+b+c+d=4 のとき、-48≦(a+b)(b+c)(c+d)(d+a)≦144.
(3) a,b,c,d,e>-1, a+b+c+d+e=5 のとき、-512≦(a+b)(b+c)(c+d)(d+e)(e+a)≦288.

980 :132人目の素数さん:2018/12/17(月) 06:11:59.37 ID:X9YWLOp7.net
〔予想〕
 a_1, a_2, …, a_n ≧ -1, a_1+a_2+…+a_n = n, のとき

・n:奇数 (n≧5) ならば
 -(2^n)(n-1)^2 ≦ Π(a_j + a_{j+1}) ≦ 2^{n-2} (n-2)^2 (n-1),

・n:偶数 ならば
 -2^{n-2} (n-2)^2 (n-1) ≦ Π(a_j + a_{j+1}) ≦(2^n)(n-1)^2,

「良そう」「止そう」と意見が割れるかも知りませぬが…

(不等式スレも平成のうちに2桁に到達でござる。思えば長い道でござった。)

981 :132人目の素数さん:2018/12/17(月) 15:34:10.00 ID:zsFrN7jo.net
第1章が2003年。
スレ立て放置されていた不等式スレを占拠して15年も経つのか…。

982 :132人目の素数さん:2018/12/17(月) 17:41:33.58 ID:X9YWLOp7.net
[初代スレ.019]
いいよ!

[初代スレ.034]
正の数 a,b,x,y,X,Y に対して
 (axX+byY)^3 ≦ (a^3+b^3)(x^3+y^3)(X^3+Y^3),

(右辺) - (左辺)
= (axY)^3 + (ayX)^3 + (bxX)^3 -3(byY)(axX)^2
 + (bxY)^3 + (byX)^3 + (ayY)^3 -3(axX)(byY)^2
= (axY)^3 + (ayX)^3 + (bxX)^3 -3(axY)(ayX)(bxX)
 + (bxY)^3 + (byX)^3 + (ayY)^3 -3(bxY)(byX)(ayY)
≧ 0,   (AM-GM)
スッキリ。

[初代スレ.039]
(参考書)
秋山 仁 + ピーター・フランクル「[完全攻略] 数学オリンピック」日本評論社 (1991/Nov)
p.24-25 [例4-3]

983 :132人目の素数さん:2018/12/17(月) 20:12:55.21 ID:zsFrN7jo.net
懐かしい。当時は tanスレや nCrスレや、おいらには解けないスレとかあったよなぁ…。
パソコンを何度も買い替えて、今となっては過去ログが見れないが。

984 :132人目の素数さん:2018/12/18(火) 03:40:20.65 ID:e1oKVpnI.net
>>979
(1) a+b, b+c, c+a のうち負は高々1個。

a+b, b+c, c+a ≧0のとき、AM-GMより、
0 ≦ (a+b)(b+c)(c+a) ≦ [ {(a+b)+(b+c)+(c+a)}/3]^3 = 8.

1つだけ負のとき、対称性から a+b < 0 ≦ b+c, c+a としてよい。
このとき、条件より 3<c≦5 で、AM-GMより、
0 ≦ -2(a+b)(b+c)(c+a) ≦ [ {-2(a+b)+(b+c)+(c+a)}/3]^3 = (c-1)^3 ≦ 64.
(証明終)

985 :132人目の素数さん:2018/12/18(火) 07:27:44.02 ID:TqVGX/9j.net
〔補題〕
正の実数a,b,cに対して
[1] (b/a) + (c/b) + (a/c) - {a/(b+c) + b/(c+a) + c/(a+b)} ≧ (ab+bc+ca)^2 /{2(a+b+c)abc} ≧ 3/2,
[2] a/(b+c) + b/(c+a) + c/(a+b) ≧ (a+b+c)^2 /2(ab+bc+ca) ≧ 3/2,

(略証)
[1]
(左辺) - (右辺)
 = ab/(c(b+c)) + bc/(a(c+a)) + ca/(b(a+b))
 = (ab+bc+ca)^2 /{abc[(b+c) + (c+a) + (a+b)]}
 ≧ (ab+bc+ca)^2 /{2(a+b+c)abc},   (←コーシー)
 JMO-2004、[初代スレ.058]

[2] もコーシーで出る。

なお、1/a = A, 1/b = B, 1/c = C とおくと
 (ab+bc+ca)^2 /{(a+b+c)abc} = (A+B+C)^2 /(AB+BC+CA),
 (a+b+c)^2 /(ab+bc+ca) = (AB+BC+CA)^2 /{(A+B+C)ABC},

(類) Nesbitt-Igarashi  >>627 >>835

986 :132人目の素数さん:2018/12/18(火) 18:31:13.09 ID:e1oKVpnI.net
>>980
偶奇で異なることを見抜いたのは、何かコツがあるの?

987 :132人目の素数さん:2018/12/18(火) 19:23:44.68 ID:TqVGX/9j.net
>>985
 辺々たして…

[3] (b/a) + (c/b) + (a/c) ≧ tt/2su + ss/2t ≧ √(st/u) ≧ 3,
 ここに s = a+b+c, t = ab+bc+ca, u = abc.

 st-9u = a(b-c)^2 + b(c-a)^2 + c(a-b)^2 ≧ 0,

988 :132人目の素数さん:2018/12/18(火) 19:41:13.00 ID:TqVGX/9j.net
>>986

{-1,……,-1,2n-1} のとき (-1)^n・(2^n)(n-1)^2
{-1,…,-1,n-1,n-1} のとき - (-1)^n・2^{n-2} (n-2)^2 (n-1)
かとオモタ。

989 :132人目の素数さん:2018/12/18(火) 21:15:32.22 ID:e1oKVpnI.net
>>979
よく見たら、条件式のa,b,c>-1のところが、書き間違ってるな。
正しくは a,b,c≧-1など。

990 :132人目の素数さん:2018/12/18(火) 21:42:52.76 ID:e1oKVpnI.net
>>979
(1)
a+1=A, b+1=B, c+1=C とおくと、問題は
『A,B,C≧0, A+B+C=6 のとき、32≧(A-4)(B-4)(C-4)≧-8』.

A+B+C=s(=6), AB+BC+CA=t, ABC=u とおくと、不等式は 0≦4t-u≦40.

う〜む、非同次は難しい。
s=6だから、無理やり同時にすると、0≦2st-3u≦120.
これではダメか…。

991 :132人目の素数さん:2018/12/18(火) 23:13:50.02 ID:TqVGX/9j.net
>>990 (1)
s, t, u で表わせば
(与式) = (a+b)(b+c)(c+a)
 = (A+B - s/3)(B+C - s/3)(C+A - s/3)
 = (2s/3 - C)(2s/3 - A)(2s/3 - B)
 = -4(s/3)^3 + (2/3)st -u,

(与式) = - 4(s/3)^3 + (1/9){5st+(st-9u)} ≧ -4(s/3)^3,
(与式) = (s/3)^3 - (2/27)s(ss-3t) - (1/9)(s^3 -4st+9u) ≦ (s/3)^3,

992 :132人目の素数さん:2018/12/19(水) 04:07:16.90 ID:K5b8go44.net
>>990の続きだけど、これで合ってるかな?

> (1) a,b,c>-1, a+b+c=3 のとき、-32≦(a+b)(b+c)(c+a)≦8.
>
> a+1=A, b+1=B, c+1=C とおくと、問題は
> 『A,B,C≧0, A+B+C=6 のとき、32≧(A-4)(B-4)(C-4)≧-8』.
>
> A+B+C=s(=6), AB+BC+CA=t, ABC=u とおくと、不等式は 0≦4t-u≦40.


s=6を使って、同次化して、0 ≦ 9(2st-3u) ≦ 5s^3.
これを証明する。

右側 : 5s^3 - 9(2st-3u) = 5(s^3-4st+9u) + 2(st-9u) ≧0.
左側 : 9(2st-3u) = 15st + 3(st-9u) ≧0.

993 :132人目の素数さん:2018/12/19(水) 06:03:45.97 ID:5zoTD2o3.net
>>992
 正解です!

なお、
st - 9u = a(b-c)^2 + b(c-a)^2 + c(a-b)^2 ≧ 0,

s^3 - 4st + 9u = a(a-b)(a-c) + b(b-c)(b-a) + c(c-a)(c-b)
= {ab(aa-bb)^2 + bc(bb-cc)^2 + ca(cc-aa)^2}/{(a+b)(b+c)(c+a)}
≧ 0,

http://dic.nicovideo.jp/a/シューアの不等式

994 :132人目の素数さん:2018/12/19(水) 06:36:06.17 ID:K5b8go44.net
>>979
(2)を>>984のように場合分けして解こうとしているんだけど、うまくいかん。

995 :132人目の素数さん:2018/12/19(水) 06:44:01.18 ID:5zoTD2o3.net
>>984 >>986 >>989 >>990 >>992 >>994
 続きは次スレで…

   ___
 ./  ≧ \
 |::::  \ ./ |
 |::::: (● (● | グッジョブ!
 ヽ::::... .ワ....ノ    n  
 ̄ ̄   \    ( E)
フ     /ヽ ヽ_//

[初代スレ.998]

996 :132人目の素数さん:2018/12/19(水) 06:44:55.05 ID:5zoTD2o3.net
        ハァハァ   ∩
               ( ⌒)_   ∩_ _ グッジョブ!!
グッジョブ!! .___  //,. ノ≧ \ .i .,,E)__
     / nCr  \| / /\ ./ |/ / cos \
  _n  .|::::\ ./  |/ /(● (● | ノ\ ./ |
 ( l  |::●) ●) .| /:::... .ワ ....ノ/(● (● |   グッジョブ!!
  \ \ヽ:::::.∀   .ノ      /ヽ:::::... .▽....ノ  n
   ヽ__ ̄   ノ ヽ      |  ̄     \    ( E)
     /    /    \    ヽ フ    / ヽ ヽ_//

[第5章.991]

997 :132人目の素数さん:2018/12/19(水) 06:45:32.33 ID:5zoTD2o3.net
    ___
  ./  ≧ \   神降臨キタ━(゚∀゚)━!!!
  |::::  \ ./ | ハァハァ
  |::::: (● (● |
  ヽ::::... .∀....ノ /  チン ☆
 _(  ⊃  ⊃  チン ☆
 |\ ̄ ̄ ̄ ̄旦 ̄\
 | | ̄ ̄ ̄ ̄ ̄ ̄ ̄|
 \|  愛媛みかん |
    ̄ ̄ ̄ ̄ ̄ ̄ ̄
[第5章.996]

998 :132人目の素数さん:2018/12/19(水) 06:46:20.72 ID:5zoTD2o3.net
 "; ;ヾ; ;ヾ; ;メヾ "ゞ ;ヾ ;ゞ ;" "ゞ ; ; ; ゞ ;" "ゞ";ヾ ; ヾ ;ゞ; ;ゞ ;ゞ ;" "ゞ        /.             ヽ
 ;" "ゞ ; ; ; ゞ ; ;ヾ ; ; ヾ ;ゞ;ヾ ; ;";ヾ; ;"/" ; ;ヾ ;ヾ; ヾ ; ヾ ;ゞ; ;ゞ ;" ";ゞ ; ;ヾ      l             l
 " ;ヾ ; ;";ヾ; ;"/" ; ;ヾ ;ヾ;ヾ ; ;ヾ ; ; ヾ ;ゞ  " ;ヾ ; ;";ヾゝゝ" ;ヾゞ           ヽ            /
,." ;ヾ ; ;";ヾ; ;"/" ; ;ヾ ;ヾ;ヾ ; ;ヾ ; ; ヾ ;ゞ  " ;ヾ ; ;";ヾゝゝ" ;ヾ ; ; ヾ ;ゞ;        \        /
 ゞヾ ; ;" ; ; ;; ;"iiiiii;;;;;::::: :)_/ヽ,.ゞ:,,ヾゞヾゞ__;::/        `      `        `   ー ─ ' `
   ゞヾゞ;\\iiiiii;;;;::::: :|;:/ヾ; ;ゞ "ゝゞ ; ;`
 " ;゛ ; ;" ; ;ゞ "|iiiiii;;;;::: : |:/ ヾゞ        `         `      ` `
  `      ,|i;iiiiiii;;;;;;::: :| `    `         `     `      ` `   `
        ,|iiii;iiii;;;;:;_ _: :| ___  秋の夜長に不等式    `        `        `,
   `    |iiiiiii;;;;;;((,,,):::|/  ≧ \                    ヾ从//"
    `   |iiiiiiii;;;;ii;;;;;;;;::|::::: (● (● |           `  ゙  `    ヾ'./"
         |iiiiii;iii;;;;i;;:: ::::|ヽ::::......ワ...ノ                 ○     .||.       ,
    `   |iii;;iiiii;::;:;;;;::::::| ( つ且 ~      `              ○○   | |
  , , .,.. ,..M|M|iMii;;ii:i;;i:i;:; ゝ つつ.,.. ,...... ,.... ,,,.,.. ,.... ,,,.,.. ,..,,,,.,...,..,.,| ̄ ̄|,.,..(  ).. ,,,..,,.. ,.... ,,,.,...,.. .. ,.... ,,,.,.. ,.... ,,,

[前スレ.998]

999 :132人目の素数さん:2018/12/19(水) 06:47:12.97 ID:5zoTD2o3.net
_| ::|_
 ̄| ::|/|           ┌──┐
  | ::|  |     .┌──┐| ∧_∧  いいな、俺たちの誰かが殉職したら・・
/|_|  |┌──┐| ∧_∧|(・ω・` )
  |文|  | | ∧_∧(    )⊂   )
  | ̄|  | | (    )⊂   ) (_Ο Ο :::
  | ::|  | | ⊂   ) (_Ο Ο わかってる、生き延びた奴が
  | ::|/ .|_ (_Ο Ο ::::::::: :::::: 不等式を収集し、証明する !
  | ::| :::::::::::::::::::::::::::::::: 俺たちゃ死んでも仲間だぜ !!

[前スレ.999]

1000 :132人目の素数さん:2018/12/19(水) 06:48:05.42 ID:5zoTD2o3.net
  ┏━━━┓
  ┃ Q.E.D. ┃
  ┗━┳━┛
( ゚∀゚) ノ

[前スレ.1000]

1001 :2ch.net投稿限界:Over 1000 Thread
2ch.netからのレス数が1000に到達しました。

総レス数 1001
438 KB
掲示板に戻る 全部 前100 次100 最新50
read.cgi ver 2014.07.20.01.SC 2014/07/20 D ★